Vous êtes sur la page 1sur 696

Exámenes de la

OLIMPIADA NACIONAL DE QUÍMICA

MÉXICO
San Luis Potosí 2006

Toluca 2007

Oaxaca 2008

Campeche 2009

Veracruz 2010

Toluca 2011

Guadalajara 2012

Chihuahua 2013

Distrito Federal 2014

Guadalajara 2015

“Guanajuato 2016”

Compilación: Comité Académico de la Olimpiada de Química del


Distrito Federal

2016
2015

********************************************************************************

OLIMPIADA NACIONAL DE QUÍMICA

En el marco del programa Olimpiadas Nacionales de la Ciencia, la Academia Mexicana de Ciencias


organiza la Olimpiada Nacional de Química (ONQ) en colaboración con la Facultad de Química de
la UNAM, Asociación Mexicana de Química Analítica y la Sociedad Química de México.

Este Concurso anual tiene como propósito elegir a los mejores estudiantes preuniversitarios de
química del país a integrar las delegaciones que representarán a México en la Olimpiada
Internacional de Química y en la Olimpiada Iberoamericana de Química.

Otros objetivos primordiales de la ONQ son:

 Promover el estudio de las Ciencias Químicas y estimular el desarrollo de jóvenes talentos


en esta Ciencia.
 Contribuir a la profundización de la amistad entre los estudiantes participantes y crear un
marco propicio para fomentar la cooperación, el entendimiento y el intercambio de
experiencias.
 Destacar la importancia de la enseñanza experimental de la Química y promover que todas
las escuelas de educación media del país cuenten con laboratorios bien equipados y
funcionales.

Este concurso se organiza anualmente y consta de tres etapas:

a) Concursos estatales.
b) Concurso nacional.
c) Selección y entrenamiento de las delegaciones que representan a México en las olimpiadas
internacionales e iberoamericanas.
En las dos primeras etapas los estudiantes, que deben haber estado inscritos en alguna institución
mexicana de enseñanza pre-universitaria en el año que participan y al menos dos años anteriores,
pueden competir en dos niveles diferentes según su nivel de estudio (Nivel A: Hayan cursado o
estén cursando el último año de los programas de enseñanza de nivel medio superior; Nivel B:
Hayan cursado o estén cursando el penúltimo año de los programas de enseñanza de nivel medio
superior, o estén cursando cualquier año de enseñanza media). La inscripción es individual y
gratuita y debe hacerse a través del delegado estatal o regional correspondiente.

En la etapa del concurso nacional, la competencia consta de varios exámenes teóricos y


experimentales, estos últimos tienen un peso del 40% en la calificación global. En esta fase se elige
un grupo de estudiantes que reciben entrenamiento y del cual, con base en diversos exámenes, se
seleccionan las delegaciones de seis estudiantes (cuatro titulares y dos suplentes). Cuatro estudiantes
de cada una de estas selecciones representan a México en las competencias internacional e
iberoamericana.

México participó por primera vez en la XXIV Olimpiada Internacional de Química, celebrada en los
Estados Unidos de América en julio de 1992; desde entonces a la fecha, en las veinticuatro
olimpiadas en que se ha competido, los estudiantes mexicanos han obtenido 8 medallas de plata, 34
de bronce y 15 menciones honoríficas.

México ha participado desde la primera Olimpiada Iberoamericana celebrada en Mendoza,


Argentina en 1995. Los estudiantes mexicanos que han representado al país en estos concursos han
obtenido alguna distinción: 14 menciones honoríficas, 58 medallas de bronce, 41 de plata y 19 de
oro.

En cada estado hay uno o dos delegados que son los responsables de la organización del concurso
regional en su entidad. Esta etapa consta de exámenes teóricos y, en ocasiones, de un examen
experimental.

Comité Académico de la Olimpiada Nacional de Química

Carlos Mauricio Castro Acuña


Ramiro Eugenio Domínguez Danache
María Antonia Dosal Gómez
Juan Carlos Hernández Chacón
Fernando León Cedeño
Mercedes Llano Lomas
Armando Marín Becerra
José Manuel Méndez Stivalet
Gloría Pérez Cendejas
Eugenio Octavio Reyes Salas

Academia Mexicana de Ciencias

Ma. Carmen Quintanar Martínez


Alejandra Piña Arias
Renata Villalba Cohen
Jaime Urrutia Fucugauchi
********************************************************************************
Delegados Estatales de la Olimpiada de Química en México (2015)

Aguascalientes: Irma Adriana Castro Gallo


Baja California: Rogelio Ortiz Salinas y Mario Alberto Ramírez Cruz
Baja California Sur: Aurora Rebolledo Martínez
Campeche: Sergio Monroy Cornejo
Chiapas: Silvia Rincón Serrano
Chihuahua: Daniela Yenthile Rodríguez Hernández
Coahuila: Edith Colunga Urbina e Ileana de la Garza
Colima: Roberto Atilano Coral
Distrito Federal: Blas Flores Pérez y Héctor García Ortega
Durango: J. Enrique Torres Cabral y Raúl Olvera Corral
Estado de México: María Dolores Arce Granados y José Enrique Robles Soto
Guanajuato: Víctor Manuel Mejía Cobos y Jesús Raúl Lugo Martínez
Guerrero: Guadalupe Toache Berttolini
Hidalgo: María Elena Páez Hernández y Gloria Sánchez Cabrera
Jalisco: Gabriel Palacios Huerta y Eulogio Orozco Guareño
Michoacán: Luis Nieto Lemus
Morelos: Eduardo Ángel García Ramírez
Nayarit: Luis Armando Ríos Magallanes
Nuevo León: Cruz Imelda Páez Garza y Perla Elizondo Martínez
Oaxaca: Laura Bello Fernández L. y Marco A. Sánchez Medina
Puebla: Alfredo Cesar Benítez Rojas
Querétaro: Alfonso Pérez Buenrostro y Carlos Gerardo Zavala Porto
Quintana Roo: Julián Atilano Díaz
San Luis Potosí: Norma Elisa Gazcón Orta
Sinaloa: Juan Bernardo Castañeda Sánchez e Isidro Jiménez Osuna
Sonora: David Octavio Corona Martínez
Tabasco: Carlos Lobato García y Armando Escobar Ramos
Tamaulipas: Enrique González Rodríguez
Tlaxcala: Moisés Merlo Cortina
Veracruz: Edith Aideé Cárdenas Uribe
Yucatán: Manuel Barceló Quintal y Ángel Trejo Irigoyen
Zacatecas: Jorge Enrique Núñez Monreal y José Carranza Téllez

Felicidades a todos, y sobre todo al Comité Académico de la Olimpiada


Nacional de Química, por estos 25 años de Olimpiadas.
Hablamos de química, pero… ¿En dónde se encuentra la química?

La química la podemos encontrar en todos lados, en la ropa que vestimos, en el esmog del Distrito
Federal (magnífica ciudad), en las medicamentos que tomamos, en los discos compactos que
utilizamos, en los superconductores, en los anticonceptivos, en los celulares, en los alimentos que
ingerimos, en las emociones, el miedo y el dolor que sentimos y en muchos otros asuntos que
también tienen que ver con la química.

Lo anterior intenta reflejar el hecho de que la química se encuentra en todas partes, y es una
construcción humana realizada a lo largo de la historia. Pero todavía hay mucho por hacer y para
nosotros como químicos (que muchas veces vestimos una bata medio limpia y con algunos
agujeros), la química es muy importante y a la vez divertida.

La historia de la química es la historia de hombres y mujeres que buscan entender y transformar el


mundo, y esperamos que en un futuro no muy lejano algunos de los jóvenes participantes en los
concursos de las Olimpiadas de Química, sigan formando parte de esta grandísima historia, ya que
uno de los objetivos de estos eventos es el de contribuir a elevar el nivel de enseñanza de la Química
y a que exista una mejor comprensión de los grandes beneficios que esta ciencia ha aportado a la
humanidad.

Finalmente, el Comité Académico de la Olimpiada de Química del Distrito Federal quiere agradecer
el apoyo brindado para realizar estos eventos a:

La Facultad de Química de la UNAM, a los Departamentos de Química del CINVESTAV y de la


UAM-Iztapalapa, a la Academia Mexicana de Ciencias, a la Asociación Nacional de la Industria
Química, a los Académicos del Comité de la Olimpiada Metropolitana de Química, a los Delegados
Estatales de las Olimpiadas de Química en México, al Comité Académico de la Olimpiada Nacional
de Química, a los alumnos que han participado en este concurso, y a los papás y maestros de los
mismos por el impulso que han dado a esta olimpiada durante estos XXVI años.

Comité Académico de la Olimpiada de Química del Distrito Federal

Armando Ariza Castolo


Blas Flores Pérez
Héctor García Ortega
José Reyes Alejandre Ramírez
María del Rocío Santillana Hinojosa

(José Andrés Cedillo Ortiz, Víctor Manuel Ugalde Saldivar, Elizabeth Nieto Calleja, Juan Marcos
Esparza Schulz).

http://depa.fquim.unam.mx/olimpiada/
TEMARIO

QUÍMICA ORGÁNICA. NIVEL B.


1. Alcanos. Nomenclatura IUPAC. Propiedades físicas.
2. Alquenos. Nomenclatura IUPAC.
3. Alquinos. Nomenclatura IUPAC.
4. Haluros de alquilo. Nomenclatura.
5. Compuestos aromáticos. Benceno: estructura, aromaticidad. Derivados del benceno.
Nomenclatura.
6. Alcoholes y fenoles. Nomenclatura y clasificación.
7. Aldehídos y cetonas. Estructura y nomenclatura.
8. Ácidos carboxílicos. Nomenclatura IUPAC.
9. Aminas. Nomenclatura y clasificación.
10. Aminoácidos. Estructura iónica de los aminoácidos.
11. Proteínas. Estructura básica de las proteínas.
12. Ácidos grasos y grasas. Jabones y detergentes.
13. Carbohidratos. Glucosa y fructosa.

QUÍMICA ORGÁNICA. NIVEL A.


1. Además del temario del Nivel B.
2. Alcanos. Estructura. Nomenclatura IUPAC y común. Hibridación sp3. Obtención. Reacciones
principales de los alcanos: halogenación, oxidación y pirólisis.
3. Cicloalcanos. Estructura. Nomenclatura IUPAC. Conformación de silla y bote. Estabilidad de los
sustituyentes de los cicloalcanos: enlaces ecuatoriales y axiales, isómeros cis-trans.
4. Alquenos. Estructura. Nomenclatura IUPAC y común. Hibridación sp2. Isomería cis-trans (E y
Z). Métodos de obtención en el laboratorio. Reacciones principales de los alquenos: reducción,
adición (regla de Markovnikov) y ozonólisis.
5. Alquinos. Estructura. Nomenclatura IUPAC y común. Hibridación sp. Métodos de obtención en
el laboratorio. Reacciones principales de los alquinos: adición, oxidación y reducción. Acidez de
los alquinos.
6. Haluros de alquilo. Estructura. Nomenclatura IUPAC y común. Métodos de obtención.
Reacciones de sustitución nucleofílica (SN2 y SN1) y de eliminación (E2 y E1). Reacciones de
identificación de derivados halogenados.
7. Compuestos aromáticos. Nomenclatura IUPAC y común. Benceno: estructura y aromaticidad
(resonancia). Reacciones de sustitución y eliminación. Reacciones de sustitución electrofílica y
efecto del sustituyente. Alquilbencenos.
8. Alcoholes y fenoles. Estructura. Nomenclatura IUPAC y común. Identificación. Síntesis de
alcoholes. Obtención de fenoles. Acidez de alcoholes y fenoles. Reacciones de alcoholes: ruptura
del enlace C-OH, ruptura del enlace O-H. Reacciones de fenoles: Acidez. Formación de éteres y
ésteres. Reacciones de identificación de alcoholes y fenoles.
9. Aldehídos y cetonas. Nomenclatura IUPAC y común. Métodos de obtención de aldehídos:
oxidación de alcoholes primarios y metilbencenos, reducción de cloruros de ácido. Métodos de
obtención de cetonas: oxidación de aldehídos y de metilcetonas (reacción de haloformo),
reducción, adición nucleofílica (Grignard, cianuros, derivados de amoníaco). Enlace cetal,
hemicetal, su importancia en los azúcares. Acidez de los hidrógenos al grupo carbonilo:
tautomería ceto-enol. Condensación aldólica. Reacciones de identificación de grupo carbonilo.
10. Ácidos carboxílicos. Estructura. Nomenclatura IUPAC y común. Fuerza ácida, efectos
inductivos. Métodos de obtención: oxidación: hidrólisis de nitrilos, de ésteres y carbonatación
de reactivos de Grignard. Reacciones de ácidos carboxílicos: conversión a cloruros de ácido,
ésteres y amidas. Ácidos di- o tricarboxílicos. Ácidos aromáticos y su obtención. Reacciones de
identificación de grupo carboxilo.
11. Derivados de ácidos carboxílicos.
Cloruros de ácido. Estructura. Nomenclatura IUPAC y común. Obtención y reacciones
(obtención de ácidos, amidas, ésteres y acilación de Friedel-Crafts).
Anhídridos de ácido. Estructura. Nomenclatura IUPAC y común. Obtención y reacciones:
hidrólisis, obtención de amidas, ésteres y acilación de Friedel-Crafts.
Ésteres. Estructura. Nomenclatura IUPAC y común. Obtención e hidrólisis.
Amidas. Estructura. Nomenclatura IUPAC y común. Obtención e hidrólisis.
12. Aminas. Nomenclatura IUPAC y común. Reacciones de identificación de aminas primarias,
secundarias y terciarias (prueba de Hinsberg). Métodos de obtención: reducción de grupos nitro
o nitrilo, y reacción de halógenos con amoníaco. Basicidad de aminas aromáticas y alifáticas.
Reacciones: conversión a amidas, reacciones con ácido nitroso. Obtención y reacciones de sales
de diazonio (colorantes).
13. Estereoquímica. Confórmeros. Proyecciones de Fischer y de Newman. Estereoisómeros.
Enantiómeros. Diastereoisómeros. Configuraciones R y S. Isomería óptica.
14. Aminoácidos y péptidos. Punto isoeléctrico. Clasificación en grupos de los veinte
aminoácidos. Enlace peptídico.
15. Proteínas. Desnaturalización por cambio de pH, temperatura, metales y EtOH.
16. Ácidos grasos y grasas. Estructura. Nomenclatura IUPAC desde el C4 al C18. Micelas.
17. Carbohidratos. Glucosa y fructuosa, sus proyecciones de Fischer y de Haworth. Diferencias
entre la α y β D-glucosa. Reacciones de las aldosas. Formación de glucósidos, mutarrotación de
la glucosa. Enlace glucosídico de los disacáridos.
18. Macromoléculas. Polímeros naturales: almidón, celulosa. Polímeros sintéticos, propiedades.
Polímeros de adición: polietileno y poliestireno. Polímeros de condensación: poliamidas,
poliésteres. Reciclaje de polímeros.

FISICOQUÍMICA. NIVEL B.
1. Termodinámica. Sistema y alrededores. Primera ley de la termodinámica. Energía, calor y
trabajo. Capacidad calorífica (definición).
2. Gases. Ley del gas ideal. Definición de presión parcial. Ley de Dalton.
3. Sistemas de fases. Líquidos puros: Dependencia de la presión de vapor de un líquido con
respecto a la temperatura. Ecuación de Clausius Clapeyron, ley de Raoult, regla de Trouton,
Ecuación de Antoine.
4. Soluciones ideales. Propiedades coligativas. Elevación del punto de ebullición. Depresión del
punto de congelación. Determinación de masa molar. Presión Osmótica.
5. Equilibrio químico. Modelo dinámico del equilibrio químico equilibrio expresado en término de
concentraciones relativas y de presiones parciales relativas. Relación entre la constante de
equilibrio para gases ideales expresada en términos diferentes (concentraciones, presiones,
fracción molar). Aspectos cualitativos (principio de LeChatelier).
6. Equilibrio iónico. Teoría de Arrhenius y de Bronsted-Lowry de ácidos y bases. Potencial
estándar de electrodo. Ecuación de Nernst. Leyes de Faraday.
7. Cinética de reacciones homogéneas. Factores que afectan la rapidez de reacción de 1er orden.
Coordenada de reacción y la idea básica del estado de transición.
FISICOQUÍMICA. NIVEL A.
1. Además del temario del Nivel B.
2. Termodinámica. Relación entre entalpía y energía. Funciones de estado. Diferencia entre Cp y
Cv. Ley de Hess. Uso de las entalpías estándar de formación. Entalpías de combustión de
solución y solvatación. Energías de enlace (definición y usos). Ley de Kirchhoff (ΔH a otras
temperaturas).
3. Segunda ley. Ciclo de Carnot y otros procesos. Definición de entropía (qrev/T). Entropía y
desorden (Aspecto fenomenológico de la entropía y cálculo de ΔS en procesos: isotérmicos,
isobáricos, isocóricos, adiabáticos, reversibles y no reversibles, cálculo de ΔS, cambio de estado
PVT, y procesos físicos: fusión, vaporización, sublimación, transición y en mezclado de gases).
Relación ΔG = ΔH - TΔS. ΔG y direccionalidad de los cambios.
4. Gases. Propiedades críticas. Desviación de la idealidad: Z de compresibilidad.van der Waals:
Cálculo de P, T, a y b. Propiedades críticas: Propiedades reducidas, Z generalizado.
5. Sistemas de fases.
6. Soluciones ideales. Ley de Henry. Ley de Raoult.
7. Equilibrio químico. Relación entre la constante de equilibrio para gases ideales expresada en
términos diferentes (concentraciones, presiones, fracción molar). Dependencia de K con la
temperatura (ΔHr y ΔSr constantes): Analítico y gráfico.
8. Equilibrio iónico. Equilibrio de electrodos. Definición de fuerza electromotriz. Electrodos de
primera clase. Electrodos de segunda clase.
9. Cinética de reacciones homogéneas. Ecuación de rapidez (diferencial). Constante de rapidez
(definición). Orden de reacción (concepto). Reacción de 1er orden. Dependencia del tiempo y la
concentración en reacciones de primer orden. Vida media. Relación entre vida media y constante
de rapidez. Paso determinante de la rapidez de reacción. Molecularidad. Definición de energía de
activación y ecuación de Arrhenius. Cálculo de la rapidez y constante de rapidez para reacciones
de 1er orden.

QUÍMICA INORGÁNICA. NIVEL B.


1. Estructura atómica y tabla periódica. Configuración electrónica, principio de exclusión de
Pauli, primera regla de Hund. Grupos principales. Tendencias generales de los parámetros
atómicos dentro de los principales grupos en la Tabla Periódica: tamaño atómico, estados de
oxidación, carácter metálico.
2. Formulación y nomenclatura IUPAC. Los elementos naturales, sustancias simples, compuestos
de los elementos de los grupos principales, compuestos de metales de transición del bloque "d".
3. Masa atómica relativa. Isótopos. Conteo de nucleones (número de masa, número atómico) y
abundancias relativas.
4. Estequiometría. Balance de ecuaciones. Relaciones de masa y volumen. Fórmulas empíricas.
Masa molar. Constante de Avogadro.
5. Elementos del bloque "s". Reacción de los metales con oxígeno. Reacción de los metales con el
agua; basicidad relativa. Reacción de los metales con los halógenos. Hidruros.
6. Elementos del bloque "p". Reacción de estos elementos con O2, H2 y halógenos. Compuestos
con halógenos y en oxoaniones de los elementos siguientes, con los estados de oxidación citados:
B(III), Al(III), Si(IV), N(V), P(V), S(IV ó VI), O(II), F(I), Cl(I, III, V ó VII), Pb(II) y Bi(II).
Reacción de óxidos no metálicos con el agua y estequiometría de los ácidos resultantes. Reacción
de los halógenos con el agua. Hidróxidos con propiedades anfóteras. Pasivación del aluminio.
7. Elementos del bloque "d". Compuestos con los estados de oxidación referidos para los
siguientes metales de este bloque: Cr(III ó VI), Mn(II, IV ó VII), Fe(II ó III), Co(II), Ni(II), Cu(I
ó II), Ag(I), Zn(II), Hg(I ó II). Colores en disolución acuosa de los iones de los referidos metales
del bloque "d" y la valencia de los cationes que se forman. Hidróxidos con propiedades anfóteras.
8. Enlace covalente. Elementos que forman compuestos covalentes, propiedades de compuestos
covalentes. Estructuras de Lewis, regla del octeto Carga formal, resonancia.
9. Enlace iónico. Características y propiedades generales de compuestos iónicos. Elementos que
forman compuestos iónicos.
10. Ácidos y bases. Óxidos ácidos y básicos. Conceptos de ácidos y bases según Bronsted-Lowry.

QUÍMICA INORGÁNICA. NIVEL A.


1. Además del temario del Nivel B.
2. Estructura atómica y tabla periódica. Tendencias generales de los parámetros atómicos dentro
de los principales grupos en la Tabla Periódica: tamaño iónico, primera energía de ionización,
electronegatividad, electroafinidad, número de oxidación máximo, metales, semi-metales y no-
metales. Efectos de diamagnetismo y paramagnetismo en las propiedades periódicas.
3. Formulación y nomenclatura IUPAC. Compuestos de coordinación: complejos metálicos de
los cationes de los bloques s, p y primera serie de transición, y número de coordinación.
4. Masa atómica relativa. Isótopos radiactivos: decaimiento radiactivo (alfa, beta, gamma),
reacciones nucleares (alfa, electrones, positrones, protones, neutrones).
5. Estequiometría.
6. Elementos del bloque "s".
7. Elementos del bloque "p".
8. Elementos del bloque "d". Pasivación del hierro y del cromo.
9. Química de coordinación de los elementos del bloque “d”. Estados de oxidación más comunes
para los elementos del bloque d. Compuestos de coordinación, nomenclatura. Concepto de
número de coordinación. Concepto de ligante y sus diferentes tipos. Geometrías en los
compuestos de coordinación. Teoría de campo cristalino y teoría de enlace valencia, propiedades
magnéticas.
10. Enlace covalente. Estructura molecular. Teoría de enlace valencia (TEV), formación de
orbitales híbridos. Modelo de RPECV y predicción de la estructura. Moléculas polares y no
polares. Teoría de orbitales moleculares (TOM). Concepto de orden de enlace, diagramas de
orbitales moleculares para moléculas diatómicas hetero y homonucleares. Afinidad electrónica
y potencial de ionización en diagramas de orbitales moleculares para moléculas diatómicas.
Enlace covalente coordinado.
11. Enlace iónico.
12. Ácidos y bases. Conceptos de ácidos y bases según Lewis .Acidez relativa de oxianiones.
Basicidad relativa de bases como hidróxidos y derivados de amoniaco. Anfoterismo.
13. Estado sólido. Estructura y propiedades físicas de sólidos cristalinos. Características generales
de cristales iónicos, covalentes, moleculares y metálicos. Sistemas cristalinos. Sistema cúbico:
estructura primitiva, estructura centrada en el cuerpo, estructura centrada en las caras, relaciones
entre tamaño atómico ó tamaño iónico y parámetros de celda, y densidad.
14. Oxidación y reducción. Predicción de las reacciones de oxidación con base en los valores de
potenciales normales de reducción. Solubilización de los metales en medio ácido diluido.
Aniones comúnmente usados como oxidantes y reductores: propiedades reductoras del HNO2 y
sus sales, propiedades oxidantes del HNO3 y sus sales, reacción del Na2S2O3 con yodo.
Disminución de la reactividad y poder oxidante de los halógenos del F2 al I2. Productos de la
reducción del permanganato en función del pH.
15. Obtención industrial de productos inorgánicos importantes. H2SO4, NH3 (proceso de
Haber-Bosch), HNO3 (método de Ostwald), Na2CO3 (método de Solvay), Cl2, NaOH, H3PO4 y
aluminio (proceso de Bayer y proceso de Hall-Héroult). Fertilizantes inorgánicos: nutrientes
primarios y nutrientes secundarios.
16. Ciclos naturales. Agua, carbono, nitrógeno y oxígeno.

QUÍMICA ANALÍTICA. NIVELES A Y B.


1. Disoluciones. Definición. Diferentes formas de expresión de la concentración. Cálculo de
concentraciones.
2. Equilibrio ácido-base. Definición de ácidos y bases según el concepto de Bronsted-Lowry.
Definición de pH. Relación entre el producto iónico del agua y los valores de pKa y pKb.
Predicción cualitativa y cuantitativa de reacciones ácido-base. Cálculo de pH de ácidos y bases
fuertes, ácidos y bases débiles. Cálculo del pH de disoluciones de anfolitos y de disoluciones
amortiguadoras. Preparación de disoluciones amortiguadoras.
3. Equilibrio redox. Agentes oxidantes y reductores. Ley de Nernst. Fuerza relativa de oxidantes y
reductores. Predicción cualitativa y cuantitativa de reacciones redox. Cálculos de potencial de
disoluciones que contengan oxidantes y/o reductores.
4. Equilibrio de formación de complejos sencillos (relación estequiométrica 1:1). Definición de
constantes de disociación y formación de complejos. Predicción cualitativa y cuantitativa de
reacciones de formación de complejos.
5. Equilibrio de formación de solubilidad. Definición de Ks y pKs. Relación entre solubilidad y la
constante Ks. Efecto del ion común. Predicción cualitativa y cuantitativa de reacciones de
formación y solubilización de precipitados. Cálculos de concentración de especies involucradas
en el equilibrio de precipitación.
6. Identificación de cationes y aniones. Identificación de los cationes de los bloques “s” y “p” de
la primera serie de elementos de transición. Identificación de aniones de usa más frecuente:
halogenuros, nitrato, sulfuro, sulfato, carbonato y oxalato.
7. Valoraciones. Valoraciones ácido-base, redox y de formación de complejos. Uso de los
indicadores visuales de fin de reacción.
8. Ley de Lambert-Beer. Aplicaciones.
9. Cromatografía. Principios básicos.

2015
ÍNDICE

Exámenes XV ONQ – San Luis Potosí – 2006 1

Exámenes XVI ONQ – Toluca – 2007 51

Exámenes XVII ONQ – Oaxaca – 2008 105

Exámenes XVIII ONQ – Campeche – 2009 157

Exámenes XIX ONQ – Veracruz – 2010 203

Exámenes XX ONQ – Toluca – 2011 255

Exámenes XXI ONQ – Guadalajara – 2012 309

Exámenes XXII ONQ – Chihuahua – 2013 365

Exámenes XXIII ONQ – Distrito Federal – 2014 409

Exámenes XXIV ONQ – Guadalajara – 2015 471

Respuestas XV ONQ – San Luis Potosí – 2006 531

Respuestas XVI ONQ – Toluca – 2007 547

Respuestas XVII ONQ – Oaxaca – 2008 565

Respuestas XVIII ONQ – Campeche – 2009 585

Respuestas XIX ONQ – Veracruz – 2010 597

Respuestas XX ONQ – Toluca – 2011 611

Respuestas XXI ONQ – Guadalajara – 2012 625

Respuestas XXII ONQ – Chihuahua – 2013 637

Respuestas XXIII ONQ – Distrito Federal – 2014 651

Respuestas XXIV ONQ – Guadalajara – 2015 663


 
 
EXÁMENES
Estos exámenes fueron realizados por el
Comité Académico de la Olimpiada Nacional de Química
y fueron compilados por el
Comité Académico de la Olimpiada de Química del Distrito Federal
y puestos en este formato por Héctor García Ortega
XV Olimpiada Nacional de Química
1er Examen Nivel A y B.
Total: 30 preguntas. Tiempo asignado: 90 minutos. San Luis Potosí 2006.

Un aspecto importante para que obtengas buenos resultados en esta competencia es que no sólo uses
tus conocimientos de química sino también leas cuidadosamente todas las instrucciones y apliques
tu razonamiento lógico. En esta primera parte, anota en el recuadro correspondiente el NÚMERO
ATÓMICO del elemento que corresponde a la frase que lo describe. No escribas el nombre del
elemento o su símbolo. Tienes una tabla periódica donde puedes consultar los números atómicos. Si
consideras que ningún elemento es la respuesta correcta, escribe una “X” en el recuadro. No dejes
ningún recuadro en blanco.

1) Se llama potencial de primera ionización a la energía necesaria para arrancar un


electrón de un cierto átomo. Entre los cinco primeros elementos de la Tabla Periódica
el que tiene el menor potencial de primera ionización es:

2) Entre los primeros diez elementos de la Tabla Periódica el que tiene mayor
electronegatividad es:

3) Este elemento de los alcalino térreos se combina con el oxígeno para producir un
compuesto cuya masa molar es 40.31 g mol-1:

4) Es el primer elemento que tiene un electrón en el nivel 3d:

5) El aire consta de un 79% de este elemento:

6) 10 g del carbonato de este elemento se descomponen para producir el óxido


correspondiente y 4.4 g de CO2:

7) 1 mol del hidróxido de este elemento reacciona con 3 mol de ácido clorhídrico y se
obtiene 1 mol del cloruro del elemento más 54.05 g de agua. La masa molar del
cloruro obtenido es 133.33 g mol-1. El elemento cuyo hidróxido se usa como reactivo
es:

8) El permanganato de este elemento tiene una masa molar de 158.032 g mol-1. Su


número atómico es:

1
9) Al disolver 6.512 g de un compuesto “X” en 500 mL de agua, se obtiene una
disolución 0.2 molal. Si “X” es un cianuro de cierto elemento, el número atómico de
este elemento es:

10) En la molécula del etanol, el elemento con menor % en masa es:

11) En la reacción 8HNO3 + H2S  8NO2 + 4H2O + H2SO4, el número atómico del
elemento que se reduce es:

12) En la reacción de la pregunta 11, el número atómico del elemento que se oxida es:

13) La descomposición térmica de 54.15 g de un óxido metálico (“XO”) se utiliza para


obtener 4 g de oxígeno gaseoso en el laboratorio. El elemento “X” es:

14) Entre el grupo de elementos (Au, Zn, Ag, I, Na, H), el que reacciona violentamente al
contacto con el agua es:

15) De acuerdo a la Ley de los gases ideales, pV=nRT, a una temperatura de 0 0C y una
presión de 101.325 kPa (R=0.08314 L bar mol-1 K-1, 1 bar=100 kPa), 13.13 g de este
gas noble ocupan un volumen de 2.24 L:

16) Para preparar 50 mL de una disolución acuosa 0.1 M de este nitrato se utilizaron
0.85 g del sólido. Este nitrato es del elemento:

17) En el compuesto “K2ZCl6”, la letra Z representa un elemento de transición. La


densidad de este compuesto sólido es igual a 3.5 g cm-3 y 1 mol del mismo ocupa un
volumen de 0.1389 L. El número atómico del elemento “Z” es:

18) Dentro de los primeros quince elementos de la Tabla Periódica es el más utilizado en
la fabricación de celdas solares. Sus óxidos forman parte de muchos minerales. Este
elemento es:

19) En la reacción X2O5 + SO2  X2O4 + SO3, el producto sólido tiene un % en masa de
oxígeno de 38.58%. El número atómico del elemento “X” es:

20) Las cetonas son compuestos orgánicos que se caracterizan por tener un doble enlace
de este compuesto entre un átomo de carbono y un átomo de:

2
21) Este elemento se combina con el oxígeno en una proporción 3:4. La masa molar de
este compuesto es 685.6 g mol-1:

22) El borax es el tetraborato decahidratado (diez moléculas de agua) de este elemento.


La masa molar del borax es 381.37 g mol-1. El elemento es:

23) El trióxido de este lantánido tiene una masa molar de 352 g mol-1:

24) En una mezcla de 56 g de nitrógeno gaseoso con 40.358 g de otro elemento gaseoso
cuyas moléculas son monoatómicas, la fracción mol del elemento desconocido es 0.5.
El número atómico de este elemento es:

25) La densidad de este elemento es 7.86 g cm-3. 1 mol de este elemento ocupa un
volumen de 7.106 cm3:

26) El elemento “X” al combinarse con cloro forma un compuesto hexahidratado cuya
masa molar es 203.31 g mol-1. El elemento “X” es:

27) La configuración electrónica de este elemento es 1s22s22p43s2:

28) Todos los óxidos de los metales alcalinos reaccionan con el agua para formar los
hidróxidos correspondientes. En una reacción de este tipo se obtuvieron 5 g del
hidróxido, lo que equivale a 0.209 mol. El óxido que se utilizó es del elemento cuyo
número atómico es:

29) En la electrólisis del agua, el elemento del compuesto que se obtiene en el cátodo es:

30) En la siguiente reacción, las letras griegas “”, ””, ”” representan elementos
químicos 24 + 2NH3  (NH3)22 + 2KCl. La masa del compuesto 24 es
415.1 g mol-1. Por lo tanto, “” representa al elemento cuyo número atómico es:

FIN DEL EXAMEN

3
4
XV Olimpiada Nacional de Química. 2do Examen Nivel A y B.
Nivel B: Sólo deben contestar las primeras 19 preguntas.
Nivel A: Deben contestar las 22 preguntas.
Tiempo asignado: 90 minutos. San Luis Potosí 2006.

1. En la combustión de gas propano en una hornilla se consumen 20 cm3 de gas cada segundo,
medidos a una presión de 1.013 bar y a una temperatura de 25 ºC. Durante la combustión de 1
mol de propano se desprenden 2 220 kJ por mol de propano. Si la combustión se realiza con aire
que contiene 20% en volumen de oxígeno gaseoso y 80% en volumen de nitrógeno gaseoso,
calcula: (MUESTRA TUS CÁLCULOS EN CADA CASO)
C3H8 (g) + 5 O2 (g)  3 CO2 (g) + 4 H2O (g)

1.1) La cantidad de oxígeno gaseoso (en moles) consumido en la hornilla después de haber estado
encendida durante una hora.

Cantidad de gas oxígeno = mol

1.2) La cantidad de aire (en moles) necesarios para la combustión.

Cantidad de aire = mol

1.3) La masa (en gramos) de nitrógeno gaseoso que pasan a través de la hornilla.

Masa de nitrógeno = g

5
1.4) La cantidad de agua (en moles) formada en la combustión.

Cantidad de agua = mol

1.5) La fracción mol de CO2 formado en la combustión con aire.

Fracción mol de CO2 =

1.6) El por ciento en masa de nitrógeno gaseoso después de la combustión.

% en masa de nitrógeno gaseoso =

2. En una serie de datos experimentales se tienen los siguientes valores:


0.2724  0.016

25.00

El resultado debe expresarse como:


a) 0.010 b) 0.0109 c) 0.010832 d) 0.01083 e) 0.0108

6
3. Si el valor de la Ka de un ácido HX es 7.2×10–4, esto significa que:
a) HX se ioniza ampliamente en disolución acuosa.
b) El H2O es una base más fuerte que X-.
c) HX es un ácido más fuerte que H3O+.
d) Una disolución de NaX deberá ser neutra.
e) Debe formarse HX al mezclar disoluciones acuosas de NaX y HCl.

4. De las siguientes ecuaciones balanceadas, indica cuáles implican procesos redox:


I) 2 OH– (ac) + Cr2O72– (ac)  2 CrO42– (ac) + 2 H2O (l)
II) 3 NO2 (g) + H2O (l)  2 HNO3 (l) + NO (g)
III) PBr3 (l) + 3 H2O (l)  H3PO3 (ac) + 3 HBr (ac)
IV) 2 H2SO4 (ac) + 2 NaBr (s)  Br2 (l) + SO2 (g) + Na2SO4 (ac) + 2 H2O (l)

a) I y II b) I y III c) I y IV d) II y III e) II y IV

5. Una muestra de agua potable tiene una concentración de ion nitrato (NO3-) de 1.4 x 10-4 mol L-1.
El nitrato puede afectar a los glóbulos rojos y reducir su capacidad de transportar oxígeno al
cuerpo, particularmente en los niños pequeños (provoca en los bebes el llamado síndrome del
bebé azul). El máximo aceptable es de 10 ppm (mg L-1). Esta muestra de agua:
a) Es apta para su consumo porque contiene 14 ppm.
b) No es apta para su consumo porque contiene 8.68 ppm.
c) No es apta para su consumo porque contiene 14 ppm.
d) Es apta para su consumo porque contiene 8.69 ppm.
e) No es posible saberlo si no se conoce el catión del nitrato.

6. La siguiente reacción redox (NO BALANCEADA) ocurre en una disolución ácida:

MnO4– + Cl– → Mn2+ + Cl2


Cuál de las afirmaciones siguientes es verdadera para esta reacción:
a) Cl2 es el agente reductor.
b) MnO4– se oxida.
c) Cl– es el agente reductor.
d) Cl– se reduce.
e) El número de oxidación del Mn se incrementa en 3 unidades.

7. Se desea conocer la pureza de un carbonato cálcico mineral y para ello se disuelve una muestra
de 0.750 g en 50.0 cm3 de HCl 0.15 mol L-1. Esto supone un exceso de ácido y este exceso
requiere para su neutralización un volumen de 4.85 cm3 de NaOH 0.125 mol L-1. El porcentaje
(en masa) de CaCO3 que contiene la muestra es:
a) 0.4595 % b) 0.92 % c) 45.95 % d) 68.90 % e) 91.91 %

7
8. Al mezclar disoluciones de un halógeno con un haluro se obtienen los siguientes resultados:

Cl2 + Br-  disolución naranja I2 + Br-  disolución café rojiza

Br2 + Cl-  disolución naranja Br2 + I-  disolución café rojiza

Estas observaciones permiten deducir que:


a) El bromo es más oxidante que el cloro.
b) El cloro oxida al yoduro.
c) El bromo oxida a los cloruros.
d) El reductor más fuerte es el cloruro.
e) El reductor más fuerte es el bromuro.

81
9. Indica el inciso en el que se encuentra la configuración electrónica correcta del 35 Br .
a) 1s22s22p63s23p63d104s24p5
b) 1s22s22p63s23p63d104s24p6
c) 1s22s22p63s23p63d104s24p7
d) 1s22s22p63s23p63d104s24p4
e) 1s22s22p63s23p63d104s24p65s1

10. Ordena de mayor a menor potencial de ionización los siguientes elementos:


Cs F Li Br O B

> > > > >

11. La escala de electronegatividad de Pauling está expresada ¿en qué unidades?

a) kJ/mol b) kcal/mol c) No tiene d) Newtons e) Coulombios

12. En una familia, cuando el número cuántico principal se incrementa:


a) El radio atómico aumenta y la electronegatividad es constante.
b) La electronegatividad disminuye y el radio atómico disminuye.
c) La electronegatividad aumenta y el radio atómico aumenta.
d) El radio atómico disminuye y la electronegatividad aumenta.
e) El radio atómico aumenta y la electronegatividad disminuye.

8
13. Completa los espacios denotados del 1 al 5 para que puedas conocer el nombre del elemento que
se encuentra en la vertical que se inicia en el espacio 5 y que se refiere al segundo nombre del
elemento también llamado tungsteno.
5

Horizontales
1. Raro elemento radiactivo que lleva el nombre del país de la científica que participó en su
descubrimiento. Tiene 125 neutrones.
2. Aparece en muchos minerales (como galena, pirita, yeso, etc.); su hidrácido se puede
encontrar en aguas naturales.
3. Presente en todos los compuestos orgánicos; uno de sus isótopos se utiliza para conocer la
edad arqueológica de los objetos.
4. Se encuentra abundantemente en la naturaleza; uno de sus compuestos es el cuarzo.

14. En cual de los siguientes incisos están ordenadas las moléculas CSe, CO, C2 y CS en orden
decreciente de polaridad:
a) C2  CS  CSe  CO
b) CO  CSe  CS  C2
c) C2  CSe  CS  CO
d) CO  CS  CSe  C2
e) CO  CSe  C2  CS

15. ¿Cuál de las siguientes sustancias sería menos soluble en agua?


a) NaCl b) KCl c) LiCl d) MgO e) CaO

16. El compuesto que NO cumple con la regla del octeto es:


a) CO2 b) PH3 c) H3PO4 d) COCl2 e) CCl4

9
17. En una disolución acuosa, ¿cuál ion metálico NO presentaría coloración en alta concentración?
a) Cu2+ b) Mn7+ c) Fe3+ d) Zn2+ e) Ru3+

18. Dibuja en los tres cuadros las estructuras de Lewis que contribuyen a la resonancia del ion
carbonato. “Nota: No olvides escribir todos los electrones de valencia para todos los átomos”.

19. Cuál de las siguientes sales, en disoluciones de igual concentración, elevará más el valor de pH:
a) NaClO4 b) NaClO3 c) NaClO2 d) NaClO e) NaCl

De aquí en adelante sólo deben continuar los del nivel A.

20. El arreglo geométrico de la molécula Fe(CO)5 es:


a) Pirámide base b) Tetraédrica c) Cuadrada d) Bipirámide e) Trigonal
cuadrada trigonal

21. De los siguientes ejemplos, cuál es un caso de tautomería:


a)
CH3 CH3

b)
H

H3C C N CH3 H3C C N CH3 + HCl

H H
Cl:

c)
H
O O

10
d)
Na

Na

e)
H3C O H3C O:
N N
H3C H H3C H

22. Clasifica las siguientes reacciones como de sustitución (S), eliminación (E), adición (A) o
transposición (T):

O O
a)
-
H3C C O:
- + H3C I H3C C O CH3 + I:

OH
HCl
b) O + H2O

OH

Br
c)
+ + H2O + KBr
KOH

SH
d)
+ H2S

e)
+ HCl Cl
OH
O

f) Br EtOH OEt
+ + HBr

11
Calor
g) O OH

Coloca en cada recuadro la letra S, E, A, T según corresponda.

REACCIÓN (a) REACCIÓN (b) REACCIÓN (c) REACCIÓN (d)

REACCIÓN (e) REACCIÓN (f) REACCIÓN (g)

FIN DEL EXAMEN

12
XV OLIMPIADA NACIONAL DE QUÍMICA TERCER EXAMEN NIVEL B
Total: 17 preguntas Tiempo asignado: 180 minutos San Luis Potosí 2006

1. En los consultorios dentales es necesario contar con aire a presión para accionar la turbina de aire
con la que se desbastan y pulen las piezas dentales. El aire se comprime y se almacena en un
tanque para utilizarlo posteriormente. Un compresor opera durante 30 minutos con una potencia
de 800 J/s para introducir aire a un tanque de almacenamiento de 30.0 L. La temperatura del aire
después de la compresión aumenta hasta 35 ºC, lo que produce una transferencia de energía en
forma de calor hacia los alrededores hasta que se alcanza nuevamente la temperatura ambiente de
20 ºC y la presión en el interior del tanque de almacenamiento es de 9.5 bar. Considerando que
para el aire: Cp = 7/2 R, Cv = 5/2 R y R = 8.314 J/mol K, calcula:

a) El trabajo desarrollado por el compresor.

Trabajo = J

b) La cantidad de sustancia de aire (en moles) dentro del tanque de almacenamiento al final de la
compresión.

Cantidad de sustancia de aire = mol

c) La presión del gas (en bares), si la temperatura al final de la compresión es de 45 ºC.

13
Presión = bar

d) La variación de energía interna del aire después de la compresión.

U = J

e) La cantidad de calor cedida a los alrededores cuando el gas se enfría desde 45 ºC hasta la
temperatura ambiente de 20 ºC.

Q= J

********************************************************************************

2. Una de las propiedades que tienen tanto los líquidos como los sólidos es la de tender a pasar a la
fase de vapor. En el primer caso decimos que el líquido se evapora, en tanto que en el caso de los
sólidos decimos que se subliman. Una vez que las moléculas han saturado la fase de vapor,
producen una presión que se conoce como presión de vapor. La presión de vapor varía con la
temperatura, aumentando con lentitud a temperaturas bajas y luego muy rápidamente a

14
temperaturas altas. Las ecuaciones de Clausius Clapeyron y la de Antoine permiten evaluar la
presión de vapor de muchas sustancias, una vez que se conocen las constantes correspondientes.
Cuando la presión de vapor alcanza la del medio ambiente, entonces los líquidos “hierven” a su
temperatura de ebullición, y los sólidos “subliman” a su temperatura de sublimación.

Ecuación de Clausius Clapeyron.


T = Temperatura Absoluta
 P2  Hvap  1 1 Hvap = Entalpía molar de vaporización
ln      
 P1  R  T 2 T1  P = Presión de vapor de la sustancia
R = 8.314 J/mol K

Ecuación de Antoine.
B A, B, C son constantes
log 10 P  A 
t C “t” = Temperatura, en grados Celsius.
P = Presión de vapor, en mmHg

La presión de vapor del agua es de 92.51 mmHg a 50 ºC y de 760 mmHg a 100 ºC.
Utilizando la ecuación de Clausius Clapeyron, calcula:

a) La entalpía de vaporización del agua.

Hvap = J/mol

b) La temperatura de ebullición del agua en una ciudad donde la presión ambiental es de 400
mmHg. Considera el valor reportado en tablas de Hvap = 40656 J/mol.

Temperatura de ebullición del agua = ºC

Para el etanol las constantes de la ecuación de Antoine son: A = 8.321, B = 1718, C = 237.52.

15
c) Calcula la temperatura de ebullición del etanol en una ciudad donde la presión ambiental es de
500 mmHg.

Temperatura de ebullición del etanol = ºC

********************************************************************************

3. Cuando se calienta un hidrato de sulfato de cobre (II) sufre una serie de cambios. Una muestra de
2.574 g de CuSO4xH2O se calienta a 140 ºC, se enfría y se pesa. El producto sólido resultante se
calienta a 400 ºC, se enfría y se pesa. Finalmente este sólido se calienta a 1000 ºC, se enfría y se
pesa por última vez. Se obtuvieron los siguientes registros de las pesadas efectuadas:

Muestra original = 2.574 g


Después de calentar a 140 ºC = 1.833 g
Después de calentar a 400 ºC (se elimina toda el agua de hidratación) = 1.647 g
Después de calentar a 1000 ºC = 0.812 g

a) Si se supone que toda el agua de hidratación se elimina a 400 ºC, ¿cuál es la fórmula del hidrato
original?

Fórmula:

b) ¿Cuál es la fórmula del hidrato obtenido cuando el original se calienta sólo hasta 140 ºC?

16
c) El residuo que se obtiene a 1000 ºC es un óxido de cobre. ¿Cuál es su composición porcentual y
su fórmula empírica?

Fórmula empírica: % de cada elemento:

********************************************************************************

4. La información siguiente fue tomada de un periódico. “Ayer, a primera hora de la mañana, un


camión de la empresa San Luis de los Hondos, sufrió una avería en la boca de uno de sus tanques
en los que transportaba seis mil litros de ácido clorhídrico. Se utilizaron cien bolsas de cal para
neutralizar los efectos nocivos del líquido; posteriormente se lavó con agua para anular la
contaminación y el agua utilizada se canalizó hacia un río próximo. Se supone que se derramó
todo el contenido del camión y que contenía ácido clorhídrico al 37% masa ( = 1.19 g mL–1).

a) La reacción balanceada que corresponde a la reacción de neutralización empleada para contener


el efecto del ácido derramado es:

b) ¿Cuántas bolsas de 20 kg de Ca(OH)2 se requieren para neutralizar el ácido derramado?

No. mínimo de bolsas:

17
c) De acuerdo con tus cálculos, las 100 bolsas de Ca(OH)2, ¿fueron suficientes para neutralizar el
todo el ácido o se contaminó el río con HCl?

********************************************************************************

5. Una muestra de un compuesto MSO4 que tiene una masa de 0.1131g reacciona completamente
con cloruro de bario acuoso produciendo 0.2193 g de BaSO4:

MSO4 + BaCl2 (ac)  BaSO4 (s) + MCl2 (ac)

Se conoce que la única fuente de iones sulfato es el compuesto MSO4, ¿cuál debe ser la masa
atómica de M? ¿De qué elemento se trata?

Elemento:

********************************************************************************

6. La concentración del ácido “muriático” (ácido clorhídrico) que se vende comúnmente en


tlapalerías y que se emplea para remover manchas de cemento y limpiar pisos de cerámica,
usualmente, se suele expresar en % en masa. Para determinar la concentración real de HCl en un
producto cuya etiqueta declara contener 18% de ácido muriático ( = 1.1 g mL–1), se tomaron
25.00 mL del producto y se llevaron a 1000 mL en un matraz volumétrico. De esta disolución se
tomaron 25.00 mL y se titularon con 37.95 mL de una disolución acuosa 0.100 M de NaOH.
Indica cuál es la concentración del producto (en mol/L) y si es correcta la información en la
etiqueta del HCl.

********************************************************************************

18
7. Una muestra que contiene NaCl, Na2SO4 y NaNO3 muestra el siguiente análisis elemental: Na:
32.08%; O: 36.01%; Cl: 19.51%. Calcula el porcentaje en masa de cada compuesto en la mezcla.

% de cada sal:

********************************************************************************

8. El hijo de uno de los trabajadores de la Universidad de San Luis Potosí se metió sin permiso en el
laboratorio de Química. Creyendo que se trataba de una golosina ingirió el polvo blanco de un
ácido que iba a ser utilizado para limpiar los pisos. Casualmente tú entras al laboratorio en ese
momento y, al ver al niño, buscas alguna sustancia para neutralizar la acidez en el estómago del
pequeño. Encuentras un anaquel de nueve repisas; en cada una de ellas hay un recipiente con
sustancias diferentes (ver dibujo) y un instructivo para identificar las sustancias que contienen.
Aunque en varios recipientes hay reactivos capaces de neutralizar al ácido, algunos de ellos
pueden ser más perjudiciales que el propio ácido.

2 1
3

1
2 3

1
3 2

Instructivo:
 En un recipiente marcado con el número 3 hay NaCl.
 Hay un recipiente con mercurio debajo de otro que contiene NH4Cl.
 A la izquierda de un recipiente alto hay agua destilada.
 En la fila inferior, en un recipiente igual al que contiene el NaCl, más abajo que el del agua
destilada, hay cianuro de sodio.

19
 En la fila superior está una disolución de sosa concentrada.
 En la columna izquierda hay ácido acético.
 En un recipiente con el número 2 hay NH4Cl.
 Hay arsénico en un recipiente redondo que no está cerca del cianuro.
 Otro recipiente contiene NaHCO3.

Completa el siguiente cuadro indicando cuál es la sustancia que se encuentra en cada recipiente e
indica cuál es la adecuada para darle al pequeño.

********************************************************************************

9. La configuración electrónica del Ga3+ es:

a) [Ar]3d104s24p1 b) [Ar]3d104s14p1 c) [Ar]3d104s24p3


d) [Ar]3d104s24p2 e) [Ar]3d10

********************************************************************************

10. Ordene las especies de mayor a menor tamaño.


Ru0 Ru4+ Ru-1 Ru2+ Ru3+ Ru6+

> > > > >

********************************************************************************
37
11. La configuración electrónica del 17 Cl es:

a) [Kr] 3s23p5 b) [Ar] 3s2 3p5 c) [He] 3s23p5


d) [Ne] 3s23p5 e) [Ne] 3s23p7

********************************************************************************

12. De los compuestos enunciados a continuación, en cuál de ellos la electronegatividad del


átomo de hidrógeno es mayor que la del átomo con el que forma el enlace:

a) HF b) NaH c) NH3 d) H2O e) CH4

20
********************************************************************************

13. En que opción se presenta el clorato de potasio:

a) KClO b) KClO2 c) KClO4 d) KClO3 e) KClO7

********************************************************************************

14. En las siguientes reacciones, indique que sustancias corresponden a A, B, C, y escriba los
coeficientes estequiométricos “X” y “ Z”. Si se sabe que A es un elemento, que el compuesto B
tiene %Cl = 77.45 (137.19 g/mol) y que el compuesto C tiene %Cl = 85.13 (208.24 g/mol).

1 A + 6Cl2  X B

1 A + 10 Cl2  Z C

A= B= C= X= Z=

********************************************************************************

15. En la molécula PH4+ la carga formal fósforo y del hidrógeno, QP y QH, son:

a) QP = 5+ y QH = +1 b) QP = -5 y QH = -1 c) QP = +1 y QH = 0

d) QP = -1 y QH = 0 e) QP = 5+ y QH = -1

********************************************************************************

16. El compuesto en el cual el estado de oxidación del Br es el más positivo es:

a) HBr b) NaBrO c) NaBrO3

d) Br2 e) C6H5Br

********************************************************************************

17. En un enlace ___________ se presentan altos puntos de fusión debido a la fuerza y


omnidireccionalidad del enlace, aunque esto también traiga como consecuencia fragilidad.

a) iónico b) covalente c) metálico

d) tricéntrico e) sigma

FIN DEL EXAMEN

21
22
XV OLIMPIADA NACIONAL DE QUÍMICA TERCER EXAMEN NIVEL A
Total: 20 preguntas Tiempo asignado: 180 minutos San Luis Potosí 2006

1. Cuando se calienta un hidrato de sulfato de cobre (II) sufre una serie de cambios. Una muestra de
2.574 g de CuSO4·xH2O se calienta a 140 ºC, se enfría y se pesa. El producto sólido resultante se
calienta a 400 ºC, se enfría y se pesa. Finalmente este sólido se calienta a 1000 ºC, se enfría y se
pesa por última vez. Se obtuvieron los siguientes registros de las pesadas efectuadas:

Muestra original = 2.574 g


Después de calentar a 140 ºC = 1.833 g
Después de calentar a 400 ºC (se elimina toda el agua de hidratación) =1.647 g
Después de calentar a 1000 ºC = 0.812 g

a) Si se supone que toda el agua de hidratación se elimina a 400 ºC, ¿cuál es la fórmula del hidrato
original?
Escribe aquí tus cálculos:

Fórmula:

b) ¿Cuál es la fórmula del hidrato obtenido cuando el original se calienta sólo hasta 140 ºC?
Cálculos:

Fórmula:

23
c) El residuo que se obtiene a 1000 ºC es un óxido de cobre. ¿Cuál es su composición porcentual y
su fórmula empírica?
Cálculos:

Fórmula: %:

********************************************************************************

2. Una muestra de un compuesto MSO4 que tiene una masa de 0.1131 g reacciona completamente
con cloruro de bario acuoso produciendo 0.2193 g de BaSO4:

MSO4 + BaCl2 (ac)  BaSO4 (s) + MCl2 (ac)

Si se sabe que la única fuente de iones sulfato es el compuesto MSO4, ¿cuál debe ser la masa
atómica de M? ¿De qué elemento se trata?
Cálculos:

Elemento:

********************************************************************************

3. A un volumen de 100 cm3 de una disolución de Cu2+ se le añade una cierta cantidad de CuCl2
sólido. Se mide el potencial de un alambre de cobre sumergido en la disolución (mediante una
celda electroquímica formada con un electrodo de referencia adecuado) antes y después de la
adición y se registra un incremento de 9 mV. ¿Cuál es la masa de CuCl2 añadida?

Datos: Masa molar del CuCl2 = 134.5 g mol-1


Valor de Eo del sistema Cu2+/Cu = 0.34 V
Cálculos:

Masa:

24
********************************************************************************

4. Una cierta cantidad de cromato de plomo fue vertida accidentalmente en una reserva de agua. Los
ingenieros responsables de la reserva necesitan conocer que tan contaminada está el agua para
después tratar de eliminar la contaminación. El producto de solubilidad Kps del cromato de
plomo a 18 oC es 1.77 x10-14.

a) ¿Cuál es la solubilidad del cromato de plomo en agua?


Cálculos:

b) Algunos ingenieros piensan que el plomo en el agua puede ser removido tratando el agua con
cromato de potasio. ¿Cuál es la solubilidad del cromato de plomo en una disolución 0.1 mol/L
de cromato de potasio?
Cálculos:

c) Otros ingenieros argumentan que la adición del cromato de potasio (K2CrO4) es también una
forma de contaminación por lo que proponen eliminar este reactivo precipitándolo con BaCl2 en
forma de cromato de bario, ¿cuál es la solubilidad del cromato de bario en una disolución de
cloruro de bario 0. 1 mol/L? ( Valor de pKs de BaCrO4 = 9.7)
Cálculos:

d) Si estos dos compuestos (cromato de potasio y cloruro de bario) se adicionan en forma sucesiva
¿qué concentración de cromato y de plomo puede quedar en el agua?
Cálculos:

********************************************************************************

25
5. Una muestra que contiene NaCl, Na2SO4 y NaNO3 muestra el siguiente análisis elemental: Na:
32.08%; O: 36.01%; Cl: 19.51%. Calcula el porcentaje en masa de cada compuesto en la mezcla.
Cálculos:

Porcentajes:

6. Una disolución de un ácido monoprótico de pKa = 6.0 se neutraliza completamente con 40.00 mL
de NaOH de concentración aproximada 0.2 mol/L. ¿Cuántos mL de esa misma disolución tendrán
que añadirse si se desea preparar una disolución amortiguadora (buffer) de pH = 6.3?
Cálculos:

Volumen:

********************************************************************************

7. Un compuesto A tiene la fórmula molecular C7H8O. Este compuesto es insoluble en agua, ácido
clorhídrico diluido y en una disolución acuosa de bicarbonato de sodio. Sin embargo, fue soluble
en una disolución acuosa de hidróxido de sodio. Al tratar el compuesto A con agua de bromo a
temperatura ambiente, se observó la formación, rápidamente, del compuesto B, con fórmula
molecular C7H5OBr3. ¿Cuáles son las estructuras de los compuestos A y B?

Compuesto A Compuesto B

********************************************************************************

8. a) El alcaloide Quinina es un fármaco antipalúdico que se extrae de la corteza de la quina. Indica


cuál de las siguientes estructuras corresponde a la Quinina, teniendo en cuenta que dicha
molécula presenta cuatro centros quirales.

26
N N
O HO HO N

CH 3O CH 3O CH 3O
a) b) c)
N N N

b) ¿Cuántos estereoisómeros puede presentar la Quinina?

********************************************************************************

9. Escribe los reactivos y las condiciones de reacción que son necesarios para llevar a cabo cada una
de las siguientes transformaciones:
OH

A
Br
B

C
OH

OH
OH

RESPUESTA A RESPUESTA B

RESPUESTA C RESPUESTA D

27
********************************************************************************

10. En los consultorios dentales es necesario contar con aire a presión para accionar la turbina de
aire con la que se desbastan y pulen las piezas dentales. El aire se comprime y se almacena en un
tanque de almacenamiento para utilizarlo posteriormente. Un compresor opera durante 30
minutos con una potencia de 800 J/s e introduce aire a un tanque de almacenamiento de 0.030 m3.
La temperatura del aire después de la compresión aumenta hasta 35 ºC, lo que produce una
transferencia de energía en forma de calor hacia los alrededores hasta que se alcanza nuevamente
la temperatura ambiente de 20 ºC y la presión en el interior del tanque de almacenamiento es de
9.5 bar. Considerando que para el aire: Cp = 7/2 R, Cv = 5/2 R, y R = 8.314 J/mol K, R = 8.314
Pa m3/mol K.
Calcula:

a) La cantidad de sustancia de aire (en moles) dentro del tanque de almacenamiento al final de la
compresión.

moles de aire = mol

b) El volumen que ocuparía el aire a la presión y temperatura ambientes (1 bar y 20 ºC).

Volumen de aire a 1 bar y 20 ºC = m3

c) La temperatura que alcanzaría el aire suponiendo que la compresión fuera adiabática reversible
(para el aire  = 1.4).

Temperatura = K

28
d) La presión del aire (en bares), si la temperatura realmente al final de la compresión es tan solo de
45 ºC.

Presión = bar

e) La cantidad de calor cedida a los alrededores cuando el aire se enfría desde 45 ºC hasta la
temperatura ambiente de 20ºC.

Q cedido a alrededores = J

f) El cambio de entropía en los alrededores cuando el aire se enfría desde 45 ºC hasta la


temperatura ambiente de 20 ºC.

S de los alrededores = J/K

g) El cambio de entropía del aire cuando se enfría desde 45 ºC hasta la temperatura ambiente de 20
ºC.

S del aire = J/K

29
********************************************************************************

11. La configuración electrónica del Ga3+ es:

a) [Ar]3d104s24p1 b) [Ar]3d104s14p1 c) [Ar]3d104s24p3

d) [Ar]3d104s24p2 e) [Ar]3d10

********************************************************************************

12. Para las siguientes reacciones químicas:


a) B  B   e 
b) C  C   e 
c) N  N   e 
d ) O  O   e

Escribe en los paréntesis el potencial de ionización correspondiente.

( ) 0.80 MJ/mol ( ) 1.31 MJ/mol ( ) 1.40 MJ/mol ( ) 1.08 MJ/mol

********************************************************************************

13. En las siguientes reacciones, indica que sustancias corresponden a A, B, C, y escribe los
coeficientes estequiométricos “X” y “Z”. Si se sabe que A es un elemento, que el compuesto B
tiene %Cl = 77.45 (137.47g/mol) y que el compuesto C tiene %Cl = 85.13 (208.24g/mol).

1 A + 6 Cl2  X B
1 A + 10 Cl2  Z C

A= B= C= X= Z=

********************************************************************************

14. Para las siguientes moléculas escribe en el paréntesis la disposición geométrica de cada una.
Nota: Las disposiciones geométricas se pueden repetir en varias moléculas.

( ) O3 ( ) NO- ( ) B(OH)3 ( ) NH3 ( ) PF5

( ) PF6- ( ) NO ( ) XeF4 ( ) IF5 ( ) XeF6

a) Octaédrica b) Tetraédrica c) Cuadrada plana

d) Forma de V e) Lineal f) Bipirámide trigonal

g) Trigonal h) Piramidal i) Pirámide base cuadrada

30
********************************************************************************

15. Para las siguientes moléculas escribe en el paréntesis la hibridación del átomo central que le
corresponde. Nota: La hibridación del átomo central se puede repetir en varias moléculas.

( ) NH4+ ( ) PF6- ( ) BF3

( ) BeCl2 ( ) NO3- ( ) ClO4-

a) sp b) sp2 c) sp3 d) dsp3 e) d2sp3

********************************************************************************

16. Para el diagrama de orbitales moleculares de la molécula O2+, completa en los cuadros, los
orbitales correspondientes.

********************************************************************************

31
17. El orden de enlace para la molécula O2+ es:

a) 1.0 b) 1.5 c) 2.0 d) 2.5 e) 0.75

********************************************************************************

18. Para el siguiente compuesto de coordinación [Ru(NH3)6]Cl2 contesta la siguientes peguntas:

Nombre del compuesto ______________________________________________


Número de coordinación del metal ______ Estado de oxidación del metal ________
Geometría de la molécula ___________________

********************************************************************************

19. El número de electrones desapareados para el [Fe(H2O)6](NO3)2 es

a) Cero b) 1 c) 5 d) 6

********************************************************************************

20. En el empaquetamiento cúbico centrado en las caras para una sustancia NaCl el número de
veces que se repite la fórmula es:

a) 2 b) 3 c) 1 d) 6 e) 4

Cl-
Na+

FIN DEL EXAMEN

32
XV OLIMPIADA NACIONAL DE QUÍMICA
EXAMEN INTERNACIONAL
9 PREGUNTAS Tiempo: 4 HORAS San Luis Potosí 2006

1. Para resolver este problema deberás acomodar nueve diferentes elementos químicos en las nueve
casillas que les corresponden en este esquema:

1 2 3

4 5 6

7 8 9

Cada una de las letras que forman la palabra OLIMPIADA corresponde a la letra con la que
empieza el símbolo químico de uno de los nueve elementos. Como puede haber muchas
combinaciones posibles, te damos algunos requisitos que deben de cumplirse para completar
correctamente la tabla.

 El gas noble y el elemento de mayor número atómico dentro de los considerados en este
problema, no están ni en la misma fila ni en la misma columna.
 La suma de los números atómicos de los elementos ubicados en una de las columnas da un total
de 151.
 Hay tres elementos cuyos números atómicos son consecutivos pero no están ni en la misma fila
ni en la misma columna.
 El halógeno está en una esquina, en una posición más abajo del gas noble.
 El lantánido está justo debajo de un elemento gaseoso muy importante para la vida, mismo que
está justo a la izquierda del elemento alcalino térreo.
 El metal alcalino está en la casilla que tiene el número igual al de su número atómico y no está
en la misma columna que el halógeno.

33
2. Uno de los constituyentes del aceite de turpentina (aguarrás) es el -pineno, que tiene la fórmula
molecular C10H16. En el siguiente esquema se ilustran varias reacciones del -pineno. Determina
la estructura del -pineno y la de los productos de reacción A al E.
A
(C10H16Br2)

Br2, CCl4

D PhCO 3H Br2, H 2O B
 -pineno
(C10H 16O) CH 2Cl2 (C10H17BrO)

1) O 3 H2SO 4, CALOR
H 2SO 4 (20%) 2) (CH3)2S

E C
O
(C10H18O 2) (C 10H15Br)
C
H

C O
CH3

 -PINENO COMPUESTO A COMPUESTO B

COMPUESTO C COMPUESTO D COMPUESTO E

34
3. a) Proponga un mecanismo para la siguiente reacción:

O
-+
CH3O: Na
OCH3 + Na+ -:Br
CH3OH O
Br

R ESPU ESTA

b) ¿Qué tipo de reacciones se están llevando a cabo?


i) Adición seguida de una eliminación.
ii) Sustitución seguida de una adición.
iii) Adición seguida de una sustitución nucleofílica intramolecular.
iv) Sustitución nucleofílica intramolecular seguida de una eliminación.
v) Eliminación seguida de una sustitución nucleofílica bimolecular.
R E S P U E S T A

35
4. Las configuraciones relativas de los estereoisómeros del ácido tartárico se determinaron a partir
de las siguientes síntesis:
O H
C
HCN
a) H OH A + B
D IASTER EOISÓMER OS
OH
SEPAR AD OS
D -(+)-gliceraldehído

1) B a(OH )2 , H 2 O
b) A C
2) H C l, H 2 O

1) B a(OH )2 , H 2 O
B D
2) H C l, H 2 O

H N O3
c) C (-)-Ácido tartárico
H 2O

H N O3
D (meso)-Ácido tartárico
H 2O

a) Tomando como base la configuración del D-(+)-gliceraldehído, utilice las proyecciones de


Fischer para representar las configuraciones de los compuestos A, B, C y D.

COMPUESTO A COMPUESTO B

COMPUESTO C COMPUESTO D

36
b) Represente las configuraciones de los tres estereoisómeros del ácido tartárico: ácido (+)-
tartárico, ácido (-)-tartárico y ácido (meso)-tartárico, utilizando las proyecciones de Fischer

ÁCIDO (+)-TARTÁRICO ÁCIDO (-)-TARTÁRICO

ÁCIDO (meso)-TARTÁRICO

5. El producto de adición electrofílica de ácido clorhídrico a isobuteno es:

a) Cloruro de n-butilo
b) Cloruro de isobutilo
c) Cloruro de sec-butilo
d) Cloruro de ter-butilo
e) Cloruro de n-propilo

R E S P U E S T A

37
6. El ciclo de Carnot es un ciclo termodinámico, ideal y reversible, en el cual el rendimiento es el
máximo posible y sirve como patrón comparativo para todos los demás ciclos.

El ciclo de Carnot Consta de cuatro procesos o transformaciones reversibles es:

P
a
 Expansión isotérmica (ab)
 Expansión adiabática (bc) Tc
b
 Compresión isotérmica (cd)
 Compresión adiabática (da)
d
c
Tf
V

Durante la expansión isotérmica la máquina térmica absorbe calor de la fuente caliente y una
parte de éste lo transforma en trabajo durante el ciclo (llamado trabajo del ciclo). La otra parte del
calor absorbido no se aprovecha ya que lo cede a la fuente fría.

La eficiencia () de la máquina térmica se calcula como la fracción del calor absorbido que es
convertido en trabajo, o bien, a partir de las temperaturas de las fuentes térmicas.

 Wciclo Tcaliente  Tfría


 
Qabsorbido Tcaliente

En los procesos isotérmicos se cumple la ley de Boyle: PiVi = PfVf

En tanto que en los adiabáticos reversibles se cumplen las relaciones:

P1V1  = P2V2  y T1(V1)  1 = T2(V2)  1


Donde  es la relación Cp/Cv. Para un gas diatómico: Cp = 7/2 R y Cv = 5/2 R

∆U = nCv∆T ∆H = nCp∆T ∆U = Q + W

R = 0.082 L atm mol-1 K-1 =1.9872 cal mol-1 K-1 = 8.314 J mol-1 K-1

∆S = nCv ln(T2/T1) + nR ln(V2/V1) = nCp ln(T2/T1) – nR ln(P2/P1)

W = nRT ln(V2/V1) para un proceso isotérmico reversible

38
Una máquina de Carnot opera con 2.75 moles de un gas diatómico (modelo ideal), entre dos
fuentes térmicas. La fuente caliente está a 700 K y la fuente fría está a 500 K.

En la primera etapa se expande a 700 K desde un volumen de 20.0 L hasta otro de 55 L.


En la segunda etapa se expande adiabáticamente hasta alcanzar 500 K.
En la tercera etapa se comprime a 500 K hasta alcanzar una presión de 2.43 atm.
En la cuarta etapa se comprime adiabáticamente hasta cerrar el ciclo.

a) Calcula la eficiencia de la máquina térmica.

Eficiencia  =

b) Calcula el calor absorbido durante la expansión isotérmica.

Calor absorbido = J

c) Calcula el calor cedido a la fuente fría.

Calor cedido = J

d) Calcula el volumen al final de la expansión adiabática.

V= L

39
e) Calcula la variación de entropía del gas en cada etapa del ciclo y la total en el ciclo:
e.1) en la expansión isotérmica.

S = J/K

e.2) en la expansión adiabática.

S = J/K

e.3) en la compresión isotérmica.

S = J/K

e.4) en la compresión adiabática.

S = J/K

40
e.5) en el ciclo.

S = J/K

f) Calcula la variación de entropía de la fuente caliente.

S = J/K

g) Calcula la variación de entropía de la fuente fría.

S = J/K

41
7. El Co(II) y el ligante R forman un complejo de color rojo que tiene un máximo de absorción en la
longitud de onda de 550 nm. Para investigar la relación estequiométrica del complejo, un
estudiante olímpico (que recordaba que la ley de Lambert y Beer es A =  l C), preparó una serie
de disoluciones en las cuales mantuvo constante la concentración del catión (2.5 x 10-5 mol/L) y
varió la del ligante R. Los datos de absorbancia que obtuvo (leídos y representados
gráficamente), al efectuar las medidas en una celda de 1 cm son los siguientes:

Conc. ligante Absorbancia Conc. ligante Absorbancia Conc. ligante Absorbancia


(mol/L) (A) (mol/L) (A) (mol/L) (A)
1.5 x 10-5 0.107 7.5 x 10-5 0.500 13.5 x 10-5 0.535
3.0 x 10-5 0.214 9.0 x 10-5 0.523 15.0 x 10-5 0.535
4.5 x 10-5 0.321 10.5 x 10-5 0.533
6.0 x 10-5 0.441 12.0 x 10-5 0.534

0.6

0.5
Absorbancia (u. a.)

0.4

0.3

0.2

0.1

0.0 1.5 3.0 4.5 6.0 7.5 9.0 10.5 12.0 13.5 15.0
-5
Concentración de R (x 10 mol/L)

a) Esto le permitió deducir cuál(es) de estas especies químicas (el Co(II), el ligante R o el complejo
CoRx) absorbe(n) en esta longitud de onda.

b) Pudo calcular que la relación del catión y el ligante para el complejo es:

c) y que el valor de  para el complejo es igual a:

42
d) Del valor experimental leído y el teórico cuando las líneas se interceptan pudo calcular que el
porcentaje de disociación del complejo era:
Cálculos:

% de disociación del complejo:

e) y que la constante de disociación del complejo es:


Cálculos:

Valor de la constante Kd=

8. La determinación gravimétrica de los cloruros se realiza precipitando el cloruro de plata y


pesando el sólido después de haberlo secado; sin embargo, debe tenerse mucho cuidado de que
no incida la luz sobre le precipitado porque puede reducirse de acuerdo con la siguiente reacción:

2 AgCl  2 Ag + Cl2

A su vez, el cloro con el agua bismuta para dar clorato y cloruro, de acuerdo con la reacción:
3 Cl2 + 3 H2O  ClO3- + 5 Cl- + 6 H+

Con el exceso de nitrato de plata el cloruro formado vuelve a precipitar mientras que el clorato de
plata es soluble.
a) ¿Cuál es el error cometido en la determinación cuando el 10 % del AgCl inicialmente precipitado
se descompone con la luz?
Cálculos:

% de error:

9. Las moléculas con simetría plana pueden ser representadas en un eje cartesiano con coordenadas
(x, y). A partir de esta representación y con ayuda de relaciones trigonométricas es posible
calcular algunos parámetros importantes tales como la distancia y el ángulo entre los átomos.
Como ejemplo se tiene el NO3-, donde (las coordenadas para el nitrógeno N (0,0) y para los
oxígenos son O1 (1.060, 0.6125) y O2 (-1.060, 0.6125), O3 (0.0, -1.225), el ángulo de enlace es
de 120° tal como se muestra en la siguiente figura:

-
120

2O O1
120 N 120

O3

43
Adicionalmente se puede conocer la carga parcial de cada átomo (Q) en la estructura.
En el caso de la molécula BBrICl, se conoce la siguiente información:

Átomo Coordenadas (x, y) Q (carga del átomo)


B (0, 0) 0.274
I (0.0, 1.270) X
Cl (1.413, -0.803) -0.068
Br (-1.710, -0.479) -0.102

a) Con esta información calcula:


a) La distancia de enlace B-Br
b) La distancia de enlace B-Cl
c) La distancia de enlace B-I
d) El ángulo de enlace Br-B-Cl
h) La carga X del átomo de I

Las siguientes relaciones trigonométricas pueden ser útiles:


y x y
r  (x 2  x 1 ) 2  (y 2  y1 ) 2 sen θ  cos θ  tan θ 
r r x

FIN DEL EXAMEN

44
XV Olimpiada Nacional de Química
Examen Experimental de Química Analítica San Luis Potosí 2006

Número de código: __________________________________________ No. de muestra: _________

La Universidad de San Luis Potosí recibió recientemente un donativo con reactivos para
laboratorios. Entre ellos se encontraba un lote con cinco frascos que contenían hidróxido de sodio y
algunas sales de este mismo catión. Desafortunadamente las etiquetas no indica que compuestos se
encuentran en cada frasco ni tampoco su pureza.

Algunas pruebas preliminares indican que cada frasco puede contener, ya sea un compuesto puro o
mezclas de dos de ellos. Las posibilidades son:

a) NaOH b) Na2CO3 c) NaHCO3 d) NaOH + Na2CO3 e) Na2CO3 + NaHCO3

Tu tarea esta tarde será ayudarnos a determinar el contenido de cada frasco y la pureza de el(los)
compuesto(s) que contenga(n). Para lograrlo nosotros hemos preparado disoluciones con cantidades
exactamente pesadas del contenido de cada uno de los frascos y te hemos proporcionado una
muestra de una de las disoluciones. Tu deberás indicarnos cuál(es) es(son) la(s) sustancia(s) que
contiene tu muestra y cuál es la concentración de la(s) misma(s).

Cuentas para ello con una disolución de HCl valorada (cuya concentración exacta está indicada en la
etiqueta), material volumétrico apropiado y goteros que contienen dos indicadores (fenolftaleína y
anaranjado de metilo).

Datos: pKa del H2CO3 = 6.0 y 10.3

Intervalo de vire de los indicadores:


Fenolftaleína: 8.0 a 9.6 Anaranjado de metilo: 3.1 a 4.4

Información adicional.
El punto de equivalencia para la neutralización de la sosa puede ser detectado con cualquiera de los
dos indicadores.

La fenolftaleína permite detectar el punto de equivalencia de la primera neutralización del carbonato


y el anaranjado de metilo el de la segunda.

Consejo: Efectúa la valoración con cada uno de los indicadores (utilizando alícuotas diferentes).

En una hoja blanca describe brevemente el procedimiento que te permitirá conocer cuáles son los
componentes en la disolución que te hemos proporcionado. (No olvides poner tu número de
código y número de muestra).

Entrega tu procedimiento a uno de los asesores quien te dará a cambio la hoja en que deberás anotar
tus resultados.

45
HOJA DE RESPUESTAS

Número de código: __________________________________________ No. de muestra: _________

Procedimiento y resultados

Toma una alícuota de 5.0 mL de tu muestra y colócala en el vaso d precipitados; titula con la
disolución de ácido clorhídrico (cuya concentración se indica en la etiqueta) usando el indicador
apropiado. Agita suavemente después de cada adición y evita cualquier pérdida del líquido. Indica
los volúmenes de HCl utilizados en las valoraciones efectuadas.

Indicador utilizado Volumen gastado Indicador utilizado Volumen gastado

Volumen que utilizarás en tus cálculos.

Indicador utilizado Volumen gastado


Fenolftaleína
Anaranjado de metilo

El(los) compuesto(s) que se encuentran en tu muestra son:

Y su concentración es:

46
XV Olimpiada Nacional de Química
Examen Experimental de Química Inorgánica San Luis Potosí 2006

Número de código: _________________________________________ No. de muestra: _________

Pregunta Experimental 1

“Síntesis de un compuesto de coordinación de Ni(II)”

Los compuestos de coordinación se asocian principalmente a la química de los metales de


transición. Una de sus características principales son los colores que estos presentan; los cuales
dependen del estado de transición del ion metálico y de los gigantes que estén unidos.

Las rutas se síntesis, de algunos de estos compuestos puede ser tan sencilla como mezclar
disoluciones de ion metálico y de los gigantes.

Se pueden obtener diferentes compuestos de un mismo ion metálico con el mismo estado de
oxidación y un mismo ligante, cambiando la relación estequiométrica.

Esta tarde tu labor será sintetizar un compuesto de coordinación de Ni(II) con el ligante bidentado
etilendiamina “en” (NH2CH2CH2NH2) en una relación estequimétrica (1:3).

“Para lograrlo necesitas hacer un cálculo previo”:


¿Cuántos mililitros de una disolución (0.1 M) de etilendiamina debes adicionar a 2 mL de una
disolución de níquel (0.1 M) para obtener el compuesto de relación estequiométrica (1:3)?

Cálculos:

Volumen necesario =

Una vez realizado este cálculo pídele a alguno de los supervisores que te entreguen el
procedimiento experimental.

47
Número de código: _________________________________________ No. de muestra: _________

Pregunta Experimental 2

“Síntesis de un compuesto de coordinación de Ni(II)”

Procedimiento

Coloca 2 mL de la disolución de níquel (0.1 M) en un vaso de precipitados de 50 mL. A esta misma


disolución agrégale 6 mL de una disolución (0.1 M) de etilendiamina. Realiza esta adición,
lentamente con agitación suave. Anota tus obervaciones.

Al término de la adición agrega 15 mL de acetona en el vaso que contien la mezcla de reacción.


Anota tus observaciones.

Coloca el vaso en el recipiente con hielo durante 10 minutos para que finalice la precipitación y
filtra el producto en el papel filtro que te proporcionamos en la bolsa. Evita que el producto quede
pegado en las paredes del vaso de reacción arrastrándolo con un poco más de acetona. Deja secando
el producto en el embudo mientras contestas las preguntas de la siguiente hoja y realiza la pregunta
experimental 2.

Cuando este completamente seco el producto, con mucho cuidado dobla el papel y deposítalo en la
bolsa asignada, para entregarlo a uno de los supervisores.

48
Número de código: _________________________________________ No. de muestra: _________

Preguntas

1. Durante la adición del ligante, si observaste cambios en la disolución, en tu opinión estos se


deben a:

a) Sólo a la formación de un precipitado.


b) No hubo cambios notables.
c) La formación del compuesto de coordinación entre el Ni(II) y en con relación
estequiométrica (1:3).
d) La formación de varios compuestos de coordinación entre el Ni(II) y en con diferentes
relaciones estequiométricas.
e) Efectos de la luz sobre la mezcla de reacción.
f) Efecto de la disminución de la solubilidad del compuesto de coordinación de Ni(II) y
en con relación estequimétrica (1:3).

2. Escribe la ecuación química balanceada, correspondiente a la formación del producto obtenido en


este experimento.

3. Propón una estructura para el compuesto obtenido, tome un cuenta que esta partiendo de una
disolución de NiSO4.

49
50
XVI Olimpiada Nacional de Química

1er Examen Nivel A y B.


Total: 30 preguntas. Tiempo asignado: 75 minutos. Toluca 2007

Actualmente el desarrollo de las ciencias impone un modelo multidisciplinario en el que la química se


relaciona con otras áreas del conocimiento como las matemáticas, la física, la biología y la medicina. Debes
anotar en el recuadro correspondiente la letra del inciso que contesta correctamente cada pregunta. Si
consideras que ningún inciso es el correcto deberás anotar una letra X. Recuerda que siempre es bueno leer
con atención cada pregunta.
NOTA ACLARATORIA: Al hacer tus cálculos puedes encontrar una pequeña diferencia, en general
no mayor de unas décimas, entre tu resultado y alguno de los propuestos. Esto es normal y por eso sólo
deberás usar la “X” cuando ninguna respuesta sea muy cercana a la obtenida por ti.

************************************************************************************

Para complementar su alimentación, muchas personas ingieren pastillas que contienen vitaminas y minerales.
En un producto comercial encontramos la siguiente información: Cada tableta con una masa de 0.80 g
contiene las siguientes sustancias:

Fosfato de calcio 160 mg Sulfato ferroso 50.9 mg


Yoduro de potasio 0.20 mg Sulfato de cobre (II) pentahidratado 2.5 mg
Sulfato de manganeso 3.10 mg Óxido de zinc (ZnO) 15 mg
Oxido de magnesio 100.0 mg Vitamina B2 5 mg
Vitamina B6 5 mg Vitamina C (ácido ascórbico) 90 mg
Vitamina E (d-alfa-tocoferol) 30 U.I. Selenato de sodio 0.05 mg
El resto es una sustancia inerte.

U.I. significa “unidad internacional” pero no pertenece al Sistema Internacional de Unidades. El término U.I.
está asociado a una actividad biológica más que a una masa determinada. Por esta razón, el significado de
U.I. es diferente para cada sustancia, por ejemplo, una U.I. de vitamina C equivale a 0.050 mg de ácido
ascórbico y una U.I. de vitamina E corresponde a 0.667 mg de d-alfa-tocoferol.

1) La fórmula química de la vitamina E es C29H50O2, ¿Cuántas tabletas se deberían tomar para


ingerir 0.001 moles de esta sustancia?

A Menos de 15 tabletas B Entre 16 y 18 tabletas


C Entre 20 y 22 tabletas D Más de 24 tabletas

2) La masa molar del sulfato de cobre (II) pentahidratado es, en g / mol:

A Menos de 240 B Entre 240 y 260


C Entre 261 y 280 D Más de 280

51
3) Si una persona ingiere dos de estas tabletas, ¿cuántas U.I. de vitamina C habrá ingerido?

A Menos de 3000 B Entre 3100 y 3400


C Entre 3500 y 3700 D Más de 3800

4) El sulfato férrico o sulfato de hierro (III), Fe2(SO4)3, es un compuesto cuya masa molar es,
aproximadamente:

A 400 g mol-1 B 200 g mol-1


C 800 g mol-1 D 112 g mol-1

5) En una mezcla del mismo número de gramos de FeSO4 y de Fe2(SO4)3, la fracción mol del
sulfato ferroso o sulfato de hierro (II) es:

A Menor de 0.5 B 0.5


C Entre 0.55 y 0.7 D Mayor a 0.7

6) Una de estas tabletas se disuelve totalmente en agua hasta tener 100 mL de disolución, la
concentración molar de selenato de sodio (Na2SeO4) será. en mol L-1:

A Menor a 3 x 10-6 B 2.6 x 10-5


C 2.6 x 10-2 D Mayor a 3 x 10-2

7) Un análisis demostró que el compuesto de calcio contenido en las tabletas es realmente CaHPO4.
El % en masa de Ca en este compuesto es:

A Menor al 25 % B Entre 26 y 35 %
C Entre 36 y 45 % D Mayor al 45 %

8) El sulfato de zinc se puede obtener haciendo reaccionar zinc metálico con ácido sulfúrico. Otro
de los productos de esta reacción es:

A Oxígeno B Hidrógeno
C Azufre D Agua

9) Para producir el óxido de zinc se hace reaccionar este metal con oxígeno gaseoso. ¿Cuántos
moles de oxígeno se requieren, aproximadamente, para producir 16 mg de este compuesto?

A 3.1 x 10-3 B 9.83 x 10-5


C 4.9 x 10-5 D 1.55 x 10-3

10) ¿Cuál de estos elementos tiene la mayor electronegatividad?

A Flúor B Oxígeno
C Neón D Litio

52
11) De acuerdo a la ley de los gases ideales, 16 gramos de oxígeno gaseoso, a una temperatura de
10ºC y una presión de 0.5 atmósferas, ocuparían un volumen de:
(R = 0.082 L atm / mol K)

A Menos de 12 L B Entre 12 y16 L


C Entre 17 y 22.8 L D Más de 22.8 L

12) El ácido nítrico es un líquido cuya densidad es 1.574 g / cm3. Por lo tanto, la masa contenida en
medio litro de ácido nítrico es:

A 15.74 g B 157.4 g
C 7.87 g D 78.7 g

13) Si consideramos que el ácido nítrico (HNO3) concentrado tiene un 90 % en masa del ácido y el
resto es agua, al calcular la molalidad (mol kg–1) del ácido suponiendo al agua como el
disolvente, obtenemos un valor igual a:

A 14.28 m B 142.8 m
C 90 m D 9.0 m

14) Un matraz aforado de 100.0 mL se pesa vacío y la lectura es 25.83 g. Este matraz se llena hasta
el aforo con un líquido “X” y se pesa nuevamente. Ahora la lectura es 108.93 g. La densidad del
líquido “X” es, en g /cm3:

A 0.00831 B 0.0831
C 0.831 D 83.1

15) En la cifra 0.000421, el número de cifras significativas es:

A seis B cinco
C cuatro D tres

16) Si se mezcla medio litro de disolución acuosa 0.2 molar (mol L-1) de cloruro de sodio con
250.0 mL de una disolución 0.5 molar del mismo compuesto, la concentración molar del cloruro
de sodio en la disolución obtenida es:

A 0.7 M B 0.5 M
C 0.3 M D 0.225 M

17) Al balancear la ecuación siguiente, el coeficiente estequiométrico para el oxígeno es:


FeCr2O4 + K2CO3 + O2  2 Fe2O3 + K2CrO4 + CO2

A 12 B 7
C 6 D 14

53
Para contestar las preguntas 18 a 23, considera los datos que aparecen en la siguiente tabla:

Nombre de la sustancia Fórmula Punto de fusión Punto normal de ebullición


(1 atmósfera)
Metanol CH3OH - 98 0C 65 0C
Etanol C2H5OH - 130 0C 78 0C
Benceno C6H6 5.5 0C 80 0C
Agua H2O 0 0C 100 0C
1atm = 1.01325 x 105 Pa = 760 mmHg

18) Si tomamos una muestra de 100.0 g de cada una de estas sustancias, ¿en cuál de las muestras
tendríamos una menor cantidad de sustancia (moles)?

A Metanol B Etanol
C Benceno D Agua

19) Si preparamos una disolución con la misma cantidad de gramos de agua y de etanol, la fracción
mol del etanol en esta disolución es:

A 0.5 B mayor a 0.5


C menor a 0.5 D No se puede calcular sin
conocer la masa total de la
disolución

20) A una temperatura de 348 K y P = 760 mmHg, ¿cuántas de estas sustancias se encuentran en
estado líquido?

A Todas B Tres
C Una D Ninguna

21) ¿Cuál de estas sustancias tiene una densidad menor en estado sólido que en estado líquido?

A Metanol B Etanol
C Benceno D Agua

22) ¿Cuál de estas sustancias tiene una molécula en forma de ciclo?

A Metanol B Etanol
C Benceno D Agua

23) Si tenemos una muestra de etanol y nos encontramos a una altura de 3000 metros sobre el nivel
del mar, ¿cuál podría ser la temperatura de ebullición de esta sustancia?

A 68 oC B 78 oC
C 88 oC D 93 oC

54
24) ¿Cuál de los siguientes óxidos al reaccionar con agua produce un ácido fuerte?

A SO3 B K2O
C CO2 D Na2O

25) Al reaccionar 100.0 mg de sodio con 100 mL de agua, se produce hidróxido de sodio e
hidrógeno. El volumen de hidrógeno desprendido (medido a 1 atmósfera y a 273.15 K) es, en
mL:

A 22400 B 224
C 49 D 98

26) La concentración de la disolución de hidróxido de sodio formada en la reacción indicada en la


pregunta anterior es:

A 4.3510-3 M B 4.3510-2 M
C 4.3510-1 M D 4.35102 M

27) Si disolvemos 1 x 10-3 moles de NaOH en 100 mL de agua, el pH de esta disolución será:

A -3 B 3
C 7.3 D 11

28) De acuerdo con el modelo de Lewis identifique el grupo de especies químicas con carácter
básico.

A NaOH, HCl, Na2SO4 B NaOH, NH3, PCl3


C Al(OH)3, H2SO4 D CO, Cl-, NO2-

29) En la siguiente reacción, las letras griegas ““, ““, ““ representan elementos químicos:
4 3  2 2 + 4 2 + 2
El elemento  es de mucha importancia para la vida y en estado gaseoso su molécula es
diatómica. La masa molecular del compuesto 3 es 68.94 g / mol y este compuesto contiene
nitrógeno. El elemento  es:

A H B Li
C Be D Na

30) El carbonato del elemento  (mismo de la pregunta anterior) se descompone calentándolo a


800 0C. Los productos obtenidos son el óxido del elemento  y CO2. Por cada mol del carbonato
que se descompone, se obtienen de CO2:

A 0.5 moles B 1 mol


C 2 moles D 3 moles

FIN DEL PRIMER EXAMEN A y B

55
56
XVI Olimpiada Nacional de Química. 2do Examen Nivel A y B.
Nivel B: Sólo deben contestar las primeras 8 preguntas.
Nivel A: Deben contestar las 12 preguntas.
Tiempo asignado: 75 minutos. Toluca 2007

1. Un carro tanque de ferrocarril de 30 m3 de capacidad transporta gas L. P. (gas Licuado de


Petróleo). El gas se transporta a presión en forma líquida, pero al descargar el tanque a
temperatura ambiente de 25 °C, sólo queda la mezcla gaseosa con una composición de 30 % en
masa de propano (C3H8) y el resto de butano (C4H10) a una presión ambiental de 650 mmHg. Al
terminar la descarga se inicia un incendio que pone en peligro al tanque para lo cual se rocía con
agua. Sin embargo, se estima que la temperatura del tanque puede ascender hasta 90 °C. A esta
temperatura los empaques de las válvulas del tanque resisten una presión máxima de 3 atmósferas.

R = 8.314 J/mol K = 8.314 Pa m3/mol K = 0.082 atm L/mol K; 1atm = 760 mmHg = 101325 Pa

Considerando comportamiento ideal de los gases, contesta lo siguiente:


a) La cantidad de sustancia (en mol) de propano en el tanque al finalizar la descarga.

npropano = mol

b) La cantidad de sustancia (en mol) de butano en el tanque al finalizar la descarga.

nbutano = mol

57
c) La fracción mol de propano en el tanque.

d) La masa de propano (en kg) en el tanque.

Masa de propano: kg

e) La masa de butano (en kg) en el tanque.

Masa de butano: kg

f) La presión total (en Pa) en el tanque a 90 °C.

P= Pa

58
g) La presión parcial del butano (en Pa) en el tanque a 90 °C.

P= Pa

h) La densidad de la mezcla (en kg/m3) en el tanque a temperatura ambiente de 25 °C

Densidad a 25 °C = kg/m3

i) La densidad de la mezcla (en kg/m3) en el tanque a la temperatura de 90 °C

Densidad a 90 °C = kg/m3

j) ¿Resistirán los empaques de las válvulas? Marca con una X y justifica tu respuesta.

Resisten: SI: NO:


Explicación:

59
2. La fracción molar de dióxido de azufre (SO2) disuelto en una mezcla de gases que contiene 120 g
de SO2 en cada 1500 g de dióxido de carbono es:
Cálculos:

Fracción molar de SO2 =

3. Cuando un estudiante disolvió 0.1 mol de un ácido HA en un litro de disolución, obtuvo un valor
de pH=2.4. Esta observación le permitió indicar que el valor del pKa de este ácido es:
Cálculos:

Valor de pKa=

4. Los electrodos de un acumulador de plomo están constituidos, respectivamente, por láminas de


plomo recubiertas por plomo esponjoso y por PbO2, ambos sumergidos en H2SO4 4.1 mol/L.

a) Las reacciones balanceadas que ocurren sobre cada uno de los electrodos en el proceso de
descarga del acumulador y la polaridad de cada electrodo en cada proceso son:

b) Si dicho acumulador suministra durante su descarga 10 A h, calcula la cantidad de PbO2


transformado a Pb2+ luego de una hora de descarga.
Cálculos:

Cantidad de PbO2 =

Datos: Valor del Faraday 96484 C.

60
5. El envenenamiento por cadmio produce problemas pulmonares, óseos, etc., debido a que
interfiere en el metabolismo de algunos oligoelementos (tales como Zn, Cu, Fe y Ca). El Cd2+ se
vierte en descargas de diferentes tipos de industrias (baterías, pigmentos, semiconductores,
estabilizadores de PVC, etc.). Una de estas industrias vierte un contenido de Cd2+ igual a 0.003
moles por cada litro de agua; el técnico responsable afirma que si se alcaliniza el agua hasta un
pH=8, el cadmio quedará insolubilizado en forma de sulfato y/o de hidróxido y que, por tanto,
mediante filtros podría ser eliminado de las aguas de descarga. Calcula la composición de la
disolución para demostrar si en esas condiciones es cierto lo que el técnico asegura (sin
considerar las concentraciones de otras posibles especies en disolución ni los equilibrios de
acidez del ion SO42–).

Datos: pKs (CdSO4) = 1.59; pKs (Cd(OH)2) = 13.8.

[H+]=_____________ mol/L; [OH–]=________________mol/L ;

[Cd2+]=______________ mol/L; [SO4–2-]=________________mol/L ;

Por lo que_______________________________________________________________
queda disuelto precipita como sulfato o como hidróxido

6. De los siguientes ácidos del cloro: HClO, HClO2, HClO3 y HClO4, el más fuerte es el:

a) HClO b) HClO2 c) HClO3 d) HClO4 e) No hay diferencia en la fuerza ácido-base.

Respuesta: ___________

7. La fuerza de estos ácidos:

a) es mayor cuanto mayor sea el número de oxidación del elemento central.


b) es mayor cuanto menor sea el número de oxidación del elemento central.
c) no depende del número de oxidación del elemento central.
d) es la misma cuando se trata del mismo elemento central.
e) es menor cuando aumenta el número de átomos de oxígeno.

Respuesta: _____________

8. Completa y balancea la siguiente reacción iónica en medio básico:

_____ Mn2+ + _____ MnO4- + _____  _____ MnO42- + _____________

61
DE AQUÍ EN ADELANTE SÓLO DEBEN DE CONTINUAR LOS DEL NIVEL A.

9. Indica cuantos isómeros estructurales, incluyendo estereoisómeros, existen con la fórmula C4H8.

a) 3 b) 4 c) 5 d) 6

10. ¿Cómo se conoce a la orientación en el espacio de los átomos de una molécula?

a) Constitución b) Configuración c) Conformación d)


Composición

11. Indica como se podrían llevar a cabo las siguientes transformaciones:


CH 3
H 3C C O H
a CH 3
H
b
H 3C C CH 2 Br

c CH 3

d H 3C C CH 2 O H
CH 3

OH
H 3C C CH 2 O H
CH 3

Respuesta a Respuesta b

Respuesta c Respuesta d

62
12. Escribe la estructura del producto principal de cada una de las siguientes reacciones:

RESPUESTA a
H
Et
H
KOH, EtOH
a) H Me
H Cl
H CALOR
H
H H

RESPUESTA b
Br
KOH, EtOH
H H
b) CALOR
H 3C
CH3

RESPUESTA c
-+
O O O: K
S
c) O
CALOR
CH 3

FIN DEL EXAMEN

63
64
XVI OLIMPIADA NACIONAL DE QUÍMICA
TERCER EXAMEN NIVEL A y B Tiempo asignado: 180 minutos Toluca 2007

1. Se disolvió 1.0 mL de ácido clorhídrico concentrado (37% m/m,  = 1.19 g mL ) en un matraz de


–1

500.0 mL y se diluyó con agua destilada hasta el nivel del aforo. De la disolución así preparada, se
tomaron 20.0 mL, se vertieron en un erlenmeyer de 250 mL al cual se le agregaron 25 mL de agua.
¿Qué volumen (expresado en mL) de NaOH (ac) 0.100 M se necesitan para neutralizar el ácido
contenido en el erlenmeyer y cuál es la concentración (expresada en mol L–1) de la disolución
concentrada inicial?

Cálculos:

Volumen de NaOH __________________________ mL

Concentración inicial ______________________ mol L-1

2. Cuando dos placas de un mismo metal M, con igual masa y superficie se sumergieron, una en una
disoluciones de AgNO3 y otra en una de Cu(NO3)2, se observó el depósito de Ag y Cu en cada
una de las respectivas placas. Por otra parte, en ambas disoluciones se demostró la presencia de
iones M2+. Una vez terminado el depósito, las dos placas metálicas se secaron y pesaron; se
observó una disminución del 5.51 % de la masa en la que había estado sumergida en la disolución
de nitrato de cobre (II) y un aumento del 9.70 % en la sumergida en la de nitrato de plata.
Indica

a) Las reacciones (balanceadas) que ocurren en cada una de las placas son:

65
b) El elemento del cual están constituidas las placas es:
Cálculos:

El metal es:

c) Esta información te permite deducir que el valor del potencial estándar del sistema M2+/M es

__________________________ que el del sistema Cu2+/Cu, y que el del sistema Ag+/Ag es


mayor o menor

__________________________ que el del sistema M2+/M.


mayor o menor

3. Se añade lentamente nitrato de plata sólido a una disolución acuosa que es 0.0010 mol/L en
NaCl y en NaBr. Datos (a 298.15 K): Ks (AgBr) = 3.3×10–13;s Ks(AgCl) = 1.8×10–10.

a) Calcula la [Ag+] requerida para iniciar la precipitación de cada uno de los haluros de plata.

[Ag+] para que precipite el bromuro:

[Ag+] para que precipite el cloruro:

b) Calcula el porcentaje de AgBr precipitado antes de que precipite el AgCl.


Cálculos:

66
4. Para el ion tiosulfato indica:

a) La estructura de Lewis:

b) El estado de oxidación y la carga formal de cada uno de los átomos de azufre es:

c) La hibridación propuesta para el átomo central es:

5. Se tienen cinco jeringas etiquetadas del 1 al 5, cada una contiene uno de los siguientes gases: He,
Cl2, N2, O2, H2.

A partir de la información siguiente, identifica el gas que está contenido en cada jeringa.

i) Los gases en las jeringas 1, 2, 3 y 5 no presentan propiedades ácido-base en disolución acuosa.


El contenido en la jeringa 4 se comporta como ácido en disolución acuosa.
ii) Cuatro de los gases son incoloros. El que es colorido reacciona con una disolución de yoduro
de potasio.
iii) Al hacer pasar los gases a través de un tubo caliente que contiene óxido de cobre, éste
cambia de color sólo con el gas de la jeringa número dos.
iv) Al pasar los gases a través de un tubo caliente que contiene cobre metálico, éste se cubre de
un polvo negro con el gas contenido en la número tres.
v) Al burbujear los gases en una disolución jabonosa y acercar un cerillo, dos de ellos presentan
reacción positiva.
vi) El gas tres contenido en las burbujas es atraído por un imán.
vii) Para discriminar entre los dos gases que son inertes, se cuenta con los datos de la masa de
cada jeringa llena y vacía. El gas número uno es el menos denso.

67
Jeringa 1 2 3 4 5

Gas

6. Los gases que se mencionan se burbujean en agua desionizada. Considere los datos de la
siguiente tabla y contesta las preguntas que se plantean a continuación. En todos los casos
justifique su respuesta.

Gas N2 O2 CO2 NH3 HCl


Solubilidad
0.018 0.039 1.45 470 695
g/kg (1 atm)
Densidad
0.89 1.2
kg/L
Masa molar
28 32 44 17 36.5
(g/mol)

a) ¿Cuál(es) (de las) disolución(es) obtenida(s) conduce(n) la corriente eléctrica?

b) Escribe todas las ecuaciones que representan la reacción química que se lleva a cabo entre cada
uno de los gases y el agua.

c) Indica cuál(es) disolución(es) tiene(n) comportamiento ácido.

68
d) Indica cuál(es) disolución(es) tiene(n) comportamiento básico.

e) Al preparar una disolución saturada de cada gas, ¿cuál será la concentración molar de cada una de
las especies presentes en cada una de ellas?

********************************************************************************

LAS PREGUNTAS 7 Y 8 SÓLO DEBEN SER CONSTESTADAS POR LOS DE NIVEL B (Y


CON ESTAS PREGUNTAS TERMINA EL EXAMEN PARA LOS DEL NIVEL B).

LOS DE NIVEL A CONTINÚEN CON LAS PREGUNTAS DE LA 9 A LA 13.

7. Las máquinas de vapor operan con pistones que reciben el vapor generado por la caldera que
llevan al frente. Al entrar el vapor al pistón lo empuja provocando que la rueda se mueva. La
primera máquina de vapor operó con un pistón dispuesto en forma vertical.

Al quemar el carbón en la caldera, se liberan 393.505 kJ/mol. Parte de ese calor liberado se
aprovecha en calentar el agua (Qsensible) hasta su temperatura de ebullición para después, con más
calor, convertirlo en vapor (Qlatente).

69
Qsensible  mc esp T
y
Qlatente  mvaporización
donde m es la masa.

La capacidad térmica específica del agua es cesp = 4.184 J/g°C y el calor necesario para
transformar un gramo de agua en vapor es vaporización = 2260 J/g.

R = 8.314 Pa m3/mol K = 8.314 J/mol K = 1.987 cal/mol K; 1 atm = 101325 Pa.


Calcula:

a) La cantidad de calor liberado (en joules) cuando se quema una tonelada de carbón.

Calor liberado = J

b) La cantidad de calor (en joules) necesaria para transformar 500 g de agua desde líquido a
temperatura ambiente (25 °C) hasta vapor a su temperatura normal de ebullición (100 °C).

Calor necesario = J

c) Partiendo de agua líquida a 25 °C, el vapor en kg producido al quemarse una tonelada de carbón
si sólo se aprovecha el 87% del calor liberado en la combustión.

Vapor producido = kg

70
d) El volumen de vapor generado (en m3) cuando un litro de agua (agua = 1.0 g/mL) se transforma
en vapor a su temperatura normal de ebullición. (Considera un comportamiento ideal de los
gases).

Volumen generado = m3

e) El trabajo producido cuando el vapor generado por un litro de agua a 100 °C se expande contra
una presión de oposición constante de 1 bar. (1 bar = 105 Pa). W =  PopV.

Trabajo producido = J

f) El trabajo producido por el pistón de diámetro d = 20 cm, cuando se desplaza la distancia a = 10


cm contra una presión de oposición constante de 12 bar.
d

Trabajo producido = J

71
8. Una forma de verificar la pureza de algunas sustancias es a través de la determinación de sus
propiedades físicas como son la presión de vapor, la temperatura de fusión y la de ebullición.

Tf = Kf m Tb = Kb m Pvap = P*vap puro x Donde m es la molalidad

La presencia de impurezas, así como la adición de un soluto a un disolvente, altera


considerablemente estas propiedades. La medición en el cambio de estas propiedades permite
determinar no sólo la pureza del disolvente sino también algunas propiedades del soluto como
son su masa molar y su capacidad para disociarse en disolución.

En el laboratorio se preparó una disolución que contiene 1568 g de agua como disolvente y 258
g de un soluto cuya composición es: Carbono 40%, Hidrógeno 6.67 % y Oxígeno 53.33 % en
masa. La disolución registró una temperatura de fusión de – 1.7 °C.

La temperatura de ebullición del agua en la Cd. de México a una presión de 585 mmHg es de
93 °C. Para el agua la constante crioscópica es Kf = 1.86 K kg/mol y la constante ebulloscópica
Kb = 0.51 K kg/mol.
Calcula:

a) El aumento en la temperatura de ebullición de la disolución.

Tb = °C

b) La masa molar del soluto.

Msoluto = g/mol

c) La presión de vapor de la disolución a la temperatura de ebullición de la Cd. de México.

Pvapor = mmHg

72
d) La fórmula mínima del soluto.

Fórmula mínima:

e) La fórmula molecular del soluto.

Fórmula molecular:

9. Los calores producidos por la combustión completa de muchos compuestos orgánicos se


determinan quemando la sustancia en una bomba de combustión y midiendo el calor generado,
con lo que se obtiene el calor de combustión a volumen constante. Los calores de combustión se
emplean para calcular los calores de formación de compuestos orgánicos.

Si el compuesto sólo contiene carbono e hidrógeno, los productos de la combustión son agua
líquida y bióxido de carbono gaseoso y a partir de las entalpías de formación de estos
compuestos y junto con el calor de combustión es posible determinar la entalpía de formación
del compuesto.

TABLA 1: Entalpías estándar a 25 °C


Sustancia Entalpía de combustión
Benceno C6H6 (l) Hcomb = – 780 980 cal/mol
Benceno C6H6 (g) Hcomb = – 787 200 cal/mol

Sustancia Entalpía de formación


H2O(l) Hf = – 68 320 cal/mol
H2O(g) Hf = – 57 800 cal/mol
CO2 (g) Hf = – 94 050 cal/mol

73
A partir de las entalpías de la tabla anterior calcula:

a) La entalpía de formación del benceno líquido.

Hf = cal/mol

b) La entalpía de formación del benceno gaseoso.

Hf = cal/mol

c) La entalpía de vaporización del benceno.

Hf = cal/mol

d) La entalpía de vaporización del agua.

Hf = cal/mol

e) El calor liberado en la combustión de un gramo de benceno líquido.

Q= cal

74
f) La cantidad de agua en gramos producida en la combustión de un gramo de benceno.

Agua producida = g

10. Escribe las ecuaciones para la reacción del sulfato ácido de p-nitrobencendiazonio con cada una
de las siguientes sustancias:
a) m-Feniléndiamina (1,3-diaminobenceno).
b) p-Cresol (p-metilfenol).
c) KI.
d) HBF4, Calor.

RESPUESTA a

RESPUESTA b

RESPUESTA c

RESPUESTA d

75
11. La Novocaína es un anestésico local. Su fórmula molecular es C13H20N2O2. Es insoluble en agua
y en disolución acuosa de NaOH, pero si es soluble en HCl diluido. Forma un sólido con un color
intenso cuando se trata con NaNO2 y HCl y después con –naftol.
Al calentar la Novocaína con una disolución acuosa de NaOH se observa la disolución lenta del
compuesto. De la disolución anterior se hace una extracción líquido-líquido con éter etílico para
aislar uno de los compuestos orgánicos de la mezcla de reacción. La fase acuosa se acidula y se
obtiene un precipitado (Compuesto A) el cual se aísla por medio de una filtración al vacío.
Cuando el Compuesto A se trata con más ácido clorhídrico, este se vuelve a disolver. El
Compuesto A tiene la fórmula molecular C7H7NO2.
Cuando se elimina el disolvente de la fase etérea por destilación, se obtiene un líquido
(Compuesto B), el cual tiene como fórmula molecular C6H15NO. El Compuesto B se disuelve en
agua, dando una disolución que tiñe de color azul al papel tornasol. Cuando se trata el Compuesto
B con anhídrido acético, se obtienen el Compuesto C con fórmula molecular C8H17NO2. El
Compuesto C es insoluble en agua y en disolución acuosa de NaOH, pero si es soluble en una
disolución de HCl diluido. Se encontró que el Compuesto B es idéntico al compuesto que se
obtiene cuando se hacen reaccionar la dietilamina con óxido de etileno.

¿Cuál es la estructura de la Novocaína? Escriba la estructura de los Compuestos A al C.

COMPUESTO A COMPUESTO B COMPUESTO C

NOVOCAÍNA

76
12. ¿Qué alcohol reacciona en cada caso, con oxicloruro de fósforo y piridina para dar los
correspondientes alquenos?

a) ALCOHOL

POCl3

b) ALCOHOL

POCl3

c) ALCOHOL

POCl3

77
13. Escribe la estructura de los compuestos intermediarios de la siguiente secuencia sintética:

O
1) CH 3Li 1) O3, CH2Cl2 NaOH
A B C
2) HCl, H 2O,  2) (CH 3)2S CH 3OH
 1) (CH 3)2CuLi
2) CH 3I

H 2N NH 2 . H 2O
E D
KOH
O
HO OH

COMPUESTO A COMPUESTO B COMPUESTO C

COMPUESTO D COMPUESTO E

FIN DEL EXAMEN

78
XVI OLIMPIADA NACIONAL DE QUÍMICA
EXAMEN INTERNACIONAL
8 PREGUNTAS Tiempo: 4 HORAS Toluca 2007

PROBLEMA 1.

Fritz Haber (1868-1934) nació en Breslau Alemania. En 1905 reportó la producción de pequeñas
cantidades de amoníaco a partir de N2 e H2 a una temperatura de 1000 °C, utilizando hierro como
catalizador. Posteriormente, habiendo realizado muchas pruebas con otros catalizadores logró la
síntesis de amoniaco a una temperatura de 500 °C y a presiones entre 150-200 atm. Las
modificaciones al proceso de Haber permitieron la fabricación de sulfato de amonio que se utiliza
como fertilizante. Haber fue galardonado con el premio Nóbel de química en 1918.

½ N2(g) + 3/2 H2(g) ⇄ NH3(g)

Transcurrido el tiempo suficiente, la reacción alcanza la condición de equilibrio y se aprecia que las
presiones de cada reactante ya no cambian. La relación entre las presiones de productos y reactivos
permite calcular la constante de equilibrio. Por ejemplo, para la reacción: aA + bB ⇄ yY + zZ

pYy pZz
Kp 
p Aa p Bb

y esta a su vez se relaciona con el cambio de energía libre de Gibbs:

Gro   RT Ln Kp

y también: Gro  H ro  T S ro

La constante de equilibrio cambia con la temperatura, y se puede calcular a partir del H° de la
reacción y de una constante de equilibrio a una temperatura conocida:

K  H ro 1 1
Ln  2      
 K1  R  T2 T1 

Las propiedades termodinámicas de las especies reactantes a 25 °C son:

Sustancia H°f/kJ mol–1 S°/J K–1 mol–1


NH3(g) – 45.94 192.670
H2(g) 0 130.570
N2(g) 0 191.502

79
En un reactor de laboratorio de 20 L de capacidad se llevó a cabo la reacción de Haber a 25 0C
introduciendo 140 g de nitrógeno y 30 g de hidrógeno. Si la reacción se lleva a cabo y alcanza el
equilibrio, calcula:

1a) El cambio de entalpía de la reacción a 25 °C en kJ/mol.

H°r = kJ/mol

1b) El cambio de entropía de la reacción a 25 °C en J/mol K.

S°r = J/mol K

1c) El cambio de energía libre de Gibbs de la reacción a 25 °C en J/mol.

G°r = J/mol

1d) La constante de equilibrio Kp a 25 °C.

Kp25°C = .

80
1e) La constante de equilibrio a 75 °C.

Kp75°C = .

Al modificar la temperatura se encontró que al llegar al equilibrio han reaccionado 4.911 moles de
nitrógeno (de las que había originalmente), alcanzándose una presión total de equilibrio de 13.485
atmósferas. Calcula:

1f) La temperatura absoluta a la que ocurre el equilibrio.

Temperatura absoluta = .

1g) La constante de equilibrio en estas últimas condiciones.

Kp = .

81
PROBLEMA 2. DETERMINACIÓN DE HIERRO EN UN FÁRMACO.

La deficiencia de hierro en el organismo provoca la enfermedad denominada anemia, que


usualmente se combate con tratamientos de compuestos que contienen este elemento
(especialmente en forma de Fe(II) por tener mayor efecto terapéutico que el Fe(III)). Cabe
mencionar que el Fe(II) es un agente reductor que puede ser fácilmente oxidable a Fe(III).
Para analizar el contenido de Fe(II) y Fe(III) en un fármaco que contiene fumarato de hierro
(preferentemente en forma de Fe(II)), se siguió el procedimiento descrito a continuación:

 Se midieron 1.876 g de una muestra homogénea del fármaco y se disolvieron en 100.0 mL


de agua destilada.
 Se tomó una alícuota de 50 mL de la disolución anterior y se le añadió un oxidante adecuado
para transformar todo el hierro a Fe (III).

2a) En el cuadro de la izquierda marca con una X cuál(es) de los siguientes compuestos
permitiría(n) oxidar cuantitativamente el hierro hasta Fe3+.

Forma oxidada Forma reducida E, V/ENH


Fe3+ Fe2+ +0.77
H2O2 (H+) H2O +1.77
I2 I– +0.54
CO2 H2C2O4 -0.43

2b) Escribe la reacción que se produce y calcula su constante.

Valor de Ka =

2c) Calcula cuál sería el pH de una disolución que contuviera FeCl3 0.1 mol/L. Considera que el
Fe3+ es un catión ácido con una constante de disociación igual a Ka = 6.3 x 10–3.

pH=

 Después de oxidar el hierro presente en la alícuota de 50.0 mL hasta Fe(III) se precipitó éste
en forma de hidróxido férrico y se calcinó a 1000 oC. La masa obtenida de Fe2O3 fue igual a 428.9
mg.

82
2d) Calcula qué pH sería necesario para iniciar la precipitación del hidróxido férrico en la
disolución anterior (FeCl3 0.1 mol/L).
El producto de solubilidad del Fe(OH)3 es Kps = 6.3 x 10–38.

pH=

2e) ¿En qué valor de pH sería cuantitativa la precipitación del Fe(OH)3 a partir de los 100.0 mL de
FeCl3 0.1mol/L? Considera la precipitación como cuantitativa si no quedan más de 0.1 mg de
Fe3+ en la disolución.

pH=

2f) Calcula la cantidad de sustancia (en moles) de Fe(III) total precipitado de la alícuota de 50.0 mL
que dieron una masa de 428.9 mg de óxido férrico y los existentes en la muestra inicialmente
pesada.

moles en la alícuota

y ____________________________ en la muestra inicial

 El Fe(II) puede ser determinado en presencia de Fe(III) por titulación con KMnO4 en medio
ácido. Las disoluciones de KMnO4 se descomponen con el tiempo, razón por la cual la disolución
titulante fue preparada y valorada el mismo día. Esta titulación se realiza con un estándar primario
(sustancia pura de composición definida). En este caso se utilizó ácido oxálico (H2C2O4) del cual se
tomó una masa exacta que se disolvió en agua, se adicionó ácido sulfúrico 1:1 y se tituló en caliente
con la del permanganato previamente preparada (de concentración aproximada 0.02 mol/L).

2g) Escribe la reacción que ocurre entre el ácido oxálico y el permanganato (en medio ácido).

83
 Para titular una masa de 0.04960 g de ácido oxálico, disuelto en agua y con adición de ácido
sulfúrico 1:1 se gastaron 10.4 mL de la disolución de KMnO4 para observar la aparición de una
ligera coloración violeta.

2h) Calcula la concentración exacta de la disolución del KMnO4.


Cálculos:

La concentración de la disolución de KMnO4 es igual a ___________mol/L

 Para determinar el contenido de Fe(II) se tomaron alícuotas de 10.0 mL de la disolución


inicialmente preparada (1.876 g de la muestra en 100.0 mL de agua) y cuando se observó la
aparición de la ligera coloración violeta, se habían añadido 9.60 mL de la disolución de KMnO4
valorada anteriormente.

2i) Calcula la cantidad de sustancia (en moles) de Fe(II) contenidas en la alícuota de 10.00 mL y en
la masa de fármaco total.
Cálculos:

moles de Fe(II) en los 10.0 mL de disolución = ________________________

y moles en los 100 mL de disolución = ______________________________

2j) Calcula la fracción molar de Fe(II) y de Fe(III) contenida en la muestra del fármaco.

___________de Fe(II) y _________ de Fe(III)

84
2k) El porcentaje de Fe total en la muestra del fármaco es igual a:

_____________________ % de hierro en la muestra


original

PROBLEMA 3. Responde el cuestionario que se presenta a continuación, referente al siguiente


procedimiento experimental.

Se hace reaccionar mediante calentamiento una lámina de cobre que se encuentra dentro de un
erlenmeyer cerrado que contiene yodo sublimado. La lámina con la película que se le forma encima
se lava con tiosulfato de sodio para eliminar el producto formado. Se obtienen los siguientes datos:

Masa de la lámina
1.1424
original en gramos
Masa de la lámina después
1.3028
de la reacción en gramos
Masa de la lámina después
de lavarla con tiosulfato en 1.0622
gramos

3a) ¿Cuál es la masa de yodo en gramos que reaccionó?

3b) ¿Cuál es la masa de cobre en gramos que reaccionó?

3c) Determine la fórmula mínima del compuesto obtenido por la reacción entre el cobre y el yodo.

85
3d) Escribe la ecuación de la reacción que se llevó a cabo.

En vez del proceso descrito anteriormente, se obtiene el mismo producto al hacer reaccionar 10 mL de
Cu(NO3)2 0.1M con 12 mL de KI 0.2M. Contesta las siguientes preguntas:

3e) ¿Cuántos gramos del producto se obtienen si la reacción es completa?

3f) Escriba la ecuación de la reacción que se lleva a cabo.

Durante el proceso el color de la disolución cambia a café rojizo.

3g) ¿A qué especie se debe esta coloración? ¿Con qué reactivo sería posible valorar esta especie?
¿Qué indicador se utilizaría?

Especie:

Reactivo:

Indicador:

86
PROBLEMA 4.

En el siguiente esquema se muestran escondidos cuatro compuestos (A, B, C y D) que intervienen


en ocho procesos químicos:

A(s) B(aq) A(s)


(a) (c) (e) (g)

Fe(s) D(S)
(b) (f) (h)
(d)
C(s) C(aq)
B(aq)

4a) Escribe la fórmula de los compuestos A, B, C y D que identifiques y asigna las letras de la (a) a
la (h), indicadas en el esquema, a los procesos que correspondan.
A B C D
Compuesto

Letra que Proceso


corresponde
Calentar la sustancia al aire a elevadas temperaturas.

Calentar la sustancia a elevadas temperaturas en un flujo de


hidrógeno.
Calentar la sustancia a elevadas temperaturas en un flujo de cloro.

Reacción con ácido sulfúrico diluido.

Reacción con ácido clorhídrico diluido.

Reacción con una disolución acuosa de hidróxido de sodio.

Reacción con peróxido de hidrógeno en disolución ácida seguida por


una reacción con una disolución acuosa de hidróxido de sodio. (Dos
reacciones).
Reacción en disolución acuosa con magnesio en polvo.

4b) Escribe las ecuaciones balanceadas de las reacciones (a) - (h).


a

87
d

PROBLEMA 5.

¿Cuáles de las siguientes fórmulas de Fischer representa un par de enantiómeros?

CH3 CH3 C2H5 CH3

HO H H Cl Cl H HO H

H Cl HO H HO H Cl H

C2H5 C2H5 CH3 C2H5


I II III IV

a) I y II
.
b) III y IV
.
c) I y IV
.
d) II y III
.

PROBLEMA 6.

Para determinar la estructura de la lactosa, se utilizaron los siguientes datos experimentales. Utiliza
esta información para establecer su estructura, y escribe las ecuaciones de las reacciones de los
incisos (a), (c) y (d).

a) La hidrólisis de la lactosa mediante la enzima emulsina (una -glicosidasa) produce cantidades


equivalentes de D-glucosa y D-galactosa.
b) La lactosa es un azúcar reductor.
c) La metilación de la lactosa con yoduro de metilo y óxido de plata, seguida de una hidrólisis,
conduce a la 2,3,6-tri-O-metil-D-glucosa y a la 2,3,4,6-tetra-O-metil-D-galactosa.
d) La oxidación suave de la lactosa con agua de bromo, seguida de una metilación e hidrólisis,
conduce al ácido 2,3,6-tri-O-metilglucónico y a la 2,3,4,6,-tetra-O-metil-D-galactosa.

88
H O
C
H OH
HO H
ESTRUCTURA DE LA D-GALACTOSA
HO H
H OH
OH

REACCIÓN INCISO (a)

REACCIÓN INCISO (c)

REACCIÓN INCISO (d)

ESTRUCTURA DE LA LACTOSA

89
PROBLEMA 7.

Uno de los constituyentes del aceite de turpentina (aguarrás) es el -pineno, que tiene la fórmula
molecular C10H16. En el siguiente esquema se ilustran varias reacciones del -pineno. Determina
la estructura del -pineno y la de los productos de reacción A al E.

A
(C10H16Br2)

Br2, CCl4

D PhCO 3H Br2, H 2O B
 -pineno
(C10H 16O) CH 2Cl2 (C10H17BrO)

1) O 3 H2SO 4, CALOR
H 2SO 4 / H 2O (20%) 2) (CH3)2S

E C
O
(C10H18O 2) (C 10H15Br)
C
H

C O
CH3

COMPUESTO A COMPUESTO B COMPUESTO C

COMPUESTO D COMPUESTO E  -PINENO

90
PROBLEMA 8.

Escribe la estructura de los compuestos intermediarios de la siguiente secuencia sintética:


A 1) Hg(OAc)2, H2O B CrO3
C
(C6H10) (C6H12O) H2SO4 (C6H10O)
2) NaBH 4
Da (+) la prueba H2C P(C6H5)3
Br2 / CCl4
del yodoformo

G Na2Cr2O7 1) BH3, THF D


F E
(C6H10Br2) H 2O (C7H12)
(C7H 12O2) (C7H14O) 2) H2O2, NaOH
H2SO4

COMPUESTO A COMPUESTO B COMPUESTO C

COMPUESTO D COMPUESTO E COMPUESTO F

COMPUESTO G

FIN DEL EXAMEN

91
92
XVI Olimpiada Nacional de Química
Examen Experimental de Fisicoquímica Toluca 2007

Determinación de la entalpía de disolución de KNO3.

El nitrato de potasio se disuelve en agua y se disocia de acuerdo con la siguiente reacción:

KNO3 (s) + H2O (l) ⇄ K+ (ac) + NO3- (ac)

La constante de equilibrio para esta reacción está dada por la constante del producto de solubilidad
Kps y se calcula a partir de la solubilidad (mol/L).

Keq = Kps = [K+][NO3-] = (S)(S) = S2

La solubilidad “S” se puede determinar experimentalmente llevando una disolución hasta su punto
de saturación. Para muchas sustancias la solubilidad aumenta con la temperatura, y si se enfría una
disolución cuidadosamente se le puede llevar hasta la temperatura de saturación en la que se observa
la aparición de cristales. Conocida la cantidad de sal y el volumen total de la disolución se puede
calcular la solubilidad:

cantidad de sustancia del soluto (mol)


Solubilidad  S 
volumen total de la disolución (L)

Una vez conocida la solubilidad se puede calcular la constante de equilibrio Keq = Kps.

La constante de equilibrio (al igual que la solubilidad) depende de la temperatura:

H r 1
Ln Keq  -  C
R T

Una gráfica de Ln Keq (ordenadas) contra (1/T) (abscisas) resulta en una recta que muestra la
dependencia de la constante de equilibrio de la reacción con la temperatura:

Ln Keq Ln Keq
pendiente = - Hr / R
(endotérmica)

pendiente = - Hr / R
(exotérmica)

1/T 1/T

93
La pendiente se puede calcular tomando dos puntos de la recta que mejor se ajuste a los datos
experimentales:

y 2 - y1
pendiente 
x 2  x1

A partir de la pendiente se puede obtener la entalpía de reacción: Hr = - (pendiente)(R).

Materiales y reactivos

Materiales Reactivos
1 Probeta graduada de 50 mL con base de plástico KNO3 (R. A.)
1 Termómetro digital Agua destilada
1 Jeringa de plástico de 5 o 10 mL con manguera para
usar como propipeta
1 Pipeta graduada de 5 mL
1 Baño maría (vaso de precipitados de 600 mL, tela de
asbesto, tripié y mechero o resistencia eléctrica de
calentamiento)
1 Agitador de vidrio

Procedimiento experimental

Antes de iniciar el procedimiento, lee completamente todas las instrucciones, una vez que las hayas
leído podrás iniciar el experimento.

1. Transfiere el nitrato de potasio de su contenedor (20 ± 0.02 g) a la probeta graduada de 50 mL.


Registra en la hoja de respuestas la cantidad en gramos de nitrato de potasio indicada y calcula la
cantidad de sustancia (moles) correspondientes.
2. Añade al nitrato de potasio 15 mL de agua destilada con la pipeta graduada (usa la jeringa como
propipeta), y calienta la probeta a baño maría agitando suavemente, con el agitador de vidrio,
hasta que se disuelva totalmente el nitrato de potasio.
3. Retira el agitador de vidrio y anota en la tabla de datos el volumen total de la disolución de
KNO3 de la probeta.
4. Retira la probeta del baño, introduce el agitador de vidrio y el termómetro en la probeta y permite
que se enfríe la disolución agitando suavemente.
5. Anota la temperatura en que aparecen los primeros cristales.
6. Agrega 5 mL de agua destilada y calienta en baño maría hasta disolución total de KNO3. Retira
el agitador de vidrio y el termómetro. Anota en la tabla de datos el volumen total de la
disolución.
7. Repite los pasos del 4 al 6 hasta un total de seis determinaciones.
8. Completa la tabla haciendo los cálculos correspondientes.
9. Traza la gráfica, determina la pendiente y la entalpía de reacción.

94
Hoja de respuestas

Masa de KNO3 = g
Masa de KNO3 = mol

Dato Vol. de Vol. total Temp. Temp. 1/T Solubilidad Keq Ln Keq
agua de la
No. agregado disolución (0C) (K) (K-1) (mol/L)
(mL) (mL)
1 15
2 5
3 5
4 5
5 5
6 5
7 5

En la hoja que se te proporciona construye la gráfica de Ln Keq vs. 1/T y traza la recta que mejor se
aproxime a los puntos experimentales.

Indica claramente las unidades que corresponden a las ordenadas y a las abscisas.

Para determinar la pendiente elige dos puntos de la recta (márcalos con un círculo pequeño) y
escribe las coordenadas en los siguientes recuadros:

Punto Abscisas
P1 X1 = Y1 =
P2 X2 = Y2 =
Unidades

y 2 - y1
pendiente 
x 2  x1

Constante universal R = 8.314 J/mol K

H de reacción = J/mol

95
96
XVI Olimpiada Nacional de Química
Examen Experimental de Química Analítica Toluca 2007

Problema No.: ___________________

Una industria farmacéutica envió a esta Universidad varios frascos de un mismo lote del ácido
salicílico (C7H6O3) que utilizan como materia prima para preparar al ácido acetilsalicílico (C9H8O4)
que se requiere en la fabricación de las aspirinas. En su envío solicitaban que se realizara el análisis
de la pureza del reactivo (que se supone es mayor del 85%) e indicaban que, una vez realizado éste,
la Universidad podría disponer del reactivo restante.

En el laboratorio de Química Orgánica pensaron que la materia prima excedente podría ser utilizada
en el examen práctico de la Olimpiada de Química y que los participantes también podían sintetizar
el ácido acetilsalicílico como parte del examen experimental del concurso. Las pruebas previas
realizadas fueron un éxito, el ácido acetilsalicílico se obtuvo con buenos resultados de pureza y
rendimiento (mayores del 85%) pero –desafortunadamente- se confundieron las etiquetas de los
frascos que contenían el reactivo del ácido salicílico y el ácido acetilsalicílico obtenido.

La industria farmacéutica requiere con urgencia los resultados del análisis y tu labor esta mañana
será ayudarnos a identificar el ácido contenido en cada frasco y cuantificar su pureza (ya sea de la
materia prima o del producto obtenido).

NIVEL B: Para lograrlo cuentas con aproximadamente 40 mL de una disolución de uno de los dos
ácidos contenido en uno de los dos frascos, con disoluciones de HCl valoradas (cuya concentración
exacta se indica en la etiqueta del frasco correspondiente) y con una disolución de NaOH que
deberás valorar para conocer su concentración (utilizando fenolftaleína como indicador).

NIVEL A: Para lograrlo cuentas con aproximadamente 40 mL de una disolución de uno de los dos
ácidos contenido en uno de los dos frascos, con una disolución de NaOH valorada (cuya
concentración exacta se indica en la etiqueta del frasco correspondiente) y con dos indicadores de
los que deberás elegir el más adecuado para realizar la titulación.

Además de estos reactivos tienes el material volumétrico y el de vidrio necesarios para que realices
la valoración por triplicado e indiques los resultados del análisis.

En la etiqueta de la disolución problema se indica la masa de ácido que fue medida para preparar un
litro de disolución. Considera que los valores de pKa de los ácidos salicílico y acetilsalicílico son
3.0 y 3.5, respectivamente.

97
NIVEL B. HOJA DE RESPUESTAS.

Problema No.: ___________________

La concentración (en g/L) del ácido de tu muestra es: _______________

Valoración del NaOH.


No. de titulación Volumen de NaOH Volumen de HCl

1ª.

2ª.

3ª.

Volumen utilizado en tus cálculos: ________________________

La concentración de NaOH es: ___________________________

Valoración del ácido del problema.


No. de titulación Volumen de muestra Volumen de NaOH en el punto
final
1ª.

2ª.

3ª.

Volumen utilizado en tus cálculos: ________________________

Cálculos efectuados:

El ácido que contiene tu muestra es: ____________________________

El contenido (% de pureza) del ácido de tu muestra problema es: __________________

98
NIVEL A. HOJA DE RESPUESTAS.

Problema No.: ___________________

Indicador utilizado: _________________________________

La concentración (en g/L) del ácido de tu muestra es: _______________

Valoración del problema.


No. de titulación Volumen de muestra Volumen de NaOH en el punto
final

1ª.

2ª.

3ª.

Volumen de NaOH utilizado en tus cálculos: ________________________

Cálculos efectuados:

El ácido que contiene tu muestra es: ____________________________

El contenido (% de pureza) del ácido de tu muestra problema es: __________________

99
100
XVI Olimpiada Nacional de Química
Examen Experimental de Química Orgánica Toluca 2007

Número de código: __________________________________ No. de muestra: ___________

INTRODUCCIÓN.
El Ácido Acetilsalicílico es una droga maravillosa por excelencia. Se utiliza ampliamente como
analgésico (para disminuir el dolor) y como antipirético (para bajar la fiebre). También reduce la
inflamación y aún es capaz de prevenir ataques cardiacos. Debido a que es fácil de preparar, la
aspirina es uno de los fármacos disponibles menos costosos. Es producida en grandes cantidades, de
hecho la industria farmoquímica de los Estados Unidos de Norteamérica, produce cerca de 200
toneladas de este farmoquímico cada año.
El nombre comercial de Aspirina deriva del vocablo Spirea, que en botánica designa una familia de
plantas y de ahí la sílaba "spir". La letra "A" indica el proceso de acetilación al que se somete el
ácido salicílico para convertirse en ácido acetilsalicílico. La sílaba final "in" era una terminación
empleada con frecuencia para los medicamentos, en aquella época.

OBJETIVOS.
Que el alumno:
a) Efectúe la síntesis de un derivado de un ácido carboxílico como lo es un éster, y en particular
el ácido acetilsalicílico, el cual es muy importante en nuestra vida diaria.
b) Sintetice el ácido acetilsalicílico a nivel microescala.
c) Conozca la filosofía de la química verde y maneje las técnicas de estos procesos.

PROCESO POR QUÍMICA VERDE (CATÁLISIS BÁSICA).


El objetivo de la química verde es desarrollar tecnologías químicas benignas al medio ambiente,
utilizando en forma eficiente las materias primas (de preferencia renovables), eliminando la
generación de desechos y evitando el uso de reactivos y disolventes tóxicos y/o peligrosos en la
manufactura y aplicación de productos químicos.
Para determinar que tan benéfica al medio ambiente es una reacción o proceso, se pueden calcular
varios parámetros, dentro de los cuales está el porcentaje de economía atómica, que se define de la
siguiente manera:

 Masa Molar Producto 


Porcentaje de economía atómica    x 100
 Masa Molar Reactivos 
101
En esta práctica se va a llevar a cabo una reacción en la que se ilustra un proceso de química verde, a
través del cual ocurre una reacción de acilación de un fenol, utilizando como sustrato el ácido
salicílico para obtener el éster correspondiente, el ácido acetilsalicílico, empleando anhídrido acético
como agente acilante y en presencia de bases de metales alcalinos (e. g. NaOH, KOH o K2CO3). La
reacción se lleva a cabo a temperatura ambiente.

REACCIÓN.

O
OH O
O O 1) NaOH O O
O
OH + 0
25 C
O +
OH
2) HCl OH

Ácido Salicílico Anhídrido Acético


C7H6O3 Ácido Acético
C4H6O3 Ácido Acetilsalicílico
MM = 138.12 g/mol MM = 102.08 g/mol C9H8O4
d = 1.08 g/mL MM = 180.16 g/mol

PROCEDIMIENTO.
En el vaso de precipitados de 100 mL ya se encuentran pesados aproximadamente 0.5 g (3.6
mmol) del ácido salicílico (la cantidad exacta está indicada en la etiqueta del vaso y la debes de
anotar en el espacio indicado para tal fin más adelante, dentro de las preguntas de este examen).
Adiciona 1.2 mL de anhídrido acético (1.296 g, 12.696 mmol) (líquido que se encuentra en la
cantidad exacta dentro del FRASCO GOTERO A). Con una varilla de vidrio mezcla bien los dos
reactivos. Una vez que se obtenga una mezcla homogénea, con la ayuda de la espátula de acero
inoxidable debes de adicionar, en porciones y lentamente, 0.5 g de hidróxido de sodio (NaOH) (los
cuales se encuentran dentro del FRASCO B) y en cada una de las adiciones debes de tener el
cuidado de agitar nuevamente la mezcla de reacción con ayuda de la varilla de vidrio. Una vez que
termines de adicionar toda la base, continúa la agitación por 5 minutos más. Agrega lentamente 1.5
mL de agua destilada y posteriormente adiciona lentamente, una solución de ácido clorhídrico al 50
% hasta que el pH de la solución sea de 3 (dispones de unas tiras de papel indicador universal dentro
de la bolsa de plástico). La mezcla se deja enfriar en un baño de hielo (aproximadamente unos 10
minutos hasta que precipite el producto crudo). El producto crudo lo debes de aislar por medio de
una filtración al vacío, utilizando un matraz Kitazato y un embudo Büchner (el papel filtro se
encuentra en la bolsa de plástico), para ello baja los cristales del ácido acetilsalicílico con agua fría
(es importante que el agua esté bien fría, ya que el ácido acetilsalicílico se puede redisolver en
agua tibia). Coloca la fase acuosa en los FRASCOS DE DESECHOS ÁCIDOS, que se encuentran
en las campanas del laboratorio.

AISLAMIENTO Y PURIFICACIÓN.
Purifica el producto crudo por medio de una recristalización utilizando 10 mL de acetato de etilo (ya
se encuentra la cantidad exacta dentro del FRASCO C) y 10 mL de hexano (ya se encuentra la
cantidad exacta dentro del FRASCO D), adiciona unas piedritas de carburo de silicio (hay un frasco
en cada mesa conteniendo estas piedritas) para regular la ebullición. Una vez que se disuelvan los
cristales del producto crudo, elimina el disolvente por evaporación, concentrando el volumen del
disolvente aproximadamente a la mitad. En este momento retira el vaso de precipitados de la parrilla
y coloca éste dentro de un baño de hielo. Los cristales puros del ácido acetilsalicílico los puedes
aislar por medio de una filtración al vacío, utilizando un matraz Kitazato y un embudo Büchner

102
(coloca el filtrado en los FRASCOS de DISOLVENTES ORGÁNICOS, que se encuentran en las
campanas del laboratorio).
Tu producto lo debes de colocar dentro de la bolsa de plástico. No olvides anotar tu clave. Entrega
el sólido y tu examen escrito al profesor encargado en el laboratorio.

Preguntas.
Si el porcentaje de economía atómica se define de la siguiente manera:

 Masa Molar Producto 


Porcentaje de economía atómica    x 100
 Masa Molar Reactivos 

1. ¿Cuál es el porcentaje de economía atómica para la reacción que realizaste en esta práctica?

2. ¿Por qué consideras que el porcentaje de economía atómica sea moderado?


a) La reacción es poco eficiente, ya que el anhídrido acético es poco reactivo.
b) La reacción es poco eficiente, ya que el ácido salicílico es poco reactivo.
c) Se pierde masa en el subproducto de la reacción.

3. La manera como se obtiene este importante compuesto en la industria y en las prácticas descritas
en la literatura para prepararlo en el laboratorio, es por medio de la siguiente reacción:
O
OH O
O O
O O
O
OH + H3PO4
O +
OH
70 a 85 0C OH

Ácido Salicílico Anhídrido Acético


Ácido Acético
Ácido Acetilsalicílico

El proceso de química verde es el siguiente:


O
OH O
O O 1) NaOH O O
O
OH + 0
25 C
O +
OH
2) HCl OH

Ácido Salicílico Anhídrido Acético


Ácido Acético
Ácido Acetilsalicílico

103
3.1. Con respecto al uso del disolvente:
a) De acuerdo a la cinética, la reacción es rápida, ya que al no utilizar disolvente, no se propicia una
mayor superficie de contacto entre los reactivos.
b) De acuerdo a la cinética, la reacción es lenta, ya que al no utilizar disolvente, no se propicia una
mayor superficie de contacto entre los reactivos.
c) De acuerdo a la cinética, la reacción es muy rápida, ya que se están utilizando los dos reactivos a
su concentración más alta.

3.2. Con respecto a la temperatura a la cual se lleva a cabo la reacción:


a) La reacción procede a temperatura ambiente, ya que se está utilizando un catalizador básico para
formar un fenóxido, el cual es un excelente nucleófilo.
b) La reacción procede a temperatura ambiente, ya que se está utilizando un catalizador básico para
formar un carboxilato de sodio, el cual es un excelente nucleófilo.
c) La reacción procede a temperatura ambiente, ya que se está utilizando un catalizador básico para
incrementar la reactividad de uno de los grupos carbonilo del anhídrido acético.

3.3. ¿Por qué es importante la catálisis ácida en la reacción que se lleva a cabo en la industria?
a) Para incrementar la reactividad del fenol como nucleófilo y de esta manera se incrementa la
rapidez de la reacción.
b) Para incrementar la reactividad del carbonilo del anhídrido acético como electrófilo y de esta
manera se hace más rápida la reacción.
c) Como disolvente en la reacción para favorecer la superficie de contacto entre los dos reactivos y
de esta manera se incrementa la rapidez de la reacción.

3.4. ¿Por qué es importante la catálisis básica en la reacción que llevaste a cabo en este experimento
(proceso de química verde)?
a) Para incrementar la reactividad del fenol como nucleófilo e incrementar la rapidez de la reacción.
b) Para incrementar la reactividad del carbonilo del anhídrido acético como electrófilo y hacer más
rápida la reacción.
c) Como una base capaz de favorecer la superficie de contacto entre los dos reactivos.

4. ¿Cómo calcularías el rendimiento de la reacción con el que obtuviste el ácido acetilsalicílico?

NOTA: Tú no vas a pesar el producto, lo van a pesar los maestros responsables del
experimento de química orgánica. Tú sólo nos vas a indicar cómo se hacen los cálculos para
hacer esta determinación.

La cantidad que utilicé de ácido salicílico fue de: ________________________


(Cantidad que estaba escrita en el vaso de precipitados).

Cálculos:

104
XVII Olimpiada Nacional de Química
1er Examen Nivel A y B.
Total: 33 preguntas. Tiempo asignado: 75 minutos. Oaxaca 2008

En el siglo XXI es común encontrar problemas que involucran a varias ciencias y por esta razón en
esta primera parte del examen, encontrarás preguntas que relacionan a la química con otras ciencias
como las matemáticas, la física y la biología. Además, deberás leer con todo cuidado cada pregunta.
Debes anotar en el recuadro correspondiente la letra del inciso que contesta correctamente cada
pregunta.
No olvides que si consideras que ningún inciso es el correcto deberás anotar una letra X.
NOTA ACLARATORIA: Al hacer tus cálculos puedes encontrar una pequeña diferencia, en
general no mayor de unas décimas, entre tu resultado y alguno de los propuestos. Esto es
normal y por eso sólo deberás usar la “X” cuando ninguna respuesta sea muy cercana a la que
tú obtengas.

1) La densidad del benceno (C6H6) es 873.81 kg / m3. ¿Cuál es la masa de un litro de


esta sustancia?

A 8.7381 kg B 0.87381 kg
C 87.381 kg D 0.087381 kg

2) En las jeringas normalmente se indica la medida en mililitros (1 mL = 1 cm3).


¿Cuántos moles de benceno hay en 1000 cm3 de esta sustancia?

A 1.12 x 10-2 B 1.12 x 10-3


C 1.12 x 10 D 1.12 x 102

3) La densidad del agua es 1000 kg/ m3. Si un recipiente que, totalmente lleno, puede
contener 100 moles de agua se vacía y se llena totalmente con benceno, ¿cuántos
moles de benceno puede contener?
A 2.016 B 0.02016
C 100 D 20.16

4) La constante de Avogadro es 6.023 x 1023. ¿Cuántos átomos de helio hay en 2 ng de


este gas? (1 ng = 1 x 10-9 g)
A 6.023 x 1023 B 3.0115 x 1014
C 6.023 x 1014 D 3.0115 x 1023

5) Las ppm (partes por millón) son unidades de concentración, muy utilizadas en
estudios de contaminación de aguas o concentración de elementos en fluidos de
seres vivos. En gramos, una ppm equivale a un gramo en un millón de gramos. Si
se lograra disolver un mol de benceno en 100 m3 de agua, la concentración de
benceno en esta mezcla sería en ppm:
A 0.078 B 0.78
C 7.80 D 78.0

105
6) En un litro de plasma sanguíneo hay 3.25 g de iones Na+. ¿Cuál es la concentración
aproximada de iones Na+, en ppm, en el plasma sanguíneo, cuya densidad es
prácticamente la misma que la del agua (1 g / cm3)?
A 3250 B 32500
C 325 D 32.5

7) La configuración electrónica de un ion Na+ es:

A 1s2 2s2 2p6 3s1 B 1s2 2s2 2p6 3s2


C 1s2 2s2 2p5 3s1 D 1s2 2s2 2p6

8) Para producir ácidos carboxílicos en el laboratorio en ocasiones se utiliza una


disolución acuosa de permanganato de potasio (KMnO4). El estado de oxidación
del manganeso en este compuesto es:
A +7 B +5
C +3 D +1

9) Si se hace reaccionar el KMnO4 con ácido sulfúrico concentrado se obtiene un


compuesto que puede explotar al contacto, el Mn2O7. En la reacción propuesta se
obtiene además agua y KHSO4. Al balancear esta reacción se encuentra que por
cada mol de permanganato, reaccionan de H2SO4:
A 0.5 moles B 1.0 mol
C 1.5 moles D 2.0 moles

10) Si la reacción anterior se lleva a cabo en disolución acuosa, se propone la siguiente


ecuación:
KMnO4(aq) + H2SO4(aq) → MnSO4(aq) + K2SO4(aq) + H2O(l) + O3(g)
Por cada mol de permanganato que reacciona se obtienen de ozono:
A Menos de un mol B Un mol
C Dos moles D Más de dos moles

11) ¿Cuál de estas sustancias tiene una densidad menor en estado sólido que en estado
líquido?
A Agua B Benceno
C Fenol D Ácido benzoico

12) Si en la molécula de benceno se sustituye un hidrógeno por un grupo OH se


produce fenol, una sustancia cuya estructura es:

OH

Se pueden disolver hasta 83 g de fenol en un litro de agua. ¿Cuántos litros de agua


se necesitarían para disolver un mol de fenol?
A 11.33 B 8.3
C 0.1133 D 0.882

106
13) Para quemar un mol de fenol y producir CO2 y H2O, ¿cuántos moles de oxígeno
gaseoso se requieren?

A 15 B 14
C 7.5 D 7

14) La combustión de compuestos aromáticos suele ser incompleta y se produce


también CO además de CO2 y H2O. Si al quemarse un mol de fenol se producen la
misma cantidad de moles de CO y CO2, ¿cuántos moles de oxígeno gaseoso se
consumen?
A Más de 15 B Entre 10 y 15
C Entre 6 y 9.5 D Menos de 6

15) El ácido benzoico también es un derivado del benceno y se obtiene al sustituir un


hidrógeno por un grupo –COOH. El % en masa de oxígeno en la molécula del
ácido benzoico es:
A Menos de 20% B Entre 20% y 25%
C Entre 25.1% y 30% D Más de 30%

16) El ácido benzoico y sus sales se utilizan como conservadores en alimentos pero su
eficiencia depende del pH de los alimentos. El pH del jugo gástrico es:

A Menor a 3 B Entre 5 y 6
C Cercano a 7 D Mucho mayor a 7

17) Un estudiante toma 200.0 mL de una disolución acuosa de HCl 2.0 M y le agrega
agua destilada hasta tener 500.0 mL de disolución. El pH de la disolución
preparada es:
A – 0.097 B 0.397
C 0.097 D 3.97

18) Cuando un gramo de agua es adicionado a una mezcla de ácido y agua, la nueva
mezcla es 25% en masa de ácido. Cuando 2 gramos de ácido son adicionados a la
nueva mezcla, la mezcla ahora contiene 50% en masa de ácido. El % en masa de
ácido en la mezcla original era:
A Menos de 25% B Entre 25% y 30%
C Entre 31% y 36% D Más de 36%

19) La ley de los gases ideales es PV = nRT, donde:


(R = 0.082 L atm / mol K) (R = 8.314 J / K mol) (1 J = kg m2 s-2)
(1 atm = 101.325 kPa) (1 Pa = 1 N m-2) (1 N = 1 kg m s-2)
A 25 oC y 100.0 Pa, un mol de un gas ideal ocupa un volumen de:
A Menos de 30 L B Entre 30 L y 2000 L
C 2.0 x 104 L D Más de 2.0 x 104 L

107
20) En una fábrica se produce cobre metálico a partir de una disolución que contiene
iones Cu2+, utilizando una reacción electroquímica. El enunciado correcto es:

A El Cu metálico se obtiene en el ánodo por reducción de los iones Cu2+


B El Cu metálico se obtiene en el ánodo por oxidación de los iones Cu2+
C El Cu metálico se obtiene en el cátodo por reducción de los iones Cu2+
D El Cu se obtiene en el cátodo por oxidación de los iones Cu2+

21) En un recipiente de 2.0 L de capacidad se coloca la misma cantidad en gramos de


nitrógeno, oxígeno y CO gaseosos. La fracción mol de nitrógeno en esta mezcla es:

A Menor a 0.3 B Entre 0.3 y 0.4


C Entre 0.4 y 0.5 D Mayor a 0.5

22) Indica cuál es el enunciado verdadero con relación a la reacción:


2 K2CrO4 + H2SO4  K2Cr2O7 + H2O + K2SO4
A Es una reacción redox y los iones B Es una reacción redox y los iones
cromo se reducen azufre se oxidan
C Es una reacción redox y los iones D No es una reacción redox
cromo se oxidan

23) En la reacción: 3   + 2, las letras griegas “”, “”, “” representan
elementos químicos. El compuesto 2 es uno de los productos principales en la
combustión de muchas sustancias orgánicas. La masa molecular del compuesto 
es 40.3 g/mol. El elemento  es:
A Li B Mg
C H D Ca

24) La corriente que circula en una celda electroquímica se puede medir en amperes
(A). Un ampere equivale a una carga de un coulomb (C) que pasa cada segundo por
un punto del circuito. La carga de un electrón es 1.6 x 10-19 C. Si en una celda
electroquímica circula una corriente de 5 mA, significa que el número de electrones
que pasan por un punto del circuito cada segundo son:
A 3.125 x 10-16 B 3.2 x107
C 3.125 x 1016 D 3.2 x 10-17

25) La constante de Faraday es la carga contenida en un mol de electrones, por lo tanto


su valor es:

A Menos de 50 mil coulombs B Entre 50 mil y cien mil coulombs


C Entre 100 mil y 200 mil coulombs D Más de 200 mil coulombs

26) En una celda electroquímica, para reducir un mol de iones plata Ag+ y convertirlos
en átomos de plata metálicos, se requiere que circule por la celda una carga total de:
A 96368 C B 289104 C
C 192736 C D 48184 C

108
27) Masas; electrón: 9.1095 x 10-28 g, neutrón: 1.67495 x 10-24 g, protón:
1.67252 x 10-24 g. La masa de un átomo de helio es:
A 3.346 x 10-24 g B 5.0218 x 10-24 g
C 5.0242 x 10-24 g D 6.696 x 10-24 g

28) De acuerdo a la expresión “X”  20682 Pb +  podemos concluir que el átomo


radiactivo “X” es: (Las partículas alfa son núcleos de helio)
A 21083 Bi B 210
81 Tl
210 210
C 85 At D 84 Po

29) La densidad del oro es 19.30 g/cm3. La densidad del plomo es 11.30 g/cm3.
Área de la esfera: 4πr2. Volumen de la esfera: 4/3 (πr3).
Si tenemos dos esferas de idéntico tamaño, una de oro y otra de plomo, y el área de
la esfera de oro es de 1.00 cm2, la masa de la esfera de plomo es:
A Menos de 0.8 g B Entre 0.8 y 1.2 g
C Entre 1.2 y 2.0 g D Más de 2.0 g

30) Para obtener un litro de HCl 6 N a partir de una disolución “1” de HCl 12 N y una
disolución “2” de HCl 4 N, se deben mezclar:
A 1/3 L de 1 + 2/3 L de 2 B 2/3 L de 1 + 1/3 L de 2
C 3/4 L de 1 + 1/4 L de 2 D 1/2 L de 1 + 1/2 L de 2

31) Una disolución saturada a 25 oC con LiCl tiene una masa de 100 g y contiene
35.5 g de LiCl disuelto. La densidad de la dilución es de 1.52 g/cm3. Con respecto a
esta disolución la única opción falsa es que:
A contiene 550.4 g de LiCl en 1 kg de B contiene 1520 g de LiCl en 1 L de
H2O disolución
C contiene 539.6 g de LiCl en 1 L de D la fracción molar del LiCl en la
disolución disolución saturada es igual a 0.19

32) ¿Cuál es la solubilidad del hidróxido de cobre (expresada en mol L–1) del Cu(OH)2
y el pH de la disolución saturada de Cu(OH)2 en agua pura?
Datos: Ks del Cu(OH)2 = 2.210–20.
A 1.810–7 mol L–1 y B 2.8 10–7 mol L–1 y
pH = 7.6 pH = 7.3
–7 –1
C 2.810 mol L y D 1.810–7 mol L–1 y
pH = 7.6 pH =7.3

33) La presencia del ion Fe3+ en una disolución acuosa se detecta por la aparición de
una coloración rojo “sangre” cuando se añaden unas gotas de una disolución de
algún tiocianato alcalino. Esta identificación se debe a:
A La reacción de reducción del Fe3+ B La reacción de complejación del
por el SCN- Fe3+ con el SCN-
C Un cambio de pH al añadir el D La precipitación del Fe(SCN)3
tiocianato

FIN DEL EXAMEN

109
110
XVII Olimpiada Nacional de Química. 2do Examen Nivel B.
Tiempo asignado: 90 minutos. Oaxaca 2008

Fisicoquímica:

1. Una barra de cobre de 500 g inicialmente a 80 ºC se pone en contacto con una barra de grafito de
100 g a una temperatura de 20 ºC. Para el cobre el Cp = 5.85 cal/mol K y para el grafito el
Cp = 2.07 cal/mol K. La temperatura de equilibrio es:

A Entre B Entre C Entre D Entre


57 y 58 ºC 63 y 64 ºC 69 y 71 ºC 75 y 77 ºC

2. Un recipiente de 35 litros contiene una mezcla de oxígeno y argón a una presión total de 4.15 bar.
Si la mezcla contiene 40 % en masa de oxígeno, la presión parcial del argón es:

A Entre B Entre C Entre D Mayor de


1.5 y 1.6 bar 2.2 y 2.3 bar 2.4 y 2.5 2.5 bar
bar

3. La reacción química CH3OH(g) ⇄ CO(g) + 2 H2(g) alcanza el equilibrio cuando el CH3OH se ha


disociado 75 %. Si la presión inicial del CH3OH(g) es de un bar, la constante de equilibrio es:

A Entre B Entre C Entre D Entre


0.08 y 0.09 0.16 y 0.17 4.4 y 4.6 6.7 y 6.8

4. La constante de equilibrio se relaciona con la temperatura absoluta y la entalpía de reacción a


través de la siguiente fórmula:
K  H ro 1 1
ln  2      
 K1  R  T2 T1 
Donde: K = constante de equilibrio; H ro  entalpía de reacción ;
T = temperatura; R = 8.314 J mol– 1 K– 1

La constante de disociación del óxido de mercurio es 2.035x10–2 a 420 ºC y 1.866x10– 1 a 450 ºC.
La entalpía de disociación del óxido de mercurio es:

A Menor de B Entre 116 y C Entre 307 y D Mayor de


116 kJ/mol 117 kJ/mol 308 kJ/mol 308 kJ/mol

111
Las preguntas 5, 6 y 7 sólo son para los de Nivel B

5. Si se mezclan 20 moles de N2 con 2 moles de H2, en un recipiente de 100 litros y se determina


que la presión interna es de 101.325 kPa (1 atm, 1.01325 bar), la temperatura en el interior del
recipiente es: (PV = nRT), (R = 0.083 L bar/mol K).

A Menor a B Entre 50 K C Entre 100 K D Mayor a


50 K y 100 K y 500 K 500 K

6. A una temperatura de 500 K, se hacen reaccionar 20 moles de N2 y 2 moles de H2 para formar


amoniaco (NH3). La reacción alcanza el equilibrio cuando ha reaccionado el 80% del H2
presente originalmente. Los moles de amoniaco formado son:

A Menos de B Entre 0.5 y C Entre 0.9 y D Mayor a 1.2


0.5 0.9 1.2

7. En la reacción del inciso anterior, la cantidad de H2 a un tiempo “t”, [H2]t, se relaciona con la
cantidad de H2 inicial [H2]0 por la ecuación:

Ln [H2]0 - Ln [H2]t = K t K = 0.2 s-1

A los 4 segundos de iniciada la reacción, la cantidad de moles de H2 que ha reaccionado es:

A Menos de B Entre 0.2 y C Entre 0.5 y D Más de 1.0


0.2 0.5 1.0

********************************************************************************

Química Analítica:

8. Completa el crucigrama (y ESCRIBE también la palabra en la línea correspondiente).

Horizontales

1. Papel que juega el agua con muchos solutos _____________________________

3. Sustancia que en agua tiene propiedades ácidas y básicas ___________________

7. Agua que contiene sales de calcio y magnesio ____________________________

9. Desinfectante utilizado para potabilizar el agua ___________________________

10. Descubridor de las leyes de la electrólisis _______________________________

Verticales

2. Método que permite, a partir del agua, obtener hidrógeno en el cátodo ___________________

112
4. Añadir una base a un ácido _________________________________________

5. Papel que juega el agua en el equilibrio: F- + H2O ⇄ HF + OH- _______________________

6. Agua cuya fórmula es H2O2 _________________________________________

8. Papel que juega el agua al reaccionar con Na0 ___________________________

1 2

3 4

5
6
7
8
9

10
M i c h a e l

9. Los estimuladores cardiacos (marcapasos) son pilas de litio cuyas características se mencionan a
continuación:

Los sistemas redox que intervienen son:


Li+/Li Eo = - 3.02 V/ENH

I2/I - Eo = 0.53 V/ENH

Indica cuál es la reacción que ocurre en cada electrodo cuando la pila funciona y si se trata de
una reacción de oxidación o de reducción.

Reacción Tipo de reacción


ánodo Li –e- Li +

cátodo I2 +2e-- 2 I-

113
10. Indica por qué el electrolito que se utiliza debe estar completamente anhidro (libre de agua).

_____________________________________________________________________________

********************************************************************************

Química Inorgánica:

11. En este problema tendrás que acomodar nueve de los primeros 18 elementos de la Tabla
Periódica en el diagrama que se muestra a continuación. Toma en cuenta los datos que se te
proporcionan y escribe en cada recuadro el símbolo del elemento que le corresponde:

1 2 3

4 5 6

7 8 9

Los primeros 18 elementos son:

H He Li Be B C N O F Ne Na Mg Al Si

P S Cl Ar

Cuando termines de acomodar los elementos, notarás que el número atómico siempre va en
aumento desde la casilla 1 hasta la 9.

Datos:
1.- En la columna de en medio están el único halógeno considerado en este problema y el gas
que se considera más importante para la vida.
2.- En la fila inferior está un elemento que se presenta en varias formas alotrópicas, como
rómbico y monoclínico.
3.- En la columna derecha está el único metal alcalino considerado en este problema y el
elemento que es la base de la química orgánica.
4.- Seis de los elementos son gases a temperatura ambiente y presión de una atmósfera; ninguno
de los dos gases nobles incluidos en este problema está en la columna izquierda.
5.- El elemento más abundante en el aire que respiramos está en una casilla con número par.

114
12. Los habitantes del planeta X en el que existen menos elementos que en la Tierra, los han
agrupado en una tabla periódica, muy similar a la nuestra pero más condensada, que se muestra
a continuación:

De acuerdo a las características de cada uno, coloca en la tabla los símbolos de los elementos del
planeta X.

1.- El elemento B tiene un electrón 1s1.


2.- Los elementos E, Ba y Ra forman iones con carga -1. El elemento E es más electronegativo
que Ra pero menos electronegativo que Ba.
3.- Los elementos B, Y y S están dispuestos en forma creciente de sus radios atómicos y forman
iones con carga +1.
4.- Los elementos O, Te y Pe forman, con el elemento Ba, compuestos cuyas fórmulas son OBa2,
TeBa2 y PeBa2, respectivamente.
5.- El número atómico de Te es mayor que el de O pero menor que el de Pe.
6.- El elemento M es el único de su grupo que no es metal, aunque una de sus características es
que ya tiene propiedades conductoras.

115
Química Orgánica (Sólo Nivel A):

13. El Compuesto A (C7H14) es ópticamente activo y tiene la configuración absoluta S. La


reacción con ozono y después con agua oxigenada del compuesto A produce ácido acético y un
ácido carboxílico B (C5H10O2), el cual es ópticamente activo. Escriba las estructuras de A y B,
con la estereoquímica apropiada.

RESPUESTAS:

Estructura de A Estructura de B

14. ¿Cuál es el producto principal de la siguiente reacción?


-+
CH 3O: Na
CH3 CH CH2 CH 3
Br CH 3OH
CALOR

CH 3 CH3 CH3 H
a) CH3 CH CH2 CH 3 b) H2C CH CH2 CH 3 c) C C d) C C
O H H H CH 3
CH3

RESPUESTA: ________

15. El aspartame es uno de los edulcorantes que más se utiliza en los productos comerciales
NutraSweet y Equal:

116
El aspartame es 160 veces más dulce que la sacarosa.
¿Qué productos se obtendrían si el aspartame se hace reaccionar con una disolución de HCl 8 M
y calentamiento a reflujo?

O
:O O
N CH 3
O NH 3 H O

ASPARTAME

RESPUESTAS:

PRODUCTO A PRODUCTO C
PRODUCTO B

********************************************************************************

FIN DEL EXAMEN

117
118
XVII OLIMPIADA NACIONAL DE QUÍMICA
TERCER EXAMEN NIVEL A Tiempo asignado: 180 minutos Oaxaca 2008

1. La Colina es un constituyente de los fosfolípidos (los cuáles son ésteres fosfóricos similares a las
grasas, de gran importancia fisiológica) tiene la fórmula C5H15 NO2. Se disuelve con facilidad en
agua para dar una disolución con pH básico. La Colina se puede preparar por la reacción entre el
óxido de etileno y la trimetilamina, en presencia de agua.

a) ¿Cuál es la estructura de la Colina?


b) ¿Cuál es la estructura de su derivado acetilado? La Acetilcolina, C7H17NO3, es importante por su
acción como neurotransmisor del sistema nervioso central y del sistema nervioso periférico.

RESPUESTAS:

a) Estructura de la Colina b) Estructura de la Acetilcolina

119
2. Complete la siguiente secuencia sintética:
O

H C
NH2
N CH3

a H2SO4
B
SO3

O S O
HNO3
OH
H2SO4 H2SO4
E D exceso
H2O (20%)
(C6H5N3O4)
calor

RESPUESTAS:

Condiciones a Compuesto B

Compuesto D Compuesto E

120
3. Un compuesto desconocido A (C11H14) se trató con hidrógeno gas en presencia del catalizador de
Adams (Pt) para formar el compuesto B (C11H22), el cual se identificó como cis-4-n-propil-1-
etilciclohexano. El compuesto A no se vio afectado con una disolución de plata amoniacal
[Ag(NH3)2OH] y tampoco reaccionó con hidrógeno gas en presencia del catalizador de Ni/B.
Cuando se trató el compuesto A, primero con ozono y después con sulfuro de dimetilo, se
obtuvieron los siguientes compuestos:

H O

O H O
O
+ + 2 C C C
C C
H H H3C H O H H

a) ¿Cuáles son las estructuras de los compuestos A y B?

RESPUESTAS:

Estructura del compuesto A Estructura del compuesto B

121
4. Los haluros de alquilo se han usado ampliamente como insecticidas desde el descubrimiento del
DicloroDifenilTricloroetano (DDT) en 1939. El DDT fue el primer compuesto al que se le
descubrió una alta toxicidad para los insectos y una toxicidad relativamente baja para los
mamíferos. En 1972 se prohibió el uso del DDT en los Estados Unidos ya que es un compuesto
muy estable, duradero, y su amplio uso estaba causando la acumulación del mismo en
concentraciones apreciables en la vida silvestre. El Clordano es un haluro de alquilo que se usa
como insecticida para proteger las construcciones de madera contra las termitas. El Clordano se
puede sintetizar a partir de dos reactivos en una reacción de un solo paso.

a) ¿Qué tipo de reacción se lleva a cabo?


b) Uno de los reactivos es el hexaclorociclopentadieno. ¿Cuál es el otro reactivo?

Cl

Cl Cl Cl
Cl

Cl
Cl
Cl
CLORDANO

RESPUESTAS:

a) Tipo de reacción que se lleva a cabo b) Estructura del segundo reactivo

122
5. ¿Cuál es el valor de pH de la disolución que resulta al disolver 10.0 litros de cloruro de hidrógeno
gaseoso (que se encuentra en condiciones normales de temperatura y presión) en 750 mL de
agua?
(R = 0.082 L atm K–1 mol–1 = 8.31 J K–1 mol–1).

pH =

6. Para obtener 15.54 g de un metal a partir de la reducción de una disolución acuosa de sus iones
divalentes se requiere una carga total de electricidad de 14475 C. ¿Cuál es el metal?
Datos: Faraday = 96485 C mol-1.

Metal =

7. Para determinar el contenido de nitrato de potasio en una mezcla que contiene también cloruro de
potasio se disuelven 4.00 g de la mezcla en 50.0 mL de agua destilada; a continuación se añaden
100 mL de una disolución acuosa de AgNO3 1.00 mol/L y se obtienen 2.08 g de un precipitado
blanco. ¿Cuál es el porcentaje en masa de cada uno de los componentes de la mezcla?

% de KCl =

% de KNO3 =

8. En un laboratorio de la Universidad Autónoma B. J. de Oaxaca se requirió preparar 6.00 litros de


una disolución acuosa que tenga una concentración 1.92 mol/L de HCl y 8.33 mol/L de NaCl. Para
prepararlas se cuenta con dos disoluciones: una de HCl al 37.5 % m/m (d = 1.13 g/mL) y otra de
NaCl 41.2 % m/m (d = 1.42 g/mL). ¿Qué volúmenes de cada una de estas disoluciones se requiere
mezclar? Suponer que los volúmenes son aditivos.

VHCl = litros VNaCl = litros

123
9. ¿Qué masa de H2C2O4·2H2O se necesita para preparar 1.0 L de una disolución de concentración
tal que 1.00 mL precipite en forma de oxalato de cerio (III) con 1 mL de una disolución
0.100 mol/L de CeCl3?

10. Estudia la siguiente lista de compuestos y busca el resultado que le corresponde en la lista de
reacciones que podrían obtenerse cuando estos compuestos se calientan suavemente en un tubo
de ensaye. En cada caso escribe el número de la reacción correspondiente en la casilla de
REACCIÓN:

No. DE No. DE
COMPUESTO COMPUESTO
REACCIÓN REACCIÓN
DICROMATO DE ÓXIDO DE SILICIO
AMONIO (IV)
CARBONATO DE CARBONATO DE
COBRE (II) SODIO HIDRATADO
BICARBONATO DE
NITRATO DE SODIO
SODIO
CLORURO DE CARBONATO DE
HIERRO (III) ZINC
NITRATO DE
ÓXIDO DE ZINC
PLOMO (II)
ÓXIDO DE
ÓXIDO DE MAGNESIO
MERCURIO (II)

1 Se obtiene un polvo negro. 2 Se forma vapor de agua.


3 Se forma un gas comburente. 4 Se obtiene un metal.
5 Después de enfriar da un polvo 6 Se obtiene un gran volumen de un
amarillo. polvo verde.
7 Se obtiene un polvo blanco que 8 Se obtiene nitrógeno.
cuando se trata con HCl diluido
produce CO2.
9 Se obtiene un polvo que es amarillo 10 Se obtiene un sublimado.
cuando está caliente pero es blanco
cuando se enfría.
11 No hay ningún cambio químico ni 12 Ninguna de las respuestas es
físico (aparte del calentamiento). correcta.

124
11. El CO es uno de los principales contaminantes generados en motores de combustión interna y por
ello se utilizan catalizadores para convertirlo a CO2. Cada segundo, cada uno de los cilindros
completa 25 ciclos de quemado y consume un total de 0.400 g de combustible (C8H18). La
relación de compresión del cilindro es de 1:8 (esto es la relación entre el menor volumen y el
mayor volumen que los gases ocupan durante el movimiento del pistón dentro del cilindro en
cada ciclo de quemado). Para que se lleve a cabo la combustión del combustible, al cilindro entra
una cierta cantidad de moles de combustible (Nc) y otra cantidad de moles de aire (Na). El
combustible gasificado y el aire se introducen al cilindro cuando su volumen es máximo, ambos a
una temperatura de 100.0 oC hasta que se alcanza una presión en el cilindro de 101.0 kPa. El aire
contiene un 21% en volumen de oxígeno y el resto es nitrógeno. Se supone que el 10% del
combustible presente produce CO durante la combustión, que el resto produce CO2 y que el
nitrógeno permanece inerte. PV = nRT, R = 8.314 Pa m3 / mol K.

a) Escribe la reacción de combustión balanceada que muestre el proceso que ocurre dentro del
cilindro. Escribe la ecuación para diez moles de combustible.

_______________________________________________________________________

b) Por cada mol de combustible, se consumen de moles de aire: ___________

A) Menos de 55.0 mol


B) Entre 55.0 y 56.9 mol
C) Entre 57.0 y 59.0 mol
D) Entre 59.1 y 61.0 mol
E) Más de 61.0 mol

c) Calcula el volumen de aire que entre al motor para llevar la combustión, en m3/s.
El volumen de aire es: ___________

A) Menos de 5.00 x 10-3 m3s-1


B) Entre 5.10 x 10-3 m3s-1 y 6.40 x 10-3 m3s-1
C) Entre 6.40 x 10-3 m3s-1 y 7.00 x 10-3 m3s-1
D) Entre 7.00 x 10-3 m3s-1 y 8.00 x 10-3 m3s-1
E) Más de 8.00 x 10-3 m3s-1
14
12. El tiempo de vida media para el decaimiento del C es de 5730 años. Si una muestra de madera
14
contiene sólo un 25 % del C que contiene un árbol vivo, la antigüedad de la muestra de madera
es: ________________
(Coloca la letra del inciso correcto en la línea).

A) Menor a 2860 años


B) Entre 2860 y 5700 años
C) Entre 5700 y 8500 años
D) Entre 8500 y 11420 años
E) Mayor a 11420 años

FIN DEL EXAMEN

125
126
XVII OLIMPIADA NACIONAL DE QUÍMICA
TERCER EXAMEN NIVEL B Tiempo asignado: 180 minutos Oaxaca 2008

La adición de un soluto a un disolvente puro trae como consecuencia que se modifiquen las
propiedades del disolvente. Mientras más concentrada sea una disolución, más se modifican las
propiedades del disolvente. Las propiedades de las disoluciones diluidas que dependen del número
de moléculas de soluto y no del tipo de especies presentes se denominan propiedades coligativas.
Si se determinan experimentalmente estas propiedades, es posible conocer la composición de la
disolución y a partir de ésta determinar otras propiedades como son la masa molar del soluto y la
pureza del disolvente.
Para disoluciones diluidas se pueden aplicar las siguientes expresiones para calcular estas
propiedades:

Propiedad Coligativa
Disminución de la presión de vapor P  x 2 P 
Disminución de la temperatura de fusión T fusión  K c m
Aumento de la temperatura de ebullición Tebullición  K b m
Presión osmótica   RTC

Donde:
P* = presión de vapor del disolvente puro
x2 = fracción molar del soluto
Kc = constante crioscópica (o constante de descenso de la temperatura de fusión Kf)
Kb = constante ebulloscópica (o constante de aumento de la temperatura de ebullición)
 = presión osmótica
C = concentración de la disolución (mol de soluto/L de disolución)
m = Concentración molal de la solución (mol de soluto/kg de disolvente)
R = 8.314 J mol– 1 K– 1 = 0.082 L atm mol– 1 K– 1
1 atm = 76 cm Hg = 101325 Pa
1 cm Hg = 13.59 cm H2O

1. Si 12.5 g de un compuesto “A” no volátil se disuelven en 520.8 g de etanol (C2H5OH), la presión


de vapor del disolvente puro de 56.18 Torr se reduce a 55.24 Torr. La masa molar del compuesto
“A” es en g/mol:
A Menor a B Entre C Entre D Mayor a
64 64 y 66 66 y 68 68

2. El naftaleno puro (C10H8; 128.17 g/mol) tiene un punto de fusión de 353.35 K. Si el punto de
fusión de una muestra impura de naftaleno es 351.85 K y la constante crioscópica del naftaleno es
Kc = 7.00 K kg mol– 1; la pureza de la muestra en porcentaje molar es:
A Entre B Entre C Entre D Mayor a
21 y 22 % 78 y 79 % 97 y 98 % 98%

3. Las constantes crioscópica y ebulloscópica para el agua son Kc = 1.86 K kg mol– 1


y
Kb = 0.51 K kg mol– 1. Considere la densidad del agua de 1.0 g/mL.

127
3.1. Una disolución que contiene 15.0 g de sacarosa (C12H22O11) disueltos en 75.0 g de agua tiene
una temperatura de fusión:
A Mayor de B Entre C Entre D Menor de
– 0.04 ºC – 0.04 ºC y – 0.58 ºC y – 1.0 ºC
– 0.58 ºC – 0.90 ºC

3.2. Una disolución que contiene 30.0 g de glucosa (C6H12O6) en 150 g de agua tiene una variación
de temperatura de ebullición (Teb)…
A Mayor que B Igual que la C Menor que D El doble que
la disolución la la disolución
disolución de sacarosa disolución de sacarosa
de sacarosa de sacarosa

4. Una disolución, a 25 oC, que contiene 0.713 g de una sustancia desconocida en 100 mL de agua
se introduce en un osmómetro y la presión osmótica resultante es de 27.0 cm de agua. La masa
molar de la sustancia es:
A Entre B Entre C Entre D Mayor de
64 y 66 92 y 93 654 y 655 6600 g/mol
g/mol g/mol g/mol

5. ¿Cuál es el valor de pH de la disolución que resulta al disolver 10.0 litros de cloruro de hidrógeno
gaseoso (que se encuentra en condiciones normales de temperatura y presión) en 750 mL de
agua?
(R = 0.082 L atm K–1 mol–1 = 8.31 J K–1 mol–1).

pH =

6. Para obtener 15.54 g de un metal a partir de la reducción de una disolución acuosa de sus iones
divalentes se requiere una carga total de electricidad de 14475 C. ¿Cuál es el metal? Faraday =
96485 C mol-1.

Metal:

128
7. Para determinar el contenido de nitrato de potasio en una mezcla que contiene también cloruro de
potasio se disuelven 4.00 g de la mezcla en 50.0 mL de agua destilada; a continuación se añaden
100 mL de una disolución acuosa de AgNO3 1.00 mol/L y se obtienen 2.08 g de un precipitado
blanco. ¿Cuál es el porcentaje en masa de cada uno de los componentes de la mezcla?

% de KC l=

% de KNO3 =

8. En un laboratorio de la Universidad Autónoma B. J. de Oaxaca se requirió preparar 6.00 litros de


una disolución acuosa que tenga una concentración 1.92 mol/L de HCl y 8.33 mol/L de NaCl.
Para prepararlas se cuenta con dos disoluciones: una de HCl al 37.5 % m/m (d = 1.13 g/mL) y
otra de NaCl 41.2 % m/m (d = 1.42 g/mL). ¿Qué volúmenes de cada una de estas disoluciones se
requiere mezclar? Suponer que los volúmenes son aditivos.

VHCl = litros VNaCl = litros

9. ¿Qué masa de H2C2O4·2H2O se necesita para preparar 1.0 L de una disolución de concentración
tal que 1.00 mL precipite en forma de oxalato de cerio (III) con 1 mL de una disolución
0.100 mol/L de CeCl3?

129
10. Estudia la siguiente lista de compuestos y busca el resultado que le corresponde en la lista de
reacciones que podrían obtenerse cuando estos compuestos se calientan suavemente en un tubo
de ensaye. En cada caso escribe el número de la reacción correspondiente en la casilla de
REACCIÓN:

COMPUESTO No. DE COMPUESTO No. DE


REACCIÓN REACCIÓN
DICROMATO DE ÓXIDO DE SILICIO
AMONIO (IV)
CARBONATO DE CARBONATO DE
COBRE (II) SODIO HIDRATADO
BICARBONATO DE NITRATO DE SODIO
SODIO
CLORURO DE CARBONATO DE
HIERRO (III) ZINC
NITRATO DE ÓXIDO DE ZINC
PLOMO (II)
ÓXIDO DE ÓXIDO DE
MAGNESIO MERCURIO (II)

1 Se obtiene un polvo negro. 2 Se forma vapor de agua.


3 Se forma un gas comburente. 4 Se obtiene un metal.
5 Después de enfriar da un polvo 6 Se obtiene un gran volumen de un
amarillo. polvo verde.
7 Se obtiene un polvo blanco que 8 Se obtiene nitrógeno.
cuando se trata con HCl diluido
produce CO2.
9 Se obtiene un polvo que es amarillo 10 Se obtiene un sublimado.
cuando está caliente pero es blanco
cuando se enfría.
11 No hay ningún cambio químico ni 12 Ninguna de las respuestas es
físico (aparte del calentamiento). correcta.

FIN DEL EXAMEN

130
XVII OLIMPIADA NACIONAL DE QUÍMICA
EXAMEN INTERNACIONAL
9 PROBLEMAS Tiempo: 4 HORAS Oaxaca 2008

1. Una medicina que se inyecta en forma intramuscular, llega al torrente sanguíneo de acuerdo a una
cinética de primer orden caracterizada por una constante de rapidez de absorción KA.
Posteriormente se elimina de la sangre y pasa al interior de un órgano por otra reacción de primer
orden que tiene una constante de rapidez de extinción KE.

[S0] [S*] [especie absorbida en el órgano]


KA KE

[S0] es la concentración inicial que se inyecta, [S*] es la concentración en la sangre y depende


del tiempo “t” de acuerdo a la ecuación:

S *  S K K
0 A E K A e K t  e K
A Et

Para [S0] = 10 (unidades arbitrarias), KA = 3KE = 2 minutos-1

1.1. Representa en una gráfica el cambio de [S*] en función del tiempo y determina ¿en qué tiempo
se alcanza la máxima concentración de [S*]?

Tiempo en que [S*] tiene el valor máximo = ____________ minutos

131
1.2. Anota al menos cinco valores de tiempo y la correspondiente [S*] que utilizaste para construir
la gráfica del inciso anterior

t [S*]

1.3. Si la medicina se inyectara directamente al torrente sanguíneo por vía intravenosa,


¿qué valores de las constantes describirían mejor este proceso? Pon en el recuadro
la letra que corresponda al inciso correcto

A KA = 1 B KA = - 100
KE = 100 KE = 1

C KA = 100 D KA = - 1
KE = 1 KE = 100

1.4. De acuerdo al inciso anterior, ¿cuál sería la ecuación que describiría [S*] en función del
tiempo?

_______________________________________________________________

Con base en los datos de KA = 3KE = 2 minutos-1, contesta las siguientes preguntas:

1.5. Para el caso de inyección intramuscular, ¿en qué tiempo se ha eliminado del torrente sanguíneo
la mitad de la concentración inicial [S0]?

t = _____________ minutos

1.6. Para el caso de inyección intravenosa, ¿en qué tiempo se ha eliminado del torrente sanguíneo la
mitad de la concentración inicial [S0]?

t = _____________ minutos

132
2. Complete usted la siguiente secuencia sintética, a través de la cual se obtiene el Acetaminofeno
un analgésico que es el componente activo del Tylenol:
H2 SO4
A O2 , L U Z B (20 % ) C + D
H2 SO4 (C9 H 1 2) CALO R (C9 H 1 2O2) CALO R (C6 H 6 O) (C3 H 6 O)
1) KNO3, H2SO4
2) DESTILACIÓN POR
ARRASTRE CON VAPOR

RESIDUO DESTILADO

ACETAMINOFENO 1) H2, Pd(C)


E F
(C8H9NO2)
2) Ac2O, Et3N (C6H5NO3) (C6H5NO3)

RESPUESTAS:
A B C

D E F

ACETAMINOFENO

133
3. Complete la siguiente secuencia sintética:
NaCN 1) CH3MgBr, Et2O NaBH 4
CH3CH2CH2 I A B C
DMSO 2) HCl, H2O CH3OH
1) CH3MgBr, Et2O
2) HCl, H 2O
LiAlH 4
I
CH3OH
D F
LiAlH4 H 2SO4 (cat.)
SOCl2 H
Al HO
NH 4OH CH3OH OH
J E F G H
PIRIDINA H 2SO4 (cat.)

AcOH (cat.)

K + H 2O

RESPUESTAS:
A B C D

E F G H

I J K

134
4. Un compuesto desconocido A, C13H16O4, es ópticamente activo, el cual no reacciona con una
disolución de bromo en tetracloruro de carbono. Es insoluble en agua, pero soluble en base
diluida. Al hidrolizar el compuesto A, se obtienen dos nuevos compuestos B y C. El compuesto
B, C4H6O4, es ópticamente inactivo e insoluble en agua. Sin embargo, es muy soluble en
presencia de una base diluida. Cuando el compuesto B se calienta, este pierde fácilmente
anhídrido carbónico para dar el compuesto D, C3H6O2. El compuesto C, C9H12O, es un
compuesto aromático ópticamente activo. Cuando se trató el compuesto C con sodio metálico se
desprendió del tubo de ensayo un gas. Así mismo, se hizo reaccionar el compuesto C con el
reactivo de Lucas (HCl, ZnCl2) y no se observó ninguna turbidez. Cuando el compuesto C se
calentó con permanganato de potasio en presencia de hidróxido de potasio, y luego se neutraliza
con ácido clorhídrico, se obtiene el compuesto aromático E, C7H6O2. Posteriormente, el
compuesto C se calienta con ácido sulfúrico concentrado, y se obtiene el compuesto F, C9H10. El
compuesto F es ópticamente inactivo y decolora una disolución de bromo en tetracloruro de
carbono. Cuando el compuesto F se trata con ozono y después con sulfuro de dimetilo, se
obtienen formaldehído y una cetona aromática G, C8H8O, la cual al hacerla reaccionar con una
disolución acuosa de yodo en hidróxido de sodio, forma un precipitado.

Proponga usted estructuras para los compuestos A al G, Tenga en cuenta que debe indicar la
fuente de quiralidad en los compuestos que presenten la actividad óptica.

RESPUESTAS:
A B C

D E F

135
5. ¿Cuál de las siguientes estructuras es el hemiacetal cíclico que se forma a partir del
4-hidroxiheptanal?

CH3 O OH
O OH
I.

IV.

CH3CH2CH2
CH3CH2 O OH

II.

CH3CH2CH2

V.

III. CH3CH2CH2 O OH
OH

A) I
B) II
C) III
D) IV
E) V
RESPUESTA:
INCISO:

136
6. Se tiene un oligosacárido del que se conoce el siguiente comportamiento:
a) Al ser hidrolizado por la maltasa (una -glicosidasa) produce galactosa y un disacárido A. La
galactosa es el epímero en C4 de la glucosa.
b) Al ser hidrolizado por la emulsina (una -glicosidasa) produce fructosa y un disacárido B. La
fructosa es una 2-cetohexosa que forma la misma osazona que la glucosa.
c) Reacciona con la fenilhidracina.
d) La metilación exhaustiva del oligosacárido y posterior hidrólisis del mismo, produce 2,3,6-
tri-O-metil-D-glucosa; 1,3,6-tri-O-metil-D-fructosa y 2,3,4,6-tetra-O-metil-D-galactosa.

Escriba cada una de las reacciones efectuadas, dando las estructuras de los productos y
represente la estructura del oligosacárido.

Reacción inciso a:

Reacción inciso b:

Reacción inciso c:

Reacción inciso d:

137
ESTRUCTURA DEL OLIGOSACÁRIDO:

H O
C

OH

HO

OH

OH

OH

D-GLUCOSA

7. El esmalte de los dientes está constituido principalmente de una sustancia inorgánica insoluble
denominada hidroxiapatita. El equilibrio de solubilidad de la hidroxiapatita se indica en la
siguiente reacción química no balanceada:

Ca5(PO4)3OH (s) ⇄ ___Ca2+ (ac) + ___PO43- (ac) + ___OH- (ac) Kps = 6.8x10-37

7.1. ¿Cuál es la solubilidad de la hidroxiapatita?

Cálculos:

Solubilidad ________________mol/L

138
7.2. Esta solubilidad aumenta en presencia de ácidos debido a la siguiente reacción global que hace
intervenir dos equilibrios: el de solubilidad y la reacción de neutralización del ion OH-. La
reacción global no balanceada es:
Ca5(PO4)3OH (s) + H+ (ac) ⇄ ___Ca2+ (ac) + ___PO43- (ac) + H2O (l)

Indicar la expresión de la constante de esta reacción global y calcular su valor.


K=

Valor de K: ____________

Al aumentar la solubilidad, la posibilidad de caries aumenta; para evitarlas muchas pastas


dentales contienen fluoruros. La presencia de fluoruro favorece la formación de fluoroapatita
Ca5(PO4)3F, que es más insoluble que la hidroxiapatita.

7.3. Calcula la solubilidad de la fluoroapatita cuyo equilibrio de solubilidad, no balanceado, es:

Ca5(PO4)3F (s) ⇄ ___Ca2+ (ac) + ___PO43- (ac) + F- (ac) Kps = 2.5x10-60

Solubilidad ________________mol/L

La reacción de formación de la fluoroapatita a partir de la hidroxiapatita se indica a


continuación y es muy cuantitativa:
Ca5(PO4)3OH (s) + F- (ac) ⇄ Ca5(PO4)3F (s) + OH- (ac)

7.4. Calcula el valor de su constante a partir de los productos de solubilidad de ambas apatitas.

Indica tus cálculos:

Valor de K =

139
8. La conductimetría es una técnica analítica que permite determinar la concentración iónica total
en una disolución por medio de un equipo sencillo que mide la resistencia al paso de la
corriente entre dos electrodos y que proporciona una señal que se expresa en siemens/centímetro
(S/cm) que se denomina conductividad específica (). La fórmula de la conductividad es:
  i C i 0i
0i es una constante cuyo valor es diferente para cada ion y se denomina conductividad
equivalente límite (sus unidades son Scm2 eq-1).

C i es la concentración de los iones (en eq/cm3). Esta concentración se obtiene multiplicando la


concentración del ion (en mol/cm3) por el valor absoluto de su carga.

Recientemente esta técnica fue utilizada en el laboratorio de Química Analítica de la


Universidad Autónoma Benito Juárez de Oaxaca para determinar el valor de la constante de
acidez de un ácido monoprótico recién sintetizado, el cual denotaremos como HA. De este
ácido HA se preparó una disolución acuosa midiendo una masa de 0.0496 g del ácido y
disolviéndola en 100 mL de agua pura (disolución R).

El valor de la conductividad específica de la disolución R fue de 647 S cm-1 y la del agua con
la cual fue preparada 10.0 S cm-1.
Datos: Valores de 0: Ion H+ = 350 S cm2 eq-1; Ion A- = 54 Scm2 eq-1

a) Para neutralizar una alícuota de 10.0 mL de la disolución R se gastaron 10.8 mL de NaOH


0.01 mol/L. Calcular la masa molar del ácido.

Cálculos:

Masa molar =

b) Calcular la concentración del ácido HA en la disolución R.

Concentración de HA = mol/L

140
c) ¿Qué iones contiene la disolución R?

d) ¿Cuál es la conductividad específica de los iones que provienen de la disociación del ácido?

= S cm-1

e) ¿Cuál es la concentración de los iones que provienen de la disociación del ácido HA?

f) ¿Qué fracción (%) del ácido HA se encuentra disociado?

g) ¿Cuál es el valor de la constante de disociación del ácido HA cuya disolución se midió


conductimétricamente y dio un valor de conductividad específica igual a 647 S?

Valor de Ka =

9. Por extrapolación para los datos conocidos para cromo, molibdeno y wolframio, el científico
Seaborg estimó los siguientes valores de potencial redox para el elemento 106 (M):
MO3 ⇄ M2O5 E° = 0.6 V MO2 ⇄ M3+ E° = 0.7 V

M2O5 ⇄ MO2 E° = 0.1 V M3+ ⇄ M E° = 0.0 V

En todos los casos siguientes escribe el equilibrio químico correspondiente y marca en el cuadro
de la derecha, poniendo un SI o un NO, si la reacción se lleva a cabo cuantitativamente.
141
a) Al elemento M se le añade HCl 1 M.

b) MCl3 se pone en disolución acidificada de sulfato de hierro (II).

c) MO2 y M se ponen en contacto en disolución ácida.

d) M3+ y MO3 se ponen en contacto en disolución acuosa.

e) El elemento M es tratado con HNO3 concentrado.

Para cada equilibrio debes considerar la posibilidad de producción de hidrógeno u oxígeno, de


dismutación o anfolización y reacciones de óxido-reducción. Toma en cuenta los aniones y
cationes presentes en cada caso. Considera la siguiente información adicional:

Fe3+ ⇄ Fe2+ E° = 0.77 V


NO3 ⇄ NO
-
E° = 0.93 V
SO4 2-
⇄ H2SO3 E° = 0.17 V

FIN DEL EXAMEN

142
XVII Olimpiada Nacional de Química
Examen Experimental de Química Orgánica Oaxaca 2008

Número de código: _____________

En la actualidad los analgésicos que se encuentran en el mercado, contienen en su formulación


alguno de los siguientes compuestos:
O
O
H
N
O O

OH

OH
ÁCIDO ACETILSALICÍLICO PARACETAMOL

y en presencia (o ausencia) de cafeína:


CH3

O N N

N
H3C N

CH3
O
CAFEÍNA

Dentro de las marcas comerciales que se pueden encontrar en estos momentos en nuestro País, se
encuentran las siguientes:
NOMBRE PRODUCIDO POR: COMPOSICIÓN POR TABLETA:
COMERCIAL
ASPIRINA Bayer de México, S. A. de ÁCIDO ACETILSALICÍLICO 500 mg
C. V.
CAFIASPIRINA Bayer de México, S. A. de ÁCIDO ACETILSALICÍLICO 500 mg
C. V. CAFEÍNA 30 mg
PANADOL SmithKline Beecham, PARACETAMOL 500 mg
México, S. A. de C. V.
XL-DOL Selder, S. A. de C. V. PARACETAMOL 500 mg
SARIDÓN Productos Roche, S. A. de PARACETAMOL 500 mg
C. V. CAFEÍNA 30 mg

En todas las presentaciones se utiliza como excipiente al almidón o bien a la metilcelulosa, en la


cantidad suficiente para formar la tableta.
143
En este experimento, tú vas a tener que identificar, por pruebas de solubilidad, por reacción
característica de un grupo funcional y por un análisis por cromatografía en capa fina, los 5 sólidos
que se encuentran en tu mesa de trabajo, los cuales están en los recipientes de plástico que tienen las
letras A, B, C, D y E.
Los compuestos A, B y C, están puros y pueden ser:
O
O H
N
O O

OH

OH

ÁCIDO ACETILSALICÍLICO PARACETAMOL

CH3
O N N

N
H3C N
O CH3

CAFEÍNA
El sólido D es una tableta, que puede ser de Aspirina o Panadol.
Finalmente el sólido E es una tableta que puede ser de Saridón o Cafiaspirina.

PRIMERA PARTE.

IDENTIFICACIÓN DE LOS COMPUESTOS A, B y C.


Sigue con cuidado cada uno de los pasos que se indican a continuación, para que puedas identificar
a los compuestos A, B y C:
Coloca la punta de una cucharita del compuesto A en tres tubos de ensayo (tubos 1, 2 y 3).
1. Al tubo No. 1 agrega 15 gotas de solución de HCl al 5% y agita. Anota tus observaciones.
2. Al tubo No. 2, agrega 15 gotas de solución NaHCO3 al 5% y agita. Anota tus observaciones.
3. Al tubo No. 3, agrega 15 de gotas de agua y adiciona 5 gotas de una solución de cloruro de
hierro (III) al 3%. Anota tus observaciones. (NOTA: un fenol da una coloración azul
intensa).

Repite los tres pasos anteriores para los compuestos B y C.

144
Pruebas de COMPUESTO
identificación (+) se disolvió o dio OBSERVACIONES
reacción colorida

A B C
a) HCl 5%

b) NaHCO3 5%

c) Prueba con FeCl3


(cloruro de hierro (III))

RESULTADOS DE LAS PRUEBAS DE SOLUBILIDAD,


Y REACCIÓN CARACTERÍSTICA DE UN GRUPO FUNCIONAL.
Por lo tanto, con base en los experimentos anteriores, las estructuras de los compuestos A, B y C
son:

A B

145
SEGUNDA PARTE.
IDENTIFICACIÓN DE LAS TABLETAS D y E.

INTRODUCCIÓN TEÓRICA.
CROMATOGRAFÍA EN CAPA FINA.
La cromatografía en capa fina es una técnica de adsorción sólido-líquido, utilizada en química
orgánica para realizar un análisis cualitativo eficiente y rápido de compuestos desconocidos o para
determinar la composición de una mezcla de compuestos.
Esta técnica cromatográfica consiste en la utilización de una fase estacionaria (alúmina o sílica-gel)
y de una fase móvil, que son los disolventes orgánicos de diferente polaridad.
El proceso de separación se basa en que la fase móvil asciende a través de la fase estacionaria, la
elución de la muestra problema está en función de su polaridad, es decir de la afinidad que presente
con alguna de las dos fases.
La fase estacionaria consiste de una película delgada (100 m) de un material disperso sobre una
superficie plana (de vidrio, aluminio o celulosa).
Las ventajas del método son la rapidez para llevar a cabo el análisis (un análisis normal toma de 2 a
10 minutos) y se pueden detectar cantidades de material de 2 a 20 g.
La secuencia de pasos que se siguen para llevar a cabo el análisis por cromatografía en capa fina
son:
1) Se traza una línea paralela a la base de la cromatoplaca (aproximadamente a 0.5 cm de la base de
la misma) sobre la superficie del adsorbente con un lápiz (NOTA: no recargues mucho la punta
del lápiz, ya que se puede llegar a romper la superficie de la fase estacionaria). La muestra que se
va a analizar (1 mg o menos) se coloca en un tubo de ensayo, y se adicionan unas gotas del
disolvente hasta observar la disolución de la muestra. Con un tubo capilar se aplica una pequeña
fracción de la solución sobre la placa.

parte superior 0.5 cm


límite de ascensión
del disolvente

X X punto de aplicación
0.5 cm
parte inferior

2) La cromatografía se lleva a cabo colocando la cromatoplaca sobre la cual se aplicó la muestra,


dentro de un frasco de vidrio con tapa (son ideales para este fin los frascos de los alimentos
infantiles), el cual ya debe contener la fase móvil (unos 3 mL). El frasco se cierra con la finalidad
de mantener una atmósfera saturada con el disolvente de la fase móvil. Efectuadas estas
operaciones, la fase móvil asciende rápidamente por capilaridad sobre el adsorbente de la fase
estacionaria hasta la marca superior. Inmediatamente se saca la placa de la cámara de elución y se
deja evaporar el disolvente. La muestra se puede encontrar en cualquier punto.

146
3) Para localizar los compuestos de la muestra problema, se utiliza un revelador. En este caso vamos
a utilizar vapores de yodo. Una medida física de la polaridad de la muestra es el Factor de
Retención (RF), el cual se determina dividiendo la distancia que recorrió la muestra entre la
distancia que recorrió el disolvente, el RF siempre es menor a 1. Bajo las mismas condiciones
de análisis cromatográfico, el RF es constante y permite identificar cualitativamente un
compuesto o bien determinar si se encuentra presente en una mezcla problema:

parte superior
línea hasta la que recorrió el eluyente

a
RF =
b b

X punto de aplicación de la muestra


parte inferior

PARTE EXPERIMENTAL.
Para identificar las tabletas D y E, se debe de extraer el principio activo de la tableta problema. Para
esto, adiciona al frasco ámbar de cada tableta 10 mL de diclorometano (los cuales ya están medidos
y se encuentran en el frasco etiquetado) y se agita la mezcla con la varilla de vidrio durante 5
minutos. La mezcla se filtra por gravedad por medio de un embudo de vidrio con tallo corto,
recibiéndose el filtrado de cada tableta en sus respectivos tubos de ensayo (disoluciones D y E).
En la parte superior e inferior de la placa se trazan con un lápiz dos líneas horizontales como se
indica en la siguiente figura. La línea inferior se divide en 5 partes equidistantes entre sí, se marcan,
y sobre las tres primeras marcas se aplican las soluciones de los tres estándares (se te proporcionan
sus soluciones en diclorometano) de ácido acetilsalicílico, paracetamol y de cafeína. En los
siguientes dos puntos aplica las soluciones en diclorometano de las tabletas D y E respectivamente.

0.5 cm
parte superior

parte inferior X X X X X
0.5 cm

A B C D E

ESTÁNDARES MUESTRAS PROBLEMA

147
Una vez que aplicaste todas las soluciones sobre la fase estacionaria (tanto de los 3 estándares como
las de las dos tabletas), se deja evaporar el diclorometano y la placa se introduce en la cámara de
elución (frasco Gerber con tapa) que contiene una mezcla de hexano-acetato de etilo 20:80, la cual
va a ser la fase móvil (eluyente). Se coloca la tapa sobre la boca del frasco y se gira para un cierre
hermético. Una vez que dejaste eluir la mezcla de disolventes hasta la parte superior de la placa (la
cual está marcada con lápiz), con cuidado saca la cromatoplaca y deja evaporar la mezcla de
disolventes.
Para revelar la posición de los estándares y la composición de las dos tabletas, la placa se coloca
dentro de otro frasco de Gerber que contiene yodo metálico como revelador. La placa se deja
adentro de la cámara reveladora por tres minutos y con cuidado se retira la placa de la cámara
reveladora, marcando con un lápiz las manchas.
Determina el RF de los estándares, y con base en estos, decide la composición de las tabletas,
concluyendo para la tableta D si se tenía Aspirina o Panadol y en el caso de la tableta E, si se tenía
Cafiaspirina o Saridón.

REPORTE DE LOS RF
ESTÁNDARES

A B

RF = RF =

RF = RF =

RF =

RF =

TABLETAS

D E
Compuesto 1 Compuesto 2

RF = RF = RF =

RF = RF = RF =

148
Por lo tanto, con base en el análisis por cromatografía en capa fina, la Tableta D que me toco
identificar es:

ANALGÉSICO RESPUESTA:

Por lo tanto, con base en el análisis por cromatografía en capa fina, la Tableta E que me toco
identificar es:

ANALGÉSICO RESPUESTA:

149
150
XVII Olimpiada Nacional de Química
Examen Experimental de Química Inorgánica Oaxaca 2008

En la Universidad Autónoma Benito Juárez de Oaxaca, los profesores tienen una gran afición por el
juego de mesa “Adivina Quién”, sin embargo, en la Facultad de Química las tarjetas del juego
sufrieron un misterioso accidente, por lo que los académicos decidieron hacer su propia versión
utilizando sus conocimientos y los reactivos químicos con los que cuentan. Dicha versión del juego
consiste en escoger tres aniones de la siguiente lista:

OH- CO32- NO3- Cl- S2- Br- CrO42- S2O32-

y proporcionarle al contrincante disoluciones de compuestos que incluyan en su estructura alguno de


los aniones escogidos. Cada jugador debe identificar los aniones que se encuentran en las
disoluciones que le tocaron con ayuda de pruebas a la gota realizadas con disoluciones que
contienen los siguientes cationes:

Fe3+ H+ Ag+

Al enterarse de que los estudiantes del nivel B no presentarían el examen experimental de química
orgánica y que, por tanto, tendrías tiempo libre en el laboratorio, los profesores decidieron invitarte
a jugar. Para ello colocaron tres frascos gotero de uso común en la mesa en la que te encuentras,
etiquetados con el catión que contienen, te proporcionaron tres tubos con las muestras incógnitas
que te tocaron y en tu material incluyeron unas placas para pruebas a la gota. Con este material
deberás realizar las pruebas que consideres pertinentes para identificar tus aniones.

En la siguiente tabla indica que esperarías observar en cada caso (color, precipitado,
desprendimiento de gases, nada). Como ayuda te hemos llenado la que corresponde al anión
tiosulfato y te proporcionamos las reacciones de este anión con el ion Fe3+:

2 Fe3+ + 2 S2O32-  2 [FeS2O3]+  2 Fe2+ + S4O62-

151
Fe3+ H+ Ag+

OH-

CO32-

NO3-

Cl-

S2-

Br-

CrO42-

S2O32-

Ahora llena la siguiente tabla con las observaciones que realizaste en tus muestras:

Fe3+ H+ Ag+

Muestra 1

Muestra 2

Muestra 3

¿De qué aniones se trata?

Muestra 1: ____________ Muestra 2: ____________ Muestra 3: ____________

Si es el caso, indica las reacciones involucradas en la identificación de tus aniones.

152
XVII Olimpiada Nacional de Química
Examen Experimental de Química Analítica Oaxaca 2008

El ácido ascórbico (C6H8O6) es un ácido orgánico cuya estructura se muestra a continuación. Dicha
estructura fue determinada por primera vez por Walter Haworth quien logró con ello obtener el
premio Nobel en 1937.

HO
O
O
HO

HO OH Estructura del ácido ascórbico.

El enantiómero L (levógiro) del ácido ascórbico se conoce también como vitamina C (el nombre
ascórbico procede de su propiedad de prevenir y curar el escorbuto). Albert von Szent-Gyorgyi
Nagyrapolt se hizo también acreedor al premio Nobel de Medicina por sus estudios acerca de las
funciones biológicas del ácido ascórbico.
El ácido ascórbico se oxida fácilmente a ácido dihidroascórbico de acuerdo con la siguiente reacción
general:

C6H8O6 - 2e- - 2 H+  C6H6O6

Al departamento de Química Analítica de la Universidad Autónoma de Oaxaca han llegado varias


muestras de lotes de ácido ascórbico comercial cuya pureza debemos determinar urgentemente. Tu
labor en esta mañana será ayudarnos a realizarlo mediante una valoración con un oxidante adecuado.

Para lograrlo hemos preparado disoluciones midiendo cantidades conocidas de cada una de las
diferentes muestras comerciales del ácido ascórbico. Cada una de las muestras ha sido disuelta en
agua destilada y hemos completado con agua hasta obtener un volumen de 100.0 mL. De esta
disolución se te proporcionará un volumen aproximado de 40 mL. La masa exacta medida se indica
en el frasco que contiene dicha disolución.

En otro frasco encontrarás aproximadamente 100 mL de una disolución que contiene yodo en
concentración 0.10 mol/L a la que se ha añadido un exceso d yoduro de potasio y la cual podrás
utilizar como oxidante para determinar el ácido ascórbico.

En la siguiente página, escribe en forma esquemática y simple, tanto el procedimiento que deberás
seguir como lo que esperas observar para la detección del punto de equivalencia de la valoración.
Considera para ello el material del que dispones en forma individual y el que compartirás con los
otros participantes que se encuentran en la misma mesa (como el agua destilada y la disolución de
almidón).

153
PROCEDIMIENTO.

1. Llena la bureta con la disolución de yodo/yoduro de potasio.


2. Toma, con la pipeta volumétrica, una alícuota de la disolución problema y colócala en el matraz
erlenmeyer; añádele un volumen aproximado de 10 mL de agua destilada y unas gotas de la
disolución de almidón.
3. Titula tu muestra problema con la disolución de yodo/yoduro de potasio contenida en la bureta. El
punto final se detecta cuando aparece una coloración azul.
4. Repite los pasos 2 y 3 y realiza la titulación por triplicado. NO ES NECESARIO QUE
VUELVA A LLENAR LA BURETA para realizar estas dos titulaciones pero sí es
imprescindible que ANOTES EL VOLUMEN DONDE INICIASTE CAD UNA.

RESULTADOS.
Titulación Volumen de la alícuota de Volumen gastado de la
muestra problema (mL) disolución de yodo/yoduro de
potasio (mL)

Volumen que utilizarás en tus cálculos: ____________________________

Pureza del ácido ascórbico:


Cálculos:

¿Por qué es necesario añadir yoduro de potasio a la disolución de yodo?

¿Ocurre alguna reacción? Si tu respuesta es afirmativa indica cuál es.

¿A qué se debe la coloración azul observada?

En una escala de potencial electroquímico, indica la posición relativa de los pares I2/I- y
C6H6O6/C6H8O6.

154
HOJA PARA ESCRIBIR EL PROCEDIMIENTO Y LA FORMA DE DETECCIÓN DEL PUNTO
DE EQUIVALENCIA (Tienes 10 minutos para realizar esta parte. Al terminar debes entregar esta
hoja al supervisor quien te entregará otras hojas).

155
156
XVIII Olimpiada Nacional de Química
1er Examen Nivel A y B.
Total: 25 preguntas. Tiempo asignado: 60 minutos. Campeche 2009

En el siglo XXI es común encontrar problemas que involucran a varias ciencias y por esta razón en
esta primera parte del examen, encontrarás preguntas que relacionan a la química con otras ciencias
como las matemáticas, la física y la biología. Además, deberás leer con todo cuidado cada pregunta.
Debes anotar en el recuadro correspondiente la letra del inciso que contesta correctamente cada
pregunta.
No olvides que si consideras que ningún inciso es el correcto deberás anotar una letra X.

ESCRIBE CLARAMENTE LAS LETRAS PARA EVITAR CONFUSIONES AL


CALIFICAR.
SI NO ESCRIBES TUS RESPUESTAS CON TINTA, TU EXAMEN PODRÍA SER
ANULADO.

1) Las proteínas están formadas por la unión de moléculas llamadas aminoácidos. El


grupo funcional ácido que caracteriza a estas moléculas es:

A - CNH+ B - COOH
C - CH2+ D - CHOH

2) El aminoácido más simple es la glicina. Su fórmula condensada es C2H5NO2. Por lo


tanto, en 100 g de glicina, ¿cuántos moles tenemos de esta sustancia?

A Menos de 1 mol B Entre 1 mol y 1.5 mol


C Entre 1.5 mol y 2 mol D Más de 2 mol

3) La glicina tiene una densidad de 1.1607 g/cm3 y la del agua es 1.0 g/cm3. Si un
recipiente con capacidad de un litro se llena totalmente con glicina, ¿cuántos gramos
de glicina puede contener?
A Menos de 500 g B Entre 500 g y 750 g
C Entre 750 g y 1000 g D Más de 1000 g

4) En gramos, una ppm (partes por millón) equivale a un gramo en un millón de gramos.
Si se pueden disolver 250 g de glicina en 1 kilogramo de agua, ¿cuál es la
concentración de glicina en ppm?
A Menos de 100000 ppm B Entre 100000 ppm y 200000 ppm
C Entre 200000 ppm y 300000 ppm D Más de 300000 ppm

5) Si se lograra disolver un milimol (0.001 moles) de mercurio en 100 m3 de agua, la


concentración de mercurio en esta mezcla sería en ppm:

A Menos de 0.005 ppm B Entre 0.005 ppm y 0.010 ppm


C Entre 0.010 ppm y 0.015 ppm D Más de 0.015 ppm

157
6) En un recipiente de 3.0 L de capacidad hay la misma cantidad en gramos de nitrógeno,
oxígeno y neón gaseosos. La fracción mol de neón en esta mezcla es:

A Menor a 0.3 B Entre 0.3 y 0.4


C Entre 0.4 y 0.5 D Mayor a 0.5

7) La ley de los gases ideales es PV=nRT, (R = 0.082 L atm/mol K), (R = 8.314 J/K mol),
(1 J = kg m2 s-2), (1 atm = 101.325 kPa), (1 Pa = 1 N m-2), (1 N = 1 kg m s-2). ¿En qué
condiciones de P y T, un mol de gas ideal ocupa un volumen de 8.0 L?
A 1 atm y – 224.35 oC B 50.66 kPa y 48.8 K
o
C 0.25 atm y – 224.35 C D 101.324 kPa y 48.8 K

8) Por descomposición de carbonato de calcio se obtiene óxido de calcio y CO2. ¿Cuántos


gramos del carbonato se requieren para obtener 22.4 litros de CO2 a una P de 1
atmósfera y a una T de 100 oC?
A Menos de 75 g B Entre 75 g y 85 g
C Entre 85 g y 95 g D Más de 95 g

9) La constante de Avogadro es 6.0221 x 1023 partículas/mol y la carga de un electrón es


1.60 x 10-19 coulombs (C). ¿Cuál es la carga de un mol de electrones?

A Menos de 80000 C B Entre 80000 C y 90000 C


C Entre 90000 C y 100000 C D Más de 100000 C

10) La fenilalanina, otro aminoácido, tiene en su molécula un anillo similar al del benceno
(C6H6). Generalmente este compuesto se representa con ¿cuántos enlaces C = C?

A Menos de 2 B 2
C 3 D Más de 3

11) El ácido benzoico también es un derivado del benceno y se obtiene al sustituir un


hidrógeno por un grupo –COOH. El % en masa de oxígeno en la molécula del ácido
benzoico es:
A Menos de 20% B Entre 20% y 25%
C Entre 25.1% y 30% D Más de 30%

12) Cuando un gramo de agua es adicionado a una mezcla de ácido y agua, la nueva
mezcla es 25% en masa de ácido. Cuando 2 gramos de ácido son adicionados a la
nueva mezcla, la mezcla ahora contiene 50% en masa de ácido. El % en masa de ácido
en la mezcla original era:
A Menos de 30% B Entre 30% y 35%
C Entre 36% y 49% D Más de 49%

13) Un estudiante toma 2.0 mL de una disolución acuosa de HCl 3.0 M y le agrega agua
destilada hasta tener 250.0 mL de disolución. El pH de la disolución preparada es:

A Menor a 1 B Entre 1 y 1.5


C Entre 1.5 y 2 D Mayor a 2
158
14) Si se disuelven 14 g de permanganato de sodio en agua suficiente para tener 250 mL
de disolución, la concentración molar de este compuesto es:

A Menor a 0.38 mol L-1 B Entre 0.38 mol L-1 y 0.42 mol L-1
C Entre 0.42 mol L-1 y 0.44 mol L-1 D Mayor a 0.44 mol L-1

15) El estado de oxidación del manganeso en el permanganato de potasio (KMnO4) es:

A Mayor a + 5 B +5
C +3 D Menor de + 3

16) El permanganato de potasio reacciona con hidróxido de potasio y se obtiene el


manganato de potasio (K2MnO4), agua y oxígeno gaseoso. Al balancear la ecuación se
encuentra que por cada mol de permanganato que reacciona, se obtienen de oxígeno
gaseoso:
A Menos de un mol B Un mol
C Dos moles D Más de dos moles

17) En la reacción de la pregunta anterior, se puede afirmar que el elemento que se oxida
y el que se reduce son, respectivamente:

A K, O B Mn, O
C O, K D O, Mn

18) La configuración electrónica del manganeso es:

A 1s2 2s2 2p6 3s2 3p6 3d7 B 1s2 2s2 2p6 3s2 3p6 3d6 4s1
C 1s2 2s2 2p6 3s2 3p6 4s2 3d5 D 1s2 2s2 2p6 3s2 3p6 4s2 4p5

19) Cuando se deposita plata metálica a partir de una disolución que contiene iones Ag+,
utilizando una reacción electroquímica. El enunciado correcto es:

A La plata metálica se deposita en el ánodo por oxidación de los iones Ag+


B La plata metálica se deposita en el ánodo por reducción de los iones Ag+
C La plata metálica se deposita en el cátodo por oxidación de los iones Ag+
D La plata metálica se deposita en el cátodo por reducción de los iones Ag+

20) El gas LP consta básicamente de propano (C3H8) y butano (C4H10). Si suponemos que
contiene el 75% en masa de propano. ¿Cuántos moles totales habría,
aproximadamente, en un tanque que contiene 25 kg de la mezcla?
A Menos de 525 mol B Entre 525 mol y 540 mol
C Entre 540 mol y 550 mol D Más de 550 mol

159
21) Si se quema totalmente una mezcla de 100 moles de propano y 50 moles de butano,
de tal manera que sólo se produce CO2 y agua, ¿cuántos moles de CO2 se producen?

A 300 moles B 700 moles


C 150 moles D 500 moles

22) Se disuelve una cierta cantidad “x” de KCl en agua destilada para preparar 500 mL de
la disolución “1”. Se disuelve la mitad de “x” de KCl en agua destilada para preparar
200 mL de la disolución “2”. Al mezclar volúmenes iguales de las disoluciones 1 y 2
se obtiene una nueva disolución cuya molaridad de KCl es 0.422 M. ¿Cuántos gramos
de KCl hay en la cantidad “x”?
A Menos de 12 g B Entre 12 g y 15 g
C Entre 15.1 g y 20 g D Más de 20 g

23) En la reacción 3   + 2, las letras griegas ““, ““, ““ representan
elementos químicos. La masa molar del compuesto  es 70.937 g/mol. El compuesto
2 es un producto de la combustión de muchas sustancias orgánicas. El elemento  es:
A Calcio B Magnesio
C Manganeso D Potasio

24) La corriente que circula en una celda electroquímica se puede medir en amperes (A).
Un ampere equivale a una carga de un coulomb (C) que pasa cada segundo por un
punto del circuito. Si una corriente de 2 mA circula en una celda electroquímica, el
número de electrones que pasan por un punto del circuito cada segundo son:
A Menos de 1.3 x 10 16 B Entre 1.3 x 10 16 y 1.3 x 10 20
C Entre 1.3 x 10 20 y 1.3 x 10 23 D Más de 1.3 x 10 23

25) De acuerdo a la expresión “X”  23490Th +  , podemos concluir que el átomo


radiactivo “X” es: (Las partículas alfa son núcleos de helio)

A 234
91 Pa
B 231
91 Pa
C 244
94 Pu
D 238
92 U

FIN DEL EXAMEN

160
XVIII Olimpiada Nacional de Química. 2do Examen Nivel A y B.
Tiempo asignado: 90 minutos. Campeche 2009

NIVEL A: TIENEN QUE CONTESTAR TODAS LAS PREGUNTAS.


NIVEL B: SÓLO TIENEN QUE CONTESTAR DE LA PREGUNTA 5 A LA 24.

1. Indica qué tipo de reacción de sustitución (SN1 o SN2) se está llevando a cabo en cada una de las
siguientes transformaciones:

H2O OH
Cl RESPUESTA
a)

H H H
Br CH3 H3CO H3C OCH3
CH3
CH3OH
b) +
RESPUESTA

45 % 55 %

H H
I H3C CN RESPUESTA
CH3
c) NaCN

DMSO

2. Se necesitan obtener los compuestos que se encuentran en la segunda columna, a partir de la


misma materia prima: el 2-metil-1-buteno. Escribe en el paréntesis el (o los) reactivo(s)
necesario(s), de acuerdo a la letra que le corresponda al reactivo (primera columna) para efectuar
cada una de las transformaciones.

161
H3C C CH2CH3 2-METIL-1-BUTENO
H C H

REACTIVOS COMPUESTO A RESPUESTA


OBTENER
a) HBr / Peróxidos CH3 ( )
H3C C CH2CH3
Br
b) H2 / Pd(C) H3C CH2CH3 ( )
C
O
c) HBr Br H ( )
H2C C CH2CH3
CH3
d) 1) O3, 2) Me2S CH3 ( )
H3C C CH2CH3
H
e) H2O, HCl Cl Cl ( )
H2C C CH2CH3
CH3
f) H2CrO4 CH3 ( )
H3C C CH2CH3
OH
g) Cl2 Br OH ( )
H2C C CH2CH3
CH3
h) H2N-NH2, KOH

i) Br2 / H2O

3. ¿Cuál de las siguientes fórmulas representa al (E)-3,6-dicloro-6-metil-3-hepteno?

Cl Cl Cl

Cl
Cl Cl Cl
A B C Cl D

a) A
RESPUESTA
b) B
c) C
d) D

162
4. El siguiente compuesto, ¿cuántos centros estereogénicos tiene?
CH3

Cl

Cl

CH3
a) 0
RESPUESTA
b) 1
c) 2
d) 3

********************************************************************************

5. En todo este problema considera que los gases se comportan idealmente.


La ley de los gases ideales es PV = nRT, donde:
(R = 0.082 L atm / mol K), (R = 8.314 J / K mol), (1 J = kg m2 s-2)
(1 atm = 101.325 kPa), (1 Pa = 1 N m-2), (1 N = 1 kg m s-2)

El carbonato de sodio tiene un punto de fusión de 854 oC y puede obtenerse por descomposición
del bicarbonato de sodio (NaHCO3) a 270 oC; además se produce agua y CO2. Considera que a
270 oC, en un pequeño horno se descomponen 840.0 g del bicarbonato.

5 a) Calcula el volumen que ocupan los gases producidos, si la presión es de 1 atm.

El volumen expresado en litros es: (marca la casilla correcta con una “X”)

Menor a 200.0 A
Entre 200.0 y 299.0 B
Entre 300.0 y 399.0 C
Entre 400.0 y 499.0 D
Entre 500.0 y 599.0 E
Más de 599.0 F

5 b) Si posteriormente los gases obtenidos se enfrían hasta una temperatura de 25 oC, calcula
nuevamente el volumen de los gases suponiendo que la presión es constante y que la presión de
vapor del agua a esta temperatura es despreciable.

El volumen expresado en litros es: (marca la casilla correcta con una “X”)

Menor a 50.0 A
Entre 50.0 y 99.0 B
Entre 100.0 y 149.0 C
Entre 150.0 y 199.0 D
Entre 200.0 y 249.0 E
Más de 249.0 F

163
5 c) El carbonato de amonio (NH4)2CO3 se descompone a 60 oC y produce bicarbonato de amonio y
amoniaco. A su vez, el bicarbonato de amonio se descompone en amoniaco, CO2 y agua.

Calcula la cantidad en gramos de carbonato de amonio que tendrían que descomponerse para,
considerando las dos reacciones de descomposición, obtener 110.0 litros de amoniaco a una
temperatura de 60 oC y una atmósfera.

Los gramos de carbonato necesario son:

Menos de 50.0 A
Entre 50.0 y 99.0 B
Entre 100.0 y 149.0 C
Entre 150.0 y 199.0 D
Entre 200.0 y 249.0 E
Más de 249.0 F

5 d) Al obtenerse estos 110.0 litros de amoniaco también se obtienen otros productos. Si la mezcla
gaseosa se enfría hasta 25 oC, ¿cuál es la fracción mol de amoniaco en la mezcla gaseosa?

Menos de 0.40 A
Entre 0.40 y 0.50 B
Entre 0.51 y 0.60 C
Entre 0.61 y 0.70 D
Entre 0.71 y 0.80 E
Más de 0.80 F

********************************************************************************

En las siguientes preguntas, escribe en el recuadro la letra del inciso que consideras es
correcto.

6. ¿Cuál(es) de las disoluciones acuosas de las siguientes sustancias presenta(n) carácter ácido?

a) NaCl b) NH3 c) NH4Cl


d) KNO3 e) Al(NO3)3

7. El símbolo Kb de la especie HS- corresponde a la constante de equilibrio de la reacción:

a) HS- + HO-  S2- + H2O b) HS- + H2O  H2S + OH-


c) HS- + H2O  S2- + H3O+ d) HS- + H3O+  H2O + H2S
e) HS- + HS-  H2S + S2-

164
8. Si la Ka del ácido cianhídrico es 6.2x10-10 y la Kb del amoniaco es 1.8x10-5, el pH de una
disolución acuosa igual a 0.1 mol/L del cianuro amónico será:

a) pH = 7 b) pH = 6.9 c) pH < 7
d) pH = 0 e) pH = 9.2

9. En un volumen de 20 cm3 de una disolución de NaOH 2 M hay:


a) 1.6 g de NaOH b) 0.04 g de NaOH c) 0.08 g de NaOH
d) 3.2 g de NaOH e) 0.16 g de NaOH

10. Una disolución de un ácido débil monoprótico y de una de sus sales tendrá la máxima eficacia
reguladora de pH cuando:
a) El pH sea 7.
b) La concentración del ácido y de la sal sean iguales y elevadas.
c) La concentración del ácido sea elevada.
d) El pH sea alcalino.
e) La concentración de la sal sea elevada.

11. Dada la reacción de oxidación-reducción siguiente:


2MnO4- (ac) + 5H2O2 (ac) + 6H+ (ac)  2Mn2+ (ac) + 5O2 (g) + 8H2O (l), es cierto que:
a) El número de electrones puesto en juego en este proceso es de 2.
b) La especie O2 es la que resulta de la reducción de H2O2 debido al agente reductor MnO4-.
c) La especie MnO4- es el agente reductor y se oxida a Mn2+.
d) El ion MnO4- es el agente oxidante que produce la oxidación del H2O2 a O2.
e) El agua oxigenada se oxida y reduce a O2 y H2O.

12. Si los valores de potenciales normales de los sistemas Cl2/Cl- y I2/I- son 1.36 y 0.54 V,
respectivamente, podemos afirmar que:
a) El yodo oxida al ion cloruro. b) El cloro oxida al ion yoduro.
c) El cloro es más básico que el yoduro. d) El cloro reduce al ion yoduro.
e) Hay equilibrio y no hay ninguna reacción.

13. ¿Cuántos faradios son necesarios para reducir 0.20 moles de MnO4- a Mn2+ ?
(Datos: F= 96484 C)
a) 0.20 b) 3.00 c) 0.40 d) 1.00 e) 5.00

165
14. ¿Qué masa de cobre se deposita en media hora con una corriente de 2 A que pasa por una
disolución acuosa que contiene el ion Cu2+?
(Datos: F= 96484 C; Masa molar Cu = 63.5 g mol-1)
a) 0.037 g b) 1.18 g c) 2.36 g
d) 0.0198 g e) 0.0187 g

15. La solubilidad del fluoruro de magnesio en agua a 18 °C se encuentra en tablas como 0.0076 g
por 100 mL. El producto de solubilidad de esta sal es:
a) 7.3 x 10-12 b) 3.6 x 10-9 c) 7.7 x 10-9
-9 -6
d) 7.3 x 10 e) 1.5 x10

16. Los valores de pKd de los complejo de los cationes Ca2+, Mg2+, Cd2+, La3+ y Fe3+ con EDTA a
pH = 12 son respectivamente, 10.7, 8.8, 16.5, 15.5 y 25.5. De acuerdo a esta información es
cierto que:
a) El complejo más estable es el de CaY2-
b) El complejo más estable es el de FeY-
c) El complejo menos estable es el de FeY-
d) Al valor de pH 12 sólo se forma el FeY-
e) No hay diferencias de estabilidad de los complejos

17. Para determinar la concentración de una disolución de Ni2+ se tomó una alícuota de 50.00 mL de
la disolución a la que se añadieron 25.00 mL de EDTA 0.0500 mol/L; (volumen suficiente para
el complejo con todo el níquel presente en la disolución). El exceso de EDTA se valoró por
retroceso y se consumió un volumen de 10.0 mL de Zn2+ 0.025 mol/L. La concentración de
níquel en la muestra de partida es igual a:
a) 0.02 b) 0.025 c) 0.015 d) 0.010 e) 0.20

********************************************************************************

En las siguientes preguntas, escribe en el recuadro la letra del inciso que consideras es
correcto.

18. ¿Cuál de los siguientes elementos tiene el mayor radio atómico?


a) Li b) B c) N d) O e) F

19. ¿Cuál es la configuración electrónica del Pb?


a) [Xe]6s26p2 b) [Xe]4f145d106s26p2
c) [Xe]5d106s26p2 d) [Xe]6s26p66d104f10
2 6 10 14
e) [Xe]6s 6p 6d 4f

166
20. ¿Cuáles son las fórmulas del bicarbonato, el hidrógeno sulfito, el nitrato y el nitrito?
a) HCO3-, HSO3-, NO3-, NO2-
b) CO32-, H2SO3-, NO3-, NO2-
c) HCO3-, HSO3-, HNO3- NO2-
d) CO32-, HSO3-, NO3- HNO2-
e) CO32-, HSO3-, NO3-, NO2-

21. ¿Cuál de los siguientes enunciados describe correctamente a un ion Zn2+ que tiene una masa de
65 uma?
a) Contiene 28 electrones, 30 protones y 35 neutrones
b) Contiene 30 electrones, 30 protones y 35 neutrones
c) Contiene 32 electrones, 30 protones y 35 neutrones
d) Contiene 35 electrones, 35 protones y 30 neutrones
e) Contiene 37 electrones, 35 protones y 30 neutrones

22. La estructura de Lewis correcta para el CH2Cl2 contiene:


a) 2 enlaces sencillos, 2 enlaces dobles y ningún electrón de no-enlace
b) 4 enlaces sencillos y ningún electrón de no-enlace
c) 4 enlaces sencillos y 4 electrones de no-enlace
d) 2 enlaces sencillos, 2 enlaces dobles y 8 electrones de no-enlace
e) 4 enlaces sencillos y 12 electrones de no-enlace

23. En que inciso se indican correctamente los números de oxidación del: cloro en el HClO3, azufre
en el Li2SO4, manganeso en el KMnO4, fósforo en el KH2PO4 y yodo en el HI.
a) +5, +6, +7, +5, -1
b) +3, +6, +7, +5, +1
c) +6, +4, +6, +3, +1
d) -1, +4, +7, +3, +1
e) +5, +4, +7, +5, -1

24. El enlace en el CO es:


a) covalente puro
b) iónico
c) metálico
d) covalente polar
e) covalente coordinado

FIN DEL EXAMEN

167
168
XVIII OLIMPIADA NACIONAL DE QUÍMICA
TERCER EXAMEN NIVELES A y B Tiempo asignado: 180 minutos Campeche 2009

FISICOQUÍMICA

Una forma de categorizar a los combustibles fósiles con relación al calentamiento global es a través
de la energía emitida por gramo de CO2 expelido a la atmósfera. Mientras mayor sea la energía
liberada por gramo de CO2 emitido, mejor será el combustible.

La combustión de C8H18 (gasolina) libera 1309 kcal/mol, en tanto que la combustión del gas metano,
CH4, libera 213 kcal/mol y la combustión de carbón libera 94.05 kcal/mol.

1. ¿Cuántos gramos de oxígeno se requieren para la combustión de un gramo de gasolina?


A) menos de 3 gramos B) entre 3 y 4 gramos Resp

C) entre 4.1 y 5 gramos D) más de 5 gramos

2. ¿Cuántas kilocalorías se liberan por gramo de CO2 expelido a la atmósfera en el caso de la


gasolina (C8H18)?
A) menos de 2.5 kcal B) entre 2.5 y 3.5 kcal Resp

C) entre 3.6 y 4 kcal D) más de 4 kcal

3. ¿Cuántas kilocalorías se liberan por gramo de CO2 expelido a la atmósfera en el caso del gas
natural (CH4)?
A) menos de 2.5 kcal B) entre 2.5 y 3.5 kcal Resp

C) entre 3.6 y 4 kcal D) más de 4 kcal

El calentamiento de aire de una habitación requiere 1000 kcal para elevar su temperatura en 10ºC.

4. ¿Qué volumen de metano en condiciones STP (1 atm y 0 ºC) se requiere para calentar la
habitación?
A) menos de 100 L B) entre 100 y 150 L Resp

C) entre 151 y 200 L D) más de 200 L

5. ¿Cuántos gramos de CO2 se generan?


A) menos de 150 g B) entre 150 y 200 g Resp

C) entre 201 y 250 g D) más de 250 g

169
Suponiendo que el calor proviene de electricidad con 80% de eficiencia y que la electricidad fue
producida a partir de la combustión de carbón con 30% de eficiencia:

6. ¿Cuántos gramos de carbón se requieren?


A) menos de 400 g B) entre 400 y 450 g Resp
C) entre 451 y 500 g D) más de 500 g

7. ¿Cuántos gramos de CO2 se generan?


A) menos de 1500 g B) entre 1500 y 2000 g Resp

C) entre 2001 y 2500 g D) más de 2500 g

8. Si la capacidad térmica del aire en la habitación es de 6.84 cal/mol K, el volumen de aire en


metros cúbicos contenido en la habitación a una temperatura de 30 ºC y una presión de una
atmósfera es:
A) menos de 370 m3 B) entre 370 y 380 m3 Resp

C) entre 381 y 390 m3 D) más de 390 m3

9. Si el aire de la habitación contiene 20% de oxígeno y 80% de nitrógeno, la cantidad de sustancia


en moles contenidos en un m3 de aire es:
A) menos de 25 mol B) entre 25 y 35 mol Resp

C) entre 36 y 45 mol D) más de 0.045 mol

10. Los gramos de oxígeno contenidos en un litro de aire son:


A) menos de 0.2 g B) entre 0.2 y 0.3 g Resp

C) entre 0.31 y0.4 g D) más de 0.4 g

QUÍMICA ANALÍTICA (Sólo Nivel B)

11. La concentración de iones OH- en una disolución de NaOH es dos veces la concentración analítica
de la sosa. La concentración de NaOH en la disolución y el valor de su pH es igual a:

Cálculo

Concentración de NaOH = mol/L Valor de pH=

170
12. En condiciones adecuadas, el fenol (de fórmula C6H5OH) puede ser tratado con una mezcla de
ácido nítrico y sulfúrico para dar un compuesto A de fórmula (C6H5-xOH(NO2)x. El compuesto A
es un sólido cristalino de color amarillo cuya solubilidad en agua es de 7,9 g de A en 100 mL de
agua destilada y que se comporta en disolución como un ácido más fuerte que el fenol. La
titulación de una muestra de 220 mg del ácido A requiere, para su neutralización, 10,0 mL de
una disolución de KOH 0,100 mol/L; un curioso estudiante de olimpiada midió los valores de pH
de la disolución de A cuando había añadido diversos volúmenes de KOH y observó que cuando
había añadido un volumen de 9,0 mL de KOH el pH era igual a 6.

a) Indica cuál es la fórmula condensada del compuesto A y los valores de pH que debió haber
medido después de añadir 10,00 y 20,00 mL de KOH 0,100 mol/L.

La fórmula es: ____________________________

El valor de pH cuando se añadieron 10,0 mL es: ___________________________

El valor de pH cuando se añadieron 20,0 mL es: ___________________________

13. Una muestra compleja contiene dos sales: cloruro de magnesio y nitrato de bario. Para
determinar el porcentaje de cada una de ellas en la mezcla se midieron dos muestras idénticas de
2,5 g cada una y se siguió el procedimiento que se describe a continuación:

a) La primera muestra se disolvió en agua destilada y se le añadió un exceso de ácido sulfúrico. El


precipitado de BaSO4 obtenido se filtró, lavó y secó; la masa medida fue de 1,12 g.

b) La segunda muestra se disolvió en agua destilada completando el volumen a 250,0 mL. De esta
disolución se tomaron alícuotas de 25,00 mL y se titularon con una disolución de nitrato de plata
0,100 mol/L. El volumen promedio de nitrato de plata fue de 26,3 mL.

Indica las reacciones involucradas en el procedimiento seguido y calcula el porcentaje en masa de


cada uno de los componentes de la mezcla

Datos: Masas molares: BaSO4 = 233,4; MgCl2 = 95,2; Ba(NO3)2 = 261,3; AgCl = 143,35.

Las reacciones involucradas son:

El porcentaje en masa de cada una de las sales es:

171
QUÍMICA ANALÍTICA (Sólo Nivel A)

14. En una disolución la concentración de iones OH- es dos veces la concentración de NaOH. Calcula la
concentración de NaOH en la disolución y el valor de su pH.

Cálculo

Concentración de NaOH = mol/L Valor de pH=

15. En condiciones adecuadas, un ácido desconocido A puede ser preparado por nitración del fenol.
El compuesto A es un sólido cristalino cuya masa de carbono es menor del 45%. Se conoce que
A en disolución se comporta como un ácido débil y que su solubilidad es de 5,60 g en 100 mL de
agua destilada. La titulación de una muestra de 183 mg del ácido A en 10 mL requiere, para su
neutralización, 11,05 mL de una disolución de KOH 0,090 mol/L; un curioso estudiante de
olimpiada midió los valores de pH de la disolución de A cuando había añadido diversos
volúmenes de KOH y observó que cuando había añadido un volumen de 5,5 mL de KOH el pH
era igual a 3,6.

b) Indica cuál es la masa molar del compuesto A y dibuja su estructura.

c) El nombre IUPAC del compuesto A es: ___________________________________________

d) La solubilidad del ácido A expresada en mol L-1 es igual a:

e) ¿Cuál es el valor de pKa del ácido A? __________________________________________

f) Calcula los valores de pH que el curioso estudiante olímpico debió haber medido antes de iniciar
la titulación de la muestra, cuando éste se neutralizó completamente y cuando añadió un exceso
de KOH del 10 %. Completa la tabla indicando los volúmenes que corresponden a dichos valores
de pH.

172
Volumen de KOH (mL) Cálculo y valor de pH medido

16. Una muestra compleja contiene tres sales: cloruro de bario, cloruro de magnesio y nitrato de
bario. Para determinar el porcentaje de cada una de ellas en la mezcla se midieron dos muestras
idénticas de 2,13 g cada una y se siguió el procedimiento que se describe a continuación:

c) La primera muestra se disolvió en agua destilada y se le añadió un exceso de ácido sulfúrico. El


precipitado de BaSO4 obtenido se filtró, lavó y secó; la masa medida fue de 1,17 g.

d) La segunda muestra se disolvió en agua destilada completando el volumen a 250,0 mL. De esta
disolución se tomaron alícuotas de 25,00 mL y se titularon con una disolución de nitrato de plata
0,1000 mol/L. El volumen promedio de nitrato de plata fue de 25,0 mL.

Indica las reacciones involucradas en el procedimiento seguido y calcula la masa (en gramos) de
cada uno de los componentes de la mezcla.

Datos:
Masas molares: BaCl2 = 208,2; BaSO4 = 233,4; MgCl2 = 95,2; Ba(NO3)2 = 261,3; AgCl = 143,35.

173
QUÍMICA INORGÁNICA (Sólo Nivel B)

17. Completa el siguiente crucigrama y escribe las correspondientes respuestas en la línea marcada
después de cada indicación horizontal o vertical:
1 2 3

4 5 6
7 8 9
10 11

12

13
14
15 16 17
18
19
20 21

Horizontales Verticales
1. Símbolo del elemento muy abundante 1. Sistema internacional de unidades:
en la corteza terrestre: __________ _____
2. Unidad que mide la actividad 3. Unidad de masa atómica: ________
radioactiva: ____________ 4. Elemento de transición que pertenece
6. Sustancias utilizadas para neutralizar al cuarto periodo: ____________
a los ácidos: ___________ 5. País donde se efectuará la XV IChO:
7. En desorden, las letras del ion con _______________
carga negativa: ______________ 7. Se dice de una disolución que no es ni
10. La geometría de moléculas de CO2 ácida ni básica: ____________
es: ________________ 8. Símbolo del elemento necesario para
13. Terminación de sales de oxiácidos: la respiración: ____________
______________ 9. Sustancia que en disolución es
14. Símbolo de un gas noble: _______ conductora de la corriente eléctrica:
16. Fórmula alotrópica del fósforo: ________________
_____________ 11. En una pila las oxidaciones se
18. Se denominan así a los metales del realizan en el: _______________
primer grupo de la tabla periódica: 12. La sal de un hidrácido se llama:
_______________ ________________
20. Cuando los átomos tienen el mismo 15. Elemento que se forma cuando el
número de protones pero diferente radio emite una partícula alfa: ______
de neutrones presentan una relación 17. Símbolo del elemento cuya
de: _______________ configuración electrónica es
(Xe)4f145d66s2: ____________
19. Sin forma ni volumen: __________
21. Al revés, símbolo del elemento
utilizado en joyería: _____________

174
QUÍMICA INORGÁNICA (Niveles A y B)

En las siguientes preguntas, marca con una “X” la respuesta correcta.

18. ¿Cuál de los siguientes elementos tendrá la mayor energía de ionización?


a) B b) C c) N d) O e) Al

19. ¿Cuál de los siguientes átomos tiene el radio más pequeño?


a) Na b) Mg c) Si d) K e) Al

20. ¿Cuál de los siguientes iones tiene el menor radio atómico?


a) Na+ b) Mg2+ c) S2- d) Cl- e) Al3+

21. En que inciso se encuentran las fórmulas de los siguientes compuestos: fluoruro de litio, nitrato
de manganeso (II), cloruro de mercurio (I), pentóxido de dinitrógeno, ácido nítrico.
a) LiF, Mn(NO3)2, Hg2Cl2, N2O5, HNO3
b) FLi, Mn3N2, ClHg, O5N2, NH3
c) LiF, Mg(NO3)2, HgCl, N2O5, HNO3
d) FLi, Mg3N2, HgCl, NO5, H3N
e) LiF, MnNO3, Hg2Cl2, N2O5, HNO3

22. Calcula el número de protones en un átomo de potasio promedio si la masa atómica del potasio
es 39 uma y el número atómico de este elemento es 19.
a) 19 b) 20 c) 39 d) 58
e) Es imposible de determinar el número de protones de la información proporcionada

23. En un análisis por combustión una muestra de 1.532 g de un compuesto que contiene únicamente
carbono e hidrógeno produce 4.807 g de CO2 y 1.968 g de H2O. ¿Cuál es la fórmula empírica del
compuesto?
a) C2H b) CH c) CH2 d) C2H3 e) C5H2

24. ¿Cuántos electrones de valencia hay en la estructura de Lewis del ion sulfato, SO42-?
a) 32 b) 31 c) 30 d) 17 e) 16

25. ¿En cuál de los siguientes compuestos el enlace es más iónico?


a) BeF2 b) CaCl2 c) SrBr2 d) BaF2 e) CaI2

26. ¿En cuál de los siguientes compuestos el enlace es menos polar?


a) NaCl b) CH4 c) CO d) Cl2 e) HCl

27. La estructura de Lewis correcta para el ion NO2+ tiene:


a) 2 enlaces dobles y 8 electrones de no enlace
b) 1 enlace doble, 1 enlace sencillo y 12 electrones de no enlace
c) 2 enlaces sencillos y 12 electrones de no enlace
d) 2 enlaces triples y 4 electrones de no enlace
e) 1 enlace doble, 1 enlace sencillo y 8 electrones de no enlace

175
28. ¿En qué compuesto se encuentran los electrones del azufre apuntando hacia las esquinas de un
tetraedro?
a) SF3+ b) SF4 c) SF5 d) SF6 e) ambos SF4 y SF6

29. A continuación se enlistan algunas propiedades de una muestra de azufre sólido: i) Sólido
cristalino quebradizo; ii) Punto de fusión de 113.0 0C; iii) Densidad de 2.1 g/cm3; iv) Se combina
con el oxígeno para formar dióxido de azufre.
¿Cuál, si hubiera alguna, de estas propiedades sería la misma para un solo átomo de azufre
proveniente de la muestra?
a) Únicamente i y ii.
b) Únicamente iii y iv.
c) Únicamente iv.
d) Todas las propiedades serían las mismas.
e) Ninguna de las propiedades sería la misma.

30. ¿Cuál de los siguientes elementos formaría compuestos con las siguientes fórmulas Na2X, H2X,
XO2 y XF6?
a) B b) C c) N d) O e) S

31. El nitrógeno tiene un número de oxidación razonable en todos los siguientes compuestos; sin
embargo, uno de ellos no es posible, ¿cuál?
a) NF3 b) NF5 c) NO3 d) NO2 e) NO

32. Para la reacción:


3 Sn2+ (ac) + Cr2O72- (ac) + 14 H+ (ac)  3 Sn4+ (ac) + 2 Cr3+ (ac) + 7 H2O (l)
¿Cuál de las siguientes afirmaciones es correcta?
a) ambos, el Sn2+ y el H+ son agentes oxidantes
b) el Cr2O72- es el agente oxidante
c) el Sn2+ se reduce
d) el ácido no es importante en la reacción
e) no es posible que se lleve a cabo la reacción

176
QUÍMICA ORGÁNICA (Sólo Nivel A)

33. Completa la siguiente secuencia de reacciones:

OH
1) NaOH (H2O) A Zn, HCl B
(C8H9NO3) (C8H11NO)
2)
Br
NO2 NaNO2 + HCl
5oC
1) Ph OH
D NaOH C
( C14H14N2O2) 2) HCl, H O (C8H9ClN2O)
2

1) NaOH (H2O)
2) Et2SO4

E
(C16H18N2O2)

ESTRUCTURA ESTRUCTURA ESTRUCTURA


COMPUESTO A COMPUESTO B INTERMEDIARIO C

ESTRUCTURA ESTRUCTURA
COMPUESTO D COMPUESTO E

177
34. Escribe la estructura del producto principal de cada una de las siguientes reacciones:

O
Cl Zn(Hg)
a) A B
AlCl3 HCl
CALOR
O O
HNO3 Zn Cl
b) C D E
H2SO4 HCl
CALOR N
O OEt
EtONa
c) O F
EtOH
OEt
O
H C H
d) G
O CH3
H N
CH3
AcOH

ESTRUCTURA ESTRUCTURA ESTRUCTURA


COMPUESTO A COMPUESTO B COMPUESTO C

ESTRUCTURA ESTRUCTURA ESTRUCTURA


COMPUESTO D COMPUESTO E COMPUESTO F

ESTRUCTURA
COMPUESTO G

178
35. Indica la estructura del producto principal de cada una de las siguientes reacciones de sustitución
electrofílica aromática.

HNO3
a) A
H2SO4
CALOR

O O
b) Cl
N B
H AlCl3
CALOR

F
c) Cl AlCl3
+ C
CALOR

CH3
OH Br2
d) D
CH3 CHCl3

ESTRUCTURA ESTRUCTURA
COMPUESTO A COMPUESTO B

ESTRUCTURA ESTRUCTURA
COMPUESTO C COMPUESTO D

179
36. ¿Cuáles de los siguientes compuestos son aromáticos?

CH3
N N
CH2 N N
N S H3C CH3
A B C D E

N
N
N Br
Br N N N
H3C CH3 CH3
F G H I J

RESPUESTA

FIN DEL EXAMEN

180
XVIII OLIMPIADA NACIONAL DE QUÍMICA
EXAMEN INTERNACIONAL
9 PROBLEMAS Tiempo: 4 HORAS Campeche 2009

Química Orgánica

1. Escriba la estructura del producto principal de cada una de las siguientes reacciones:

CHO
H OH HNO3
A
H OH
OH
Br2 / H2O

NaBH4, CH3OH
C

Indique cuál (o cuáles) de los productos (A, B y/o C) son ópticamente activos.

ESTRUCTURA ESTRUCTURA ESTRUCTURA


COMPUESTO A COMPUESTO B COMPUESTO C

COMPUESTO A COMPUESTO B COMPUESTO C


¿PRESENTA ¿PRESENTA ¿PRESENTA
ACTIVIDAD ACTIVIDAD ACTIVIDAD
ÓPTICA? ÓPTICA? ÓPTICA?

181
2. Complete la siguiente secuencia sintética:

O
A CrO3 C
KOH
H H2SO4
(C10H16O2) CH3OH (C10H14O2)
0 oC

B H
N N
H

D + N N
(C11H16O2)

1) 2) H2O2, NaOH

B H

F TsOH E
(C10H14O2) (C11H18O3)

TsOH = SO3H

ESTRUCTURA ESTRUCTURA ESTRUCTURA


COMPUESTO A COMPUESTO C COMPUESTO D

ESTRUCTURA ESTRUCTURA
COMPUESTO E COMPUESTO F

182
3. Un compuesto A, C11H20O4, es un compuesto neutro ópticamente activo y el cual no reacciona
con la 2,4-dinitrofenilhidrazina o con una disolución de bromo en tetracloruro de carbono. Al
hidrolizar el compuesto A, se obtienen tres nuevos compuestos B, C y D. El compuesto B tiene
la fórmula molecular CH4O y como debe ser obvio para usted es ópticamente inactivo. El
compuesto C, C5H8O4, reacciona con una disolución de bicarbonato de sodio desprendiéndose
un gas y presenta actividad óptica. Al calentar el compuesto C este se convierte en el compuesto
E, C5H6O3, el cual es un compuesto neutro y con actividad óptica. Cuando el compuesto E se
trata con agua, este reacciona lentamente para volver a formar el compuesto C, ópticamente
activo. El compuesto D, C5H12O, es ópticamente inactivo y cuando se trata con ácido crómico
se obtiene el compuesto F, C5H10O. Cuando el compuesto F se trató con el clorhidrato de
hidroxilamina formó un precipitado, e igualmente reaccionó con la 2,4-dinitrofenilhidrazina. Sin
embargo, cuando el compuesto F se trató con el complejo Ag(NH3)2+ no formó el espejo de
plata. Cuando el mismo compuesto F se trató con el hipoyodito de sodio en presencia de
hidróxido de sodio, no formó precipitado alguno.

Proponga estructuras para los compuestos A al F, indicando con claridad el razonamiento que
siguió para llegar a la estructura propuesta. Tenga en cuenta que debe indicar la fuente de
quiralidad en los compuestos que presenten la actividad óptica.

ESTRUCTURA ESTRUCTURA ESTRUCTURA


COMPUESTO A COMPUESTO B COMPUESTO C

ESTRUCTURA ESTRUCTURA ESTRUCTURA


COMPUESTO D COMPUESTO E COMPUESTO F

183
4. Complete la siguiente secuencia de reacciones:
O
Br Mg 1) SOCl2
A B C
Et2O 2) HCl, H2O CALOR

1) NaNH2
D
2) C

H2, Pd (C), S8 (Lindlar)

E
O
O
O H

Cl

ESTRUCTURA ESTRUCTURA ESTRUCTURA


COMPUESTO A COMPUESTO B COMPUESTO C

ESTRUCTURA ESTRUCTURA
COMPUESTO D COMPUESTO E

ESTRUCTURA
COMPUESTO F

184
5. Dé la estructura de los productos principales de cada una de las siguientes reacciones:
H3C CH3
1) OXIMERCURACIÓN
a) H3C C H
A
H H 2) DESMERCURACIÓN

EN ESTE INCISO SE USA H2O COMO DISOLVENTE, Y


ADEMÁS SE DEBEN DE ESCRIBIR LOS REACTIVOS NECESARIOS

H3C CH3
H2SO4, H2O
b) C H
H3C B
H H

H3C CH3
c) 1) BH3, THF
C H
H3C C
H H 2) H2O2, NaOH

H3C CH3
d) C H HBr, Et2O
H3C D
H H

H3C CH3
C H HBr, Et2O
e) H C
3 E
H H O O
Ph C O O C Ph

REACTIVO REACTIVO ESTRUCTURA ESTRUCTURA


OXIMERCURACIÓN DESMERCURACIÓN COMPUESTO A COMPUESTO B

ESTRUCTURA ESTRUCTURA ESTRUCTURA


COMPUESTO C COMPUESTO D COMPUESTO E

185
Fisicoquímica

6. En los países desarrollados de todo el mundo, el elevado número de accidentes de tráfico que
tienen una relación directa con el excesivo consumo de bebidas alcohólicas es muy preocupante.
Por esta razón, han sido diseñados dispositivos específicos para que los policías de tránsito
controlen los niveles de alcohol ingeridos por los conductores y sean sancionados aquellos que
superen los niveles decretados por ley.

En un informe publicado por Lijestrand y Linde en 1930, se discute la relación entre la


concentración en sangre y en aliento. Se muestra que la cantidad de etanol contenida en 2 litros
de aliento era aproximadamente la misma que la contenida en 1 mL de sangre.

En la actualidad los límites máximos de alcohol permitidos en los países europeos son:
 Conductores en general: 0.25 mg de alcohol/L de aire (0.50 g/L en sangre)
 Conductores profesionales y principiantes: 0.15 mg de alcohol/L en aire (0.30 g/L en sangre)

La capacidad media pulmonar de una persona sedentaria es de unos 4000 mL aproximadamente


aunque en deportistas de élite puede llegar a alcanzar los 6500 mL. Sin embargo, durante la
respiración normal sólo se intercambian aproximadamente 700 mL de aire por cada aspiración-
espiración (14 respiraciones por minuto en reposo).

Si la cantidad total de sangre de una persona adulta es de 5.0 L, su capacidad pulmonar es de


4000 mL y la prueba del alcoholímetro es 0.25 mg/L de aire, entonces:

6.1. Los gramos de alcohol contenidos en sangre son:


A) menos de 0.05 g/L B) entre 0.05 y 0.15 g/L Resp

C) entre 0.16 y 1.5 g/L D) más de 1.5 g/L

6.2. Los miligramos de alcohol eliminados en la respiración en 10 minutos son:


A) menos de 2.5 mg/L B) entre 2.5 y 15 mg/L Resp

C) entre 15.1 y 30 mg/L D) más de 30 mg/L

Una persona dio una prueba del alcoholímetro de 0.75 mg/L de aire (el triple del máximo
permitido). Se dice que por cada copa de alcohol se debe esperar una hora para eliminarlo y
poder conducir un automóvil.

Supón que la desaparición de alcohol en sangre sigue una cinética de primer orden y que después
de una hora se elimina sólo el 20% del alcohol en sangre.

6.3. La constante de rapidez de eliminación de alcohol en sangre es:


A) menor a 0.2 h – 1 B) entre 0.2 y 0.3 h – 1 Resp

C) entre 0.31 y 0.4 h – 1 D) mayor que 0.4 h – 1

186
6.4. La cantidad de alcohol presente en sangre después de 90 minutos:
A) menos de 1.0 g/L B) entre 1.0 y 1.2 g/L Resp

C) entre 1.21 y 1.4 g/L D) más de 1.4 g/L

6.5. El tiempo necesario para que la persona alcance el máximo permitido de alcohol en sangre y
pueda conducir:
A) menos de 3 horas B) entre 3 y 4 horas Resp

C) entre 4 y 5 horas D) más de 5 horas

xxxxxxxxxxxxxxxxxxxxxxxxxxxxxxxxxxxxxxxxxxxxxxxxxxxxxxxxxxxxxxxxxxxxxxxxxxxxxxxx

7. Las ecuaciones que permiten conocer la presión de vapor en función de la temperatura suelen tener
la forma

Ln P = − (A / T ) + B

Donde la P está expresada en mmHg, T es la temperatura en K y las constantes A y B son


características de cada sustancia.

7.1. Indica cuáles deben ser las unidades de la constante A: ________________

7.2. Para un hidrocarburo X las constantes son A = 7000, B = 24


Para un hidrocarburo Z las constantes son A = 6800, B = 24.5
(Tú debes asignar las unidades correspondientes)

Para la sustancia X, calcula la temperatura normal de ebullición y la temperatura de


ebullición en una ciudad donde la presión atmosférica es de 585 mm Hg

Tnormal X _____________ oC T585 X _______________ oC

Tus valores deben tener una precisión de 0.1 K

7.3. Si se hace una mezcla equimolar de X y Z y suponemos comportamiento ideal*, calcula la


temperatura normal de ebullición de esta mezcla.

T normal mezcla _____________ oC

*En una mezcla ideal se puede usar la Ley de Raoult: P = P0 x

187
Química Analítica

8. La conductimetría es una técnica analítica que permite determinar la concentración total de todos los
iones presentes en una disolución. Se basa en la medida de la conductividad eléctrica  (opuesto de la
resistencia) de la disolución. En concentraciones bajas la constante de proporcionalidad entre la
conductividad y la concentración se llama conductividad equivalente límite (o) y es una constante
para cada ion. La expresión que relaciona a ambas es la siguiente:
=o C
en donde  es la conductividad eléctrica y se expresa en S cm-1 (Siemens/cm); o es la
conductividad equivalente del ion y se expresa en S eq-1 cm2, y C es la concentración del ion,
expresada en equivalentes / cm3 (carga del ion x concentración en mol/cm3). El valor de o es una
constante que depende del ion, el disolvente y la temperatura. La conductividad equivalente límite
de un electrolito se denota como o y es igual a la suma de las conductividades equivalentes
límites de los iones que lo componen.

Cuando en la disolución existen diferentes electrolitos, la conductividad total depende de las


conductividades individuales de todos los iones presentes, es decir:
 ൌ ෍ሺ௢௜ ‫ܥ‬௜ ሻ

Una de las aplicaciones de la conductimetría es para detectar el punto final en titulaciones ácido
base ya que, dependiendo de los electrolitos presentes, se pueden observar rectas con diferentes
pendientes y los cambios de pendientes coinciden con puntos de equivalencia de las reacciones.

En la siguiente figura se muestra la curva de titulación que obtuvo un brillante olímpico de


Campeche al valorar 10 mL de una mezcla de ácido clorhídrico y un ácido débil (simbolizado
como HA, pKa = 5.0) con NaOH de concentración 0.1 mol/L.

Valoración de mezcla de ácidos


 (mS)
60

50 48.5

40

30
25.6
20
14.9
10
7.6
0
0 5 10 15 20 25 30
Volumen de NaOH (mL)

188
a) Indica las reacciones que se efectúan para la titulación de la mezcla.

b) Calcula la concentración de HCl (en mol/L).

c) Calcula la concentración de HA (en mol/L).

d) Completa la siguiente tabla indicando cuál(es) es (son) el (los) electrolito(s) causante(s) de la


conductividad de la disolución en los diferentes volúmenes indicados.
Volumen de NaOH Electrolito(s)
0
6
14
28

e) Indica los valores que el brillante olímpico de Campeche obtuvo de cada uno de los electrolitos
de los que depende la conductividad de la disolución en las diferentes etapas de la titulación.
Electrolito Valor de o (en S eq-1 cm2)

189
Química Inorgánica

9. La termogravimetría es una técnica analítica que consiste en medir la pérdida de masa de una
sustancia mientras es calentada.

La siguiente gráfica muestra los cambios de masa registrados para una muestra de sulfato de cobre
(II) pentahidratado durante su calentamiento. La descomposición ocurre en las zonas de cambio
brusco, generando los productos A a F indicados en el diagrama.

0
Porcentaje de masa perdida

10 A
20
B
30 C
40

50 D

60
E
70
F

80

90

100
200 400 600 800 1000
Temperatura / 0C

a) Usando los datos de la gráfica sugiere una fórmula para los compuestos A, B y C.

A: __________________________ B: __________________________

C: __________________________

b) Durante el calentamiento de E ocurre una reacción redox. Identifica a E y F, y escribe la


ecuación química para esta reacción.

E: __________________________ F: __________________________

Reacción: ________________________________________________________

c) El compuesto D se forma cuando la mitad de C se ha descompuesto para formar E. ¿Cuál es la


fórmula empírica de D?

D: __________________________

FIN DEL EXAMEN

190
XVIII Olimpiada Nacional de Química
Examen Experimental de Fisicoquímica Campeche 2009

Cinética de una reacción.

Considera las siguientes reacciones:

(Forma iónica en disolución)


2 KI + K2S2O8 → I2 + 2 K2SO4 2 I- + S2O82- → I2 + 2 SO42-
I2 + 2 Na2S2O3 → 2 NaI + Na2S4O6 I2 + 2 S2O32- → 2 I- + S4O62-

 En la primera reacción el ion S2O82- oxida al ion I- liberando al yodo I2.


 En la segunda reacción el yodo liberado es reducido a I- por el ion S2O32-.
 La segunda reacción es mucho más rápida que la primera.
 Por esta razón, mientras haya Na2S2O3 el yoduro se regenera manteniendo su concentración
constante.
 Una vez que se haya agotado el Na2S2O3, el yodo I2, ya no se consume y da una coloración
azul en presencia de almidón.
________________________________________________________________________________

La rapidez de una reacción depende de las concentraciones de los reactivos. La ley de rapidez para
la primera reacción es:
v = k[KI][K2S2O8]

Si la [KI] permanece constante, la ley de rapidez puede escribirse como:

v = kps[K2S2O8]

siendo de primer orden respecto al persulfato de potasio.


________________________________________________________________________________

En este problema vas a determinar en forma gráfica la constante de rapidez de reacción kps para la
oxidación del yoduro de potasio con persulfato de potasio.

Material
2 tubos de ensayo de 10 mm x 100 mm.
Vaso de precipitados de 100 mL.
1 cronómetro.

En tu mesa de trabajo encontrarás 3 frascos goteros con los siguientes reactivos:

Etiqueta color Contenido


VERDE LIMÓN KI 0.2 M
ROSA K2S2O8 0.1 M
NARANJA Na2S2O3 0.1 M

Además, en la mesa de trabajo habrá un frasco gotero que contiene una disolución de almidón.

191
Procedimiento.
1. En un tubo de ensayo deposita 10 gotas de la disolución de KI 0.2 M.
2. Adiciona una gota de disolución de Na2S2O3 0.1 M y una gota de disolución de almidón.
3. Adiciona con cuidado y lo más rápidamente posible 10 gotas de la disolución de K2S2O8 0.1 M y
al terminar esta adición acciona el cronómetro y agita inmediatamente. Esto es muy importante
porque en cuanto se mezclan los reactivos da inicio la reacción.
4. Observa el tubo de ensayo y en el momento en que aparezca el color azul registra el tiempo y SIN
DETENER EL CRONÓMETRO, agrega inmediatamente una gota más de Na2S2O3 0.1 M y
agita.
5. Repite el paso número 4 hasta completar la tabla siguiente.

Número de gotas de cada disolución Tiempo transcurrido


Dato KI K2S2O8 Almidón Na2S2O3 “t”/s
0.2 M 0.1 M (disolución) 0.1 M
1
2
3
4
5

Importante: Como es lógico, al ir agregando las gotas de tiosulfato de sodio, el volumen total
del sistema reaccionante irá aumentando. Sin embargo, para facilitar tus cálculos,
considera que el columen medio total permanece constante y corresponde al de 24
gotas.

Completa la tabla siguiente:

Concentración inicial de [K2S2O8]0= ______ M. Concentración inicial de [KI]0= ______ M.


Concentración de Concentración de
Dato “t”/s K2S2O8 que ha persulfato de potasio sin Ln [K2S2O8]
reaccionado reaccionar [K2S2O8]
1
2
3
4
5

Construye la gráfica de Ln [K2S2O8] vs. t (tiempo en segundos) y determina la constante de rapidez


kps. Tu gráfica será calificada y por lo tanto deberá contener todos los elementos necesarios para
obtener la máxima puntuación.

Constante de rapidez, kps = ___________________ (Indica las unidades).

192
XVIII Olimpiada Nacional de Química
Examen Experimental de Química Analítica. Nivel A Campeche 2009

Al laboratório de alimentos del CENMART han llegado unas muestras de vitamina C de las que se
desconoce si se encuentran en su forma ácida o en forma de ascorbato. Tu labor en esta mañana será
ayudarnos a identificar la forma en que se encuentra la muestra y, a su vez, determinar su pureza.

El ácido ascórbico (C6H8O6) es un ácido orgánico diprótico cuyos valores de pKa son 4,2 y 11,3 y
cuya estructura se muestra a continuación. Dicha estructura fue determinada por primera vez por
Walter Haworth quien logró con ello obtener el premio Nobel en 1937.

HO
O
O
HO

HO OH
Estructura del ácido ascórbico

Los ascorbatos son polvos que se obtienen haciendo reaccionar el ácido ascórbico con carbonatos
minerales, en presencia de bióxidos de carbono y en disolución acuosa. Tanto el ácido ascórbico
como sus sales minerales son antioxidantes igualmente absorbibles por el organismo, pero las sales
minerales son menos ácidas que el ácido y, por lo tanto, se recomiendan a personas con problemas
gastrointestinales. Es también importante señalar que en los ascorbatos se añade una dosis del
elemento mineral correspondiente.

Los ascorbatos minerales se encuentran en diferentes formas y los más comunes son de calcio,
sodio, potasio y magnesio (aunque existen algunos de zinc y molibdeno). Los ascorbatos se utilizan
como conservadores en la industria de alimentos.

Los lotes que han llegado al CENMART pueden ser del ácido ascórbico o de algún ascorbato. Para
clasificarlos y determinar su pureza haremos uso de sus propiedades reductoras que hacen posible
que se oxiden. Las correspondientes reacciones de oxidación son:

Para el ácido ascórbico C6H8O6 - 2e- - 2H+ → C6H6O6

Y para los ascorbatos (C6H7O6)xMx+ - 2xe- - 2xH+ → xC6H5O6 + Mx+

Para adelantar el trabajo hemos preparado disoluciones midiendo cantidades conocidas de cada una
de las diferentes muestras comerciales de la vitamina C. Cada una de las muestras ha sido disuelta
en agua destilada y hemos completado con agua hasta obtener un volumne de 1,00 L. De esta
disolución se te proporciona un volumen aproximado de 25 mL. La masa exacta medida se indica en
el recipiente que contien la disolución de tu muestra. Por la urgencia de los resultados y para
adelantar tu trabajo, los profesores del CENMART han medido el pH de todas las disoluciones y
han encontrado que todas tienen un valor de pH cercano a 7,4.

193
En un vaso de precipitados encontrarás una disolución de yodo (a la que se ha añadido un exceso de
yoduro de potasio), cuya concentración deberás determinar previamente con la disolución de
tiosulfato de sodio de concentración conocida, contenida en el otro vaso de precipitados. Una vez
normalizada la disolución de yodo podrás utilizarla como oxidante para identificar y determinar la
pureza de tu muestra de vitamina.

PROCEDIMIENTO.
Titulación de la disolución de yodo.

1. Llena la bureta con la disolución que usarás como titulante (yodo o tiosulfato de sodio).
2. Toma, con la pipeta columétrica, una alícuota de 5,00 mL de la otra disolución (tiosulfato de
sodio o yodo).
3. Titula tu disolución contenida en el matraz erlenmeyer utilizando almidón como indicador para
detectar el punto de equivalencia. Añade el almidón en el momento que consideres sea el
adecuado de aucerdo al titulante que hayas elegido (de acuerdo con los incisos 1 y 2). Recuerda
que una coloración azul se debe a la formación de un complejo entre el almidón y el yodo en
exceso.
4. Repite los pasos 2 y 3 y realiza la titulación por triplicado.

Titulación de la muestra problema.

1. Llena la bureta con la disolución de yodo/yoduro.


2. Toma, con la pipeta volumétrica, una alícuota de 5,00 mL de la disolución problema y colócala en
el matraz erlenmeyer, añádele un volumen aproximado de 10 mL de agua destilada.
3. Titula tu muestra problema con la disolución de yodo/yoduro utilizando el alimidón como
indicador (añadido en el momento que consideres oportuno).
4. Repite los pasos 2 y 3 y realiza la titulación por triplicado.

HOJA DE RESULTADOS.

Código: __________ Problema No. ______________

Titulación de la disolución de yodo.

1. ¿Cuál es la reacción efectuada en esta titulación?

Alícuota de ________________ Volumen de disolución añadida


de __________________
5,0 mL
5,0 mL
5,0 mL

Volumen de titulante que utilizarás en tus cálculos: __________________

2. ¿Cuál es la concentración (en mol/L) de la disolución de yodo?


Cálculos:

Concentración _______ mol/L

194
Titulación de la disolución problema.

Volumen de la disolución Volumen gastado de la


problema disolución de yodo/yoduro (mL)
5,0 mL
5,0 mL
5,0 mL

3. Volumen de yodo/yoduro que utilizarás en tus cálculos: ____________

4. Indica cuál es la reacción de titulación que se efectúa cuando el ascorbato tiene la fórmula
C6H7O6M: ____________________________________________________________________

o C12H14O12M: _________________________________________________________________

5. De acuerdo con el pH medido, tu muestra contiene la vitamina en forma de:

( ) ácido ( ) sal

6. ¿Cuál hubiera sido el valor del pH si la muestra estuviera en la otra forma posible? ___________

7. ¿Cuál es la reacción de titulación de tu muestra con la disolución de yodo?


_____________________________________________________________________________

8. ¿Cuál es la concentración (en mol/L) de la disolución de la muestra?


Cálculos:

Concentración ______ mol/L

9. De acuerdo con la masa de sólido medida con anterioridad (indicada en la etiqueta de tu


problema) la muestra contiene: ________________________________

10. La pureza de la muestra que analizaste es:


Cálculos:

11. En una escala de potencial electroquímico, indica la posición relativa de los pares I2/I- y
C6H6O6/C6H8O6.
_____________________________________________________________________________

12. De acuerdo con tus resultados ¿cuál es el porcentaje del catión metálico presente en una muestra
pura del ascorbato identificado?

195
196
XVIII Olimpiada Nacional de Química
Examen Experimental de Química Analítica. Nivel B Campeche 2009

Al laboratório de alimentos del CENMART han llegado unas muestras de vitamina C de las que se
desconoce si se encuentran en su forma ácida o en forma de ascorbato. Tu labor en esta mañana será
ayudarnos a identificar la forma en que se encuentra la muestra y, a su vez, determinar su pureza.

El ácido ascórbico (C6H8O6) es un ácido orgánico diprótico cuyos valores de pKa son 4,2 y 11,3 y
cuya estructura se muestra a continuación. Dicha estructura fue determinada por primera vez por
Walter Haworth quien logró con ello obtener el premio Nobel en 1937.
HO
O
O
HO

HO OH
Estructura del ácido ascórbico

Los ascorbatos son polvos que se obtienen haciendo reaccionar el ácido ascórbico con carbonatos
minerales, en presencia de bióxidos de carbono y en disolución acuosa. Tanto el ácido ascórbico
como sus sales minerales son antioxidantes igualmente absorbibles por el organismo, pero las sales
minerales son menos ácidas que el ácido y, por lo tanto, se recomiendan a personas con problemas
gastrointestinales. Es también importante señalar que en los ascorbatos se añade una dosis del
elemento mineral correspondiente.

Los ascorbatos minerales se encuentran en diferentes formas y los más comunes son de calcio,
sodio, potasio y magnesio (aunque existen algunos de zinc y molibdeno). Los ascorbatos se utilizan
como conservadores en la industria de alimentos.

Los lotes que han llegado al CENMART pueden ser del ácido ascórbico o de algún ascorbato. Para
clasificarlos y determinar su pureza haremos uso de sus propiedades reductoras que hacen posible
que se oxiden. Las correspondientes reacciones de oxidación son:

Para el ácido ascórbico C6H8O6 - 2e- - 2H+ → C6H6O6


Y para los ascorbatos (C6H7O6)xMx+ - 2xe- - 2xH+ → xC6H5O6 + Mx+

Para adelantar el trabajo hemos preparado disoluciones midiendo cantidades conocidas de cada una
de las diferentes muestras comerciales de la vitamina C. Cada una de las muestras ha sido disuelta
en agua destilada y hemos completado con agua hasta obtener un volumne de 1,00 L. De esta
disolución se te proporciona un volumen aproximado de 25 mL. La masa exacta medida se indica en
el recipiente que contien la disolución de tu muestra. Por la urgencia de los resultados y para
adelantar tu trabajo, los profesores del CENMART han medido el pH de todas las disoluciones y
han encontrado que todas tienen un valor de pH cercano a 7,4.

En el vaso de precipitados encontrarás una disolución de yodo de concentración conocida, a la que


se ha añadido un exceso de yoduro de potasio, y que podrás utilizar como oxidante para identificar y
determinar la pureza de tu muestra de vitamina.

197
PROCEDIMIENTO.
Titulación de la muestra problema.

1. Llena la bureta con la disolución de yodo/yoduro.


2. Toma, con la pipeta volumétrica, una alícuota de 5,00 mL de la disolución problema y colócala en
el matraz erlenmeyer, añádele un volumen aproximado de 10 mL de agua destilada y unas gotas
de la disolución de almidón.
3. Titula tu muestra problema con la disolución de yodo/yoduro contenida en la bureta. El punto
final se detecta cuando aparece una coloración azul debida a la formación de un complejo entre el
almidón y el yodo en exceso.
4. Repite los pasos 2 y 3 y realiza la titulación por triplicado.

HOJA DE RESULTADOS.

Código: __________ Problema No. ______________

Volumen de la disolución Volumen gastado de la


problema disolución de yodo/yoduro (mL)
5,0 mL
5,0 mL
5,0 mL

1. Volumen de titulante que utilizarás en tus cálculos: ____________

2. Concentración (en mol/L) de vitamina C (como ácido ascórbico o ascorbato).


Cálculos:

Concentración ______ mol/L

3. De acuerdo con el pH medido por lo profesores del CENMART, tu muestra contiene la vitamina
en forma de: ( ) ácido ( ) sal

4. ¿Cuál hubiera sido el valor del pH si la muestra estuviera en la otra forma posible? ___________

5. Indica cuál es la reacción de titulación que se efectúa cuando el ascorbato tiene la fórmula
C6H7O6M: ____________________________________________________________________

o C12H14O12M: _________________________________________________________________

6. De acuerdo con la masa de sólido medida con anterioridad (indicada en la etiqueta de tu


problema) la muestra contiene: ________________________________

7. La pureza de la muestra que analizaste es:


Cálculos:

198
XVIII Olimpiada Nacional de Química
Examen Experimental de Química Inorgánica Campeche 2009

Introducción.

El problema que se presenta es la discriminación entre 5 gases y la identificación de 1 de ellos que te


ha sido asignado. Las posibilidades de gases son:

1. DIÓXIDO DE AZUFRE.
2. DIÓXIDO DE NITRÓGENO (en este caso se obtendrá el monóxido que reaccionará con el
oxígeno atmosférico para obtener inmediatamente el producto deseado).
3. AMONIACO.
4. CLORO.
5. DIÓXIDO DE CARBONO.

Para identificarlos contarás con los siguientes reactivos:

1. Indicador universal.
2. Indicador de azul de bromotimos (vire 6-7.8 de pH, amarillo-azul).
3. Jugo de uva.
4. Yoduro de potasio al 0.5 M.
5. Disolución saturada de ácido sulfhídrico.
6. Disolución de Fe(II) en medio ácido.
7. Disolución de hidróxido de bario al 0.5 M.
8. Disolución de cromato de potasio 0.5 M.
9. Disolución de sulfato de cobre(II) 0.5 M.
10. Disolución de sulfito de sodio 0.5 M.
11. Disolución comercial de yodo.
12. Disolución de permanganato de potasio 0.01 M acidulada con ácido sulfúrico.

De acuerdo a tus conocimientos previos, completa la siguiente tabla, indicando el cambio que
esperas al poner en contacto cada gas con los diferentes reactivos, indica el tipo de reacción de la
que se trata de acuerdo a la siguiente clave:

1. Redos (R).
2. Ácido-base (AB).
3. Formación de complejos o compuestos de coordinación (C).
4. En caso de no presentarse reacción marca la casilla con una “X”.

Reactivo 1 2 3 4 5 6 7 8 9 10 11 12
No.
SO2
NO2
NH3
CO2
Cl2

199
Para la obtención e identificación del gas asignado te proponemos la siguiente secuencia en
microescala.

Instrucciones.

En la caja de petri que se te proporciona, realiza las siguientes actividades:

a) Coloca en el centro de la base de la caja, una pequeña cantidad (una punta de espátula en caso de
ser sólido o 2 gotas en caso de ser líquido) del reactivo ___________ que se te asignó. Cuida que
el reactivo se acumule en el centro del círculo.
b) De acuerdo al esquema, coloca alrededor del círculo formado por la caja de petri, una gota de los
reactivos numerados en la parte de la introducción. Cuida de no poner un exceso de
disoluciones que propicie la mezcla de reactivos.

12
11 1

2
10

Generación 3
9 del
Gas

8 4

7
5
6

c) Coloca la tapa de la caja de petri que tiene una horadación en el centro.


d) Con una pipeta beral tomo NO más de 0.2 mL del segundo reactivo _____________. Cerciórate
de que las letras de los 2 reactivos (el reactivo central y el líquido de la pipeta) coincidan.
e) Coloca la pipeta en el orificio, tratando de que el sistema cierre herméticamente (si la pipeta es
muy larga corta la punta).
f) Presiona suavemente el bulbo de la pipeta para añadir una o 2 gotas del reactivo líquido sobre el
primer reactivo.
g) Cuida de no añadir líquido en exceso porque si se mezcla con las diferentes disoluciones deberás
volver a empezar.
h) Permite que el gas generado reaccione con los diferentes reactivos y pasados 1 ó 2 minutos,
destapa el sistema.

200
i) En el caso del reactivo 6, añade 1 gota de tiocianato de potasio y observa.
j) en el cado del reactivo 10, añade una gota de cloruro de bario y observa.
k) Registra tus observaciones en la tabla 1.

MUESTRA:
Observaciones 1 2 3 4 5 6 7 8 9 10 11 12 Características
del gas
obtenido
Condiciones
iniciales
Cambios

Notas

De acuerdo a las observaciones realizadas, el gas que te fue asignado es: ________________

Escribe las ecuaciones químicas de las reacciones que te permiten identificar a tu gas. En el caso de
reacciones redox especifica las 2 semi-reacciones.

1.

2.

3.

4.

Obtención de gases.

Partiendo de que los posibles reactivos sólidos para la obtención de gas son:
1. Nitrito de sodio.
2. Sulfito de sodio.
3. Carbonato de sodio.
4. Cloruro de amonio.
5. Hipoclorito de calcio.

Y que los reactivos líquidos son:


1. Disolución de hidróxido de sodio.
2. Disolución de ácido clorhídrico.
3. Disolución de hipoclorito de sodio.

Propón la ecuación química que representa la obtención del gas asignado y de cada uno de los otros
4 gases propuestos. Clasifica las reacciones de acuerdo a la clasificación propuesta en la
introducción de este documento.

1.
201
2.

3.

4.

5.

Datos:

E0  0,17 V E0  0,94 V E0  0,77 V


SO42  /SO2 HNO2 /NO3- Fe3 /Fe2 

E0  0,621 V E0  1,51 V E0  1,33 V


I 2 /I - MnO4 /Mn 2  CrO42  /Cr 3

E0  0,68 V E0  1,7 V E0  1,03 V


SO2 /S MnO4 /MnO2 NO2 /NO
0
E S/H 2S
 0,141 V E0  1,36 V
Cl2 /Cl -

K ps(BaSO4 )  1,5x10 -9 pKa  9 ,24 pKa2 (H 2 SO4 )  1,99


(NH 4 )

K ps(BaCO3 )  8,0x10 -9 pKa1(H 2CO3 )  6 ,35 pKa(HNO2 )  3,14


K ps(BaSO3 )  5,0x10 -10 pKa2 (H 2CO3 )  10 ,32

Vire del indicador universal


pH 1-4 5-6 7 8-9
Color Rojo Amarillo Verde Azul Azul/Índigo

202
XIX Olimpiada Nacional de Química
1er Examen Nivel A y B.
Total: 40 preguntas. Tiempo asignado: 90 minutos. Veracruz 2010

Anota en el recuadro correspondiente la letra del inciso que contesta correctamente cada pregunta.
Si consideras que ningún inciso es el correcto deberás anotar una letra X.
NOTA ACLARATORIA: Al hacer tus cálculos puedes encontrar una pequeña diferencia, en
general no mayor de unas décimas, entre tu resultado y alguno de los propuestos. Esto es
normal y por eso sólo deberás usar la “X” cuando ninguna respuesta sea muy cercana a la
obtenida por ti.
********************************************************************************
**********
Sección I. Para contestar las siguientes DIEZ preguntas, considera sólo los primeros 18 elementos
de la Tabla Periódica. Coloca en el recuadro de la derecha, el NÚMERO ATÓMICO del elemento
que corresponde a cada descripción.
1) Es un gas noble cuyo nombre está asociado a que, después de la invención de la
espectroscopía, se descubrió su existencia en el Sol.
+
2) Un ion sodio Na tiene una configuración electrónica idéntica a la del elemento:

3) El óxido de este elemento tiene una masa molar de aproximadamente 62 g / mol.

4) Un compuesto que contiene este halógeno sirve para desinfectar el agua.

5) Su molécula en estado gaseoso es diatómica y tiene un enlace triple.

6) Cuando cuatro átomos de este elemento (que no es un halógeno) se unen a un átomo


de carbono forman un compuesto de forma tetraédrica.

7) Además de hidrógeno, la molécula de amoniaco contiene este elemento.

8) En la molécula de cianuro de sodio, el elemento de menor masa atómica.

9) En la molécula de ácido sulfúrico, el elemento con mayor % en masa.

10) Tiene el mayor valor de electronegatividad en la escala de Pauling.

203
Sección II. Escribe en el recuadro la letra de la respuesta correcta. Recuerda que si consideras que
ninguna respuesta es correcta deberás escribir una “X”.
11) El sulfato férrico o sulfato de hierro (III), es un compuesto cuya masa molar es,
aproximadamente, en g mol-1:

A 152 B 278
C 352 D 400

12) En una mezcla de la misma cantidad en gramos de FeSO4 y de Fe2(SO4)3, la fracción


mol del sulfato ferroso o sulfato de hierro (II) es:

A 0.90 B 0.72
C 0.25 D 0.50

13) Si se hacen reaccionar 6.5 g de zinc metálico con 500 mL de una disolución acuosa
0.1 M de ácido sulfúrico (H2SO4), ¿cuántos moles se obtienen de sulfato de zinc?

A 0.65 moles B 0.05 moles


C 0.1 moles D 0.01 moles

14) En forma comercial, el sulfato de zinc se vende en forma de heptahidrato (cada


molécula del compuesto contiene siete moléculas de agua de hidratación). En este
compuesto, el % en masa de zinc es aproximadamente:
A Menos del 20% B Entre 20% y 30%
C Entre 30% y 40 % D Más del 40%

15) La ley de los gases ideales es PV=nRT, [R = 0.082 L atm / mol K; R = 8.314 J / K
mol; 1 J = kg m2 s-2; 1 atm = 101.325 kPa; 1 Pa = 1 N m-2; 1 N = 1 kg m s-2]
Si un mol de KClO3 se descompone para producir cloruro de potasio y oxígeno
gaseoso, a P = 101 kPa y T = 10 0C, este oxígeno ocupa un volumen de:
A Menos de 20.0 L B Entre 20.0 L y 30.0 L
C Entre 30.0 L y 40.0 L D Más de 40.0 L
3
16) El ácido nítrico es un líquido cuya densidad es 1.574 g / cm . Por lo tanto, la masa
contenida en medio litro de ácido nítrico es:

A 15.74 g B 157.4 g
C 7.87 g D 78.7 g

17) Si consideramos que el ácido nítrico (HNO3) concentrado tiene un 90 % en masa del
ácido y el resto es agua, al calcular la molalidad (mol kg–1) del ácido suponiendo al
agua como el disolvente, obtenemos un valor igual a:
A 14.28 m B 142.8 m
C 90 m D 9.0 m

204
18) En el ácido nítrico concentrado, mencionado en la pregunta anterior, la fracción mol
del agua es:

A 0.28 B 0.55
C 1.975 D 5.5
-1
19) Si se mezcla medio litro de disolución acuosa 0.2 molar (mol L ) de cloruro de sodio
con 250.0 mL de una disolución 0.5 molar del mismo compuesto, la concentración
molar del cloruro de sodio en la disolución obtenida es:
A 0.7 M B 0.5 M
C 0.3 M D 0.225 M

20) En la disolución que se obtiene en la pregunta anterior, el % en masa del cloruro de


sodio es:

A 35.1 % B 17.55 %
C 3% D 2.3 %

21) Al balancear la ecuación


? FeCr2O7 + ? K2CO3 + ? O2  2 Fe2O3 + ? K2CrO4 + ? CO2
el coeficiente estequiométrico para el oxígeno es:
A 14 B 12
C 7 D 6

22) En la reacción de la pregunta anterior , es cierto que:

A Se oxidan tres elementos y se B Se oxidan dos elementos y se reducen


reducen dos elementos dos elementos
C Se oxidan dos elementos y se D Se oxida un elemento y se reduce un
reduce un elemento elemento

23) ¿En qué condiciones, los resultados de la ecuación PV=nRT son más cercanos a los
reales?

A A presiones bajas y temperaturas B A presiones y temperaturas altas


altas
C A presiones y temperaturas bajas D A presiones altas y temperaturas bajas

24) Un alcohol lineal, sin enlaces dobles o triples, tiene una masa molar de unos 102 g /
mol. Esta molécula debe tener, de átomos de carbono:

A Seis B Siete
C Ocho D Más de ocho

205
25) El pH de la solución de NaOH 0.100 N es:

A 0.1 B 1
C 8 D 13

26) Para los compuestos NH4Cl y Na2CO3, su comportamiento ácido-base en agua es,
respectivamente:

A ácido, base B base, ácido


C ácido, neutro D base, neutro

27) En la reacción de bicarbonato de sodio con ácido sulfúrico se obtiene sulfato de sodio,
agua y CO2. ¿Cuántos gramos de bicarbonato de sodio se requieren para producir un
mol de CO2?
A 84.01 B 42.005
C 106 D 168.02

28) Las ppm (partes por millón) son unidades de concentración, muy utilizadas en
estudios de contaminación de aguas o concentración de elementos en fluidos de seres
vivos. En gramos, una ppm equivale a un gramo en un millón de gramos. Si se lograra
disolver un mol de benceno en 100 m3 de agua, la concentración de benceno en esta
mezcla sería en ppm:
A 0.078 B 0.78
C 7.80 D 78.0

29) Si se hace reaccionar el KMnO4 con ácido sulfúrico concentrado se obtiene un


compuesto que puede explotar al contacto, el Mn2O7. En la reacción propuesta se
obtiene además agua y KHSO4. Al balancear esta reacción se encuentra que por cada
mol de permanganato, reaccionan de H2SO4:
A 0.5 mol B 1.0 mol
C 1.5 mol D 2.0 mol

30) Si la reacción anterior se lleva a cabo en disolución acuosa, se propone la siguiente


ecuación:
KMnO4(ac) + H2SO4(ac) → MnSO4(ac) + K2SO4(ac) + H2O(l) + O3(g)
Por cada mol de permanganato de potasio que reacciona se obtienen de ozono:
A Menos de un mol B Un mol
C Dos moles D Más de dos moles

206
31) Si en la molécula de benceno se sustituye un hidrógeno por un grupo OH se produce
el fenol, una sustancia cuya estructura es:

OH

Se pueden disolver hasta 83 g de fenol en un litro de agua. ¿Cuántos litros de agua se


necesitarían para disolver un mol de fenol?
A Menos de 1.00 litro B Entre 1.00 y 1.10 litros
C Entre 1.10 y 1.20 litros D Más de 1.20 litros

32) Para quemar un mol de fenol y producir CO2 y H2O, ¿cuántos moles de oxígeno
gaseoso se requieren?

A Menos de 7.5 B Entre 7.5 y 8.0


C Entre 8.0 y 8.5 D Más de 8.5

33) La combustión puede ser incompleta y se produce también CO, además de CO2 y
H2O. Si al quemarse un mol de fenol se producen la misma cantidad en moles de CO y
CO2, ¿cuántos moles de oxígeno gaseoso se consumen?
A Más de 15 B Entre 10 y 15
C Entre 6 y 9.5 D Menos de 6

34) Un estudiante toma 200.0 mL de una disolución acuosa de HCl 2.0 M y le agrega agua
destilada hasta tener 500.0 mL de disolución. El pH de la disolución preparada es:

A – 0.097 B 0.397
C 0.097 D 3.97

35) Cuando un gramo de agua es adicionado a una mezcla de alcohol y agua, la nueva
mezcla tiene un 20% en masa de alcohol. Cuando 3 gramos de alcohol son
adicionados a la nueva mezcla, la mezcla ahora contiene 50% en masa de alcohol. El
% en masa de alcohol en la mezcla original era:
A Menos de 20% B Entre 20% y 27%
C Entre 27% y 34% D Mayor a 0.5

36) En un recipiente de 2.0 L de capacidad se coloca la misma cantidad en gramos de


nitrógeno, oxígeno y CO gaseosos. La fracción mol de nitrógeno en esta mezcla es:

A Menor a 0.3 B Entre 0.3 y 0.4


C Entre 0.4 y 0.5 D Mayor a 0.5

207
37) Indica cuál es el enunciado verdadero con relación a la reacción:
2K2CrO4 + H2SO4  K2Cr2O7 + H2O + K2SO4

A Es una reacción redox y los iones B Es una reacción redox y los iones azufre
cromo se reducen se oxidan
C Es una reacción redox y los iones D No es una reacción redox
cromo se oxidan

38) La corriente que circula en una celda electroquímica se puede medir en amperes (A).
Un ampere equivale a una carga de un coulomb (C)que pasa cada segundo por un
punto del circuito. La carga de un electrón es 1.6 x 10-19 C. Si en una celda
electroquímica circula una corriente de 5 mA, significa que el número de electrones
que pasan por un punto del circuito cada segundo son:
A 3.125 x 10-16 B 3.2 x107
16
C 3.125 x 10 D 3.2 x 10-17
+
39) En una celda electroquímica, para reducir un mol de iones plata Ag y convertirlos en
átomos de plata metálicos, se requiere que circule por la celda una carga total de:

A 96368 C B 289104 C
C 192736 C D 48184 C

40) De acuerdo a la expresión " X"  206


82 Pb  α podemos concluir que el átomo
radiactivo “X” es: (Las partículas alfa son núcleos de helio)

210 210
A 83 Bi B 81 Tl
210 210
C 85 At D 84 Po

FIN DEL EXAMEN

208
XIX Olimpiada Nacional de Química.
do
2 Examen Nivel A y B.
Tiempo asignado: 90 minutos. Veracruz 2010

NIVEL A: TIENEN QUE CONTESTAR TODAS LAS PREGUNTAS.


NIVEL B: SÓLO TIENEN QUE CONTESTAR DE LA PREGUNTA 1 A LA 16.

QUÍMICA ANALÍTICA.

ESCRIBE EN EL RECUADRO LA LETRA DE LA RESPUESTA CORRECTA.

1. Indica en qué casos se forma un precipitado cuando se mezclan las siguientes


disoluciones de dos componentes diferentes (ambas en concentración 0,1 mol/L)
I) HCl y AgNO3 II) NaOH y CuSO4 III) NH4NO3 y K2SO4
IV) NaNO3 y BaCl2 V) H2SO4 y Ba(OH)2

a) Sólo en I b) En I, II y V c) En todas se forma precipitado


d) En III y IV e) No se forma en ninguna

2. ¿Cuál de las siguientes disoluciones (C=0,1 mol/L) tiene el mayor valor de pH?

a) acetato de sodio b) ácido acético c) cloruro de amonio


d) sulfato de sodio e) ácido clorhídrico

3. Para la reacción:

(NH4)2PtCl6  Pt + N2 + NH4Cl es cierto que:
a) el nitrógeno dismuta
b) el elemento que se oxida es el platino
c) el elemento que se reduce es el platino
d) el nitrógeno se reduce
e) se reduce el cloro

4. En la valoración de un ácido débil con una base fuerte, el pH en el punto de


equivalencia es:
a) 14 b) 7,0 c) Menor que 7 d) Mayor que 7
e) Igual que el pKa del ácido débil.

5. Un gramo de un cierto ácido orgánico se neutraliza con 22,7 cm3 de una


disolución de hidróxido de sodio (NaOH) 0,5 mol/L. Es posible que el ácido
sea:
a) Un ácido monoprótico de masa molar 88 g mol-1
b) Un ácido diprótico de masa molar 88 g mol-1
c) Un ácido monoprótico de masa molar 44 g mol-1
d) Un ácido diprótico de masa molar 44 g mol-1
e) Un ácido monoprótico de masa molar 176 g mol-1

209
6. Se desea preparar una disolución en la que la concentración del ion NO3− sea
0,25 mol/L y se dispone de 500 mL de una disolución de KNO3 0,20 mol/L.
¿Qué volumen de disolución de Ca(NO3)2 0,30 M habrá que añadir?
a) 250 mL b) 35,70 mL c) 71,40 mL d) 142,80 mL e) 500 mL

7. ¿Cuál de las siguientes disoluciones de NaOH neutralizaría totalmente 10 mL


de una disolución H2SO4 0,15 mol L-1?
a) 10 mL de disolución 0,15 mol L-1
b) 20 mL de disolución 0,10 mol L-1
c) 10 mL de disolución 0,30 mol L-1
d) 5 mL de disolución 0,30 mol L-1
e) 30 mL de disolución 0,20 mol L-1

8. Al añadir unas gotas de un indicador ácido-base a una disolución acuosa


desconocida se observa un color verde. El indicador tiene un intervalo de
viraje de 3,8 a 5,4; a pH < 3,8 es amarillo, a pH > 5,4 es azul, y entre ambos
pH es verde.
¿Cuál de las disoluciones siguientes, todas ellas de la misma concentración
0,5 mol/L, puede ser la disolución desconocida?
a) Ácido nítrico
b) Hipoclorito de sodio
c) Hidróxido de potasio
d) Cloruro de amonio
e) Sulfato de sodio

9. Si el producto de solubilidad del AgBr (s) es 5,0x10–13 a 25°C, su solubilidad


en agua es:
a) 2,5x10–6 mol·L–1
b) 7,1x10–7 mol·L–1
c) 1,4x10–6 mol·L–1
d) 2,5x10–13 mol·L–1
e) El bromuro de plata es completamente insoluble.

10. Calcule el pH de una disolución saturada de Ca(OH)2 si su producto de


solubilidad es 5,5x10–6.
a) 11,28
b) 8,72
c) 12,04
d) 12,34
e) 13,00

210
FISICOQUÍMICA.
El mercurio (M=200.59 g/mol) es un elemento con una presión de vapor muy baja. A 20 ºC la
presión de vapor del mercurio es de 1.7 x 10–6 atm, y es altamente tóxico aun en pequeñas
concentraciones.

Escribe la respuesta en el cuadro vacío de la derecha.

11. Un estudiante derrama mercurio (Hg) a 20 °C, sobre el piso de un laboratorio que mide 15.2 m de
largo, 6.6 m de ancho y 2.4 m de alto.

11.1. La masa necesaria para saturar el laboratorio con vapor de mercurio es:
a) 3.4 g b) 5.1 g c) 34.2 g d) 40.1 g e) 50.1 g

11.2. La concentración de mercurio en mg Hg/m3 es:


a)1.42x 10– 3 b) 0.0142 c) 0.142 d) 14.2 e) 1.42x102

Una norma sanitaria oficial indica que la concentración de Hg en estado gaseoso no debe exceder de
0.050 mg Hg/m3. En un hospital de la Ciudad de México son aún más exigentes y su norma de
seguridad indica que en caso de derrames de mercurio, la concentración debe estar por debajo del
80% de esa norma sanitaria oficial.

12. A una enfermera se le rompe un termómetro de mercurio que tiene un bulbo con 40 mg de
mercurio en un cuarto de hospital de 4.5 m de largo, 3.5 m de ancho y 2.4 m de alto. Si se
evapora totalmente el mercurio, el porcentaje respecto a la norma del hospital es:

a) menor a b) entre c) entre 213 d) entre 2000 e) mayor que


21.2 % 21.2 y 212 % y 2000 % y 2500 % 2500 %

13. El nitrógeno forma varios óxidos gaseosos. Uno de ellos tiene una densidad de 1.27 g/L, medida
a 764 mmHg y 150 ºC. La fórmula del compuesto es:

a) NO b) NO2 c) N2O d) N2O4 e) N2O5

Un matraz contiene una mezcla de NO2 y N2O4 en equilibrio: 2 NO2 ⇆ N2O4 a una presión de un
bar. La presión parcial del primero es el doble de la del segundo.

14. La masa molar aparente de la mezcla es (en g/mol):


a) menor a b) entre c) entre d) entre e) mayor que
55.2 55.2 y 60.6 60.7 y 61.6 61.7 y 65.1 65.2

15. La constante de equilibrio es:


a) 0.167 b) 0.250 c) 0.500 d) 0.750 e) 1.333

211
QUÍMICA INORGÁNICA.

16. En el planeta PAJOYKOT, el cual es idéntico a la Tierra en todos los aspectos, sus habitantes han
encontrado solamente 88 elementos que han organizado en una tabla periódica análoga a la
nuestra. Sin embargo, los nombres que les han dado a dichos elementos son distintos a los
nuestros. Los habitantes de este planeta han mandado un mensaje oculto en este problema y para
ello te proporcionan las características de sus elementos con lo que podrás saber cómo se llaman
en la Tierra y de este modo podrás colocar sus símbolos en el lugar que le corresponda en la tabla
periódica de ese planeta. Cuando los acomodes podrás leer el mensaje enviado desde el planeta
PAJOYKOT.

Anelio (Ae). Los compuestos de este elemento son usados en la fabricación de imanes y es el
principal componente del acero. La deficiencia de este elemento en el organismo se asocia a las
anemias.
Linkono (L). En su forma elemental es un gas inflamable cuya molécula es diatómica y es el
menos denso de todos los gases.
Pictorin (P). Este elemento es un gas, es muy electronegativo y es un comburente por excelencia,
además de ser uno de los elementos más importantes para la vida.
Nautilio (Nt). Este elemento en su forma elemental es un sólido de color grisáceo que produce un
vapor violeta al ser calentado, tiene un isótopo radiactivo que es usado para el tratamiento y
diagnóstico de desordenes en las glándulas tiroideas.
Octilo (O). En su forma elemental es un gas de color amarillo verdoso, sus compuestos se usan
en la purificación del agua, en blanqueo y la fabricación de plásticos.
Atilio (At). Este elemento primero se descubrió en el sol, además debido a su ligereza se utiliza
en los globos dirigibles y tiene un punto de fusión muy bajo (4 K).
Coranio (C). Es el principal componente de los huesos y uno de los principales causantes de la
dureza del agua, además está presente en el mármol y en el yeso.
Algorio (Al). Es un elemento líquido que al vaporizarse forma un gas tóxico, se usa en los
termómetros y es formador de las amalgamas.
Sendrenio (Se). Este elemento del bloque d se utiliza en la fabricación de las monedas y baterías,
su configuración termina en d8 y la mayoría de sus sales son de color verde.
Divadin (Di). Este elemento del bloque p es un gas que se emplea en atmósferas inertes y es
vecino de periodo del octilo y ermio, su configuración termina en p6.
Abnitio (Ab). Es el no metal de mayor punto de fusión, está presente en los lápices.
Meovio (Me). Es el mejor conductor de la electricidad, se usa en joyería, en los rollos
fotográficos y en la fabricación de espejos.
Ermio (Er). El estado de oxidación más común de este elemento es de +I, se encuentra en la sal
de mesa y también es uno de los componentes del vidrio.
Leiyo (Le). En su forma elemental es un líquido café rojizo y es muy reactivo, se usa en
medicinas, fumigantes y es vecino de grupo del octilo.
Imperor (I). Este elemento se encuentra presente en muchas moléculas de importancia
bioquímica como el ADN y el ATP, además comercialmente se encuentra en fertilizantes,
detergentes y refrescos.
Lanógeno (La). Es el principal componente de la atmósfera, si se licua se vuelve un líquido
enfriador, está presente en los fertilizantes, combustibles de cohetes y en las proteínas.

212
1 2 3 4 5 6 7 8 9 10 11 12 13 14 15 16 17 18

¿Cuál es el mensaje oculto enviado por los habitantes del planeta PAJOYKOT?

________________________________________________________________

213
QUÍMICA ORGÁNICA.

17. Seleccione el producto principal de la serie de estructuras que se encuentran en la parte inferior,
para la reacción del 1,2-dimetilciclohexeno con cada uno de los siguientes reactivos. Tenga en
cuenta la estereoquímica de los productos obtenidos, la cual está determinada por el mecanismo
de la reacción.

Reacción con
a) HCl
b) H2O y H2SO4
c) 1) BH3; 2) H2O2, NaOH
d) Br2, CCl4
e) H2, Pt

Las estructuras que puede seleccionar son las siguientes. Si la estereoquímica no se muestra, puede
asumir que la estructura representa una mezcla de estereoisómeros.
CH3 CH3 CH3 CH3

CH3

CH3 CH3 CH3


D OH
A B C

CH3 CH3 CH3


CH3

OH OH Cl

CH3
E CH3 CH3 CH3
F G H

CH3 CH3 CH3 CH3

CH3 Cl Br CH3

I Cl J CH3 K CH3 Br
L
CH3 CH3 CH3
CH3

Br Br Br
Br
CH3 CH3 Br

M CH3
N Br O Br P CH3

RESPUESTAS
a) b) c)

d) e)

214
18. Para la siguiente reacción:

H
NaOH
CH3

Br
H

H OH

CH3 CH3 CH3


CH3 H
HO H H H
H H H
A B C D

¿Cuál será el producto E2 y cuál será el producto SN2?


a) C, A b) D, A c) C, B d) D, B

RESPUESTA

FIN DEL EXAMEN

215
216
XIX OLIMPIADA NACIONAL DE QUÍMICA
TERCER EXAMEN NIVEL A y B Tiempo asignado: 180 minutos Veracruz 2010

QUÍMICA ANALÍTICA

1. Mediante una técnica denominada termogravimetría es posible registrar los cambios en masa
cuando se calienta una sustancia.
La siguiente gráfica muestra las variaciones en masa de un oxalato de calcio hidratado
(CaC2O4xH2O).

A partir de estos datos anteriores identifica:


¿Cuál es el compuesto D?

¿Cuál es el compuesto E?

¿Cuál es el compuesto F?

La sustancia que se pierde entre


140 y 200 oC es:

La sustancia que se pierde entre


450 y 500 oC es:

La sustancia que se pierde entre


600 y 700 oC es:

217
Escribe todas las ecuaciones correspondientes, con estado de agregación, efectuadas durante el
calentamiento del oxalato de calcio hidratado.

2. En el laboratorio se encontró un frasco, no etiquetado, que contiene un polvo blanco que


corresponde a una sustancia orgánica pura con propiedades ácidas. Para poder identificarlo un
estudiante olímpico decidió realizar una titulación ácido base y registrar los valores de pH
obtenidos en el transcurso de la misma. El procedimiento elegido se describe a continuación:
 Midió con exactitud una masa de 5.800 g del polvo blanco, lo disolvió en agua destilada y lo
llevó a un volumen de 100.00 mL.
 Tomó una alícuota de 20.0 mL de esta disolución, la tituló con una disolución de KOH de
concentración 1.0 mol/L y registró los valores de pH obtenidos después de cada adición. Los
resultados obtenidos se muestran en la siguiente figura.

a) De su primera observación dedujo que se trataba


de un ácido: ____________________________
monoprótico, diprótico, triprótico

b) La concentración del ácido en la disolución es


igual a: mol L-1

c) El (los) valor(es) del pka del ácido es (son)


igual(es) a:

d) La masa molar del ácido es igual a:

218
e) Lo que le permitió deducir que la fórmula
condensada del ácido era igual a:

f) Lo que le permitió deducir que la fórmula


semidesarrollada del ácido podría ser:

El inciso g) sólo es para el Nivel A.

g) Esta fórmula puede corresponder a dos isómeros diferentes. Indica cuáles son los posibles
ácidos.

La pregunta 3 sólo es para el Nivel B.

3. Balancear la siguiente reacción redox:


___ (NH4)2PtCl6  ___ Pt + ___ N2 + ___ NH4Cl + ____ HCl

FISICOQUÍMICA

En el recuadro de la derecha escribe la letra de la respuesta correcta.


1. Se preparó una disolución acuosa con 20% en masa de glucosa (C6H12O6), la fracción mol de
glucosa es:
Cálculos:

a) 0.024 b) 0.20 c) 0.24 d) 0.25 e) 0.75

2. Si la fracción mol de glucosa en una disolución acuosa es Xglucosa = 0.05, el % en masa de glucosa en
la disolución es:
Cálculos:

a) 5 % b) 10 % c) 11.1 % d) 34.5 % e) 65.5 %


219
Se tienen 250 g de una disolución de urea CO(NH2)2 0.75 molal en agua.
3. El % en mol de la urea en la disolución es:
Cálculos:

a) 0.07 % b) 0.3 % c) 1.33 % d) 4.3 % e) 18 %

4. La masa de la urea en la disolución es:


Cálculos:

a) 0.175 g b) 0.75 g c) 3.325 g d) 10.76 g e) 45.0 g

Una disolución contiene 35 g de un soluto desconocido en 450 g de agua.


Para el agua T°fus = 0 °C, T°eb = 100 °C, Kf = 1.86 K kg/mol; Kb = 0.51 K kg/mol; agua = 1.0 g/mL. Si
la temperatura de fusión de la disolución es Tfus = – 2.41 °C entonces:
5. La temperatura normal de ebullición de la disolución es:
Cálculos:

a) 364.36 K b) 372.45 K c) 373.81 K d) 375.44 K e) 381.94 K

6. La masa molar del soluto es:


Cálculos:

a) 50.6 g/mol b) 60.0 g/mol c) 65.6 g/mol d) 101 g/mol e) 387 g/mol

7. La presión osmótica (en Pascales), de la disolución a 20 °C es:


Cálculos:

a) 9.28 x 103 b) 2.16 x 105 c) 6.33 x 105 d) 3.16 x 106 e) 9.28 x 106

220
Formulario:
J
P1  x1 P1* T fus  K f m Teb  Kb m   RTC R  8.314
mol K
m = molalidad C = concentración en mol/L

8. Considera los datos para los siguientes líquidos:


Líquido Masa molar (g/mol) aprox. Temperatura normal de ebullición en oC
(cuando P = 760 mmHg)
A 114 126
B 100 160
C 88 137

a) Ordena estos líquidos por su presión de vapor a 125 oC de MAYOR a MENOR:

b) Si se pudiera preparar una mezcla con la misma cantidad de gramos de cada líquido, ¿cuál sería
la fracción mol del líquido B?

c) El líquido A es un alcano, mientras que B y C son alcoholes (uno de ellos cíclico). Propón las
posibles fórmulas desarrolladas para cada líquido:
A B C

d) Con base en la fórmula propuesta en el inciso anterior, escribe la ecuación balanceada que
describa la reacción de combustión completa del líquido B. (Usa la fórmula condensada).

e) Ahora escribe la ecuación balanceada de la reacción de combustión de A que, por llevarse a cabo
con menos oxígeno, produce además de CO2, también CO. Considera que se obtienen las
mismas cantidades en moles de CO y de CO2.

f) Ahora escribe la ecuación balanceada de la reacción de combustión de A que, por llevarse a cabo
con menos oxígeno, produce además de CO2, también CO. Considera que se obtienen las
mismas cantidades en gramos de CO y de CO2.

221
Coloca la letra del inciso correcto en la línea.
g) En una destilación fraccionada de una mezcla equimolar de B y C, el vapor tendría (suponiendo
comportamiento ideal):
A) Una mayor fracción mol de B
B) Una mayor fracción mol de C
C) Fracciones molares iguales de ambos __________

h) En una destilación fraccionada de una mezcla equimolar de B y A, el vapor tendría (suponiendo


comportamiento ideal):
A) Una mayor fracción mol de B
B) Una mayor fracción mol de A
C) Fracciones moles iguales de ambos __________

QUÍMICA INORGÁNICA

1. Tratando boro elemental a altas temperaturas con amoniaco se obtiene un sólido untuoso X,
conductor de la corriente eléctrica, con una composición de:

% B = 43.5 y % N = 56.5

Tratando a este sólido X a muy altas presiones se obtiene el sólido Y, que tiene la misma
composición, pero que es de una dureza comparable a la del diamante y no conduce la corriente
eléctrica.

a) La fórmula mínima de X e Y es:

b) La configuración electrónica del boro es:

c) La configuración electrónica del nitrógeno es:

El inciso d) sólo es para el Nivel A.

d) Teniendo en cuenta que dos átomos de carbono tienen el mismo número de electrones que un
átomo de boro y uno de nitrógeno, ¿cuál es la única aseveración cierta para los sólidos X e
Y?

222
A) X es un análogo del grafito, por lo tanto tiene estructura tetraédrica con ángulos de 1090 e
hibridación sp3. Y es un análogo del diamante, en el que los ángulos de enlace son de
1200, con una hibridación sp2.
B) X es un análogo del grafito, formado por láminas infinitas de anillos hexagonales. Los
ángulos de enlace son de 1200 y la hibridación de los átomos es sp2. Y es un análogo del
diamante, con estructura tetraédrica, ángulos de enlace de 1090 e hibridación sp3.

QUÍMICA ORGÁNICA (Sólo Nivel B)

1. En un laboratorio veracruzano hay NUEVE compuestos orgánicos que tendrás que acomodar en la
siguiente tabla de acuerdo a los datos que se te proporcionan.
Coloca en cada espacio, el NOMBRE y la FÓRMULA CONDENSADA del compuesto que
corresponde.

1 2 3

4 5 6

7 8 9

DATOS:
a) Hay tres ácidos carboxílicos, uno en cada columna. Uno de éstos es un compuesto aromático
de masa molar igual a 122.12 g/mol. Otro es común mezclarlo con agua.
b) A temperatura ambiente, sólo dos de los compuestos están en estado de gas o de vapor y
ambos están en casillas impares. Otro compuesto es una cetona que se encuentra en varios
productos de uso casero.
c) Sólo uno de los nueve compuestos tiene un punto de fusión mayor a 100 oC, es aromático y
está en la fila inferior.
d) En la fila de en medio hay un alcohol y junto a éste hay un compuesto aromático muy
conocido y que tiene el mismo número de átomos de carbono y de hidrógeno. Uno de los
ácidos es un derivado de este compuesto aromático.
e) El vinagre es una disolución acuosa de un ácido que está en la columna izquierda. Otro de
los ácidos es una molécula lineal con una masa molar de 88.11 g/mol, está en la columna
derecha, justo abajo del compuesto que tiene la menor masa molar.
f) En casillas impares hay dos alcoholes. El que está presente en muchas bebidas está en la fila
inferior.
g) Hay tres compuestos cuya masa molar es menor a 35 g/mol. Uno es un alcohol y los otros
dos son alcanos, ambos ubicados en la fila superior.
223
h) Si sumas los átomos de carbono presentes en la fórmula condensada de todos los nueve
compuestos considerados en este problema, el total es 28.

Notas adicionales: El grupo funcional de los ácidos carboxílicos es –COOH


El grupo funcional de los alcoholes es –CH2OH
O

El grupo funcional de las cetonas es C


R R'

Un compuesto aromático típico es el benceno

QUÍMICA ORGÁNICA (Sólo Nivel A)

1. Complete la siguiente secuencia sintética, dando la estructura de los compuestos o bien los
reactivos necesarios para llevar a cabo cada una de las reacciones indicadas:

N
O OH
C C

a, b
HNO3 c, d
C D
H2SO4

A 1) Fe, HCl, Calor


B

2) NaOH

O OCH3
C O OCH3
C

O e, f

N
NH2
F H E

RESPUESTAS
a b c d

224
e f C D

2. Escriba los reactivos o los productos necesarios en cada una de las siguientes reacciones, según
sea el caso:
H

N
AcOH N
a) A + B

H3 C H
1) D
b) C 2) HCl / H2O
3) H2SO4, 

O
H2O
c) E + F
HCl
O
N
H2O
d) G + H
HCl
N
N
OH
AcOH
e) I + J

RESPUESTAS
A B C D
⇔ ⇔

E F G H
⇔ ⇔

225
I J

3. Hay tres ácidos ciclopropanodicarboxílicos isómeros A, B y C, cada uno de los cuales tiene la
fórmula C5H6O4. El isómero A no se ve afectado por el calor. El isómero B al calentarse pierde
una molécula de CO2 para dar un nuevo compuesto D, C4H6O2. En tanto que el isómero C al
calentarse pierde una molécula de agua para producir un nuevo compuesto E, C5H4O3. Con base
sólo en estos datos:

a) Señale las estructuras de los compuestos A al E.

b) Escriba las ecuaciones para las reacciones que se llevan a cabo (NOTA: no considere la
posibilidad de tener enantiómeros).

RESPUESTAS
A B C

D E

ECUACIONES
COMPUESTO B

COMPUESTO C

226
4. El pulgón verde del durazno (Myzus Persicae) se repele utilizando su propia feromona defensiva
(aunque también se puede repeler por la presencia de otros pulgones triturados). Esta Feromona
de alarma se pudo aislar del extracto de los propios pulgones. Dicha Feromona presentó la
fórmula molecular C15H24. Para determinar su estructura se llevó a cabo una hidrogenación
catalítica empleando hidrógeno y como catalizador paladio soportado en carbono, y se determinó
que la Feromona absorbió cuatro moles de hidrógeno, obteniéndose como producto el 2,6,10-
trimetildodecano. Cuando la Feromona se hizo reaccionar con ozono y posteriormente con
sulfuro de metilo, se obtuvieron los siguientes productos:

Feromona de alarma

C15H24

1) O3, CH2Cl2, -78 0C


2) CH3-S-CH3
O
O
O O
+ + H H
2 +
H
H H
O O
O

a) Ignore la isomería Z-E y con base a los resultados de las reacciones anteriores, proponga una
estructura razonable para la Feromona de alarma.
b) Tomando en cuenta las unidades de isopreno, ¿cómo clasificaría a la Feromona de alarma?
c) ¿La estructura que está proponiendo cumple la regla isoprénica?

RESPUESTAS

a)

b) c)

FIN DEL EXAMEN

227
228
XIX OLIMPIADA NACIONAL DE QUÍMICA
EXAMEN INTERNACIONAL
9 PROBLEMAS Tiempo: 4 HORAS Veracruz 2010

Química Analítica

PROBLEMA I.
La argirodita es un mineral que contiene Ag+, S2- y un tercer elemento (X). La identidad de X fue
investigada en 1886 por un químico alemán quien descubrió que se trataba de un elemento cuya
existencia había sido predicha 15 años antes por Mendelev, indicando que X tendría un número de
oxidación de 4+.
Para reproducir los resultados que permitieron llegar a su fórmula un estudiante olímpico siguió el
siguiente procedimiento:
 Calentó al aire 1,00 g de argirodita para oxidar el sulfuro hasta SO2; el gas formado se burbujeó
en una disolución de Ba(OH)2 y obtuvo un precipitado que después de filtrar y secar tenía una
masa de 1,156 g.
 Trató con ácido nítrico diluido al residuo sólido remanente después de la eliminación del SO2 y
observó que dicho residuo (A) se disolvía parcialmente quedando aún un compuesto sólido
remanente (B).
 Una vez separado el sólido B determinó la concentración de Ag presente en la disolución filtrada
adicionando a ésta 100,0 mL de KBr (C=0,100 mol/L). Por medio de una titulación por retroceso
determinó que el exceso de KBr que no había reaccionado correspondía a 29,1 mL.
 Finalmente, encontró que el compuesto B que no fue soluble en ácido nítrico, correspondía a un
óxido anfotérico que podía disolverse en disoluciones concentradas, tanto de HCl como de
NaOH, dando lugar, en ambos casos, a disoluciones incoloras.
Reproduce los cálculos que fueron efectuados por el brillante olímpico a fin de que puedas llegar a
sus mismas conclusiones.

1. De acuerdo con las predicciones de Mendelev, ¿a qué grupo puede pertenecer el elemento X?

2. Calcula la masa de plata en 1 g de argirodita.

_____________g de plata

3. Escribe la reacción de precipitación del SO2 con Ba(OH)2. No olvides los estados de agregación.

229
4. Calcula la masa de azufre en 1 g de argirodita.

____________g de azufre

5. Calcula la masa del elemento X en 1 g de argirodita.

________________g de X

6. Indica cuál es el elemento X. Justifica tu respuesta tomando en consideración las cargas de los
elementos presentes en el mineral.

El elemento es_________________

7. ¿Cuál es la fórmula de la argirodita?

La fórmula es ________________

8. Escribe las reacciones del compuesto B con las disoluciones concentradas de HCl y NaOH.

PROBLEMA II.

El análisis volumétrico es uno de los métodos más ampliamente utilizados en determinaciones


cuantitativas en medio homogéneo o heterogéneo y consiste en añadir, desde una bureta, un reactivo
titulante que reacciona estequiométrica y cuantitativamente con la sustancia que se desea
determinar.
Con frecuencia las titulaciones se realizan añadiendo un indicador visual que cambia de color e
indica el momento en que debe ser detenida la adición del titulante.

230
Sin embargo, otra forma para determinar experimentalmente el punto final de la reacción es utilizar
un instrumento que permita medir alguna propiedad del sistema que varíe bruscamente en este punto
final; este abrupto cambio es el que permite determinar el volumen final requerido en la titulación.
De acuerdo con la reacción involucrada y con el instrumento utilizado para registrar las variaciones,
la propiedad medida puede relacionarse logarítmica (por ejemplo pH) o linealmente con la
concentración de una especie química. Cuando esto último ocurre, se obtienen rectas de pendientes
diferentes antes y después del punto de equivalencia, y el punto final de la titulación se determina
por la intersección de ambas rectas.
Tal es el caso de una valoración conductimétrica por precipitación en la cual la conductancia, G, de
una disolución (cuyas unidades son Siemens, S, y corresponden a la inversa de su resistencia al paso
de la corriente eléctrica) varía cuando los iones que se encontraban en disolución precipitan y son
sustituidos por otros que contribuyen de manera diferente a esta conductancia; cuando la
precipitación termina se registra un aumento de la conductancia si el titulante tiene iones o se
mantiene constante cuando no es iónico. En ambas situaciones se registra un cambio en la pendiente
de la recta obtenida.
En la siguiente gráfica se muestran los resultados obtenidos por Luis, curioso e inteligente estudiante
olímpico, quien registró los valores de conductancia obtenidos al añadir diferentes volúmenes de
una disolución 0,001 mol/L de BaCl2 a 100,0 mL de una disolución de sulfato de sodio cuya
concentración se requería conocer.

Titulación de sulfato de sodio


40
G (S)

36.3
35
30.3
30
26.0
25
22.0

20 18.9
16.2
15 13.5 13.3
13.5
13.4 13.2
10

0
0 1 2 3 4 5 6 7 8 9 10
Volumen agragado de cloruro de bario 0.001 mol/L (mL)

Luis observó que, en lugar de las dos rectas de pendiente diferente que se interceptarían en el
volumen correspondiente al punto final de la titulación, había obtenido una curva y que la
conductancia medida para dicho volumen era ligeramente mayor al que correspondía a la
intersección. Estaba aún preguntándose si había cometido algún error experimental cuando paso un
asesor y le dijo “recuerda que aunque la sustancia sea insoluble siempre habrá un poco de ella
disuelta”.

 ¿Podré entonces determinar cuánto está disuelto?

231
 Más aún, le contestó el asesor, podrás calcular su producto de solubilidad. La fórmula que
necesitas para hacerlo es:
G * kc = oi Ci zi

en donde o es una constante diferente para cada ion (expresada en S eq-1cm2), C es la concentración
de cada ion (expresada en moles/mililitro) y z su carga (en valor absoluto). kc es una constante del
instrumento.
DATOS
kc (para el equipo utilizado) = 1 cm-1
Valores de o para los iones (en S eq-1cm2): Na+ =52; SO42- =83; Ba2+ = 66; Cl- = 79.

1. Indica cuál es la reacción de precipitación del sulfato de bario.

2. Indica qué otros iones se encuentran en la disolución en el punto de equivalencia de la reacción


de titulación.

3. ¿Cuál debe ser la conductancia de los iones subproductos de la reacción?

4. ¿Cuál es la conductancia debida al sulfato de bario disuelto?

5. Indica cuál es la solubilidad del sulfato de bario que Luis calculó (en mol/L).

6. Calcula cuál es el valor del producto de solubilidad, Kps, del BaSO4.

232
Fisicoquímica.

PROBLEMA III.

Para bucear se utiliza aire comprimido en tanques con diferentes capacidades 5, 10, 12, 15 y 18
litros. Siendo la composición del aire normal 20% en volumen de oxígeno, 79% de nitrógeno y 1%
de otros gases. Sin embargo, en ciertos casos se utilizan otras mezclas. Una de las más usadas es la
mezcla “EAN” (Enriched Air Nitrox), la cual es aire con oxígeno extra, a menudo con 32% ó 36%
de oxígeno, y por lo tanto, con un % de nitrógeno menor.

Estos niveles de oxígeno en la mezcla EAN ayudan para prevenir los problemas de la
descompresión; sin embargo, a profundidades mayores a la Profundidad Máxima de Operación
(PMO) el aumento en la presión parcial del oxígeno puede llevarlos al nivel de toxicidad del
oxígeno. Para desplazar al nitrógeno sin incrementar la concentración de oxígeno se pueden usar
otros diluyentes, como el helio; la mezcla así resultante se le denomina “Trimix”.

Convencionalmente, la mezcla es nombrada por su porcentaje de oxígeno, su porcentaje de helio y


opcionalmente el porcentaje complementario de nitrógeno. Por ejemplo, la mezcla denominada
Trimix 10/70 consiste de 10% en volumen de oxígeno, 70% de helio y por supuesto 20% de
nitrógeno.

Al disminuir el contenido de oxígeno se incrementa la profundidad máxima de operación y la


duración de la inmersión a la cual la toxicidad del oxígeno se convierte en un factor limitante.

Los límites recomendados para buceo son:

Presión parcial de Tiempo de buceo


oxígeno (bar) (min)
1.6 45
1.5 120
1.4 150
1.3 180
1.2 210

En buceo técnico, la profundidad máxima de operación PMO se calcula con la fórmula:


 ppO 2  
PMO  33 pies    1
 FO 2  
En donde ppO2 es la presión parcial deseada del oxígeno y FO2 es el valor decimal de la fracción del
oxígeno en la mezcla. Por ejemplo, si un gas contiene 36% de oxígeno y la presión parcial máxima
del oxígeno es 1.4 bar, la PMO es: 33 pies x[(1.4/0.36)-1]=95.3 pies.
La presión de llenado de los tanques va de 232 a 300 bar.

233
Se tiene un tanque de buceo de 12.0 L y se va a cargar a una presión de 232 bar y a la temperatura
ambiente de 20.0 0C. (Suponga comportamiento ideal)

1. Si el tanque se carga con aire normal y la masa molar del aire es 29 g/mol, ¿cuántos gramos de
aire se necesitan para cargar el tanque?

a) 3257.0 g b) 3312.6 g c) 3884.3 g d) 38843.4 g e) 48554.0 g

2. Suponiendo que en el aire normal sólo hubiera un tercer gas “X” (además del oxígeno y del
nitrógeno), ¿cuál sería la masa molar aparente de este gas “X” (en g/mol)?

a) 28.52 b) 28.81 c) 30.00 d) 30.88 e) 48.00

3. Suponiendo que la masa molar del gas “X” es 48.00 g/mol, ¿cuántos gramos de este gas “X”
habrá en el tanque?
a) 53.91 g b) 54.83 g c) 539.1 g d) 642.9 g e) 803.7 g

4. Suponga que el gas “X” está constituido sólo por bióxido de carbono (CO2) y SO2, la fracción
mol de estos gases son respectivamente:

a) 0.002 y b) 0.004 y c) 0.006 y d) 0.008 y e) 0.009 y


0.008 0.006 0.004 0.002 0.001

5. La masa del CO2 en el tanque es:

a) 10.05 g b) 20.10 g c) 30.16 g d) 40.20 g e) 50.26 g

En una sesión de buceo se van a utilizar tanques de 18 L con la mezcla Trimix 10/70:

6. El porciento en masa de oxígeno en la mezcla Trimix 10/70 es:

a) 21.4 % b) 24.1 % c) 27.6 % d) 48.3 % e) 51.7 %

Se planea hacer un recorrido de 45 minutos en los arrecifes de coral. Los buzos usarán una mezcla
EAN con 32% de oxígeno:

7. La profundidad máxima a la que pueden sumergirse es:

a) 197 pies b) 170 pies c) 164 pies d) 132 pies e) 62 pies

En otro recorrido se planea usar Trimix 15/70 a una profundidad de 275 pies.

8. El tiempo recomendado para esta inmersión es:

a) 45 min b) 120 min c) 150 min d) 180 min e) 210 min

234
PROBLEMA IV.
REFINACIÓN DE COBRE.
México es un importante productor de cobre a nivel mundial. Uno de los métodos de purificación
del cobre llamado electro-refinación, se basa en un proceso electroquímico en el que un electrodo de
cobre de baja pureza actúa como ánodo y se disuelve mientras que un electrodo de titanio actúa
como cátodo, depositándose sobre el mismo una capa de cobre de alta pureza. Las impurezas que
contenía el cobre de baja pureza quedan disueltas en la disolución electrolítica (llamada electrólito).

1. En una celda de electro-refinación que consta sólo de dos electrodos, la reacción o reacciones de
reducción se llevan a cabo en el: (Marca el cuadro de la respuesta correcta)

Cátodo Ánodo

2. Si el electrólito es una disolución de CuSO4 en un medio de ácido sulfúrico diluido en agua, al


paso de una corriente eléctrica por la celda pueden esperarse al menos dos reacciones de
reducción. Escribe dos de las posibles reacciones de reducción:

_______________________________________
_______________________________________
_______________________________________

3. Para preparar el electrólito, un estudiante utiliza ácido sulfúrico concentrado que tiene una
densidad de 1.84 g/cm3 y un 96.0% en masa del ácido. Calcula la concentración molar de este
ácido concentrado:

CÁLCULOS

La concentración molar del ácido concentrado es: _________________________________

La corriente que circula por una celda electroquímica se mide en amperes (A).Un ampere equivale al
paso de una carga de un coulomb (C) cada segundo, por un punto del circuito. Tome en cuenta que
la carga de un electrón es 1.60 x 10-19 C y la constante de Avogadro (N) es 6.022 x 1023.

4. Cuando la corriente es de un miliampere, 1.0 mA, la cantidad de electrones que pasa por un punto
del circuito cada segundo es:

CÁLCULOS

Cantidad de electrones :________________________________________

5. Después de diez horas del funcionamiento de una celda electroquímica, manteniendo la corriente
constante, se determina que por un punto del circuito ha pasado un mol de electrones. Por lo
tanto, la corriente que se ha mantenido en la celda es:

235
CÁLCULOS

Corriente _______________________________________A

6. En un experimento de electro-refinación de cobre, el estudiante utiliza un cátodo de titanio cuya


masa inicial es de 10.0 g. Después de diez minutos de estar funcionando la celda, a corriente
constante de 1.0 A, extrae dicho electrodo y determina que su masa es de 10.140 g. Durante el
experimento el estudiante nota que sobre la pieza de titanio se va depositando una película de
cobre pero también ha observado la formación de un gas. Calcula la cantidad de moles de gas
producidos en el cátodo:

CÁLCULOS:

El gas producido es: ____________


Los moles de este gas producidos son: __________________

236
Química Orgánica.

PROBLEMA V.
1. Sugiera los reactivos apropiados para llevar a cabo cada uno de los pasos de la siguiente
secuencia sintética:

237
PROBLEMA VI.
1. Complete la siguiente secuencia sintética, escribiendo los reactivos necesarios para llevar a cabo
cada una de las reacciones indicadas:

RESPUESTAS

a) b) c) d) e)

f) g h) i) j)

238
PROBLEMA VII.
1. Complete la siguiente secuencia de reacciones a través de las cuales se ilustra la química de los
carbohidratos, escribiendo la estructura de cada uno de los productos formados:

O H
Br2, H2O
H OH A
HO H
CH2OH

NaBH4
B
CH3OH

HNO3
C

1) NaCN, H2O, EtOH


D + E
2) a) NaOH, H2O, CALOR
b) HCl, H2O
3) Na (Hg), H2O

RESPUESTAS

A B C D E

a) Diga si B es ópticamente activo.

239
b) Designe el descriptor estereoquímico para especificar la configuración absoluta de los carbonos
estereogénicos en la materia prima y en los productos A al E.

c) ¿Cuál es la relación que existe entre los estereoisómeros D y E?

240
PROBLEMA VIII.

1. El compuesto A tiene la fórmula C6H12O. No es ópticamente activo y no puede separarse en dos


enantiómeros. No adiciona hidrógeno en una hidrogenación catalítica. Cuando A se hace
reaccionar con el reactivo de Lucas, el precipitado blanco aparece a los 10 minutos. Cuando A se
calienta, con H2SO4, se obtienen dos productos, B y C, los cuales son enantiómeros con fórmula
C6H10.
La ozonólisis del compuesto B, seguida de una ruptura reductiva con sulfuro de dimetilo,
produce el compuesto D, el cual es un ceto-aldehído con fórmula C6H10O2. La ozonólisis de C,
seguida de una ruptura reductiva con sulfuro de dimetilo, produce el compuesto E, el cual es un
ceto-aldehído, ópticamente activo y es el enantiómero del compuesto D.

Con base sólo en estos datos, escriba las estructuras de los compuestos A al E, INDICANDO
CON CLARIDAD LA ESTEREOQUÍMICA DE TODOS ELLOS.
RESPUESTAS

A B C

D E

241
Química Inorgánica.

PROBLEMA IX.

Los electrodos tipo II se fabrican cubriendo un metal con una sal muy poco soluble del metal. El
metal recubierto se sumerge en una disolución que contiene disuelto un compuesto iónico del anión
de la sal insoluble. Los electrodos de plata/cloruro de plata (Ag∣AgCl(s), Cl-) y de calomel
(Hg∣Hg2Cl2(s), Cl-) son ejemplos de este tipo de electrodos. La fem estándar de una celda construida
con los electrodos anteriores Ag∣AgCl(s), Cl-∥Cl-, Hg, Hg2Cl2(s)∣Hg es E0 = 0,0455 V a T = 298 K.
ΔE 0
El coeficiente de temperatura para esta celda es  3,38x10  4 V ; F = 96485 coulombios .
ΔT K mol e _

1. Da las ecuaciones de las reacciones que tienen lugar en los electrodos de la celda y la reacción
general de la celda.

Ánodo:

Cátodo:

Global:

0
2. Calcula el cambio de energía libre de Gibbs (G ) para el proceso que tiene lugar en la celda.
¿Qué implica su signo?

Cálculos:

Signo:
0
G =

242
ΔE 0
3. Si ΔS  nF , calcula el cambio de entalpía para el proceso a 298 K.
ΔT

Cálculos:

H =

4. Sabiendo que el potencial estándar del electrodo de Ag ∣ Ag es E = 0,799 V y el producto de


+ 0

solubilidad para AgCl es Kps = 1,73x10-10, calcula el valor para el potencial estándar del electrodo
de plata/cloruro de plata. Deduce una expresión que muestre la relación ente E0 (Ag∣Ag+) y
0
E (Ag∣AgCl(s), Cl-).

Cálculos:

E0 (Ag ∣ AgCl(s), Cl-) = Expresión:

5. Calcula el producto de solubilidad de Hg2Cl2, sabiendo que el potencial estándar del electrodo del
par Hg22+/Hg es E0 = 0,789 V.

Cálculos:

Kps =

FIN DEL EXAMEN

243
244
XIX Olimpiada Nacional de Química
Examen Experimental de Química Inorgánica Veracruz 2010

Problema.
Identificar los 15 reactivos asignados utilizando el comportamiento químico de cada uno de ellos al
ponerlos en contacto entre si.
Instrucciones experimentales.
Se te proporcionan 15 frascos goteros con sustancias en disolución acuosa (CONSULTA LA LISTA
DE POSIBILIDADES ANTES DE INICIAR EL TRABAJO EXPERIMENTAL), las cuales
identificarás haciéndolas interactuar entre ellas ya que únicamente podrás utilizar adicionalmente
indicador universal para determinar el pH o en su defecto papel pH suficiente para su identificación.
Como material adicional se te proporciona una hoja cuadriculada en cuya superficie colocarás un
acetato que te permitirá observar reacciones a la gota.
Cuida los volúmenes que utilizas ya que NO se te proporcionarán reactivos extra si te acabas alguno
de ellos. ÚNICAMENTE EN CASOS EXCEPCIONALES Y APLICANDO UNA
PENALIZACIÓN).
UTILIZA UNA GOTA DE CADA UNO DE ELLOS PARA LAS PRUEBAS QUE SE TE
PROPONEN.

Procedimiento experimental.
1. Determina el pH de cada una de las disoluciones propuestas y regístralos en la tabla 1. Utiliza
únicamente UNA GOTA de cada reactivo.
2. Si por sus características físicas (color u organolépticas) puedes formular alguna hipótesis de qué
reactivo se trata, también anótalo en la tabla.
3. A la disolución 12 añádele por separado los reactivos 4 y 5, y observa los precipitados formados.
4. Al reactivo 13 añádele el reactivo 3 y observa si hay algún cambio.
5. Al mismo reactivo 13 añádele por separado los reactivos 4 y 5, y observa el comportamiento al
mezclarlos.
6. Para asegurar la identificación de 13 añádele a una gota de este reactivo otra gota del reactivo 1 y
observa el precipitado característico que se forma.
7. Por separado añade el reactivo 13 a algunas otras de las disoluciones asignadas (especialmente a
la 14) y saca conclusiones.
8. Para identificar el reactivo 6, se te sugiere poner una gota de éste y añadirle una gota del reactivo
2.
9. Para comprobar la identidad de 6, añádele una gota del reactivo 4 y observa el cambio de color
debido a la formación de un compuesto de coordinación característico.
10. El reactivo 11 reacciona por separado con los reactivos 1, 6, 7, 8, 9, 10, 12, 14 y 15 dando
precipitados característicos.
11. Los reactivos 15 y 17 reaccionan con el reactivo 5 dando precipitados que se redisuelven en
exceso del reactivo 5.
12. Los reactivos 1 y 14 reaccionan con el reactivo 3 dando precipitados blancos característicos.
Uno de ellos es soluble en caliente.
13. También los reactivos 1 y 14 reaccionan con el reactivo 2 dando también precipitados
característicos.
14. Realiza algunas otras interacciones que te permitan identificar los reactivos asignados.

245
LAS 15 DISOLUCIONES A IDENTIFICAR SON:
Nitrato de cobre (II), nitrato de zinc, nitrato de aluminio, nitrato de plomo, nitrato de hierro (III),
nitrato de manganeso (II), nitrato de plata, cromato de potasio, sulfuro de sodio, hidróxido de sodio,
ácido clorhídrico diluido, yoduro de potasio, nitrato de cadmio, amoniaco en disolución acuosa y
nitrato de níquel (II).

Tabla 1.
No. de Hipótesis pH Especie Ecuaciones que representan el proceso
muestra (Fórmula) identificada (Mínimo 2 por cada especie)
(Fórmula)
1
2
3
4
5
6
7
8
9
10
11
12
13
14
15

Cuestionario.
1. Una vez identificado el sulfuro de sodio, ¿qué pH presentaba y a qué se debe ese
comportamiento? Justifícalo con las ecuaciones correspondientes.

2. ¿Cuáles eran las especies coloridas y a qué se debe ese color?

3. ¿Qué diferencia se observa entre los reactivos 4 y 5, y a qué se debe esa diferencia?

4. ¿Qué sulfuros NO son negros? ¿Cuáles de ellos obtuvo experimentalmente?

5. ¿Cuáles de las especies asignadas se comportaron como anfóteras?

6. ¿Qué especies dan reacciones básicas y a qué se debe este comportamiento?

7. ¿Qué especies dan reacciones ácidas y a qué se debe este comportamiento?

8. Mencione 4 sulfuros negros obtenidos durante el proceso experimental.

9. ¿A qué se deben los cambios de intensidad y/o color de la especie cromato en medio ácido y
básico?

10. Mencione al menos 2 cationes que precipitan como cloruros.

246
XIX Olimpiada Nacional de Química
Examen Experimental de Química Orgánica Veracruz 2010

Identificación de alcoholes primarios desconocidos por medio de la 2,4-dinitrifenilhidrazona


del aldehído correspondiente.

Alcohol problema No. ______

Objetivos.
a) Que el alumno prepare el agente oxidante necesario para realizar una reacción redox con un
alcohol primario.
b) Que el alumno efectúe una reacción de oxidación de un alcohol primario para obtener el aldehído
correspondiente.

Ecuación.

H
R
R C OH + Oxidante C O + Reductor
Calor
H H

Equipo.
3 Tubos de ensaye 1 Mechero de alcohol
1 Tapón monohoradado para el tubo de ensayo 2 Vasos de precipitados de 100 mL
1 Tubo de desprendimiento 1 Embudo büchner
1 Pinza para tubo de ensayo 1 Matraz kitazato

Sustancias.
Carbonato de cobre (II), 1 g Alcohol problema, 2 mL
Disolución acuosa de hidróxido de calcio, 2 mL 2,4-Dinitrofenilhicracina, 2 mL
Mezcla de etanol:agua (90:10), 10 mL Hexano, 2.5 mL
Acetato de etilo, 2.5 mL

Procedimiento.
En el tubo de ensaye de 10 mL, ya se encuentra 1 g de carbonato de cobre (II). Se adapta un tapón
de hule con un tubo de desprendimiento. Utilizando unas pinzas para tubo de ensaye para sostener el
tubo, éste se calienta utilizando un mechero de alcohol con una flama suave y el gas que se
desprende se burbujea dentro de otro tubo de ensaye en el que debes colocar la disolución de
hidróxido de calcio (la cual se encuentra dentro del frasco etiquetado Disolución Ca(OH)2).
Es importante que dejes enfriar el tubo de ensaye en el que ya se encuentra formado el agente
oxidante. Agrega 2 mL del alcohol problema (se encuentran los 2 mL exactos en el frasco con la
etiqueta que dice Alcohol problema). Vuelve a sujetar el tubo (conteniendo el agente oxidante y el
alcohol problema) con las pinzas y vuelve a calentar el tubo con una flama suave utilizando
nuevamente el mechero de alcohol. El gas que se desprende (el aldehído correspondiente) se
burbujea dentro de una disolución de la 2,4-dinitrofenilhidracina (frasco etiquetado como

247
Disolución 2,4-DNF) (2 mL) la cual debes colocar en el otro tubo de ensaye que se te proporciona.
Se debe formar un precipitado (la 2,4-dinitrofenilhidrazona del aldehído), el cual se aísla por medio
de una filtración al vacío, empleando un embudo büchner y un matraz kitazato, se seca y se
recristaliza con una mezcla de etanol:agua (90:10) (frasco etiquetado como Mezcla de etanol:agua
(90:10)). Se determina de RF llevando a cabo un análisis por cromatografía en capa fina, utilizando
como eluyente una mezcla de hexano:acetato de etilo (20:80) la cual se encuentra en el frasco
etiquetado como Eluyente y dentro del cual se encuentra exactamente 5 mL de dicho eluyente y el
mismo frasco va a servir como cámara de elución. Con esto, podrás saber qué alcohol problema se te
asignó. No se necesita utilizar un revelador, ya que las 2,4-dinitrofenilhidrazonas de los 3 aldehídos
posibles, para este experimento, son muy coloridas y se pueden ver a simple vista.

Cromatografía en capa fina.

Introducción teórica.
La cromatografía en capa fina es una técnica de adsorción sólido-líquido, utilizada en química
orgánica para realizar un análisis cualitativo eficiente y rápido de compuestos desconocidos o para
determinar la composición de una mezcla de compuestos.
Esta técnica cromatográfica consiste en la utilización de una fase estacionaria (alúmina o sílica gel)
y de una fase móvil, que son los disolventes orgánicos de diferente polaridad.
El proceso de separación se basa en que la fase móvil asciende a través de la fase estacionaria, la
elución de la muestra problema está en función de su polaridad, es decir, de la afinidad que presente
con alguna de las dos fases.
La fase estacionaria consiste de una película delgada (100 m) de un material disperso sobre una
superficie plana (de vidrio, aluminio o celulosa).
Las ventajas del método son la rapidez para llevar a cabo el análisis (un análisis normal toma de 2 a
10 minutos) y se pueden detectar cantidades de material de 2 a 20 g.
La secuencia de pasos que se siguen para llevar a cabo el análisis por cromatografía en capa fina
son:
1. Se traza una línea paralela a la base de la cromatoplaca (aproximadamente a 0.5 cm de la base de
la misma) sobre la superficie del adsorbente con un lápiz (NOTA: no recargues mucho la punta
del lápiz, ya que se puede llegar a romper la superficie de la fase estacionaria). La muestra que se
va a analizar (1 mg o menos) se coloca en un tubo de ensaye, y se adicionan unas gotas del
disolvente hasta observar la disolución de la muestra. Con un tubo capilar se aplica una pequeña
fracción de la disolución sobre la placa.

Parte superior 0.5 cm


Límite de ascención
del disolvente

X X Punto de aplicación
Parte inferior 0.5 cm

248
2. La cromatografía se lleva a cabo colocando la cromatoplaca sobre la cual se aplicó la muestra,
dentro de un frasco de vidrio con tapa (son ideales para este fin los frascos de alimentos
infantiles), el cual ya debe contener la fase móvil (unos 3 mL). El frasco debe estar cerrado con la
finalidad de mantener una atmósfera saturada con el disolvente de la fase móvil. Efectuadas estas
operaciones, la fase móvil asciende rápidamente por capilaridad sobre el adsorbente de la fase
estacionaria hasta la marca superior. Inmediatamente se saca la placa de la cámara de elución y se
deja evaporar el disolvente. La muestra se puede encontrar en cualquier región de la placa.
3. Para localizar los compuestos de la muestra problema, se utiliza un revelador. En este caso vamos
a utilizar vapores de yodo. Una medida física de la polaridad de la muestra es el Factor de
Retención (RF), el cual se determina dividiendo la distancia que recorrió la muestra entre la
distancia que recorrió el disolvente, el RF siempre es menor a 1. Bajo las mismas condiciones de
análisis cromatográfico, el RF es constante y permite identificar cualitativamente un compuesto o
bien determinar si se encuentra presente en una mezcla problema.

Parte superior
Línea hasta la que recorrió
el eluyente

b
a
RF=
b

X Punto de aplicación de la muestra


Parte inferior

Preguntas.
1. Escribe las ecuaciones balanceadas de los procesos químicos que ocurrieron cuando calentaste el
carbonato de cobre (II) y el gas que se generó lo hiciste burbujear en la disolución de hidróxido
de calcio.

2. Escribe la ecuación balanceada del proceso químico que ocurrió cuando calentaste el alcohol
problema con el agente oxidante que se generó en la pregunta anterior.

3. Escribe la ecuación que explique el proceso químico de formación de la 2,4-dinitrofenilhidrazona


del aldehído.

4. Determina el RF de la 2,4-dinitrofenilhidrazona que obtuviste. Marca las distancias y muestra


cómo lo determinaste.

5. Con base a tu respuesta a la pregunta anterior, ¿cuál fue tu alcohol problema y cuál fue el
aldehído que formaste en la reacción redox?

249
Toma en cuenta la siguiente información:

O2 N RF
H
H
R
R C OH R N NO2
C O
H C N
H
H
O2 N
H
H
OH
0.33
H N NO2
O
N

O2 N

H
H
H N NO2
0.53
OH O N

O2 N

H
H
H N NO2

OH O N 0.49

250
XIX Olimpiada Nacional de Química
Examen Experimental de Química Analítica Nivel A Veracruz 2010

DETERMINACIÓN DE LA COMPOSICIÓN DE UNA MUESTRA DE BICARBONATO


Y/O CARBONATO DE SODIO.

En este laboratorio se ha recibido una donación de varios frascos que contienen un polvo blanco que
puede contener carbonato de sodio y/o bicarbonato de sodio. Se nos informó que, ocasionalmente,
pudiera contener alguna pequeña cantidad de alguna otra sal sódica sin propiedades ácido-base.

La apariencia de estos compuestos es similar pero no ocurre lo mismo con sus aplicaciones.

El bicarbonato de sodio se utiliza para repostería, para eliminar malos olores, como antiácido en la
elaboración de medicamentos, en la industria del curtido, en extintores de incendio, en el tratamiento
de lana y seda, etc.

El carbonato de sodio es usado, entre otras cosas, en la fabricación de jabón y vidrio, como
ablandador de aguas duras, en refinación de petróleos, producción de aluminio, textiles, pulpa y
papel, y en otros procesos industriales.

Por tanto, es de suma importancia que se identifique el(los) componente(s) mayoritario(s) de la


muestra y que se determine su composición porcentual. Tu labor esta mañana será ayudarnos a
lograrlo.

Hemos adelantado el trabajo midiendo una muestra de 10,00 g de uno de los frascos, la cual
disolvimos con agua destilada completando hasta un volumen de 1,00 litro.

De esta disolución te proporcionamos (en un frasco etiquetado con un número) un volumen


aproximado de 50 mL. Un segundo frasco contiene una disolución de ácido clorhídrico cuya
concentración exacta te será indicada por un asesor.

Deberás tomar alícuotas de la disolución problema y realizar titulaciones con la de HCl utilizando
los indicadores que se proporcionan para uso general y cuyos intervalos de pH de vire se indican a
continuación:

Fenolftaleína: 8 a 9,8, incolora en forma ácida y rosa bugambilia en forma básica.


Verde de bromocresol: 3,8 a 5,4, amarilla en forma ácida y azul en forma básica.

251
RESULTADOS.

Titulación de la muestra comercial.

Titulación No. Alícuota de Volumen de HCl gastado con Volumen de HCl gastado con el
la muestra el primer indicador elegido seguno inicador elegido
1
2
3

1. Indica cuál fue el primer indicador utilizado: ________________________________________

2. ¿La adición de este indicador te proporcionó alguna información? Si tu respuesta es positiva


indica qué observaste y cuál fue tu conclusión?
______________________________________________________________________________
____________________________________________________________________________

3. Señala el inciso que corresponde al contenido de los componenetes mayoritarios de la muestra


analizada:

a) Únicamente carbonatos b) Únicamente bicarbonato

c) Una mezcla de carbonato y bicarbonato

4. Cálculos efectuados para determinar la composición cuantitativa de la muestra indicada en


porcentaje de el(los) componente(s).

252
XIX Olimpiada Nacional de Química
Examen Experimental de Química Analítica Nivel B Veracruz 2010

DETERMINACIÓN DE LA PUREZA DE UNA MUESTRA COMERCIAL DE


BICARBONATO DE SODIO.

El bicarbonato de sodio es uno de los compuestos de usos más versátil. Se utiliza para repostería,
para eliminar malos olores, como antiácido en la elaboración de medicamentos, en la industria del
curtido, en extintores de incendio, en el tratamiento de lana y seda, etc.

En este laboratorio se ha recibido una donación de varios frascos que contienen bicarbonato de sodio
cuya pureza se requiere determinar. Tu labor esta mañana será ayudarnos a lograrlo.

Hemos adelantado el trabajo midiendo una muestra de 10,00 g de uno de los frascos, la cual
disolvimos con agua destilada completando hasta un volumen de 1,00 litro.

De esta disolución te proporcionamos (en un frasco etiquetado con un número) un volumen


aproximado de 50 mL del cual deberás tomar alícuotas y titular con la disolución de ácido
clorhídrico cuya concentración exacta te será indicada por un asesor. Como indicador deberás poner
una o dos gotas de verde de bromocresol.

Datos: El intervalo de vire del indicador es de 3,8 a 5,4, amarillo en forma ácida y azul en forma
básica.

RESULTADOS.

Titulación de la muestra comercial No. ________

Titulación No. Alícuota de la muestra Volumen de HCl gastado


1
2
3

Volumen que utilizarás en tus cálculos: ______________________

Cálculos efectuados y resultado, para obtener el porcentaje de pureza de la muestra de bicarbonato.

253
254
XX Olimpiada Nacional de Química
1er Examen Nivel A y B.
Total: 30 preguntas. Tiempo asignado: 80 minutos. Toluca 2011

Anota en el recuadro correspondiente la letra del inciso que contesta correctamente cada pregunta.
Si consideras que ningún inciso es el correcto deberás anotar una letra X. USA LETRAS
MAYÚSCULAS Y ESCRIBE TUS RESPUESTAS CON TINTA.
NOTA ACLARATORIA: Al hacer tus cálculos puedes encontrar una pequeña diferencia, en
general no mayor de unas décimas, entre tu resultado y alguno de los propuestos. Esto es
normal y por eso sólo deberás usar la “X” cuando ninguna respuesta sea muy cercana a la
obtenida por ti.

********************************************************************************

En la química, así como en otras ciencias, es importante desarrollar la habilidad de recopilar


información y sacar conclusiones correctas. Con base en tus conocimientos y los datos de la tabla
siguiente, contesta las preguntas 1-10:

Nombre de la Fórmula Punto de fusión Punto normal de ebullición en


sustancia en oC o
C (a 1 atm)
Metanol CH3OH - 98 65
Etanol C2H5OH - 130 78
1-Propanol C3H7OH - 126.5 97.1
Agua H2O 0.0 100
Fenol C6H5OH 41 182
2-Propanol C3H7OH - 89 82.5
1 mmHg = 133.322 Pa 1 atm = 1.01325 x 105 Pa = 760 mmHg

1) Si tomamos una muestra de 18.0 g de cada una de estas sustancias, ¿en cuál de las muestras habrá
una mayor cantidad de sustancia en moles?
A Metanol B Etanol C 1-Propanol D Agua

2) Al mezclar cantidades iguales, en gramos, de metanol y agua, la fracción mol del agua en esta
disolución es:
A Menor a B Entre 0.30 y C Entre 0.61 y D Mayor de
0.30 0.60 0.80 0.80

3) A una temperatura de 310 K y P = 760 mmHg, ¿cuántas de estas sustancias se encuentran en


estado líquido?
A Seis B Cinco C Cuatro D Tres

255
4) A una temperatura de 270 K y P = 760 mmHg, ¿cuántas de estas sustancias se encuentran en
estado líquido?
A Cuatro B Tres C Dos D Una

5) ¿Cuántas de estas sustancias son isómeros?


A Cuatro B Tres C Dos D Una

6) Si disolvemos 2.0 g de fenol en 50.0 mL de agua, ¿cuál es la molaridad del fenol en esta
disolución?
A Menos de B Entre 0.15 C Entre 0.31 D Más de 0.50
0.15 mol L-1 mol L-1 y 0.3 mol L-1 y mol L-1
mol L-1 0.50 mol L-1

7) En la molécula de etanol, el % en masa de carbono es:


A Menos de B Entre 30% y C Entre 40% y D Más de 50%
30% 40% 50%

8) Cuando se quema un mol de metanol con suficiente oxígeno gaseoso se obtienen, de moles de
agua:
A 0.5 B 1.0 C 2.0 D 4.0

9) ¿Cuántos gramos de oxígeno gaseoso se requieren para la combustión completa de 92 g de


etanol? Recuerda: en la combustión completa de un hidrocarburo sólo se producen agua y
dióxido de carbono.
A Menos de 90 B Entre 90 g y C Entre 140 g D Más de 190
g 140 g y 190 g g

10) La ley de los gases ideales es PV=nRT. Receurda: (R=0.082 L atm/mol K ), (R=8.314 J/mol K),
(1 J = kg m2 s-2), (1 atm = 101.325 kPa), (1 Pa = 1 N m-2), (1 N = 1 kg m s-2). Si 6.0 g de
1-propanol están a una P = 101 kPa y T = 110 oC, esta sustancia ocupa un volumen de:
A Menos de B Entre 1.0 L C Entre 3.0 L D Más de 5.0
1.0 L y 3.0 L y 5.0 L L

11) Supón que existe un átomo X, que tiene una masa que es cuatro veces la de un átomo de
carbono. ¿Qué masa de X, en gramos, se combinará con un gramo de carbono al formar el
compuesto X2C?
A 2.0 B 4.0 C 8.0 D 48.0

12) Un elemento M, forma un bromuro de fórmula MBr2. Si este compuesto tiene un 26.9% en masa
de M, la masa molar de M, en g/mol es:
A Menos de 30 B Entre 30 y C Entre 60 y D Más de 120
60 120

256
13) En forma comercial, el sulfato de zinc se vende en forma de heptahidrato (cada molécula del
compuesto contiene siete moléculas de agua de hidratación). En este hidrato, el % en masa de
zinc es aproximadamente:
A Menos del B Entre 12 y C Entre 20 y D Más del
12% 20% 30% 30%

14) Bajo ciertas condiciones, el sulfato de calcio también forma un hidrato. Cuando una muestra de
3.5 g de este hidrato se calienta para eliminar el agua, el residuo que queda tiene una masa de
2.77 g. Por cada molécula del sulfato de calcio, ¿cuántas moléculas de agua tiene este hidrato?
A 1 B 2 C 4 D 7

15) Si se mezcla medio litro de disolución acuosa 0.2 molar (mol L-1) de cloruro de potasio con
100.0 mL de una disolución 0.5 molar del mismo compuesto, la concentración molar del cloruro
de potasio en la disolución obtenida es:
A 0.15 mol L-1 B 0.20 mol L-1 C 0.25 mol L-1 D 0.30 mol L-1

16) Una muestra de cierto hidrocarburo se quema completamente y produce 132 mg de CO2 y 27.1
mg de agua. La fórmula empírica de este hidrocarburo es:
A CH B CH2 C CH3 D C2H

17) 1.22 kg de KClO3 se descomponen para formar cloruro de potasio y oxígeno gaseoso. Cuando se
ha descompuesto un 80% del reactivo, los gramos que se han obtenido del gas son:
A Menos de B Entre 300 g C Entre 400 g D Más de 500
300 g y 400 g y 500 g g

18) Si agregamos unas gotas de una disolución de HCl 3 mol L-1 a otra disolución que contiene
iones Na+, Ca2+ y Pb2+, lo que observará es:
A Sólo se B Sólo se C Se forman D Sólo se
forma un forma un precipitados forma un
precipitado precipitado de CaCl2 y precipitado
de NaCl de CaCl2 PbCl2 de PbCl2

19) Una disolución saturada de Ca(OH)2 en agua a 25 oC tiene una concentración 0.011 mol L-1 de
Ca(OH)2. La constante del producto de solubilidad de este compuesto a 25 oC es:
A 1.1 x 10-2 B 1.2 x 10-4 C 1.3 x 10-6 D 5.3 x 10-6

20) Una mezcla de la misma cantidad en gramos de etanol y de metanol se quema en presencia de
oxígeno. Suponer que ambos compuestos se consumen con la misma rapidez (medida en
moles/s). Cuando se ha consumido todo el etanol, se han producido 396 g de CO2. ¿Cuántos
moles de metanol quedan aún sin reaccionar?
A No queda B Entre 0.10 y C Entre 1.41 y D Más de 1.60
nada de 1.40 moles 1.60 moles moles
metanol

257
21) Un alcohol lineal, sin enlaces dobles ni triples, tiene una masa molar de unos 130 g/mol. Esta
molécula debe tener, de átomos de carbono:
A 9 B 8 C 10 D 7

22) Si se utiliza una reacción electroquímica para depositar oro metálico a partir de una disolución
que contiene iones Au+3, el enunciado correcto es:
A El oro metálico se deposita en el ánodo por oxidación de los iones
Au+3
B El oro metálico se deposita en el ánodo por reducción de los iones
Au+3
C El oro metálico se deposita en el cátodo por oxidación de los iones
Au+3
D El oro metálico se deposita en el cátodo por reducción de los iones
Au+3

23) El gas LP consta básicamente de propano (C3H8) y butano (C4H10). Si suponemos que contiene
el 75% en masa de propano, ¿cuántos moles totales habría, aproximadamente, en un tanque que
contiene 25 kg de la mezcla?
A Menos de B Entre 525 y C Entre 540 y D Más de 550
525 moles 540 moles 550 moles moles

24) La corriente que circula en una celda electroquímica se puede medir en amperes (A). Un ampere
equivale a una carga de un coulomb (C) que pasa cada segundo por un punto del circuito. La
carga de un electrón es 1.6 x 10-19 C. Si en una celda electroquímica circula una corriente de 50
mA, significa que el número de electrones que pasan por un punto del circuito cada segundo
son:
A 3.125 x 10-17 B 3.2 x108 C 3.2 x 10-16 D 3.125 x 1017

25) En un electrodepósito de plata, la reacción es: Ag(CN)2- (ac) + e-  Ag (s) + 2 CN- (ac).
¿Cuántos gramos de plata se depositan después de 28.0 minutos a corriente constante de 4.50
amperes?
A Menos de B Entre 3.0 g y C Entre 7.0 g y D Más de 11.0
3.0 g 7.0 g 11.0 g g

26) El catión metálico que al reaccionar con un sulfuro produce un precipitado negro, con un cloruro
da un precipitado blanco y con un yoduro, un precipitado amarillo es:
A Ag + B Au3+ C Mn2+ D Cd2+

27) Un compuesto contiene los elementos M y N en proporción 43.63% con una composición
porcentual en masa de 43.7% de M y 56.3% de N. Se conoce además que la masa molar de M
es 1.94 veces mayor que la de N. La fórmula empírica del compuesto es:
A MN B M2 N C M2N3 D MN2

258
28) Se tiene una disolución acuosa de ácido sulfúrico (H2SO4) de concentración 2.50 M
(M = molar = mol L-1) y cuya densidad es de 1.15 g / cm3. ¿Cuántos gramos de agua hay en un
litro de este ácido diluido?
A 905 B 1150 C 1000 D 245

29. Una mezcla de nitrógeno, oxígeno y CO2, todos gaseosos, contiene un total de 3 moles. Si la
fracción mol del nitrógeno es 0.4, la masa de nitrógeno presente en la mezcla es:
A 1.2 g B 11.2 g C 33.6 g D 28 g

30. Si a la masa atómica del carbono se le asignara el valor de 50 en vez de 12, ¿cuál sería la masa
molecular del H2O consistente con ese nuevo valor?
A Menos de 15 B Entre 15 y C Entre 25.1 y D Más de 70
25 70

FIN DEL EXAMEN

259
260
XX Olimpiada Nacional de Química. 2do Examen Nivel A y B.
Tiempo asignado: 90 minutos. Toluca 2011.

1. En el planeta Arakna, muy similar al nuestro, los científicos han agrupado sus elementos en una
tabla periódica similar a la nuestra. Estos elementos son los mismos que en la Tierra pero se les han
nombrado de distinta manera, a continuación se muestra un fragmento de ella:

Sd Dn Ln Py Jn
Dl Rm Mr Tñ Gr
Dv Qr Mu St Ov

Algunos estados de oxidación de estos elementos son: Dl3+, Ov5+, Tñ2–, Ln3–, Mu5+, Qr4+, St6+,
Jn1–. Con base en esta información contesta las siguientes preguntas

a) Ordena las siguientes especies químicas en orden creciente de tamaño:


Dv3+, Rm4–, Gr1–, Tñ2– y Mr3–

b) Predice la(s) fórmula(s) del(los) posibles compuestos que se formarían entre los elementos:
(Dv y Gr)

Pon una X en la respuesta correcta:


Mayor Igual Menor
c) El elemento Dn con respecto al Qr tendrá un número de
electrones de valencia... ( ) ( ) ( )

d) El elemento Tñ con respecto al Dv tendrá un carácter


metálico... ( ) ( ) ( )

e) El elemento Rm con respecto al Gr tendrá un radio


atómico... ( ) ( ) ( )

f) El elemento Dl con respecto al St tendrá un número de


electrones de valencia... ( ) ( ) ( )

g) El hidrácido del elemento Jn con respecto al del Ov


tendrá un carácter ácido... ( ) ( ) ( )

h) El elemento Py respecto al elemento Sd tendrá una


electronegatividad… ( ) ( ) ( )

i) El elemento Jn respecto al elemento Mu tendrá una


primera energía de ionización... ( ) ( ) ( )

261
********************************************************************************

2. El Uranio (Z = 92) es un elemento radiactivo que existe en la naturaleza en la forma de 2 isótopos


naturales: 238U (99.3%, t½ = 4.46109 años) y 235U (0.7%, t½ = 7.04108 años), los cuales decaen
por emisión de partículas α. El decaimiento de ambos isótopos es distinto y se lleva a cabo por
varias secuencias de emisiones α y β terminando con los isótopos estables de plomo (Z = 82)
206
Pb y 207Pb, respectivamente. La radiación gama que aparece en varias desintegraciones no
afecta en las transmutaciones. Con esta información, calcula el número total de partículas α y β
emitidas en cada una de las dos secuencias completas de decaimiento (238U 206Pb y
235
U 207Pb). Recuerda que una partícula α es un núcleo de helio y una β es un electrón emitido
por el núcleo.

En las preguntas 3 a 8, escribe en el renglón la letra del inciso correcto.

3. Una muestra de 0,01 moles del cloruro de un elemento X reaccionan completamente con 100 cm3
de una disolución 0,2 mol/L de nitrato de plata. ¿Cuál es la identidad de dicho elemento?

A K B Ca C Al D Si

_______________

4. Al calentar 20,5 g de sulfato de cobre hidratado y perder las moléculas de agua de hidratación se
obtuvo una masa de 13,1 g. ¿Cuál es la fórmula de la sal?

A CuSO4·2 H2O B CuSO4·3 H2O C CuSO4·4 H2O D CuSO4·5 H2O

_______________

5. Cuando el pH de la sangre es menor a 7,4 se produce la llamada acidosis. El pH de la sangre se


mantiene como consecuencia del siguiente equilibrio:
CO2 (g) + H2O (l) ⇄ H+ (ac) + HCO3− (ac)

Una forma de reducir la acidosis es:


A) Tomar mucha agua para desplazar el equilibrio a la derecha.
B) Adicionar una enzima que catalice esta reacción.
C) Introducir cloruro de sodio en la sangre para cambiar el pH.
D) Provocar una respiración intensa para reducir los niveles de CO2 en la sangre.

______________

262
6. El producto de solubilidad del ioduro de plomo a 25 °C es 1·10−9. Calcular la solubilidad en
gramos por cada 100 mL.

A 0,29 g B 0,0145 g C 0.029 g D 0.145 g

______________

7. Un estudiante de olimpiada observó en el laboratorio que al añadir 500 mL de ácido clorhídrico


0,05 mol L-1 a 0,2790 g de limaduras de Fe metálico, este último se disolvía completamente. La
reacción de disolución del Fe metálico con el ácido clorhídrico y el valor del pH de la disolución
resultante son:

A) 2 Fe (s) + 6 HCl (ac) → 2 FeCl3 (ac) + 3 H2 (g) 2,0


B) Fe (s) + 2 HCl (ac) → FeCl2 (ac) + H2 (g) 1,4
C) Fe (s) + 4 HCl (ac) → FeCl2 (ac) + 2 H2 (g) + Cl2 (g) 2,3
D) Fe (s) + 2 HCl (ac) → FeCl2 (ac) + H2 (g) 1,50

Datos: Valores de Eo: Fe2+/Fe = -0,40 V/ENH; Fe3+/Fe2+ = 0.77 V/ENH

_____________

8. Una disolución contiene Fe3+ y Cu2+, ambos en concentración 0,01 mol/L. ¿Qué ocurre cuando a
100 mL de esta disolución se añaden 10 mL de una de EDTA (Y4-) 0,1 mol/L?

A) se complejan totalmente los dos cationes


B) únicamente se compleja el Cu2+
C) únicamente se compleja el Fe3+
D) se compleja el 50 % de cada uno de los cationes

Datos: Los valores de pK de formación de los complejos son: pKc CuY2+ = 18,8;
pKc FeY+ = 25,1

_______________

263
9. El alcohol como energético sustituto del gas L. P., gas natural (metano 90%, etano 10%), y las
gasolinas. R = 8.314 Pa m3/mol K; T(K) = t(°C) + 273.15

En un laboratorio de química de secundaria se acabó el


gas butano y se recurrió a las antiguas lámparas de alcohol
etílico para calentar un “baño maría” que contiene 300
mL de agua a 20 °C, para los experimentos.

Un alumno preguntó al profesor ¿qué era mejor?, si


calentar con la lámpara de alcohol o con el mechero de
Bunsen. El profesor propuso al alumno que investigara
para responder a su pregunta.

a) ¿Cuántos mililitros de alcohol etílico se requieren para calentar el agua del baño maría hasta una
temperatura de 60 °C? Datos: densidad del agua es de 1.0 g/mL; capacidad térmica específica del
agua = 4.184 J/g K; densidad del etanol = 0.789 g/mL; calor de combustión del etanol = 29.8
kJ/g.

a) 0.468 mL b) 1.3293 mL c) 1.6848 mL d) 2.135 mL e) 3.215 mL

b) ¿Cuántos gramos de agua se pueden calentar (de 20 °C a 60 °C), con 25.0 L de gas butano
(C4H10), medidos a la presión de 1 bar y una temperatura de 20 °C? El calor de combustión del
butano es – 49.2 kJ/g. (Suponga comportamiento ideal para el butano.)

a) 59.5 g b) 826.9 g c) 2 927 g d) 17 490 g e) 59 490 g

La composición del gas natural varía dependiendo del yacimiento. El gas proveniente de un
yacimiento X está compuesto por 90% en masa de metano (CH4) y 10% de etano (C2H6). El calor
de combustión del gas natural es de 54 kJ/g.

264
c) El porciento en mol de metano en esta mezcla es:

a) 33.3 % b) 56.5 % c) 75 % d) 86.5 % e) 94.4 %

d) La masa molar aparente de la mezcla es:

a) 16 g/mol b) 16.8 g/mol c) 17.4 g/mol d) 23 g/mol e) 28.6 g/mol

e) Un tanque de 50 L contiene esta mezcla a una temperatura de 30 °C y a una presión de 5.5 bar. La
cantidad de calor que se puede obtener de la combustión de esta mezcla es:

a) 5.95 kJ b) 164.8 kJ c) 183.1 kJ d) 8899 kJ e) 9887.4 kJ

********************************************************************************

265
La pregunta 10 sólo es para el Nivel B

10. Un niño se metió sin permiso en el laboratorio de Química y creyendo que se trataba de una
bebida, ingirió un poco de un ácido que iba a ser utilizado para limpiar los pisos. Casualmente tú
entras al laboratorio en ese momento y, al ver al niño, buscas alguna sustancia para neutralizar la
acidez en el estómago del pequeño. Encuentras un anaquel de nueve repisas; en cada una de ellas
hay un recipiente con sustancias diferentes (ver dibujo) y un instructivo para identificar las
sustancias que contienen. Aunque en varios recipientes hay reactivos capaces de neutralizar al
ácido, algunos de ellos pueden ser más perjudiciales que el propio ácido.

2 3 1

1 2 3

3 1 2

Instructivo:
 En un recipiente marcado con el número 3 hay NaCl.
 Hay un recipiente con mercurio debajo de otro que contiene NH4Cl.
 A la izquierda de un recipiente alto hay agua destilada.
 En la fila inferior, en un recipiente igual al que contiene el NaCl, más abajo que el del agua
destilada, hay cianuro de sodio.
 En la fila superior está una disolución de sosa concentrada.
 En la columna izquierda hay ácido acético.
 En un recipiente con el número 2 hay NH4Cl.
 Hay arsénico en un recipiente redondo que no está cerca del cianuro.
 Otro recipiente contiene NaHCO3.

Completa el siguiente cuadro indicando cuál es la sustancia que se encuentra en cada recipiente e
indica cuál es la adecuada para darle al pequeño.

La sustancia adecuada es: __________

FIN DEL EXAMEN NIVEL B.

266
********************************************************************************

Las preguntas 11 y 12 sólo son para el Nivel A

11. Escriba la estructura correcta de las siguientes moléculas:

I. (E)-2-Penteno II. cis-3-Hexeno III. (R)-2-Hexanol

IV. (S)-3-Clorooctano V. (Z)-2-Buteno

I II III

IV V

267
12. Complete el siguiente diagrama indicando los reactivos necesarios:

Br
O

I II III

+
CH2 O
IV V

OH

OH

Br

I II III IV V

FIN DEL EXAMEN NIVEL A.

268
XX OLIMPIADA NACIONAL DE QUÍMICA
TERCER EXAMEN NIVEL A y B Tiempo asignado: 180 minutos Toluca 2011

Problema No. 1

El laboratorio de Química Analítica de esta escuela recibió un donativo que contenía varias sales;
entre ellas se encontraban cinco nitratos: AgNO3, Cu(NO3)2, Fe(NO3)2, Mg(NO3)2 y Pb(NO3)2;
desafortunadamente las etiquetas de los frascos que los contenían no eran claras. El profesor
encargado del laboratorio le pidió a Luis investigar a que frasco correspondía cada uno de ellos para
lo cual le proporcionó trocitos de los metales de los cinco cationes (Ag, Cu, Fe, Pb y Mg). Luis
preparó disoluciones con los cinco nitratos, las etiquetó como 1, 2, 3, 4 y 5 y buscó la tabla que se
muestra en la que podía observar la posición relativa de los pares redox:

Mg2+ Fe2+ Pb2+ Cu2+ Ag+


Eo
Mg Fe Pb Cu Ag

Posteriormente, tomó unas gotas de cada una de las cinco disoluciones y las colocó sobre cada
trocito de metal. Anotó un signo + cuando observó que se producía alguna reacción y uno – cuando
no se veía ningún cambio.

Los resultados observados los registró en la siguiente tabla pero en ese momento no informó sus
conclusiones al profesor.

Indique a qué disolución corresponde cada una de las disoluciones:


Cu Ag Fe Mg Pb
1 - - - + -
2 - - - - -
3 + - + + +
4 - - + + -
5 - + + +

La disolución 1 contiene ______________________


La disolución 2 contiene ______________________
La disolución 3 contiene ______________________
La disolución 4 contiene ______________________
La disolución 5 contiene ______________________

Escriba las ecuaciones balanceadas que corresponden a dos de las reacciones que fueron positivas.

269
Cuando Luis llegó a su casa y le comentó a su hermano Juan, que estudia química, él le dijo que
podría haber hecho menos experimentos y haber identificado algunas de esas sales por sus
propiedades físicas. ¿Cuáles crees que hubieran sido esas sales y por qué?

Problema No. 2

El isótopo 42K tiene un tiempo de semidesintegración de 12 horas. ¿Cuál es la fracción de la


concentración inicial de dicho isótopo que queda después de 48 horas?

Problema No. 3

En el laboratorio de Química Analítica de esta Facultad se encontró un frasco de un reactivo sólido


sin etiquetar que pertenecía a un ácido monoprótico (HA). Cecilia, estudiante olímpica interesada en
el laboratorio decidió hacer algunos experimentos para tratar de identificarlo. Para lograrlo pesó 784
mg del sólido, lo disolvió con un volumen de agua destilada, que desafortunadamente olvidó anotar,
y comprobó que la disolución era ácida con un pH=2,45. Tomó esta disolución (a la que llamó
“disolución A”) y le añadió agua destilada hasta un volumen de 100,00 mL. A la nueva disolución,
etiquetada como disolución B, le midió un pH de 2,65.
Más tarde tomó una alícuota de 10,00 mL de esta disolución B y le añadió 3 mL de una disolución
de NaOH de concentración conocida igual a 0,0667 mol/L. Al medir el pH de la disolución
resultante obtuvo un valor de 3,88.

Con los datos anteriores la brillante olímpica pudo obtener la información que, sin duda, tú podrás
reproducir contestando las siguientes preguntas:

a) Indica la ecuación que representa la autodisociación de HA en medio acuoso y la expresión de su


constante de autodisociación.

270
b) Indica la reacción que ocurrió al añadir la disolución de NaOH a los 10,0 mL de la disolución B.

c) ¿Cuántos moles de NaOH añadió a los 10,00 mL de la disolución B?

d) Expresa las constantes de equilibrio que corresponderían a las disoluciones A y B, indicando los
valores numéricos de las concentraciones conocidas y denotando como CA y CB las que aún no
has calculado.

e) ¿Cuál es la concentración del ácido HA en la disolución B? Indica los cálculos efectuados.

CB = _______________________________mol/L

f) ¿Cuál es el valor de la constante Ka del ácido? En caso de que no hayas podido calcular la
concentración de HA en la disolución B considera como si ésta hubiera sido 0,042 mol/L y
continúa respondiendo las siguientes preguntas.

271
g) Calcula el valor de la concentración de la disolución A.

CA = _______________________________mol/L

h) Indica el volumen de la disolución A que la brillante estudiante olvidó anotar.

V inicial de A = ____________________mL

i) ¿Qué volumen de la disolución de NaOH hubiera sido necesario añadir para neutralizar todo el
ácido contenido en los 784 mg del sólido?

V para neutralizar el ácido HA = _________mL

j) ¿Cuál es la masa molar del ácido?

k) Un profesor que seguía interesado en las experiencias de Cecilia decidió ayudarla haciendo un
análisis elemental del sólido; encontró que únicamente contenía tres elementos y que el
porcentaje de dos de ellos eran C = 36,70% e hidrógeno = 6,12%. ¿Cuál es la fórmula
condensada del ácido HA?

272
Este inciso sólo es para el Nivel A

l) Propón la fórmula de algún ácido monoprótico al que le corresponda esta fórmula condensada e
indica su nombre.

Problema No. 4

Un problema serio al que se enfrentan los buzos que permanecen en estaciones submarinas es el
enrarecimiento del aire al disminuir la cantidad de oxígeno, pero principalmente por el aumento en
la concentración del bióxido de carbono, el cual no debe llegar a más del 7% en volumen. La
respiración química global está representada por la ecuacióN:

C6H12O6 + 6 O2 → 6 CO2 + 6 H2O + energía (ATP)

Un equipo de buzos se dirige a una estación (3 m de alto x 3 m de ancho x 6 m de largo) bajo el mar
a una profundidad de 20 m. El aire en la estación contiene 17% (en masa) de oxígeno y 83% (en
masa) de nitrógeno a una temperatura de 15 oC. Considere que a esta temperatura el agua de mar
tienen una densidad de 1.029 kg/L.

Calcule:

a) La presión a la que se encuentra la estación submarina.

a) 0.202 bar b) 0.302 bar c) 1.313 bar d) 2.019 bar e) 3.032 bar

273
b) La fracción mol de oxígeno en la estación antes de que lleguen los buzos.

a) 0.152 b) 0.296 c) 0.349 d) 0.531 e) 0.848

c) El porciento de oxígeno consumido cuando se alcanza el 7% en volumen de bióxido de carbono.

a) 3.5% b) 5.1% c) 7.0% d) 8.0% e) 8.5%

d) Suponiendo que la estación se encuentra a una presión de 3.5 bar y 15 oC, calcule la cantidad de
sustancia en moles de CO2 cuando este gas alcanza el 7% en volumen.

a) 152.2 mol b) 522.2 mol c) 788.9 mol d) 5222 mol e) 7889 mol

274
Problema No. 5

El elemento A es un metal que en la forma de catión es muy importante en muchas reacciones


bioquímicas. Cuando 0.4862 g de este elemento se hacen reaccionar con ácido sulfúrico acuoso a
30.00 °C y 760.0 mmHg de presión se obtienen 497.2 mL de un gas inflamable y un compuesto B.
Al tratarse el compuesto B con hidróxido de sodio acuoso se obtiene un precipitado blanco C (y otro
compuesto). Si se añade ácido clorhídrico concentrado al precipitado C se obtiene el compuesto D
disuelto en un líquido incoloro que también se produce en la reacción. Cuando el compuesto D se
hace reaccionar con carbonato de sodio se obtiene el compuesto E (y sal común), que al calentarse
forma un gas F y un sólido blanco G. Este sólido blanco al reaccionar con agua forma C. El
elemento A reacciona con nitrógeno para formar el compuesto H, que contiene 27.76% de
nitrógeno. La reacción de H con agua forma el compuesto C y un gas de carácter básico I. Con base
en esta información contesta lo siguiente:

1) Identifica al elemento A y los compuestos B – I.

A B C D E

F G H I

2) Escribe las ecuaciones balanceadas de las siguientes reacciones incluyendo los estados de
agregación.

2.1) La reacción de A con ácido sulfúrico.

2.2) La reacción de B con hidróxido de sodio.

2.3) La reacción de C con ácido clorhídrico.

2.4) La reacción de D con carbonato de sodio.

275
2.5) La reacción de calentamiento de E.

2.6) La reacción de G con agua.

2.7) La reacción de A con nitrógeno.

2.8) La reacción de H con agua.

3) Identifica al gas inflamable producido en la reacción del elemento A con ácido sulfúrico.

Problema No. 6

La obtención de carbonato de sodio que internacionalmente se lleva a cabo por el proceso Solvay,
fue sustituido por un proceso alternativo, diseñado por mexicanos y nombrado en el país como
PROCESO SOSA TEXCOCO; el cual utilizaba como materias primas las encontradas cerca del
conjunto Texcoco.
Para el proceso interactuaban 4 reactivos (A, B, C y D). A y B eran gases, uno de ellos (B) producto
de la descomposición térmica del carbonato de calcio, y el otro (A) de olor penetrante y pH básico
en disolución acuosa. C es una disolución saturada de sal común por lo que D es el disolvente de C.
Los reactantes en contacto y a bajas temperaturas dan 2 productos: Uno soluble (E) y otro insoluble
(F). F es separado por filtración, secado y sometido a calentamiento con el fin de obtener finalmente
el carbonato G, base del proceso, el cual es soluble en agua. Como subproductos se obtiene de
nuevo B que es reciclado al proceso ya que es parte de la materia prima inicial y el líquido D en el
cual se lleva a cabo la reacción.

276
Con esta información contesta el siguiente cuestionario:

1) Identifica cada uno de los reactivos y productos del proceso (de A a G).

Compuesto A B C D E F G

Fórmula

2) Escribe correctamente las ecuaciones que representan el proceso.

3) ¿Cuál es la diferencia con el proceso Solvay? Justifica tu respuesta.

El Problema No. 7 sólo es para el Nivel B.

Problema No. 7
Cada hora, 400 kg de una mezcla líquida A, agua/acetona (CH3COCH3), al 50% en masa, se
alimentan a un equipo separador denominado torre de destilación, que al calentar la mezcla la separa
en una parte de vapor y una parte que permanece en estado líquido. La mezcla que sale en forma de
vapor B se enriquece del compuesto más volátil que en este cado es la acetona. Esta mezcla B tiene
una fracción mol de acetona a 0.50. En el líquido C que resta, la fracción mol de agua es de 0.80.

A B (vapor)

C (líquido)

Calcule la cantidad de sustancia total en moles presentes en cada una de estas mezclas.
Muestre sus cálculos:

277
Moles totales presentes en cada corriente:

A: __________ ; B: __________ ; C: __________

FIN DEL EXAMEN NIVEL B

********************************************************************************

De aquí en adelante sólo contestan los de Nivel A.

Problema No. 8

Escriba la estructura de los intermediarios y del producto final de la siguiente secuencia sintética:

OH

HNO3 CH3CH2I Fe
I II III
NaOH HCl

1) NaNO2 / HCl

2) CuCN

IV

(C12H15NO)

I II III IV

278
Problema No. 9

Escriba la estructura de los intermediarios y del producto final de la siguiente secuencia sintética:

HBr 1) Mg 1) SOCl2
I II III
peróxidos 2) CO2 2) AlCl3 (C9H8O)
3) HCl

I II III

Problema No. 10

Asigne un descriptor estereoquímico adecuado para cada molécula:

H OH Me Me
Me
Me
H OH

I II III IV

I II III IV

279
Problema No. 11

Los extinguidores de CO2 comprimido son de los más utilizados en caso de


incendios, ya que pueden aplicarse sobre equipos eléctricos e incluso
directamente sobre las personas a una distancia prudente en caso de ser
necesario.

La presión de vapor del CO2 líquido es de 67.89 bar a una temperatura de


25.0 ºC y de 73.97 bar a 31.1 ºC.

El punto triple del CO2 se encuentra a 5.178 bar a una temperatura de – 56.6
ºC, en tanto que la presión de vapor del sólido a – 78.2 ºC es de 1.013 bar.

Un extinguidor convencional tiene una capacidad de 4840 cm3 y regularmente se carga con 2 kg de
CO2.

Si la temperatura ambiente es de 25 ºC, y suponiendo que el gas se comporta idealmente calcula:

1) La presión que ejercería si sólo hubiera gas dentro del extinguidor.

a) 2.328 bar b) 23.28 bar c) 232.8 bar d) 2328 bar e) 23 280 bar

2) Calcula los gramos de CO2 que pueden estar en estado gaseoso a 25 ºC en un extinguidor antes de
transformarse en líquido.

a) 378.0 g b) 489.1 g c) 583.3 g d) 785 g e) 899.9 g

3) Calcula los gramos de CO2 en estado líquido cuando se introducen 1.1 kg de CO2 en un
extinguidor de 8.5 L de capacidad a una temperatura de 25 ºC. (Considera que el volumen del
líquido es muy pequeño.)

a) 75.7 g b) 110 g c) 440.1 g d) 938.1 g e) 1024.3 g

Suponiendo que en este intervalo de temperaturas se cumple la ecuación de Clausius-Clapeyron.


Donde H es la entalpía de vaporización en el caso del equilibrio líquido-vapor y la entalpía de
sublimación en el caso del equilibrio sólido-gas, en tanto que P1 y P2 corresponden a las presiones
de vapor a las temperaturas T1 y T2.
P H  1 1 
Ln 2     
P1 R  T2 T1 

4) Calcula la entalpía de vaporización del CO2.

a) 1.28 kJ b) 10.60 kJ c) 83.14 kJ d) 90.9 kJ e) 200.3 kJ

280
5) Suponiendo que se cumple la ecuación de Clausius-Clapeyron para el equilibrio sólido-gas,
calcula la entalpía de sublimación del CO2.

a) 2.77 kJ b) 3.55 kJ c) 15.6 kJ d) 27.7 kJ e) 35.5kJ

Problema No. 12

Para un cierto líquido puro “X”, cuya Temperatura crítica (Tc) es 400 oC, la variación de su tensión
superficial () con la temperatura (T) está dada por la fórmula:

 = o (1 – ( T / Tc))n
[o y n son constantes características para cada líquido]

Esta ecuación es válida para un intervalo de 15 oC a 85 oC.

Valores experimentales determinados para el líquido “X”:

 (dinas cm-1) 37.128 38.921 40.123


T (oC) 45 30 20

Determina la tensión superficial de este líquido a 80 oC y el valor de las constantes: o y n.

 a 80 oC : __________________ dinas cm-1

Valor de las constantes para el líquido “X“: o = _________ ; n = __________

FIN DEL EXAMEN NIVEL A

281
282
XX OLIMPIADA NACIONAL DE QUÍMICA
EXAMEN INTERNACIONAL
10 PROBLEMAS Tiempo: 4 HORAS Toluca 2011

Fisicoquímica

1. Para determinar la composición de mezclas de líquidos volátiles es necesario conocer la presión


de vapor de ambos líquidos. La ecuación de Antoine es muy útil para calcular las presiones de
vapor de sustancias volátiles.
Ecuación de Antoine
B
log 10 ( P )  A 
T C
donde P = presión de vapor (bar) y T = temperatura (K)

Las constantes A, B y C de la ecuación de Antoine se determinan experimentalmente y son


válidas en el intervalo de temperaturas donde fueron determinadas. Si éstas se utilizan para
evaluar las presiones de vapor fuera del rango en que fueron determinadas, se corre el riesgo de
obtener resultados que se alejen mucho de los valores experimentales.

Por lo anterior, es necesario prestar atención a los intervalos de temperaturas en los que se
reportan las constantes para utilizar las que correspondan a la temperatura de trabajo.

En las tablas siguientes se presentan las constantes de la ecuación de Antoine para el benceno y el
tolueno en varios intervalos de temperatura.

Benceno
Temperatura (K) A B C
333.40 - 373.40 4.726 1661 -1.461
287.70 - 354.07 4.018 1204 -53.23

Tolueno
Temperatura (K) A B C
308.52 - 384.66 4.078 1344 -53.77
273.00 - 323.00 4.142 1378 -50.51

Considerando que las mezclas de benceno-tolueno se comportan de manera ideal, éstas cumplen
con la ley de Raoult:
Pi  xi Pi*
donde xi es la fracción mol en el líquido, Pi es la presión parcial, Pi* es la presión de vapor del
compuesto puro en tanto que yi es la fracción mol en el vapor.
y i  Pi / Ptotal

283
Utiliza la ecuación de Antoine para calcular las presiones de vapor del benceno y del tolueno.

1. La presión de vapor del benceno a 15 ºC es:

a) 6600 Pa b) 7200 Pa c) 7800 Pa d) 8700 Pa e) 78500 Pa

2. La presión de vapor del tolueno a 110 ºC es:

a) 7800 Pa b) 14500 Pa c) 78500 Pa d) 94450 Pa e) 99450 Pa

3. A cierta temperatura las presiones del benceno y del tolueno son 36.1 kPa y 12.24 kPa
respectivamente.

Un recipiente contiene 5.50 mol de benceno y 3.50 mol de tolueno a esta temperatura.

3.1. La presión parcial del benceno en la mezcla es:

a) 4.76 kPa b) 22.06 kPa c) 24.17 kPa d) 26.82 kPa e) 48.34 kPa

3.2. La fracción mol del tolueno en el vapor es:

a) 0.1775 b) 0.2532 c) 0.3391 d) 0.5923 e) 0.8225

284
3.3. Considera el siguiente diagrama de equilibrio de una mezcla de benceno y tolueno a
temperatura constante. La línea recta representa la composición de la mezcla líquida en tanto
que la línea curva representa la composición del vapor en equilibrio con el líquido.

Sistema Benceno-Tolueno
Temperatura constante
1400

1300

1200

1100
Presión total / Pa

1000

9000
8000

7000

6000

5000

4000
0.0 0.1 0.2 0.3 0.4 0.5 0.60 0.7 0.8 0.9 1.0
Fracción mol de benceno X(líq) Y(vap)

3.3.1. La fracción mol del tolueno en el líquido a una presión de 8000 Pa es aproximadamente:
a) 0.30 b) 0.40 c) 0.50 d) 0.60 e) 0.70

3.3.2. La fracción mol del benceno en el vapor a una presión de 9000 Pa es aproximadamente:
a) menor que 0.50 b) 0.50 c) 0.65 d) 0.75 e) mayor que 0.75

3.3.3. Cuando la fracción mol del tolueno en el líquido es 0.2, la presión total es aproximadamente:
a) < 5000 Pa b) 6000 Pa c) 9400 Pa d) 11850 Pa e) > 12000 Pa

3.3.4. Cuando la fracción mol del benceno en el vapor es 0.6, la presión total es aproximadamente:
a) < 7000 Pa b) 7050 Pa c) 8000 Pa d) 9900 Pa e) > 9900 Pa

3.3.5. Cuando la presión es de 9000 Pa, la fracción mol del benceno en el líquido es
aproximadamente:
a) < 0.22 b) 0.22 c) 0.50 d) 0.78 e) > 0.78

3.3.6. Cuando la presión es de 11000 Pa, la fracción mol del tolueno en el líquido es
aproximadamente:
a) < 0.1 b) 0.28 c) 0.72 d) 0.89 e) > 0.89

********************************************************************************
285
Química Orgánica

4. Escriba las estructuras de los intermediarios y del producto final de la siguiente secuencia
sintética:

CO2H

4 CH3I 1) LiAlH4 1) HBr


I II III
4 NaOH 2) KCN
HO OH 2) H2O
1) LiAlH4
OH
2) H2O
Ácido gálico

IV
Mescalina
(C11H17NO3)

I II

III IV

286
5. Escriba las estructuras de los intermediarios y del producto final de la siguiente secuencia
sintética:

OH
K2Cr2O7 Ph3P=CH2 HBr
I II III
H2SO4 peróxidos

KCN

H2O
V IV
+
H ,D
(C8H14O2)

I II III

IV V

287
6. Escriba las estructuras del producto final de las siguientes reacciones:

a)
OH
1) Hg(OAc)2
I
2) NaBH4

b)
CH3 O

O O H3O+ O
II +
CH3 OH

CH2CO2H CH3

c)
1) O3

2) Zn / H+
III
3) 2 H2N-NH2
KOH, HO-CH2-CH2-OH, 

I II III

********************************************************************************

288
Química Inorgánica

7. Durante mucho tiempo se creyó que los gases nobles eran completamente inertes e incapaces de
formar enlaces químicos. Pero después de haber observado que el O2 reacciona con el PtF6 para
dar el compuesto [O2+][PtF6-], Neil Bartlett en 1962, se convenció que puesto que la entalpía de
ionización del xenón es casi idéntica a la de la molécula de oxígeno, debería existir una reacción
análoga con el xenón. Después confirmó su predicción y obtuvo de este modo el primer
compuesto de un gas noble. Algunos compuestos estables del xenón son el XeF2 y el XeF4.

7a) Usando el principio de repulsión de los pares electrónicos de la capa de valencia, prediga las
geometrías probables del XeF2 y XeF4. Dibuje las estructuras e indique la hibridación del xenón
en cada compuesto.

XeF2 XeF4

Hibridación: Hibridación:

7b) ¿Cuáles son los estados de oxidación del xenón en el XeF2 y el XeF4?

Los compuestos fluorados del xenón se hidrolizan fácilmente con el agua, ya sea de manera total
o parcial. La hidrólisis parcial del XeF4 produce XeOF2 además de HF, mientras que la hidrólisis
parcial del XeF6 produce además de HF, XeOF4. La hidrólisis total de ambos compuestos
produce XeO3, en el caso del XeF4 además del compuesto anterior se obtiene xenón gaseoso,
fluoruro de hidrógeno y flúor. En tanto que la hidrólisis del XeF6 sólo produce HF y XeO3.

7c) Escriba las ecuaciones balanceadas para:


7c.1) La hidrólisis parcial del XeF4.

7c.2) La hidrólisis total del XeF4.

7c.3) La hidrólisis parcial del XeF6.

289
7c.4) La hidrólisis total del XeF6.

7d) Usando el principio de repulsión de los pares electrónicos de la capa de valencia, prediga las
geometrías probables del XeO3, XeOF2 y XeOF4. Dibuje las estructuras, indique la hibridación
del xenón en cada compuesto y el nombre de cada geometría.
XeO3 XeOF2 XeOF4

Hibridación: Hibridación: Hibridación:

7e) ¿Cuáles son los estados de oxidación del xenón en el XeO3, XeOF2 y XeOF4?

Una de las maneras de conocer la energía del enlace entre el flúor y el xenón es hacer un ciclo
termoquímico como el siguiente:

Xe(g) + 2F(g)
-2disH°Xe-F
disH°F (g)
2

Xe(g) + F2(g) XeF2(g)


fH°XeF (g)
2

En donde fHoXeF2 es la entalpía de formación estándar del difluoruro de xenón, el cual es un


valor conocido, disHoF2 es la entalpía de disociación del enlace F–F (valor conocido) y disH°Xe-
F, es la entalpía de disociación del enlace Xe–F, la cual en el ciclo es negativa debido a que la
forma en la que se indica es el proceso de formación del enlace y el 2 indica que son dos enlaces
Xe–F. El ciclo anterior se puede resumir con la siguiente ley de Hess:

XeF2 (g) Xe (g) + F2 (g)


F2 (g) 2 F (g)
XeF2 (g) Xe (g) + 2 F (g)

290
Usando los siguientes valores que se te proporcionan y la información anterior:

fHoXeF2 disHoF2
–106 kJ mol-1 158 kJ mol-1

7f) Calcule la energía de disociación del enlace Xe–F.

7g) Usando la información anterior y el valor obtenido en el inciso f, calcule la entalpía de


formación del XeF4 (g).

8. Una mezcla que contiene NO2 y N2O4 tiene una densidad de 2.21 g L-1, a una temperatura de 100
o
C y a una presión de 1.4 bar, (1.013 bar = 1 atmósfera). Calcule la presión parcial de cada uno de
los gases y el valor de Kp para la disociación del N2O4. Suponer que esta mezcla se comporta
idealmente.

P (NO2) : ______________________________bar

P (N2O4) : ______________________________bar

Kp = ______________________

9. El carbono-14, único radioisótopo del carbono natural, se utiliza para la determinación de edades
históricas ya que de cualquier muestra orgánica se puede determinar su antigüedad basados en la
premisa de que una vez que el organismo muere, el carbono sale de su ciclo normal y el carbono-
14 empieza a decaer con una vida media de 5730 años. Esta detección se basa experimentalmente
en que el carbono-14 en muestras de carbono actuales da 15 desintegraciones por minuto por
gramo de carbono presente.

El carbono-14 se forma de la reacción nuclear del nitrógeno atmosférico (nitrógeno-14) con


neutrones de muy alta energía (neutrón-protón) transformándose en carbono-14. Con esta
información, responde el siguiente cuestionario:

9a) Si las desintegraciones por minuto de cada gramo de carbono son de 7.5, ¿cuál es la antigüedad
de la muestra? Justifique su respuesta.

291
9b) Si las desintegraciones son de 3minutogramo de carbono, podemos considerar que la muestra
es más o menos antigua que la anterior, ¿qué tan antigua es?

9c) Una muestra arqueológica se sospecha que tiene aproximadamente 2000 años de antigüedad,
¿cuántas desintegraciones por minuto por gramo de carbono debe tener?

9d) Escriba la ecuación nuclear de transformación del nitrógeno-14 al carbono-14.

9e) Escriba la ecuación que permite calcular el tiempo de decaimiento de una muestra radiactiva.

ln 2
 t
t1 / 2
N0 e
Fórmula útil: N =

********************************************************************************

Química Analítica

10. Las meteoritas son restos de materia sólida interplanetaria que proceden generalmente de la franja
de asteroides y que consisten de materia primitiva similar a la que originalmente formaba el
sistema solar. Su estudio es importante para la mejor comprensión de la formación y el origen
del universo. Hay meteoritas pétreas, metálicas y mixtas, y todas ellas contienen hierro, níquel y
trazas de otros metales. Las meteoritas metálicas representan poco más del 5 % del total que cae
cada año en la Tierra pero en masa representan más del 90 %.

292
Meteorita Toluca

Para estudiar la composición química de meteoritas se emplean técnicas instrumentales y


volumetrías. En presencia de los componentes mayoritarios (hierro y níquel) la cuantificación de
trazas de otros constituyentes metálicos presenta mayores complicaciones debido a que, para el
análisis de estas muestras únicas, sólo se dispone de una cantidad muy pequeña. Por esta razón,
se requiere emplear técnicas instrumentales de análisis que sean sensibles, precisas y con
posibilidad de detectar cantidades muy pequeñas. Entre éstas se encuentra la Polarografía, la cual
se basa en dos principios fundamentales relacionados con las leyes de Faraday:

1) Un compuesto químico se puede reducir u oxidar a un determinado potencial que le es


propio.
2) La intensidad de corriente de electrólisis (i) es proporcional a la cantidad de materia
electrolizada.

Con base en ellos, es posible establecer que para una especie dada:

i = nkDC*
en donde:
n = número de electrones que intercambia la especie considerada en el proceso redox medido
kD = constante de proporcionalidad relacionado con las condiciones experimentales
C* = concentración de la especie que se electroliza

La Meteorita Toluca, también llamada Xiquipilco por el poblado donde cayó hace muchos
años, fue investigada por Pugh en 1856 pero no hay estudios relacionados con la composición de
la misma hasta ahora en que, algunos investigadores mexicanos ligados con la Olimpiada
Nacional de Química, han iniciado su análisis.

A continuación te daremos información del trabajo realizado por estos investigadores y algunos
datos que le permitirán interpretar sus resultados.

Procedimiento

a) Se pesaron 300 mg de la Meteorita Toluca, los cuales se disolvieron con ácido nítrico
concentrado. La “disolución fuente” obtenida se transvasó cuantitativamente a un matraz
volumétrico de 10,00 mL y se completó hasta al aforo con agua destilada.
b) A una alícuota de 1,00 mL de esta disolución se le agregó 1 mL de una disolución
amortiguadora de pH= 9 y 7,00 mL de una de EDTA (Y4-) 0,100 M. El exceso de EDTA se
valoró con 2,0 mL de una disolución de Zn2+ 0,100 mol/L, usando eriocromo negro T como
indicador complejométrico.

c) Otra alícuota de 1,00 mL de la misma disolución de la meteorita se evaporó a sequedad hasta


eliminar el exceso de ácido nítrico; posteriormente se disolvió en un poco de agua destilada y
se le agregó una gota de un indicador ácido-base apropiado que permitiera detectar el punto
final donde precipitara únicamente uno de los cationes presentes. La precipitación total se
obtuvo cuando se añadieron 6,9 mL de una disolución de NaOH 0,2 mol /L.

d) Se trazaron los polarogramas para diferentes disoluciones patrón de Ni(II) y de Co(II) en una
disolución electrolítica conductora que no contenía ninguno de los elementos a determinar. Al
293
hacer las medidas de estas disoluciones a un potencial adecuado se obtuvieron las ecuaciones
de la relación lineal existente entre la intensidad de corriente (medida en nanoamperios, nA) y
la concentración (medida en ppm, es decir, considerado en mg/L) para cada una de estas
especies:
iNi = 38,40C* - 0,336
iCo = 29,61C* + 2,167

e) Por otra parte se trazó también el polarograma de una disolución que contenía 15 microlitros
de la disolución fuente de la meteorita y 10,00 mL de la misma disolución electrolítica
utilizada para las disoluciones patrón. En los mismos valores de potencial utilizados para
disoluciones patrón se obtuvieron los siguientes valores de corriente:

Para el Ni: iNi problema =122,34 nA, y para el Co: iCo problema = 6,17 nA

Datos:
Sistema Fe2+ / Fe(0) Co2+ / Co(0) Ni2+ / Ni(0) H+ / H2 Fe3+ / Fe2+ HNO3 / HNO2 Co3+ / Co2+
redox

(V/ENH) -0,44 -0,28 -0,25 0,00 0,77 0,94 1,8

Ni(OH)2 Co(OH)2 Fe(OH)2 Fe(OH)3


pKs 15,3 14,9 14,5 38,7

(ZnY)2- (NiY)2- (CoY)2- (FeY)2- (FeY)-


pKdisoc. 18,1 16 15,9 15,8 22

Conteste las siguientes preguntas en los espacios indicados más adelante:

10.a. Indique qué cationes fueron complejados al adicionar el exceso de EDTA.

10.b. ¿Cuál es la cantidad total (en micromoles) de Y4- con la cual fueron complejados esos cationes
presentes en la meteorita?

10.c. ¿Cuál es la especie catiónica Mn+ que se precipita con los OH-?

10.d. Escriba la reacción de precipitación del catión Mn+

10.e. ¿Cuál es la cantidad total (en micromoles) de Mn+ precipitados en la alícuota?

10.f. ¿Cuál es la concentración (en ppm) de Ni2+ en celda que se determinó por polarografía?

10.g. ¿Cuál es la concentración (en ppm) de Ni2+ en la disolución de la meteorita?

10.h. ¿Cuál es la concentración (en ppm) de Co2+ en celda que se determinó por polarografía?

10.i. ¿Cuál es la concentración (en ppm) de Co2+ en la disolución de la meteorita?

10.j. ¿Cuál es el porcentaje (en masa) de Co2+ en la meteorita Toluca?

294
10.k. ¿Cuál es el porcentaje (en masa) de Ni2+ en la meteorita Toluca?

10.l. ¿Cuál es el porcentaje (en masa) de Fe en la meteorita Toluca?

En las meteoritas las aleaciones de Fe-Ni pueden estar como dos polimorfos: Kamacita y
Taenita. La Kamacita contiene en promedio 90% de Fe y 6,0% de Ni , mientras que la Taenita
contiene 60% de Fe y 30% de Ni. De acuerdo con los resultados obtenidos, indique:

10.m. ¿Cuál es el porcentaje de Kamacita en la meteorita Toluca?

10.n. ¿Cuál es el porcentaje de Taenita en la meteorita Toluca?

ESPACIOS PARA LAS RESPUESTAS.

10.a. Indique qué cationes fueron complejados al adicionar


el exceso de EDTA

10.b. ¿Cuál es la cantidad total (en micromoles) de Y4- con


la cual fueron complejados esos cationes presentes en la
meteorita?

10.c. ¿Cuál es la especie catiónica Mn+ que se precipita con


los OH-?

10.d. La reacción de
precipitación del catión Mn+
es:

10.e. ¿Cuál es la cantidad total (en


micromoles) de Mn+ precipitados en la
alícuota?

10.f. ¿Cuál es la concentración (en ppm) de Ni2+ en celda


que se determinó por polarografía?

10.g. ¿Cuál es la concentración (en ppm) de Ni2+ en la


disolución de la meteorita?

10.h. ¿Cuál es la concentración (en ppm) de Co2+ en celda


que se determinó por polarografía?

295
10.i. ¿Cuál es la concentración (en ppm) de Co2+ en la
disolución de la meteorita?

10.j. ¿Cuál es el porcentaje (en masa) de Co en la meteorita


Toluca?

10.k. ¿Cuál es el porcentaje (en masa) de Ni en la meteorita


Toluca?

10.l. ¿Cuál es el porcentaje (en masa) de Fe en la meteorita


Toluca?

10.m. El porcentaje de Kamacita en la meteorita Toluca es:

10.n. El porcentaje de Taenita en la meteorita Toluca es:

FIN DEL EXAMEN

296
XX Olimpiada Nacional de Química
Examen Experimental de Química Analítica Toluca 2011

El Departamento de esta Universidad recibio recientemente una donación de varios frascos de


cloruros (calidad Q. P.) provenientes de un laboratorio que ya no los utiliza.
Todos los cloruros son de cationes divalentes (ML2), tienen color blanco y aparentemente una
pureza superior al 95%. Lamentablemente las etiquetas de los frascos están muy deterioradas y no se
puede identificar cuál es el cloruro contenido en cada frasco.

A continuación te indicamos los nombres de los posibles cloruros:

Cloruro de calcio, cloruro de cadmio, cloruro de zinc, cloruro de estroncio, cloruro de bario y cluro
de magnesio.

Datos:
pKs (AgCl) = 9,7; pKs (Ag2CrO4) = 11,95
El cloruro de plata es blanco y el cromato de plata es rojo.
Masas molares (g/mol):
Zn = 65,4 Ca = 40,1 Cd = 112,4 Sr = 87,6 Ba = 137,3 Mg = 24,3 Cl = 35,5

Tu tarea esta tarde será ayudarnos a identificar y determinar la pureza de alguno de los cloruros.
Para lograrlo cuentas con el material y reactivos que se indican a continuación:

 Bureta, pipeta volumétrica y un vaso de precipitados pequeño.


 Una disolución que se preparó disolviendo 500,0 mg de la muestra de uno de los cloruros en
un litro de agua destilada.
 Una disolución de nitrato de plata cuya concentración se indica en la etiqueta.
 Un frasco gotero con una disolución de cromato de potasio.

Procedimiento recomendado:

Toma una alícuota de la disolución de tu problema y titúlala con la del nirato de plata. El punto final
de la valoración se obtiene al observar la aparición de un precipitado de color rojo.
Efectúa la operación por triplicado y anota los volúmenes gastados para alcanzar el punto final de la
valoración.

PRECAUCIÓN:
Evita que te caiga nitrato de plata en la piel pues éste deja una mancha café que tarda varios días en
quitarse. Si ocasionalmente te cayera una gota, lávate inmediatamente para eliminarla. De
preferencia utiliza guantes para manipular esta disolución.

297
HOJA DE RESPUESTAS

Alícuota No. Volumen de la alícuota (mL) Volumen de nitrato de plata gastado al


alcanzar el punto final (mL)
1
2
3

1. Volumen de AgNO3 que utilizarás en los cálculos: _____________

2. Escribe la reacción balanceada general de la titulación de MCl2 con el nitrato de plata.

3. Explica por qué el cromato de plara puede ser utilizado como indicador.

4. Indica las moles de nitrato de plata gastadas para llegar al punto dinal de la titulación.
Cálculos

5. Indica las moles de MCl2 que tienes en la alícuota valorada.


Cálculos

6. ¿Cuál es el cloruro que contiene tu muestra?


Cálculos:

7. ¿Cuál es la pureza de tu muestra?


Cálculos

298
XX Olimpiada Nacional de Química
Examen Experimental de Química Inorgánica Toluca 2011

Se te están proporcionado 7 cationes para identificar, todos ellos en forma de nitratos disueltos en
agua.
Siguiendo la secuencia propuesta trata de identificarlos con 8 reactivos, LOS CUALES TAMBIÉN
TIENES QUE IDENTIFICAR.
Los cationes posibles son Ni(II), cobre(II), Zn(II), coblato(II), Pb(II), hierro y aluminio (III). Y
corresponden a los NÚMEROS NONES (1, 3, 5, 7, 9, 11 y 13).
Los reactivos para identificar son:
Sulfuro de sodio 1 M
Hidróxido de sodio 1 M
Amoniaco 1 M
Ácido clorhídrico 1 M
Yoduro de potasio 0.1 M
Dimetil glioxima en disolución básica
Tiocianato de potasio 0.1 M
Hexacianoferrato(II) de potasio 0.1 M
Y corresponden a los números pares (16, 14, 12, 10, 8, 6, 4 y 2).

Observa y anota en la tabla las propiedades físicas iniciales tanto de cationes como de reactivos
identificadores. Este ejercicio preliminar te permitirá hacer hipótesis acerca de algunos de ellos.

Posteriormente sigue la siguiente secuencia y anota en la tabla todas tus observaciones. Identifica los
cationes y reactivos, y justifica tu respuesta con las ecuaciones químicas correspondientes.

Secuencia: EN TODOS LOS CASOS ANOTA OBSERVACIONES Y JUSTIFICA CON


ECUACIONES QUÍMICAS.

I. Coloca una gota de cada unos de los cationes (1, 3, 5, 7, 11 y 13) por separado en la película de
plástico que se te proporcionó, colocando previamente debajo de ésta la cuadrícula como
referencia; si lo consideras necesario numéralos. Añádeles una gota del reactivo 2. Anota tus
observaciones.
II. Repite la operación anterior ahora con el reactivo 8.
III. Si hubo algunos cationes que precipitaron en el inciso anterior, trátalos por separado
añadiéndoles el reactivo 10.
IV. Coloca una gota de los cationes 7 y 13 por separado y añádeles una gota del reactivo 16.
V. Coloca por separado una gota de 7 y 11 y añádeles el reactivo 14.
VI. A todos los cationes (1 a 13) añádeles el reactivo 6 gota a gota y observa los cambios que se
producen.
VII. Al catión 3 añádele el reactivo 12.
VIII. A todos los cationes (1 a 13) añádeles por separado el reactivo 4 gota a gota y observa los
cambios que se obtienen.
IX. A los cationes 7, 9, 11 y 13 añádeles por separado el reactivo 6 al que previamente le haz
añadido cloruro de amonio sólido para tener un pH cercano a 9 (verifícalo con el papel indicador
que se te proporciona). Asegúrate de que ya no usarás el reactivo 6.
X. Al catión 1 añádele el reactivo 10 y observa si hay cambios.

299
300
No. Prop. I II III IV V VI VII VIII IX X Identificación Justificación (Ecuaciones)
Catión o Físicas
Reactivo (Color y
olor)

BUENA SUERTE
9

10

11

12

13

14

15

Nota: Si requieres más espacio para justificar tus respuestas, solicita una hoja blanca al encargado.
16
XX Olimpiada Nacional de Química
Examen Experimental de Química Orgánica Toluca 2011

“IDENTIFICACIÓN DE ALDEHÍDOS Y/O CETONAS POR PRUEBAS A LA GOTA Y


POR CROMATOGRAFÍA EN CAPA FINA”

Objetivos.
Que el alumno:
a) Efectúe la identificación del grupo carbonilo en aldehídos y cetonas.
b) Establezca las diferencias entre un aldehído y una cetona por medio de reacciones características
y fáciles de realizar.
c) Resuelva el problema de identificar un compuesto con un grupo carbonilo desconocido aplicando
las reacciones de identificación a la gota y llevando a cabo un análisis por cromatografía en capa
fina.

Para lograr lo anterior, el alumno deberá seguir el siguiente procedimiento para identificar que
compuesto desconocido se le asignó, el cual puede ser uno de 4 compuestos posibles: 2 aldehídos o
2 cetonas.
1) Llevar a cabo una serie de reacciones de identificación a la gota, a través de las cuales podrá
saber si tiene un aldehído o una cetona.
2) Un análisis por cromatografía en capa fina, para que en función del Rf que determine
experimentalmente, sea capaz de decidir cuál de los siguientes compuestos se le ha asignado:
H NO2

H N
N
H H NO2
O
H
C N
C NO2
R N Rf
R H
EtOH
NO2
H2SO4
O H H NO2

N
H N
0.40

NO2
O H H NO2

N
H N
0.60

NO2
O
H NO2

N 0.46
N

NO2
O H NO2

N
N
0.59

NO2

301
MATERIAL CANTIDAD
Tubo de ensayo con tapón de hule 5
Gradilla o un vaso de precipitados de 250 mL 1
Baño María 1
Embudo Büchner con alargadera 1
Matraz Kitazato de 125 mL con manguera 1
Espátula de acero inoxidable 1
Varilla de vidrio 1
Vaso de precipitados de 50 mL 1
Frasco Gerber™ para cromatografía 1
Frasco vial 2
Papel filtro para embudo Büchner 2
Regla de 30 cm 1
Lápiz 1
Tubo capilar 2
Pipeta Pasteur desechable de 1 mL 3
Bolsa de plástico 2
Pinza para tubo de ensayo 1
Recipiente para hielo 1

REACTIVOS
2,4-Dinitrofenilhidracina en etanol y en presencia 1 FRASCO GOTERO
de ácido sulfúrico
Nitrato de plata (disolución acuosa) 1 FRASCO GOTERO
Hidróxido de amonio concentrado 1 FRASCO GOTERO
Reactivo de Jones (dicromato de potasio disuelto en 1 FRASCO GOTERO
ácido sulfúrico)
Agua destilada 1 PISETA
Dioxano 1 FRASCO GOTERO
Hidróxido de sodio (10 %) 1 FRASCO GOTERO
Disolución de yodo-yoduro de potasio 1 FRASCO GOTERO
Acetona 1 FRASCO GOTERO
Mezcla Hexano: Acetato de etilo (eluyente) Frasco Gerber™ que va
a servir como cámara de
elución
Etanol 1 FRASCO GOTERO

302
Procedimiento.
REACCIÓNES PARA IDENTIFICAR LA PRESENCIA DE UN GRUPO CARBONILO.

a) ENSAYO CON ÁCIDO CRÓMICO (REACTIVO DE JONES).

Reacción positiva con aldehídos e hidroxicetonas y negativa para cetonas.

R O R O
+ CrO3 + H2SO4
H OH

Reacción de identificación.
En un tubo de ensayo, disolver una gota o 10 mg de la muestra problema en 1 mL de acetona, añadir
varias gotas del reactivo de Jones que se encuentra en el frasco gotero etiquetado. Un resultado
positivo vendrá indicado por la formación de un precipitado verde o azul de sales cromosas (Cr
(III)).

b) PRUEBA DE TOLLENS PARA IDENTIFICACIÓN DE ALDEHÍDOS.


Reacción positiva para aldehídos y negativa para cetonas. Se efectúa solamente en caso de obtener
una prueba positiva con ácido crómico para evitar pruebas positivas falsas.

R O R O
+ 2 Ag(NH3)2OH
H OH

Preparación del reactivo.


En un tubo de ensayo limpio se colocan 2 mL (35 gotas) de la disolución de nitrato de plata, y una a
dos gotas de NaOH al 10%. Se deberá observar la aparición de un precipitado que es el óxido de
plata. A continuación se adiciona, gota a gota y con agitación, una disolución de hidróxido de
amonio justo hasta el punto en que se disuelva el óxido de plata que precipitó, evitando cualquier
exceso. Este reactivo debe usarse recién preparado.

Identificación de aldehídos.
Al reactivo recién preparado se agregan 0.1 g (o 2 gotas) del problema, se agita y se calienta en baño
de agua brevemente. La aparición de un espejo de plata indica una prueba positiva. Si la prueba es
positiva, el tubo de ensayo se deberá entregar junto con el examen.

c) PRUEBA DEL YODOFORMO.


Reacción positiva para metilcetonas y alcoholes precursores del tipo estructural R-CH(OH)-CH3.
El acetaldehído es el único aldehído que da reacción positiva.

303
R O R O
+ I2 + NaOH
CH3 OH

Reacción de identificación.
En un tubo de ensayo se colocan 0.1 g (o de 2 a 3 gotas) del problema, se agregan 2 mL de agua y, si
el problema no es soluble en ella, se agregan 3 mL (30 gotas) de dioxano. Se adiciona 1 mL (15
gotas) de NaOH al 10% y después, gota a gota y agitando, una disolución de yodo-yoduro de potasio
justo hasta que el color café del yodo persista.
La mezcla se calienta en baño de agua durante 2 minutos. Si durante este tiempo el color café
desaparece, se agregan unas gotas más de la disolución de yodo-yoduro de potasio, hasta que el
color no desaparezca después de dos minutos de calentamiento.
La disolución se decolora agregando de 3 a 4 gotas de NaOH al 10%, se diluye con agua hasta casi
llenar el tubo, se deja reposar enfriando. La aparición de un precipitado amarillo indica prueba
positiva para metilcetonas.

INDICACIONES IMPORTANTES.
 Es importante que antes de llevar a cabo cada prueba, los tubos y el material estén limpios.
 Tener cuidado de no contaminar los reactivos al usarlos.
 Usar las cantidades de reactivos y problemas especificados en cada prueba, pues un exceso lleva
a interpretaciones falsas.

Hasta el momento, con base en las reacciones de identificación, ya puedes decir si tienes un
aldehído o una cetona (incluso si tu compuesto desconocido es una metilcetona), pero no sabes
cuál es. Para eso tienes que llevar a cabo un análisis por cromatografía en capa fina de la 2,4-
dinitrohidrazona que vas a obtener de la siguiente forma:

d) FORMACIÓN DE 2,4-DINITROFENILHIDRAZONAS.
Reacción general tanto para aldehídos como para cetonas:

Reacción de identificación.
En un vaso de precipitado, se disuelven 0.5 mL de la muestra en 5 mL de etanol y se adicionan 5 mL
de la disolución del reactivo de 2,4-dinitrofenilhidracina, se calienta en baño de agua durante 5
minutos. Se deja enfriar y se induce la cristalización. La aparición de un precipitado indica una
prueba positiva y por lo tanto la presencia del grupo carbonilo. El precipitado se filtra y se purifica
por medio de una recristalización de etanol o etanol-agua. CON ESTE SÓLIDO PURO SE VA A
REALIZAR EL ANÁLISIS POR CROMATOGRAFÍA EN CAPA FINA.

304
CROMATOGRAFÍA EN CAPA FINA
INTRODUCCIÓN TEÓRICA

La cromatografía en capa fina es una técnica de adsorción sólido-líquido, utilizada en química


orgánica para realizar un análisis cualitativo eficiente y rápido de compuestos desconocidos o para
determinar la composición de una mezcla de compuestos.
Esta técnica cromatográfica consiste en la utilización de una fase estacionaria (alúmina o sílica-gel)
y de una fase móvil, que son los disolventes orgánicos de diferente polaridad.
El proceso de separación se basa en que la fase móvil asciende a través de la fase estacionaria, la
elución de la muestra problema está en función de su polaridad, es decir de la afinidad que presente
con alguna de las dos fases.
La fase estacionaria consiste de una película delgada (100 m) de un material disperso sobre una
superficie plana (de vidrio, aluminio o celulosa).
Las ventajas del método son la rapidez para llevar a cabo el análisis (un análisis normal toma de 2 a
10 minutos) y se pueden detectar cantidades de material de 2 a 20 g.
La secuencia de pasos que se siguen para llevar a cabo el análisis por cromatografía en capa fina
son:
1) Se traza una línea paralela a la base de la cromatoplaca (aproximadamente a 0.5 cm de la base de
la misma) sobre la superficie del adsorbente con un lápiz (NOTA: no recargues mucho la punta
del lápiz, ya que se puede llegar a romper la superficie de la fase estacionaria). La muestra que se
va a analizar (1 mg o menos) se coloca en un tubo de ensayo, y se adicionan unas gotas del
disolvente (acetona) hasta observar la disolución de la muestra. Con un tubo capilar se aplica una
pequeña fracción de la dusolución sobre la placa.

parte superior 0.5 cm


límite de ascensión
del disolvente

punto de aplicación
parte inferior 0.5 cm

2) La cromatografía se lleva a cabo colocando la cromatoplaca sobre la cual se aplicó la muestra,


dentro de un frasco de vidrio con tapa (son ideales para este fin los frascos de los alimentos
infantiles), el cual ya debe contener la fase móvil (unos 3 mL). El frasco debe estar cerrado con la
finalidad de mantener una atmósfera saturada con el disolvente de la fase móvil. Efectuadas estas
operaciones, la fase móvil asciende rápidamente por capilaridad sobre el adsorbente de la fase
estacionaria hasta la marca superior. Inmediatamente se saca la placa de la cámara de elución y se
deja evaporar el disolvente. La muestra se puede encontrar en cualquier punto.
3) Para localizar el compuesto de la muestra problema, normalmente se utiliza un revelador, pero en
este caso no es necesario ya que las 2,4-dinitrofenilhidrazonas de las muestras problemas son
coloridas. Una medida física de la polaridad de la muestra es el Factor de Retención (Rf), el cual

305
se determina dividiendo la distancia que recorrió la muestra entre la distancia que recorrió el
disolvente, el Rf siempre es menor a 1. Bajo las mismas condiciones de análisis cromatográfico,
el Rf es constante y permite identificar cualitativamente un compuesto o bien determinar si se
encuentra presente en una mezcla problema:

parte superior línea hasta la que


recorró el eluyente

b a
Rf =
b

punto de aplicación
parte inferior

NOTA: No se te olvide entregar junto con tu examen la placa y la hidrazona que te sobró en
las bolsitas de plástico que están en tu lugar.

306
PREGUNTAS
1. Escribe las ecuaciones balanceadas de las reacciones que te permitieron identificar si tu
compuesto desconocido era un aldehído, una cetona o bien una metilcetona.

RESPUESTA

2. ¿Qué tipo de reacción se lleva a cabo con el reactivo de Tollens?

a) Sustitución. RESPUESTA
b) Eliminación.
c) Redox.
d) Transposición.
e) Adición nucleofílica.
f) Adición electrofílica.

307
3a. Escribe la ecuación balanceada de la reacción que explique la formación de la
2,4-dinitrofenilhidrazona de tu aldehído o de tu cetona.

RESPUESTA

3b. ¿Por qué es importante el ácido sulfúrico en la reacción anterior?

i) Como reactivo ácido que es necesario en una relación 1:1 con RESPUESTA
respecto a la base.
ii) Como catalizador ácido que es necesario en una relación 1:0.25
con respecto a la base.

4. Determina el Rf de la 2,4-dinitrofenilhidrazona del compuesto carbonílico que se te asignó.


Marque las distancias y muestra como lo determinaste.

5. Con base a tu respuesta a la pregunta anterior y los resultados de las pruebas químicas, ¿cuál fue
tu compuesto carbonílico problema de los 4 posibles?

RESPUESTA

Bibliografía.

1) Vogel, A. I.; Elementary Practical Organic Chemistry, Part. 2. Págs. 108-109 y 111-116.
2nd. Edition, Ed. Longman, London, 1966.
2) Pasto, D. J.; Jonson, C. R.; Determinación de Estructuras Orgánicas. Ed. Reverté, México,
1974.
3) Shriner, R. L.; Fusson, R. C.; Curtin, D. Y.; Identificación Sistemática de Compuestos
Orgánicos. Ed. Limusa-Wiley, México, 1966.
4) Fiegl, F.; Anger, V.; Pruebas a la Gota en Análisis Orgánico. Págs. 194-200 y 204-350, Ed.
El Manual Moderno, México, 1978.

308
XXI Olimpiada Nacional de Química

1er Examen Nivel A y B.


Total: 35 preguntas. Tiempo asignado: 75 minutos. Guadalajara 2012

Debes anotar en el recuadro correspondiente la letra del inciso que contesta correctamente cada
pregunta. Si consideras que ningún inciso es el correcto deberás anotar una letra X.

NOTA ACLARATORIA: Al hacer tus cálculos puedes encontrar una pequeña diferencia, en
general no mayor de unas décimas, entre tu resultado y alguno de los propuestos. Esto es
normal y por eso sólo deberás usar la “X” cuando ninguna respuesta sea muy cercana a la que
tú obtengas. CONTESTA CON TINTA USANDO MAYÚSCULAS.

Este año, la Olimpiada Internacional de Química se celebrará en los Estados Unidos de América,
donde todavía usan unidades diferentes a las del sistema internacional, como las libras (1 libra =
0.454 kilogramos), los pies (1 pie = 30.5 centímetros) y los galones (1 galón = 3.785 litros).

1) La densidad del agua de mar es 1025 kg/m3 y la del agua destilada es de 1000 kg/m3.
Si un recipiente que, totalmente lleno, puede contener 100 moles de agua destilada se
vacía y se llena totalmente con agua de mar, ¿cuántos kilogramos de agua de mar
puede contener?
A 0.097 B 0.975
C 1.845 D 1845.00

2) ¿Cuál es, aproximadamente, la densidad del agua destilada expresada en libras/pie3?

A 12.88 B 77632.7
C 62.5 D 7221.8

3) En una molécula de agua, la distancia entre el átomo de oxígeno y uno de los átomos
de H es de 98.5 pm (1 pm = 1x10-12 m). Si tomamos esta distancia como el radio de
una esfera, ¿cuál será el volumen de esta esfera en cm3? Vesfera = 4/3 π r3
A 4.0 x 10 – 32 B 4.0 x 10 – 62
C 4.0 x 10 – 30 D 4.0 x 10 – 24

4) Si suponemos que una molécula de N2 cuando está “pegada” a una superficie sólida
ocupa un área de 16 nm2 (1 nm = 1 x 10-9 m), ¿cuántas moléculas de N2 se requieren
para ocupar un área de un centímetro cuadrado?
A 6.25 x 10 12 B 6.25 x 10 18
10
C 1.6 x 10 D 1.6 x 10 18

309
5) Si mezclamos cantidades iguales, en gramos, de ciclohexano (C6H12) y benceno
(C6H6), la fracción mol del ciclohexano es:

A 0.481 B 0.519
C 1.077 D 0.928

6) Las ppm (partes por millón) son unidades de concentración, muy utilizadas en
estudios de contaminación de aguas o concentración de elementos en fluídos de seres
vivos. En gramos, una ppm equivale a un gramo en un millón de gramos. Si se
lograra disolver un mol de hexano (C6H14) en 1000 moles de benceno, la
concentración de hexano en esta mezcla sería aproximadamente en ppm:
A 1000 B 1103
C 906 D 861

7) La densidad del oro es de 19320.0 kg/m3. Con 100 g de oro se recubre una superficie
sólida de tal manera que el espesor del recubrimiento es de 0.001 mm. El área del
sólido que se puede recubrir es, en metros cuadrados:
A 1.932 B 19,320,000.00
C 5.176 D 0.0005176

8) En este dibujo puedes ver la estructura del coroneno, formado con seis estructuras de
benceno unidas de la manera que se muestra.

benzeno coroneno
La masa molar del coroneno es, en g/mol:
A 288.326 B 300.336
C 462.600 D 468.648

9) Si lograras disolver completamente 60.0 g de cloruro de sodio en agua destilada hasta


tener 100 cm3 de disolución, la concentración molar de la sal sería aproximadamente:
A 0.60 mol L – 1 B 10.3 mol L – 1
–1
C 1.03 mol L D 6.0 mol L – 1

10) A 20 oC, sólo se pueden disolver 35.9 g de NaCl en 100 g de agua. ¿Cuál es,
aproximadamente, la fracción mol de la sal en esta disolución?
A 0.036 B 0.36
C 0.10 D 0.01

11) Para producir ácidos carboxílicos en el laboratorio en ocasiones se utiliza una


disolución acuosa de permanganato de potasio (KMnO4). El estado de oxidación del
manganeso en este compuesto es:
A +7 B +5
C +3 D +1

310
12) Si se hace reaccionar el KMnO4 con ácido sulfúrico concentrado se obtiene un
compuesto que puede explotar al contacto, el Mn2O7. En la reacción propuesta se
obtiene además agua y KHSO4. Al balancear esta reacción se encuentra que por cada
mol de permanganato, reaccionan de H2SO4:
A 0.5 moles B 1.0 mol
C 1.5 moles D 2.0 moles

13) Considera la reacción que ocurre en una celda electroquímica:

Ce4+(ac) + Bi(s) + H2O (l) → Ce3+(ac) + BiO+(ac) + H+(ac)

Balancea esta ecuación,tomando en cuenta las cargas; se nota que por cada mol de
agua deben reaccionar, de moles de Ce4+:
A Un mol B Dos moles
C Tres moles D Más de tres moles

14) Si en la molécula de benceno se sustituye un hidrógeno por un grupo OH se produce


el fenol, una sustancia cuya estructura es:
OH

Se pueden disolver unos 314 g de fenol en un galón de agua. ¿Cuántos litros de agua
se necesitarían para disolver un mol de fenol?
A 1.13 B 0.88
C 3.34 D 0.334

15) SO2 y BaO son ejemplos de:

A Óxidos ácidos B Óxidos básicos


C Óxido básico y óxido ácido D Óxido ácido y óxido básico
respectivamente respectivamente

16) La combustión de compuestos aromáticos suele ser incompleta y se produce también


CO además de CO2 y H2O. Si al quemarse un mol de fenol se produce la misma
cantidad de sustancia en moles de CO y CO2, ¿cuántos moles de oxígeno gaseoso se
consumen?
A Más de 15 B Entre 10 y 15
C Entre 6 y 9.5 D Menos de 6

17) El ácido benzoico también es un derivado del benceno y se obtiene al sustituir un


hidrógeno por un grupo –COOH. El % en masa de oxígeno en la molécula del ácido
benzoico es:
A Menos de 20% B Entre 20% y 25%
C Entre 25.1% y 30% D Más de 30%

311
18) Un estudiante toma 200.0 mL de una disolución acuosa de HCl 2.0 M y le agrega
agua destilada hasta tener 500.0 mL de disolución. El pH de la disolución preparada
es:
A – 0.097 B 0.097
C 0.397 D 3.97

19) Cuando un gramo de agua es adicionado a una mezcla de ácido y agua, la nueva
mezcla es 25% en masa de ácido. Cuando 2 gramos de ácido son adicionados a la
nueva mezcla, la mezcla ahora contiene 50% en masa de ácido. El % en masa de
ácido en la mezcla original era:
A Menos de 25% B Entre 25% y 30%
C Entre 31% y 36% D Más de 36%

20) La ley de los gases ideales es PV = nRT, donde:


(R = 0.082 L atm / mol K) (R = 8.314 J / mol K) (1 J = 1 kg m2 s–2)
(1 atm = 101.325 kPa) (1 Pa = 1 N m–2) (1 N = 1 kg m s–2)
A 25 oC y 100.0 Pa, un mol de un gas ideal ocupa un volumen de:
A Menos de 30 L B Entre 30 L y 2000 L
C 2.0 x 104 L D Más de 2.0 x 104 L

21) En una fábrica se produce cobre metálico a partir de una disolución que contiene
iones Cu2+, utilizando una reacción electroquímica. El enunciado correcto es:

A El Cu metálico se obtiene en el ánodo por reducción de los iones Cu2+


B El Cu metálico se obtiene en el ánodo por oxidación de los iones Cu2+
C El Cu metálico se obtiene en el cátodo por reducción de los iones Cu2+
D El Cu metálico se obtiene en el cátodo por oxidación de los iones Cu2+

22) En un recipiente de 2.0 L de capacidad se coloca la misma cantidad en gramos de


nitrógeno, oxígeno y CO gaseosos. La fracción mol de nitrógeno en esta mezcla es:
A Menor a 0.3 B Entre 0.3 y 0.4
C Entre 0.4 y 0.5 D Mayor a 0.5

23) La corriente que circula en una celda electroquímica se puede medir en amperes (A).
Un ampere equivale a una carga de un coulomb (C) que pasa cada segundo por un
punto del circuito. La carga de un electrón es 1.6 x 10–19 C.
Si en una celda electroquímica circula una corriente de 5 mA, significa que el número
de electrones que pasan por un punto del circuito cada segundo son:
A 3.125 x 1016 B 3.2 x107
–16
C 3.125 x 10 D 3.2 x 10 –17

24) La constante de Faraday es la carga contenida en un mol de electrones, por lo tanto


su valor es:

A Menos de 50 mil coulombs B Entre 50 mil y 100 mil coulombs


C Entre 100 mil y 200 mil coulombs D Más de 200 mil coulombs

312
25) En una celda electroquímica, para reducir un mol de iones plata Ag+ y convertirlos
en átomos de plata metálicos, se requiere que circule por la celda una carga total de:
A 96368 C B 289104 C
C 192736 C D 48184 C

26) Masas: electrón: 9.1095 x 10–28 g; neutrón: 1.67495 x 10–24 g;


protón: 1.67252 x 10–24 g.
La masa de un átomo de helio es:
A 3.346 x 10–24 g B 5.0218 x 10–24 g
C 5.0242 x 10–24 g D 6.696 x 10–24 g

27) De acuerdo a la expresión “X” → 22286 Rn +  podemos concluir que el átomo


radiactivo “X” es: (Las partículas alfa son núcleos de helio)
A 22684 Po B 21888 Ra
226
C 88 Ra D 21884 Po

28) Para obtener un litro de HCl 6 N a partir de una disolución “1” de HCl 12 N y una
disolución “2” de HCl 4 N, se deben mezclar:
A 1/3 L de 1 + 2/3 L de 2 B 2/3 L de 1 + 1/3 L de 2
C 3/4 L de 1 + 1/4 L de 2 D 1/2 L de 1 + 1/2 L de 2

29) ¿ Cuál es la solubilidad del hidróxido de cobre (II), expresada en mol L–1, y el pH de
la disolución saturada de Cu(OH)2 en agua pura?
Datos: Ks del Cu(OH)2 = 2.210–20.
A 1.810–7 mol L–1 y pH = 7.6 B 2.8 10–7mol L–1 y pH = 7.3
C 2.810–7 mol L–1 y pH = 7.6 D 1.810–7 mol L–1 y pH = 7.3

30) La presencia del ion Fe3+ en una disolución acuosa se detecta por aparición de una
coloración rojo “sangre” cuando se añaden unas gotas de una disolución de algún
tiocianato alcalino. Esta identificación se debe a:
A La reacción de reducción del Fe3+ por el B La reacción de complejación del
SCN– Fe3+ con el SCN–
C Un cambio de pH al añadir el tiocianato D La precipitación del Fe(SCN)3

31) Balancea la siguiente ecuación química:


KMnO4 + H2O2 + HCl → MnCl2 + O2 + KCl + H2O

Por cada mol de agua oxígenada que reacciona, se obtiene de agua:


A Entre 0.5 y 1.0 moles B Entre 1.1 y 1.9 moles
C Entre 2.0 y 3.0 moles D Más de 3.0 moles

32) Un mol del hidróxido de este elemento reacciona con 3 moles de ácido clorhídrico y
se obtiene un mol del cloruro del elemento más 54 g de agua. La masa molar del
cloruro obtenido es 133.34 g/mol. El elemento cuyo hidróxido se usa como reactivo
es:
A Na B Mg
C Al D Ca

313
33) En el compuesto "K2ZCl4", la letra “Z” representa un elemento de transición. La
densidad de este compuesto sólido es igual a 3.38 g/cm3 y un mol de "K2ZCl4" ocupa
un volumen de 0.1228 litros. El elemento "Z" es:
A Ag B Pd
C Hg D Pt

34) Las cetonas son compuestos orgánicos que se caracterizan por tener un enlace doble
entre un átomo de carbono y un átomo de:
A O B N
C Ce D S

35) En la molécula de bicarbonato de sodio, el elemento con mayor % en masa es:

A H B C
C O D Na

FIN DEL EXAMEN

314
XXI Olimpiada Nacional de Química. 2do Examen Nivel A y B.
Nivel B: 16 preguntas.
Nivel A: 18 preguntas.
Tiempo asignado: 90 minutos. Guadalajara 2012

PREGUNTAS DE QUÍMICA ANALÍTICA PARA NIVEL A Y B:

Escribe la letra del inciso correcto en el recuadro, con tinta y en MAYÚSCULAS:


1. En la siguiente tabla se muestran algunos indicadores con sus correspondientes pH
de vire. ¿Cuál de ellos es el más adecuado para titular con hidróxido de sodio el
ácido acético de un vinagre comercial?
Naranja de metilo: 3,2 – 4,4 Fenolftaleína: 8,2 – 10
Eritrosina: 2,2 – 3,6 Verde de bromocresol: 3,8 – 5,4
A Naranja de metilo B Verde de bromocresol
C Fenolftaleína D Eritrosina

2. Si se mezcla cierto volumen de disolución 2,5 mol/L de cloruro sódico con el


doble de volumen de la misma disolución, la disolución de cloruro sódico
resultante será:
A 7,5 mol/L B 5 mol/L
C 2,5 mol/L D Es necesario especificar los
volúmenes

3. ¿Cuál de los siguientes iones será reducido por el ion Cr2+ (ac) en condiciones
estándar?
A Zn2+ (ac) B Cd2+ (ac)

C Al3+ (ac) D Ni2+ (ac)

DATOS para pregunta 3:

Sistema Zn2+/Zn Cd2+/Cd Al3+/Al Cr3+/Cr2+ Ni2+/Ni


E° (V)/ENH -0,79 -0,40 -1.67 -0,41 -0,23

4. Una disolución acuosa 0,01 mol L-1 de un ácido débil tiene un grado de disociación
(α) de 0,20, el pOH es:
A 2,7 B 2
C 11,3 D 12

5. El pH de la disolución resultante de mezclar dos volúmenes iguales de una


disolución de ácido clorhídrico 0,01 mol L-1 y de una disolución de hidróxido
sódico 0,02 mol L-1 es:
A pH > 7 B pH < 7
C pH = 7 D No se puede saber sin conocer los
volúmenes

315
6. De las sales citadas a continuación, ¿cuáles darán disolución ácida?
KCl, NaNO3, (NH4)2SO4, HCOOK.
A KCl y NaNO3 B HCOOK y KCl
C (NH4)2SO4 D Ninguna

7. Sabiendo que los potenciales normales de reducción son: Eº(Fe+2/Fe) = -0,44V y


Eº(Zn+2/Zn) = -0,79 V; y se emplea una varilla de hierro para agitar una disolución
de ZnSO4 puede ocurrir lo siguiente:
A Se reduce el cinc B Se oxida el Fe
C No pasa nada D Se reduce el azufre

8. Para determinar el ácido acético (CH3COOH) de un vinagre, se introducen 25 mL


de vinagre en un matraz graduado de 250 mL y se añade agua destilada para
completar el volumen hasta el aforo. Una vez homogeneizada la disolución se
neutraliza una alícuota de 20 mL con 16,0 mL de hidróxido sódico 0,10 mol L-1.
La concentración de ácido acético en el vinagre es:
A 0,08 mol L-1 B 4,8 % m/m
-1
C 1,3 mol L D 48 g/L

9. ¿Qué volumen de una disolución de K2CrO4 0,25 mol L-1 se debe diluir con agua
para preparar 250 mL de disolución de K2CrO4 0,01 mol L-1?
A 1L B 100 mL
C 10 mL D 0.1 mL

10. Cuando las autoridades comunican una alerta medioambiental por haberse
detectado una concentración de SO2 de 3000 ppm significa que:
A Hay 3000 mg SO2 / L de aire B Hay 3000 mL de SO2 / L de aire
C Hay 3000 cm3 de SO2 / m3 de aire D El 3% de un volumen de aire es
SO2

11. El producto de solubilidad del hidróxido de hierro (III) a 20 °C es 6,0 x·10–38.


¿Qué masa de este compuesto se disolverá en 100 mL de hidróxido de sodio 0,2
mol/L, suponiendo que no hay formación de complejos?
A 6·10–39 g B 7·10–37 g
C 8·10–35 g D 9·10–35 g

PREGUNTAS DE FISICOQUÍMICA PARA NIVEL A Y B:

12. Para calentar los alimentos se utiliza gas en las casas, en algunos casos es gas L. P. (gas licuado
de petróleo) y en otras gas natural. El gas L. P. es fundamentalmente una mezcla de los gases
propano y butano, en tanto que el gas natural principalmente contiene gas metano (90 a 95%) y
otros gases en menor proporción.

Un tanque de gas de 250 L contiene 20 kg de propano (C3H8) y butano (C4H10). La mezcla


contiene 40 % en masa de propano. (Haz tus cálculos suponiendo comportamiento ideal).

316
(Utiliza los cuadros para tus cálculos y marca con X la respuesta correcta). Sin cálculos NO
SE TOMARÁ EN CUENTA AUNQUE LA RESPUESTA SEA CORRECTA.

R = 8.314 J/mol K = 8.314 Pa m3/mol K

12.1. La fracción mol de propano en la mezcla es:


Cálculos:

a) 0.879 b) 0.666 c) 0.532 d) 0.468

12.2. El volumen de gas a la presión de un bar y una temperatura de 20 °C, cuando se evapora un
mol de mezcla es:
Cálculos:

a) 2.4 x 10 – 3 m3 b) 2.4 x 10 – 2 m3 c) 1.27 x 10 – 3 m3 d) 1.27 x 10 – 2 m3

12.3. La cantidad de sustancia, en moles, de gas butano cuando se evaporan 700 g de la mezcla es:
Cálculos:

a) 15.9 mol b) 12.07 mol c) 7.24 mol d) 4.83 mol

12.4. La masa de butano, en gramos, cuando se evaporan 2.5 moles de la mezcla es:
Cálculos:

a) 58 g b) 77 g c) 87 g d) 92 g

317
12.5. Si la presión manométrica del tanque es de cero, ¿cuánto gas queda dentro del tanque si está en
una ciudad donde la presión barométrica es de 1 bar y la temperatura 20 °C?
Cálculos:

a) Entre 0.0 y 2.0 mol b) Entre 2.1 y 5.0 mol c) Entre 8 y 11 mol d) Más de 11 mol

Espacio para cálculos adicionales

318
PREGUNTAS DE QUÍMICA ORGÁNICA SÓLO PARA NIVEL A:

Para las preguntas 13, 14 y 15, debes poner una palomita (√) en el inciso correcto.

13. El producto de la siguiente reacción se muestra en la opción:

Ph
CH3ONa
H Br
?
H CH3 CH3OH

Ph

A) B)
CH3 H Ph

CH2
Ph Ph
Ph

C) D)
Ph H H Ph

Ph CH3 Ph CH3

14. ¿Cuál compuesto es el producto mayoritario en la siguiente reacción?

Br2
?
h

A) B) C) D)

Br

Br
Br
Br

319
15. La mejor manera de sintetizar en el laboratorio el siguiente alcohol es:

OH

A) B)

1) (AcO)2Hg, H2O 1) KMnO4, frío


2) NaBH4 2) NaOH (ac.)

C) D)
H2O 1) BH3
H2SO4 2) H2O2, NaOH

PREGUNTAS DE QUÍMICA INORGÁNICA PARA NIVEL A Y B:

Para las preguntas 16 y 17, debes poner un tache “X” en el inciso correcto:

16. El grafito es un material ceroso que conduce la corriente eléctrica y es quebradizo. El diamante
es duro y no conduce la corriente eléctrica. En ambos casos se trata de sustancias formadas
exclusivamente por átomos de carbono. En el caso del grafito estos se encuentran en láminas
hexagonales y en el diamante forman tetraedros regulares. Las diferencias se pueden explicar
proponiendo:

A) Hibridación sp en el grafito y sp3 en el diamante.

B) Hibridación sp2 en el grafito y sp3 en el diamante.

C) Hibridación sp en el grafito y sp2 en el diamante.

D) Hibridación sp3 en el grafito y sp2 en el diamante.

E) Hibridación sp3 en el grafito y sp en el diamante.

320
17. Considera la reacción entre el ácido sulfúrico y el hidróxido de sodio. Si se hace reaccionar 35.0
mL de una solución del ácido 0.175 mol L-1 con una solución de la base 0.250 mol L-1, ¿cuántos
mililitros de la solución de la base se requieren para hacer reaccionar todo el ácido?

A) Menos de 35 mL B) Entre 35 y 45 mL C) Entre 45 y 55 mL

D) Entre 55 y 65 mL E) Más de 65 mL

18.1. La estructura de Lewis para el ion tiosulfato es:

18.2. El estado de oxidación y la carga formal de cada uno de los átomos de azufre en el ion
tiosulfato son:

18.3. La hibridación propuesta para el átomo central es:

321
PREGUNTAS DE QUÍMICA INORGÁNICA SÓLO PARA NIVEL B:

19. En este problema tendrás que acomodar nueve de los primeros 18 elementos de la Tabla
Periódica en el diagrama que se muestra a continuación. Toma en cuenta los datos que se te
proporcionan y escribe en cada recuadro el símbolo del elemento que le corresponde:

1 2 3

4 5 6

7 8 9

Los primeros 18 elementos son:

H He Li Be B C N O F Ne

Na Mg Al Si P S Cl Ar

Cuando termines de acomodar los elementos notarás que el número atómico siempre va
en aumento desde la casilla 1 hasta la 9.

Datos:
1. En la columna de en medio están el único halógeno considerado en este problema y el gas que se
considera más importante para la vida.

2. En la fila inferior está un elemento que se presenta en varias formas alotrópicas, como rómbico y
monoclínico.

3. En la columna derecha está el único metal alcalino considerado en este problema y el elemento
que es la base de la química orgánica.

4. Seis de los elementos son gases a temperatura ambiente y presión de una atmósfera; ninguno de
los dos gases nobles incluidos en este problema está en la columna izquierda.

5. El elemento más abundante en el aire que respiramos está en una casilla con número par.

FIN DEL EXAMEN

322
XXI OLIMPIADA NACIONAL DE QUÍMICA
TERCER EXAMEN Tiempo asignado: 180 minutos Guadalajara 2012
NIVEL A: 12 problemas; NIVEL B: 9 problemas

FISICOQUÍMICA
NIVELES A Y B.

Problema No. 1. El ciclo de Carnot


Durante un ciclo de Carnot, una máquina trabaja
absorbiendo una cantidad de calor (Qabsorbido) a una
FC temperatura Tc de una fuente caliente (FC). Parte de este calor
absorbido lo transforma la máquina en trabajo (Wrealizado) que
desarrolla en los alrededores y parte del calor absorbido es
Qc transferida (Qcedido) a la fuente fría (FF). Mientras más calor se
transforma en trabajo es mejor y se dice que la máquina tiene
una mayor eficiencia.
MT W
La eficiencia (), se puede calcular fácilmente a partir del
trabajo realizado y el calor absorbido:
Qf  = Wrealizado/Qabsorbido

O bien, a partir de las temperaturas de las fuentes de calor:


FF  = (Tc – Tf)/Tc, donde Tc = temperatura de la fuente caliente
en Kelvin; Tf = temperatura de la fuente fría en Kelvin.

(Utiliza los cuadros para tus cálculos y marca con X la respuesta correcta).
Respuestas correctas sin cálculos claros y correctos no serán tomadas en cuenta.

R = 8.314 J/mol K = 8.314 Pa m3/mol K; T(K) = T(0C) + 273.15

1.1. Cuando un ciclo de Carnot opera entre las temperaturas de 600 °C y 300 °C la eficiencia del
ciclo es:

a) 0.24 b) 0.50 c) 0.34 d) 0.75

Un ciclo de Carnot opera un ciclo por segundo y realiza una cantidad de trabajo de 25 kJ por cada
ciclo, transfiriendo a la fuente fría 15 kJ por cada ciclo.

323
1.2. La eficiencia de este ciclo es:

a) 0.375 b) 0.455 c) 0.625 d) 0.727

1.3. El trabajo realizado por el ciclo anterior durante una hora es:

a) 36 000 kJ b) 54 000 kJ c) 90 000 kJ d) 144 000 kJ

1.4. Si la eficiencia de otra máquina de Carnot es  = 0.475 y produce un trabajo W de 30 kJ por


ciclo cada segundo, el calor sustraído de la fuente caliente durante 15 minutos es:

a) 947 kJ b) 3 789 kJ c) 12 825 kJ d) 56 842 kJ

1.5. ¿A qué temperatura debe operar una máquina cuya fuente fría está a temperatura ambiente de 20
°C si la eficiencia  = 0.75:

a) 1 173 K b) 664 K c) 353 K d) 300 K

FISICOQUÍMICA
SÓLO NIVEL A.

Problema No. 2. Calcula el valor de la constante de equilibrio, a 298 K, para la descomposición de


metano gaseoso en la que se forman los elementos hidrógeno gaseoso y carbono en forma de grafito.

La entalpía estándar de formación del metano gaseoso es: – 74.85 kJ/mol a 298 K.

La entropía estándar de formación del metano gaseoso es: – 80.67 J/mol K a 298 K.

Recuerda que: ΔG = ΔH – TΔS y ΔG = – RT ln K R = 8.314 J/mol K

324
Cálculos:

Valor de K = ______________________________________

FISICOQUÍMICA
SÓLO NIVEL B.

Problema No. 3. El refrigerador de Carnot

FC Si un ciclo de Carnot se opera al revés, en vez de realizar


trabajo en los alrededores, son los alrededores los que
suministran trabajo a la máquina y con esto se logra sustraer
Qc calor de la fuente fría y enviarlo a la fuente caliente (como
opera un refrigerador).

Para evaluar el desempeño de un refrigerador se evalúa el


MT W coeficiente de rendimiento  que es la relación del calor
sustraído de la fuente fría entre el trabajo suministrado:

Qf  = Qabsorbido de la fuente fría / Wsuministrado

O bien, a partir de las temperaturas de las fuentes de calor:


FF  = Tf /(Tc – Tf)

(Utiliza los cuadros para tus cálculos y marca con X la respuesta correcta).
Respuestas correctas sin cálculos claros y correctos no serán tomadas en cuenta.

325
3.1. Un refrigerador de Carnot opera con un coeficiente de rendimiento  = 9.5 a una temperatura
ambiente de 20 °C. La temperatura más baja que puede alcanzar es:

a) – 7.9 °C b) – 8.5 °C c) – 10.2 °C d) – 18.2 °C

3.2. El agua congela a 0 °C y su calor latente de fusión () es  = 6025 J/mol. ¿Cuánto trabajo (en
joules), consume el refrigerador anterior si congela 1.235 kg de agua a 0 °C?

a) 7.83 x 102 J b) 7.44 x 103 J c) 4.35 x 104 J d) 4.13 x 105 J

3.3. Los refrigeradores comerciales alcanzan temperaturas de – 20 °C. El coeficiente de rendimiento


de un refrigerador comercial que opera a temperatura ambiente de 30 °C es:

a) 1.2 b) 5.06 c) 8.4 d) 10.1

326
QUÍMICA ANALÍTICA
NIVELES A Y B.

Problema No. 4. Para preparar las siguientes disoluciones se mezclan:

1) 10 mL de NaOH 0,1 mol L-1 con 25 mL de NH3 0,1 mol L-1


2) 15 mL de NaOH 0,1 mol L-1 con 50 mL de acetato sódico 0,1 mol L-1
3) 15 mL de NH4Cl 0,1 mol L-1 con 30 mL de ácido acético 0,1 mol L-1
4) 25 mL de HCl 0,1 mol L-1 con 25 mL de NH3 0,1 mol L-1
5) 25 mL de HCl 0,1 mol L-1 con 50 mL de acetato sódico 0,1 mol L-1

4.1. Indica en que caso(s) se obtiene una disolución amortiguadora del pH

A) En la disolución 5
B) En las disoluciones 4 y 5
C) En las disoluciones 1 y 2
D) En la disolución 3

Problema No. 5. Si se conocen los siguientes potenciales condicionales a pH = 7:

E0 V/ENH Co3+/Co2+= 1,81; O2/H2O2= 0,27; O2/H2O= 0,84; H2O2/H2O= 1,36

¿Qué ocurre al preparar una disolución acuosa de Co3+?

A) No pasa nada
B) Se reduce el oxígeno del aire con formación de agua oxigenada
C) Se oxida el agua con desprendimiento de O2
D) Se reduce el oxígeno del aire con formación de agua
E) Se reduce el agua con desprendimiento de hidrógeno

Problema No. 6. Al reaccionar una muestra de 0,2260 g de un sulfato XSO4 con BaCl2 en exceso se
producen 0,4382 g deBaSO4.

6.1. ¿Cuál es el elemento X?


Cálculos:

6.2. Escribe la reacción que ocurre entre XSO4 y BaCl2 en exceso:

327
Problema No. 7. Para analizar una muestra que contiene Na2CO3, Na2C2O4 y NaCl se pesaron
0,7371 g de la mezcla y se disolvieron en agua. Posteriormente se añadieron 20,00 mL de una
disolución estandarizada de HCl 0,200 mol/L y, después de calentar a ebullición, se tituló el exceso
de ácido con NaOH 0,1016 mol/L (usando fenolftaleína como indicador); se observó el vire del
indicador con un volumen de 8,3 mL de sosa.
Una segunda muestra de 0,6418 g de la mezcla se calcinaron a 600 oC. El residuo obtenido se disolvió
en agua y se le añadieron 50,00 mL de la disolución estandarizada de HCl 0,200 mol/L. La disolución
obtenida se calentó a ebullición y se tituló con 14,7 mL de la disolución 0,1016 mol/L de NaOH
(utilizando el mismo indicador de fenolftaleína).
Dato: El oxalato se descompone en dos etapas; a 600 oC se transforma en carbonato.

7.1. Escribir todas las reacciones que ocurren durante el análisis.

7.2. Calcular la composición (%) de los componentes de la mezcla.

______________________________________________________________________

______________________________________________________________________

_____________________________________________________________________

_____________________________________________________________________

328
QUÍMICA ANALÍTICA
SÓLO NIVEL A.

Problema No. 8. Calcular la solubilidad del bromuro de plata en una disolución con una
concentración de amoniaco 0,1 mol L-1.
[Kps bromuro de plata = 5,01x10-13; pKdis Ag(NH3)2+ = 7,4].

Cálculos

329
QUÍMICA INORGÁNICA
SÓLO NIVEL B.

Problema No. 9. Completa el siguiente crucigrama y encontrarás en los espacios en blanco de la


vertical y en forma invertida, el nombre de una Ciudad que juega un importante papel para el
desarrollo de la Química en México.
1
1. Producto de reacción entre H2SO4 y una base
2
2. Elemento radiactivo
3
3. Acción de formación de un compuesto sólido
4
4. Unidad de energía
5
5. Se encuentra en los huesos
6
6. Elemento abundante en la corteza terrestre
7
7. Nombre general de los hidrocarburos
saturados
8
8. La fenolftaleína es un
9
9. Forma alotrópica del carbono
10
10. Azúcar simple (monosacárido)
11
11. Estado de agregación

QUÍMICA INORGÁNICA

NIVELES A Y B.

Problema No. 10. Se tienen 7 muestras de cationes (todos son nitratos, dada su solubilidad) los
cuales hay que identificar con 3 reactivos que se te proporcionan: ácido clorhídrico, sulfuro de sodio
y una mezcla de amoniaco/cloruro de amonio, para asegurar un pH cercano a 9.
El catión uno es incoloro, no reacciona con NINGUNO DE LOS REACTIVOS que se te
proporcionaron y a la flama da un color violeta que puede verse mejor a través de un vidrio de
cobalto.
El catión 2 es amarillo, NO reacciona con el HCl pero con el sulfuro da un precipitado negro y con
la mezcla de amoniaco un precipitado gelatinoso café rojizo.
El catión 3 es incoloro también. NO reacciona con el HCl pero con el sulfuro da un precipitado rosa
que corresponde al sulfuro correspondiente y NO precipita con la mezcla de amoniaco/cloruro de
amonio.

330
El catión 4 es azul verdoso. NO precipita con el HCl pero con el sulfuro da un precipitado negro y
con la mezcla cloruro de amonio/amoniaco forma un complejo azul característico.
El catión 5 es verde. No precipita con el HCl pero da un precipitado negro con el sulfuro y NO
precipita con la mezcla cloruro de amonio/amoniaco.
El catión 6 es rosa. No precipita con el HCl, pero cambia de color. Con el sulfuro precipita negro y
con la mezcla amoniaco/cloruro de amonio NO precipita.
Finalmente, el catión 7 es incoloro. No precipita con el HCl pero con el sulfuro forma un precipitado
blanco que corresponde al sulfuro y con el amoniaco/cloruro de amonio se mantiene en disolución
aunque en algún momento se observa un precipitado gelatinoso que se redisuelve por tratarse de un
anfótero.

Indica cuáles son los cationes problema en cada caso.

CATIÓN IDENTIFICADO
1.

2.

3.

4.

5.

6.

7.

Espacio para anotaciones:

331
QUÍMICA ORGÁNICA
SÓLO NIVEL B.

Problema No. 11. En un laboratorio de Guadalajara hay NUEVE compuestos orgánicos que tendrás
que acomodar en la siguiente tabla de acuerdo a los datos que se te proporcionan.

Coloca en cada espacio, el NOMBRE y la FÓRMULA CONDENSADA del compuesto que


corresponde.

1 2 3

4 5 6

7 8 9

DATOS PARA CONTESTAR EL PROBLEMA 11:

1) Hay tres ácidos carboxílicos, uno en cada columna. Uno de éstos es un compuesto aromático de
masa molar igual a 122.12 g/mol. Otro es común mezclarlo con agua.
2) A temperatura ambiente, sólo dos de los compuestos están en estado de gas o de vapor y ambos
están en casillas impares. Otro compuesto es una cetona que se encuentra en varios productos de
uso casero.
3) Sólo uno de los nueve compuestos tiene un punto de fusión mayor a 100 oC, es aromático y está
en la fila inferior.
4) En la fila de en medio hay un alcohol y junto a éste hay un compuesto aromático muy conocido
y que tiene el mismo número de átomos de carbono y de hidrógeno. Uno de los ácidos es un
derivado de este compuesto aromático.
5) El vinagre es una disolución acuosa de un ácido que está en la columna izquierda. Otro de los
ácidos es una molécula lineal con una masa molar de 88.11 g/mol, está en la columna derecha,
justo abajo del compuesto que tiene la menor masa molar.
6) En casillas impares hay dos alcoholes. El que está presente en muchas bebidas está en la fila
inferior.
7) Hay tres compuestos cuya masa molar es menor a 35 g/mol. Uno es un alcohol y los otros dos
son alcanos, ambos ubicados en la fila superior.
8) Si sumas los átomos de carbono presentes en la fórmula condensada de los nueve compuestos
considerados en este problema, el total es 28.
332
Notas adicionales: El grupo funcional de los ácidos carboxílicos es el –COOH
El grupo funcional de los alcoholes es el –CH2OH
El grupo cetona es R-CO–R’
Un compuesto aromático típico es el benceno

QUÍMICA ORGÁNICA
SÓLO NIVEL A.

Problema 12. Marca con una √ la respuesta correcta.

¿Qué dieno y qué dienófilo deben de ser usados para obtener el biciclo[2.2.1]hept-5-eno-2,3-
dicarbonitrilo?


Dieno + Dienófilo
CN

CN
Biciclo[2.2.1]hept-5-eno-2,3-dicarbonitrilo

A) B)

+ CN
 CN
CN
CN

C) D)
CN
CH2 + NC CN
+
CH2
CN

333
Problema No. 13. Marca con una √ la respuesta correcta.

La reacción del fenilacetileno con amiduro de potasio (KNH2) genera el compuesto I. El compuesto
I al reaccionar con yoduro de etilo genera el compuesto II. Identifique a los compuestos I y II.

A)
C K

I II

B)

NH K
NH

I II

C)

CH2 K

I II

D)
C K

I II

334
Problema No. 14.

Escriba el producto principal de las siguientes reacciones:

NO2

2 KI
1) 4 Fe, 12 HCl

2) 2 NaNO2,
4 HCl
NO2

Problema No. 15.

Escriba el producto principal de las siguientes reacciones:

OCH3 1) HNO3
2) H2, Pd / C

3) CH3

O ,

CH3
H2, Pd / C

FIN DEL EXAMEN

335
336
XXI OLIMPIADA NACIONAL DE QUÍMICA
EXAMEN INTERNACIONAL
9 PROBLEMAS Tiempo: 4 HORAS Guadalajara 2012

FISICOQUÍMICA

1. Producción de Hidrógeno Industrial.

El hidrógeno es un elemento utilizado en el refinado de combustibles fósiles (por ejemplo, el


hidrocraqueo) y la producción de amoniaco (usado principalmente para fertilizantes). Actualmente,
uno de los procesos industriales para su obtención utiliza las siguientes etapas:

(Etapa 1) Reacción endotérmica catalizada entre el metano (gas natural) con vapor de agua a alta
temperatura (steam reforming) produciendo una mezcla de monóxido de carbono e hidrógeno,
llamada “syngas” o “gas de síntesis”. La mezcla gaseosa así obtenida es difícil de separar ya sea
física o químicamente (es necesario enfriar hasta –205 ºC antes de que el monóxido pueda
condensar).

CH4 (g) + H2O (g) (Ni, 800 ºC)  CO (g) + 3H2 (g) ΔH° = +206 kJ mol–1 (1)

(Etapa 2) Para aumentar el rendimiento y facilitar también la separación posterior, el monóxido de


carbono se oxida a dióxido de carbono. Para ello, la mezcla es enfriada y se le inyecta vapor de agua
de nuevo, esta vez sobre un catalizador de óxido de hierro (III), Fe2O3.

CO (g) + H2O (g) (Fe2O3, 400 ºC)  CO2 (g) + H2 (g) ΔH° = –41.2 kJ mol–1 (2)

(Marca con una X la respuesta correcta)

1.1. La reacción química global para este proceso es:

A) CH4 (g) + 2 H2O (g)  CO2 (g) + CO (g) + 3 H2 (g)

B) CH4 (g) + 4 H2O (g)  CO2 (g) + 8 H2 (g)

C) CH4 (g) + H2O (g)  CO2 (g) + 3 H2 (g)

D) CH4 (g) + 2 H2O (g)  CO2 (g) + 4H2 (g)

337
1.2. La entalpia para esta reacción, ΔH°, en kJ/mol es:
Cálculos:

A) +164.8 kJ/mol B) –164.8 kJ/mol C) 247.2 kJ/mol D) –3 247.2 kJ/mol

Como no todo el metano se convierte, se lleva a cabo una etapa adicional a 1000 °C, en presencia de
aire para convertir en hidrógeno el metano restante:
CH4 (g) + ½ O2 (g)  CO (g) + 2 H2 (g) ΔH° = +35.7 kJ mol–1 (3)

1.3. Para favorecer la reacción (1), es necesario:

A) Aumentar la presión y aumentar la temperatura

B) Aumentar la presión y disminuir la temperatura

C) Disminuir la presión y aumentar la temperatura

D) Disminuir la presión y disminuir la temperatura

1.4. Para favorecer la reacción (2), es necesario:

A) Aumentar la temperatura

B) Disminuir la temperatura

C) Disminuir la presión

D) Aumentar la presión

1.5. Para favorecer la reacción (3), es necesario:

A) Aumentar la presión y aumentar la temperatura

B) Aumentar la presión y disminuir la temperatura

C) Disminuir la presión y aumentar la temperatura

D) Disminuir la presión y disminuir la temperatura

338
Considera los datos de la tabla siguiente a 25 °C:

Compuesto H°/(kJ/mol) S°/(J/mol K)


CH4 (g) –74.85 186.2
H2O (g) –241.8 188.7
CO (g) –110.5 197.9
H2 (g) 0 130.6

Para la reacción: CH4 (g) + H2O (g)  CO (g) + 3 H2 (g)

1.6. El cambio de entropía de la reacción es:


Cálculos:

A) –177 J/mol K B) –46.4 J/mol K C) 200.4 J/mol K D) 214.8 J/mol K

1.7. El cambio de energía de Gibbs de la reacción es:


Cálculos:

A) –63840 kJ/mol B) –5164 kJ/mol C) 142.1 kJ/mol D) 200.8 kJ/mol

1.8. A 400 K la energía de Gibbs de la reacción es de 69.637 kJ/mol. La constante de equilibrio a


esta temperatura es: (indica las unidades de la constante si es necesario)
Cálculos:

A) 8.05 x 10–10 B) 3.93 x 10–6 C) 9.79 x 10–1 D) 1.24 x 109

339
2. La descomposición del N2O5 (g) para producir NO2 (g) y O2 (g) es una reacción de primer orden
con una constante de rapidez de 5.1 x 10-4 s-1 a una temperatura de 44 0C.

2.1. Si la concentración inicial de N2O5 era 0.25 mol L-1, ¿cuál es la concentración después de 3.2
minutos?
Cálculos:

Concentración: _____________ mol L-1

2.2. ¿Cuánto tiempo, en minutos, ha transcurrido desde el principio si ahora la concentración del
reactivo es 0.15 mol L-1?
Cálculos:

Tiempo: ____________ minutos

2.3. ¿Cuánto tiempo, en minutos, se requiere para que reaccione el 62% del reactivo inicial?
Cálculos:

Tiempo: ____________ minutos

340
QUÍMICA ANALÍTICA

3. La amperometría es una técnica en la cual se mide la intensidad de la corriente en función de la


concentración en disolución de una especie química que se oxide o reduzca directamente en un
electrodo, denominado indicador, al cual se le impone un potencial fijo. Esta propiedad nos permite
seguir la variación de la concentración de una especie en el curso de una reacción química y detectar
el punto final de la misma por un cambio de las pendientes de las rectas involucradas.

Por ejemplo, a un potencial constante el Zn2+ puede ser valorado por adición del ion
hexacianoferrato (III), Fe(CN)63-, realizando las medidas de intensidad de corriente en un potencial
en el cual este ion pueda ser reducido a Fe(CN)64-. En la primera parte de la valoración se forma un
precipitado de Zn3(Fe(CN)6)2 por lo que la reducción de este ion únicamente se observa cuando todo
el Zn2+ ha precipitado. En este caso se observa una curva similar a la que se muestra a continuación
y el volumen que corresponde al punto final de la valoración se encuentra por el punto de
intersección de ambas rectas.

Volumen de Fe(CN)63-

A) Escribe la ecuación balanceada de la reacción que ocurre entre el Zn2+ y el Fe(CN)63-

B) Indica, con un diagrama I vs V, cómo sería la curva de titulación que se obtendría en el caso de
que fuera el Zn2+ el que se añadiera como reactivo titulante del ion hexacianoferrato (III).

341
C) Calcula la concentración de Zn2+ si el punto final de la valoración de 10,0 mL de una disolución
de este ion se valoraran con 20 mL de una disolución de hexacianoferrato (III) de concentración
0,04 mol/L.

D) Calcula la cantidad de electricidad (coulombios) que circularon por la celda electroquímica para
titular los 10,0 mL de la disolución de Zn2+.

E) Si el zinc contenido en los 10,0 mL de la disolución de Zn2+ tuvieran que ser reducidos a su
forma elemental, ¿qué cantidad de electricidad circularía por la celda?

Considera: 1 F = 96500 C/mol

342
QUÍMICA ORGÁNICA

4. Complete el siguiente esquema, incluyendo la estructura de los productos señalados:

O O O
+

EtONa / EtOH

O 1)

NH2NH2 2)

 / HO

A (C8H14)
H2, 1 eq.

Pd-C; 1 atm
(Ph)3P=CH2

H2 / PtO2

B (C8H14)

H2 / PtO2

1) MeLi
2) H
+ 3) 
C (C8H14)
H2 / PtO2

H 1) O3
2) SMe2
O

343
5. Escriba el producto principal de las siguientes reacciones:

1) SOCl2
2) AlCl3

3) Zn, H
CO2H

6. Escriba el producto principal de la siguiente reacción:

HC O

H OH HNO3

H OH

OH

7. Escriba el producto principal de la siguiente reacción:

HC O

H OH Br2

H OH H2O

OH

8. Escriba el producto principal de la siguiente reacción:

HC O
3 NH NH2
H OH

H OH
H
OH

344
QUÍMICA INORGÁNICA

9. En el siguiente diagrama se muestran las acciones realizadas para identificar a los cationes
metálicos presentes en la fracción soluble de un mineral que se atacó con ácido clorhídrico.

Fracción del mineral soluble en HCl


(3 elementos del bloque d y uno del bloque s)

En caliente:
NH4Cl + NH4OH

Precipitado café rojizo Filtrado incoloro

Exceso de NaOH (NH4)2S

NO se redisuelve Filtrado incoloro Precipitado rosa


Se concentra y
HCl se añade (NH4)2CO3 HCl

Sí se redisuelve Precipitado blanco Se redisuelve


Se pone a ebullición
(Se elimina el H2S)
Prueba 1 Prueba 2
Hexaciano Tiocianato NaOH en exceso
ferrato (II) de amonio Coloración
de potasio naranja a Filtrado incoloro Precipitado blanco
la flama que al aire se
oscurece.
H2S Hirviéndolo
Rojo sangre con HNO3 y PbO2
Azul de Prusia + Y 4- la disolución toma
Se pierde la coloración Precipitado blanco un color púrpura
intensa

Catión 1 Catión 2 Catión 3 Catión 4

+ NaOH conc.
H4Y = Ácido etilendiamintetraacético Na2SO3
Intermediario inestable:
disolución color verde

Precipitado café oscuro

345
A la disolución de coloración purpura intensa de la última prueba (A), se le añade NaOH
concentrada y un exceso de yoduro de sodio. La disolución toma un color verde obscuro (B) que
con calentamiento se decolorar con la aparición de un precipitado café oscuro (C).

El precipitado café oscuro se trata con ácido clorhídrico obteniéndose una disolución incolora (D)
y un gas irritante con propiedades decolorantes (E).

PREGUNTAS:

9.1. Mencionar tres aniones que pudieran haber estado presentes en el mineral original.

9.2a. Escribir la configuración electrónica del catión 1.

9.2b. Dar la fórmula del precipitado café rojizo.

9.2c. Escribir la ecuación balanceada de la reacción de decoloración de la disolución rojo sangre.

9.2d. El azul de Prusia es una sal, dar la fórmula completa.

9.3a. ¿A qué familia pertenece el catión 2?

9.3b. Este catión se identifica formando una sustancia muy abundante en la Tierra, tanto en el reino
mineral como en el animal. ¿Cuál es la fórmula de este compuesto?

9.3c. Mencionar tres ejemplos de materiales naturales donde se encuentra el compuesto mencionado
en 3b.

9.4a. ¿Cuál(es) es(son) el(los) estado(s) de oxidación más común(es) del catión 3?

346
9.4b. Escribir la(s) configuración(es) electrónica(s) correspondiente(s).

9.4c. Escribir la ecuación balanceada de la reacción de este catión con sosa diluida.

9.4d. Escribir la ecuación de la reacción de este catión con sosa concentrada.

9.4e. Escribir la ecuación balanceada de la reacción de este catión con ácido sulfhídrico.

9.5. ¿Cuál es el catión 4?

9.6. Dos de los cuatro cationes son paramagnéticos, y dos son diamagnéticos, ¿cuáles son cuáles y
por qué?
Paramagnéticos Diamagnéticos

9.7. Decir cuál es la especie que forma el catión 4, permitiendo su identificación en la prueba final.

9.8. Escribir la ecuación balanceada de la reacción de formación de la disolución verde.

9.9. Escribir la ecuación balanceada de la reacción de formación del precipitado café oscuro (que
contiene 63.2% en masa del elemento metálico), a partir de la disolución verde.

9.10. Escribir la ecuación balanceada la reacción entre el precipitado café oscuro y el ácido
clorhídrico.

FIN DEL EXAMEN

347
348
XXI Olimpiada Nacional de Química
Examen Experimental de Química Orgánica Guadalajara 2012

REACCIONES PARA IDENTIFICAR ALCOHOLES Y FENOLES

OBJETIVOS.
Que el alumno:
a) Efectúe en el laboratorio pruebas características para identificar alcoholes y fenoles.
b) Realice una serie de pruebas que le permitan diferenciar entre alcoholes primarios, secundarios y
terciarios.
c) Conozca las pruebas químicas que permiten diferenciar un alcohol alifático de un fenol.

ANTECEDENTES.
En este experimento tú vas a tener que identificar, con base a una serie de reacciones a la gota, qué
tipo de alcohol se te asignó, o bien un fenol, que puede ser cualquiera de los siguientes compuestos
y que se encuentra dentro del frasco gotero marcado como Muestra problema:

OH
OH OH
OH
OH

butanol 2-butanol 2-metil-2-propanol fenilmetanol fenol

alcohol n-butílico alcohol sec-butílico alcohol ter-butílico alcohol bencílico

REACCIONES.
Para identificar a los fenoles, se llevan a cabo reacciones que permitan diferenciarlos de los
alcoholes, y en este caso concreto la presencia del anillo aromático es lo que permite diferenciarlos.
La reacción con agua de bromo y un fenol, decolora a la solución y se forma un precipitado. La
reacción con cloruro férrico, forma complejos coloridos.

349
Si las reacciones anteriores dan resultados negativos, se procede a identificar a los diferentes tipos
de alcoholes por dos reacciones. La primera de ellas es la reacción con el reactivo de Lucas, la cual
se debe de interpretar correctamente, ya que se observa la formación de turbidez en la mezcla de
reacción por la formación del cloruro de alquilo correspondiente, a diferentes tiempos de reacción,
lo cual está relacionado con la cinética de la reacción y el mecanismo de la reacción de sustitución
bajo el que se lleva a cabo esta transformación. Los alcoholes terciarios reaccionan bajo un
mecanismo SN1, en el cual se forma un carbocatión terciario, muy estable, y la reacción es
instantánea. Los alcoholes secundarios reaccionan, dependiendo de su estructura tanto por un
mecanismo SN1 (si hay dentro de su estructura un grupo funcional que estabilice al carbocatión)
como por un mecanismo SN2, con tiempos de reacción entre 2 y 5 minutos. Los alcoholes primarios
prácticamente bajo las condiciones en las que se lleva a cabo el experimento no reaccionan, siempre
y cuando no tengan dentro de su estructura algún grupo funcional que llegue a estabilizar al
carbocatión.
H
OH ZnCl2, HCl
NO HAY REACCIÓN
R
H

H H
OH ZnCl2, HCl Cl
TIEMPO: 2 A 5 MINUTOS
R R
R' R'

R
OH R
ZnCl2, HCl Cl
R' REACCIÓN INSTANTÁNEA
R'' R'
R''

La última reacción consiste en llevar a cabo reacciones de oxidación con el reactivo de Jones. En
esta reacción se da un cambio en el estado de oxidación en el cual hay una reacción de reducción del
ácido crómico (Cr(VI)) al sulfato de cromo (Cr(III)), y en el alcohol hay una reacción de oxidación,
siempre y cuando el alcohol sea oxidable:
H
OH CrO3 + H2SO4 O O
CrO3 + H2SO4
R R + Cr2(SO4)3
R
H
H OH
ALDEHÍDO ÁCIDO
CARBOXÍLICO
H
OH O
CrO3 + H2SO4
R + Cr2(SO4)3
R
R'
CETONA R'

R
OH
CrO3 + H2SO4
R' NO HAY REACCIÓN
R''

350
MATERIAL
Tubos de ensayo con tapón 4 Pinzas para tubo de ensayo 1
Vaso de precipitados de 150 mL 1 Agitador de vidrio 1
Gradilla para tubos de ensayo 1

REACTIVOS EN LOS FRASCOS GOTEROS


Cloruro férrico
Reactivo de Lucas
Reactivo de Jones
Agua de bromo
Muestra problema

PROCEDIMIENTO.
REACCIONES DE IDENTIFICACIÓN DE FENOLES.
1) PRUEBA CON AGUA DE BROMO. A 10 gotas de la solución de la Muestra problema.
Agregue gota a gota y con agitación una solución de agua de bromo hasta que permanezca la
coloración. Se considera positiva la reacción cuando desaparece la coloración y aparece un
precipitado.

DISPOSICIÓN DE LOS DESECHOS GENERADOS. En cada prueba, se adiciona bisulfito de


sodio al residuo que queda al realizar la prueba y la solución residual se coloca en los FRASCOS
DE DESECHOS ÁCIDOS, que se encuentran en las campanas del laboratorio.

2) PRUEBA CON CLORURO FÉRRICO. A un tubo de ensayo que contenga 10 gotas de la


Muestra problema. Adicione gota a gota una solución de cloruro férrico al 3%. Se considera positiva
la reacción si aparece un color intenso en la mezcla de reacción.

DISPOSICIÓN DE LOS DESECHOS GENERADOS.


Al finalizar la prueba, se coloca el residuo de esta prueba en los FRASCOS DE DESECHOS
ÁCIDOS, que se encuentran en las campanas del laboratorio.

REACCIONES DE IDENTIFICACIÓN DE ALCOHOLES.


3) PRUEBA CON SOLUCIÓN DE ÁCIDO CRÓMICO (REACTIVO DE JONES). Coloque en
un tubo de ensayo unas 10 gotas de la Muestra problema. Agregue gota a gota el reactivo de Jones y
agite el tubo hasta mezclar las dos soluciones. Se considera positiva la prueba si desaparece el color
naranja y la mezcla de reacción se torna azul-verdosa.

DISPOSICIÓN DE LOS DESECHOS GENERADOS. Al finalizar la prueba, se coloca el residuo


generado en el FRASCO DE DESECHOS DE CROMO, que se encuentra en las campanas del
laboratorio.

4) PRUEBA DE LUCAS. Coloque 15 gotas del reactivo de Lucas en un tubo de ensayo y agregue
10 gotas de la Muestra problema. Coloque un tapón de corcho en el tubo y agite durante 5 minutos.
Deje reposar a temperatura ambiente y tome el tiempo que tarda en aparecer una turbidez, lo cual
indica que la prueba es positiva. Si a los 15 minutos no hay cambio, la prueba se considera negativa.

351
DISPOSICIÓN DE LOS DESECHOS GENERADOS.
Al finalizar la prueba, se coloca el residuo de esta prueba en los FRASCOS DE DESECHOS
ÁCIDOS, que se encuentran en las campanas del laboratorio.

CLAVE: __________

PREGUNTAS
1. Llena la siguiente tabla en la que se resumen los resultados de las pruebas a la gota que tú
realizaste en este experimento.

RESULTADOS OBTENIDOS DURANTE LA IDENTIFICACIÓN DE


ALCOHOLES Y FENOLES

Pruebas Observaciones Resultados (+ o -)


(Cambios de color,
aparición de
precipitado, formación
de suspensión)
1) Prueba con agua de bromo
2) Prueba con cloruro férrico
3) Prueba con ácido crómico
4) Prueba de Lucas
Valor: 40 puntos

2. Con base en los resultados que observaste, y que se encuentran resumidos en la tabla de la
pregunta 1, ¿cuál es la estructura de tu muestra problema?

352
3. Si los alcoholes terciarios reaccionan con mayor rapidez que los alcoholes primarios, a que
conclusión puedes llegar en cuanto a los mecanismos SN1 y SN2, ¿cuáles ocurren con mayor
facilidad? Con base a lo anterior, escoge una de las siguientes opciones:

a) Las reacciones SN1 son más rápidas ya que se forma un carbocatión y conforme este sea más
estable se forma más rápido. Por el otro lado, las reacciones SN2 son más lentas ya que el
estado de transición está muy congestionado (carbono con hibridación sp2 pentavalente).

b) Las reacciones SN1 son más lentas ya que se forma un carbocatión y conforme este sea más
estable más lentamente se forma. Por el otro lado, las reacciones SN2 son más rápidas ya que
el estado de transición está muy congestionado (carbono pentavalente).

c) Tanto las reacciones SN1 como las reacciones SN2 ocurren con la misma rapidez

RESPUESTA

4. ¿Cómo esperarías que reaccionara el alcohol bencílico bajo las condiciones de la reacción de
Lucas?

a) Puede ser por cualquiera de las dos reacciones, tanto SN1 como por una SN2, ya que estas
ocurren con la misma rapidez.

b) A través de una reacción SN1, las cuales son rápidas ya que se forma un carbocatión
bencílico el cual es muy estable y se forma con rapidez.

c) A través de una reacción SN2, ya que se forma con rapidez el intermediario con el carbono
con hibridación sp2, pentavalente).

RESPUESTA

353
BIBLIOGRAFÍA.
1) R. L. Shriner; R. C. Fuson. "The Systematic Identification of Organic Compounds", John Wiley,
New York, 1935, 1940, 1948.

2) Howard J. Lucas. "A New Test for Distinguishing the Primary, Secondary and Tertiary Saturated
Alcohols", J. Am. Chem. Soc. 1930, 52, 802–804.

3) Richard A. Kjonaas; Bernie A. Riedford. "A Study of the Lucas Test", J. Chem. Educ. 1991, 68,
704.

4) I. Heilbron; E. R. H. Jones; F. Sondheimer. "315. Researches on Acetylenic Compounds. Part


XIV. A Study of the Reactions of the Readily Available Ethynyl-ethylenic Alcohol, Pent-2-en-4-
yn-1-ol", J. Chem. Soc. 1947, 1586.

5) I. Heilbron; E. R. H. Jones. "129. Researches on Acetylenic Compounds. Part XV. The Oxidation
of Primary Acetylenic Carbinols and Glycols", J. Chem. Soc. 1949, 604.

354
XXI Olimpiada Nacional de Química
Examen Experimental de Fisicoquímica Guadalajara 2012

NIVEL A.

Material que necesitas para este experimento:

Una pila cuadrada de 9.0 volts


Un vaso de precipitados de100 mL
5.0 g de sulfato de sodio (Na2SO4)
20 cm de masking tape
Dos cables con caimanes en cada extremo (de unos 15 a 20 cm de largo)
Dos clavos de hierro grandes con cabeza (7 a 10 cm de largo)
Cuchara pequeña de plástico desechable
Un cono de papel

Procedimiento: (lee todo el problema antes de iniciar) Para medir los potenciales usarás un
voltímetro que compartirás con otros estudiantes y que un asesor te enseñará a usar.

Con el masking tape fija los dos clavos en las paredes internas del vaso de precipitados, lo más lejos
posible uno del otro, de tal manera que sobresalgan las cabezas de los clavos un centímetro del
borde del vaso:

Masking tape para fijar los clavos

Usa el masking tape para fijar la pila de 9.0 V a la parte externa del vaso de precipitados.

Con el voltímetro mide el potencial de tu pila y anótalo aquí: _________ mV.


Este valor será tu Potencial “D-1”.

Utiliza los cables con caimán para conectar la pila a los clavos, un polo a cada clavo.

Con el voltímetro, mide de nuevo el potencial de tu pila y anótalo aquí: __________ mV.
Este valor será tu Potencial “D-2”.

Observa los clavos; ¿consideras que está ocurriendo alguna reacción química?

_____ Si ______ No

355
Justifica tu respuesta:

En caso afirmativo, la o las reacciones que están ocurriendo son:

____________________________________________

____________________________________________

____________________________________________

Sin desconectar los caimanes que conectan a los polos de la pila con los clavos, agrega agua al vaso,
hasta la marca del máximo volumen; los clavos deben quedar parcialmente sumergidos en el agua.

Anota el siguiente dato:


Volumen de agua contenida en el vaso de precipitados: __________ mL.

Considera que la densidad del agua es de 1 g/cm3; indica los gramos de agua que has agregado:
_________ g de H2O.

Con el voltímetro, mide de nuevo el potencial de tu pila y anótalo aquí: __________ mV.
Este valor será tu Potencial “C-1”.

Observa los clavos; ¿consideras que está ocurriendo alguna reacción química?

_____ Si ______ No

Justifica tu respuesta:

En caso afirmativo, la o las reacciones que están ocurriendo son:

____________________________________________

____________________________________________

____________________________________________

Sin desconectar los cables, agrega un tercio de cucharada de sulfato de sodio y agita para disolver,
cuidando de no mover los clavos de su posición. Supón que la cantidad de sulfato de sodio que
agregaste es de 1.5 g.

356
Con el voltímetro, mide de nuevo el potencial de tu pila y anótalo aquí: __________ mV.
Este valor será tu Potencial “C-2”.

Observa los clavos; ¿consideras que está ocurriendo alguna reacción química?

_____ Si ______ No

Justifica tu respuesta:

En caso afirmativo, la o las reacciones que están ocurriendo son:

____________________________________________

____________________________________________

____________________________________________

Manteniendo todo igual, agrega otro tercio de cucharada de sulfato de sodio (supón que son 1.5 g) y
agita para disolver.

Con el voltímetro, mide de nuevo el potencial de tu pila y anótalo aquí: __________ mV.
Este valor será tu Potencial “C-3”.

Finalmente, agrega otro tercio de cucharada de sulfato de sodio (supón que son 1.5 g) y agita para
disolver.

Con el voltímetro, mide de nuevo el potencial de tu pila y anótalo aquí: __________ mV.
Este valor será tu Potencial “C-4”.

Con los datos obtenidos, llena la siguiente tabla donde indiques los potenciales de la pila medidos
para cada concentración de Na2SO4 en % en masa.

Potencial Potencial medido en mV Concentración en % en masa de Na2SO4


D-1

D-2

C-1

C-2

C-3

C-4

357
Para la disolución final, la concentración molar de tu disolución es: ______________ mol L-1
(considera que el volumen del sólido es despreciable).

Haz la resta: Potencial “D-2” - Potencial “C-4” = Potencial “E”

_________mV _________mV _________mV

El valor obtenido es tu Potencial “E”, (indica claramente si el valor obtenido es positivo o negativo).

Considera que la resistencia total de la celda es de 100 ohms, y calcula la corriente que está
circulando con la ley de Ohm:

Intensidad de corriente (mA) = Potencial “E” (mV) / 100 ohms = _______ mA

Si en el cátodo se está obteniendo hidrógeno gaseoso, tomando en cuenta la intensidad de corriente


que has determinado, calcula:

¿Cuánto tiempo tendría que funcionar esta celda, suponiendo que la corriente se mantiene constante,
para obtener un mol de hidrógeno gaseoso?
Cálculos:

Tiempo: _________ minutos

Constante de Faraday = 96,500.00 coulombs/mol de electrones; 1 ampere = 1 coulomb/segundo

358
XXI Olimpiada Nacional de Química
Examen Experimental de Química Analítica Guadalajara 2012

NIVEL A.

INSTRUCCIONES. ¡Atención!
En todo momento y mientras estés en el laboratorio debes usar los lentes de seguridad o tus propios
lentes graduados. Además, para pipetear debes usar la jeringa o pera de succión. Si no cumples con
esto, recibirás una llamada de atención.
Si recibes una segunda llamada de atención, se considerará que no eres apto para continuar, tendrás
que salir del laboratorio y obtendrás un cero en todo el examen experimental.
Si tienes alguna pregunta o duda respecto a las reglas de seguridad, NO DUDES EN CONSULTAR
a los profesores supervisores.
Este examen consta de dos partes que podrás realizar en el orden que quieras pero considera que
tienes cuatro horas para realizar ambas.
 Por favor lee cuidadosamente el texto de cada problema experimental antes de iniciar tu trabajo
experimental.
 Escribe tu nombre y TU CÓDIGO DE IDENTIFICACIÓN PERSONAL (que está en tu lugar
de trabajo), en la esquina superior de todas las hojas de respuestas de cada problema.
 No comiences tu trabajo hasta que escuches la orden “INICIAR el examen experimental”.
 Cuando en cada parte escuches la orden de “TERMINAR” debes suspender tu actividad
INMEDIATAMENTE. Un retraso de 3 minutos en cumplir esta orden puede ocasionar la
cancelación del examen y obtendrás una calificación de cero puntos.
 Todos los resultados debes escribirlos en las áreas correspondientes de las hojas de respuesta.
Todos los datos que escribas en otra parte no serán tomados en cuenta. NO ESCRIBAS NADA
POR LA PARTE TRASERA DE LAS HOJAS DE RESPUESTA. Si requieres más papel para
trabajar o para sustituir una hoja de respuestas, pídelo a los profesores.

PROBLEMA. Determinación de la pureza del ácido acetilsalicílico.

El ácido acetilsalicílico o AAS (C9H8O4), también conocido con el nombre de Aspirina®, es un


fármaco usado frecuentemente como antiinflamatorio, analgésico, para el alivio del dolor leve y
moderado, antipirético para reducir la fiebre y antiagregante plaquetario. El laboratorio de Química
Analítica cuenta con una buena cantidad de dicho ácido (el cual fue sintetizado en el laboratorio de
Tecnología Farmacéutica por alumnos de la Universidad) cuya pureza se desconoce.

Tu labor esta mañana será ayudarnos a determinar dicha pureza mediante una titulación por
retroceso realizada después de haber hidrolizado en caliente el ácido acetilsalicílico con un volumen
de NaOH de concentración conocida. El exceso de NaOH se titula con una disolución patrón de
HCl. Las concentraciones exactas de las disoluciones de NaOH y HCl se encuentran en la etiqueta
correspondiente del frasco que los contiene.

Se te proporcionan tres matraces erlenmeyer que contienen una muestra exactamente pesada de uno
de los lotes de ácido acetilsalicílico (la masa exacta se indica en la etiqueta de cada matraz, 91-112
mg).

359
Cuentas con dos indicadores de pH para detectar el punto final:
Fenolftaleína, intervalo de vire: 8 a 9,2
Verde de bromocresol, intervalo de vire: 3,8 a 5,4, amarillo en forma ácida y azul en forma básica.
pKa del ácido salicílico: 3,0.

PROCEDIMIENTO.
Con cada uno de los matraces erlenmeyer que contienen el ácido acetilsalicílico sólido deberás
seguir los siguientes pasos:

 Añade un volumen aproximado de 20 mL de agua destilada.

 Toma una alícuota de 10,00 ml de la disolución de NaOH y adiciónala al matraz erlenmeyer que
contiene la muestra y el agua destilada.

 Calienta sin permitir que la disolución hierva (cuando aparezcan las primeras burbujas retira el
mechero y deja enfriar un poco antes de volver a calentar). Continúa el calentamiento durante
un total de 15 minutos. No permitas que se evapore toda el agua; en cado necesario añade un
poco más de agua destilada.

 Deja enfriar y titula el exceso de NaOH con la disolución de HCl utilizando el indicador del
punto final que consideres más adecuado.

360
No. de Código:

HOJA DE RESPUESTAS.

Indica la reacción balanceada que ocurre al hidrolizar el ácido acetilsalicílico con la sosa:

De acuerdo a lo anterior, ¿qué indicador es el más apropiado para la titulación?

Completa la tabla siguiente:


Titulación No. (masa utilizada) 1( ) 2( ) 3( )
Lectura inicial de la bureta con
disolución de HCl
Lectura final de la bureta con
disolución de HCl
Volumen de disolución de HCl
gastado

Volumen de disolución de HCl a utilizar en los cálculos: __________________________________

Indica las moles de NaOH en exceso en cada uno de los matraces:


Matraz 1 2 3
Moles de NaOH en
exceso

Indica cuál es la pureza del ácido acetilsalicílico de la muestra que se te proporcionó. No olvides
indicar los cálculos realizados:
Cálculos:

Pureza del ácido acetilsalicílico: ________________ %

361
362
XXI Olimpiada Nacional de Química
Examen Experimental de Química Analítica Guadalajara 2012

NIVEL B.

INSTRUCCIONES. ¡Atención!
En todo momento y mientras estés en el laboratorio debes usar los lentes de seguridad o tus propios
lentes graduados. Además, para pipetear debes usar la jeringa o pera de succión. Si no cumples con
esto, recibirás una llamada de atención.
Si recibes una segunda llamada de atención, se considerará que no eres apto para continuar, tendrás
que salir del laboratorio y obtendrás un cero en todo el examen experimental.
Si tienes alguna pregunta o duda respecto a las reglas de seguridad, NO DUDES EN CONSULTAR
a los profesores supervisores.

Este examen consta de dos partes que podrás realizar en el orden que quieras pero considera que
tienes cuatro horas para realizar ambas.

 Por favor lee cuidadosamente el texto de cada problema experimental antes de iniciar tu trabajo
experimental.
 Escribe tu nombre y TU CÓDIGO DE IDENTIFICACIÓN PERSONAL (que está en tu lugar
de trabajo), en la esquina superior de todas las hojas de respuestas de cada problema.
 No comiences tu trabajo hasta que escuches la orden “INICIAR el examen experimental”.
 Cuando en cada parte escuches la orden de “TERMINAR” debes suspender tu actividad
INMEDIATAMENTE. Un retraso de 3 minutos en cumplir esta orden puede ocasionar la
cancelación del examen y obtendrás una calificación de cero puntos.
 Todos los resultados debes escribirlos en las áreas correspondientes de las hojas de respuesta.
Todos los datos que escribas en otra parte no serán tomados en cuenta. NO ESCRIBAS NADA
POR LA PARTE TRASERA DE LAS HOJAS DE RESPUESTA. Si requieres más papel para
trabajar o para sustituir una hoja de respuestas, pídelo a los profesores.

PROBLEMA. Determinación de la pureza de una muestra de ácido sulfosalicílico.

El ácido sulfosalicílico es un ácido orgánico diprótico que se utiliza en análisis clínicos para la
determinación de proteínas. El laboratorio de Química Analítica de esta Universidad recibió un
donativo consistente en varios frascos de dicho ácido pero antes de utilizarlo se requiere conocer
cuál es su pureza. Tu labor esta mañana será ayudarnos a lograrlo.

Para ello te proporcionamos una disolución del ácido (que tiene una masa molar igual a 218 g/mol y
que simbolizaremos por H2A) que fue preparada a partir de una muestra pesada de (12,250 g; 13,502
g; 14,634 g; 16,716 g) de uno de los frascos la cual fue disuelta en un volumen de 1,000 L de agua
destilada.

Cuentas además con una disolución de NaOH, cuya concentración exacta se indica en el frasco, y
con verde de bromocresol como indicador para detectar el punto final de la valoración.

Dato: El ácido sulfosalicílico se disocia totalmente en disolución.

363
No. de Código:

PROCEDIMIENTO.

Toma una alícuota de 10,00 mL de la disolución problema y titúlala con la disolución de NaOH
hasta observar que el indicador vira a azul en una gota. Repite la operación por triplicado.

HOJA DE RESPUESTAS.

Indica la reacción balanceada que ocurre entre el ácido sulfosalicílico (representado con H2A y la
sosa):

Completa la tabla siguiente:


Titulación No. (masa utilizada) 1( ) 2( ) 3( )
Lectura inicial de la bureta con
disolución de NaOH
Lectura final de la bureta con
disolución de NaOH
Volumen de disolución de
NaOH gastado

Volumen de disolución de NaOH a utilizar en los cálculos: ________________________________

Indica cuál es la concentración de la disolución problema del ácido sulfosalicílico que se te


proporcionó:

Indica cuál es la pureza del ácido sulfosalicílico en el frasco del cual se te proporcionó una muestra:
Cálculos:

Pureza del ácido sulfosalicílico: ________________ %

364
XXII Olimpiada Nacional de Química
1er Examen Nivel A y B.
Total: 35 preguntas. Tiempo asignado: 90 minutos. Chihuahua 2013

En Chihuahua son muy famosas las Barrancas del Cobre y además, este estado tiene minas, como la
famosa mina de Naica donde se extrae plomo, zinc y plata y hay cristales de gran tamaño de sulfato
de calcio. En esta primera parte del examen trataremos temas relacionados con la metalurgia y otros
campos donde la química ha tenido un gran desarrollo desde hace muchos años.
Debes anotar en el recuadro correspondiente la letra del inciso que contesta correctamente cada
pregunta. Muy importante: Si consideras que ningún inciso es el correcto deberás anotar una
letra X. Recuerda que siempre es bueno leer con atención cada pregunta.
NOTA ACLARATORIA: Al hacer tus cálculos puedes encontrar una pequeña diferencia, en
general no mayor de unas décimas, entre tu resultado y alguno de los propuestos. Esto es
normal y por eso sólo deberás usar la “X” cuando ninguna respuesta sea muy cercana a la
obtenida por ti.
********************************************************************************
Se analizó una muestra de 84.0 g extraída de una mina y se detectaron las siguientes cantidades de
elementos y compuestos:

Carbonato de calcio 31.0 g Sulfato ferroso 5.0 g


Sulfuro de plata 250 mg Sulfato cúprico pentahidratado 2.5 g
Sulfato de manganeso 3.10 g Sulfato de zinc monohidratado 4.1 g
Oro 1.0 mg Sulfato de calcio dihidratado 38.0 g

1) El sulfato de calcio dihidratado es un compuesto cuya masa molar es 172.17 g mol-1.


¿Cuántos moles de este compuesto hay en esta muestra?

A) 4.52 x 10-3 B) 2.20 x 10-3 C) 4.52 x 10-1 D) 2.20 x 10-1

2) Un cristal de sulfato de calcio dihidratado con un volumen de 14 cm3 tiene una masa
de 16.94 g. ¿Cuál es la densidad de este compuesto en g/cm3?

A) 1.21 B) 1.21 x 10-3 C) 0.826 D) 8.26

3) Si calentamos el sulfato de calcio dihidratado que hay en la muestra y extraemos el


agua contenida en el cristal, ¿cuántos litros de agua obtendremos considerando que la
densidad del agua es de 1.0 g/cm3?

A) 7.9 B) 17.21 C) 7.9 x 10-3 D) 1.72 x 10-3

4) Después de eliminar el agua contenida en el sulfato cúprico pentahidratado


(CuSO4•5H2O) presente en la muestra, ¿cuántos gramos de sulfato cúprico anhidro
podemos obtener?

A) 1.6 g B) 1.6 mg C) 0.9 g D) 0.53 g

365
5) Las ppm (partes por millón) son unidades de concentración, muy utilizadas en
estudios de contaminación de aguas o concentración de elementos en mezclas. En
gramos, una ppm equivale a un gramo en un millón de gramos. ¿Cuál es la
concentración de oro en la muestra, expresada en ppm?

A) 119 B) 11.9 C) 8.4 D) 0.084

6) En una mezcla de la misma cantidad en gramos de FeSO4 y de Fe2(SO4)3, la fracción


mol del sulfato ferroso o sulfato de hierro (II) es:

A) 0.66 B) 0.28 C) 0.72 D) 0.5

7) El sulfato de zinc se puede obtener haciendo reaccionar zinc metálico con ácido
sulfúrico. Otro de los productos de esta reacción es:

A) Oxígeno B) Agua C) Azufre D) Hidrógeno

8) Si se hacen reaccionar 6.5 g de zinc metálico con 500 mL de una solución acuosa de
ácido sulfúrico 0.1 M, ¿cuántos moles se obtienen de sulfato de zinc?

A) 0.65 moles B) 0.05 moles C) 0.1 moles D) 0.01 moles

9) En forma comercial, el sulfato de zinc se vende en forma de heptahidrato (cada


molécula del compuesto contiene siete moléculas de agua de hidratación) o bien de
monohidrato. En el monohidrato, el % en masa de zinc es aproximadamente:

A) 17 % B) 36 % C) 40.5 % D) 65 %

10) Uno de los minerales de plata más comunes en México es el sulfuro de plata, Ag2S.
¿Cuántos moles de este compuesto hay en la muestra?

A) 1 B) 0.1 C) 0.2 D) 2

11) En un proceso inventado en México en 1555 se hacía reaccionar Ag2S con cloruro
de cobre (I) (CuCl) para obtener cloruro de plata y sulfuro de cobre. ¿Cuántos gramos
de cloruro de cobre (I) se requieren para reaccionar con el sulfuro de plata que hay en
la muestra?

A) 1 B) 0.001 C) 0.002 D) 0.2

12) La configuración electrónica del azufre es:

A) 1s2 2s2 2p6 3s2 3p4 B) 1s2 2s2 3s2 3p6 4s4
C) 1s2 2s2 2p8 3s2 3p2 D) 1s2 2s2 2p6 3s2 3p6 4s2 4p6 4d6

366
13) En estado sólido el azufre forma moléculas en forma de anillo con fórmula S8. Si
consideras la reacción de S8 con oxígeno gaseoso (O2) para formar SO3, ¿cuántos
moles de O2 se requieren para reaccionar con cada mol de S8?

A) 4 B) 8 C) 12 D) 24

14) La ley de los gases ideales es PV= nRT , donde: (R = 0.082 L atm / mol K);
(R = 8.314 J / mol K); (1 J = kg m2 s-2); (1 atm = 101.325 kPa); (1 Pa = 1 N m-2);
(1 N = 1 kg m s-2). A 25 oC y 100.0 Pa, un mol de un gas ideal ocupa un volumen
de:

A) Menos de 300 L B) Entre 300 L y 2000 L


C) Entre 2.0 x 103 y 2.0 x 104 L D) Más de 2.0 x 104 L

15) Un estudiante quema 32 g de azufre y obtiene la misma cantidad en moles de SO2 y


de SO3. ¿Cuántos gramos de O2 se requieren para esta reacción?

A) Menos de 35 B) Entre 35 y 45
C) Entre 45 y 55 D) Más de 55

16) Si en la reacción anterior se hubieran obtenido 16 g de SO2 gaseoso, ¿qué volumen


en litros ocuparía este gas a 25 oC y presión de 1 bar? (1 bar = 105 Pa)

A) Menos de 1.0 B) Entre 1.0 y 5.5


C) Entre 5.5 y 9.0 D) Más de 9.0

17) En la siguiente ecuación, las letras griegas ““, ““, ““ representan elementos
químicos: 4 3 → 2 2 + 4 2 + 2
El elemento  es de mucha importancia para la vida y en estado gaseoso su molécula
es diatómica. La masa molecular del compuesto 3 es 68.94 g/mol y este
compuesto contiene nitrógeno. El elemento  es:

A) H B) Li C) Be D) Na

18) Un matraz aforado de 100.0 mL se pesa vacío y la lectura es 25.83 g. Este matraz
se llena hasta el aforo con un líquido “X” y se pesa nuevamente. Ahora la lectura es
108.93 g. La densidad del líquido “X” es, en g/cm3:

A) 0.00831 B) 0.0831 C) 0.831 D) 8.31

19) Si se mezcla medio litro de solución acuosa 0.2 molar (mol L-1) de cloruro de sodio
(densidad = 1.012 g/cm3) con 250.0 mL de una solución 0.5 molar del mismo
compuesto (densidad = 1.03 g/cm3), la concentración molar del cloruro de sodio en la
solución obtenida es:

A) 0.30 M B) 0.35 M C) 0.40 M D) 0.225 M

367
20) En la solución que se obtiene en la pregunta anterior, el % en masa del cloruro de
sodio es:

A) 3.50 % B) 5.85 % C) 13.2 % D) 1.73 %

21) Al balancear la ecuación:


? FeCr2O4 + ? K2CO3 + ? O2 → 2 Fe2O3 + ? K2CrO4 + ? CO2
el coeficiente estequiométrico para el oxígeno es:

A) 12 B) 7 C) 6 D) 14

22) En la reacción de la pregunta anterior, es cierto que:

A) Se oxidan tres elementos y se reducen dos elementos


B) Se oxidan dos elementos y se reducen dos elementos
C) Se oxidan dos elementos y se reduce un elemento
D) Se oxida un elemento y se reduce un elemento

23) Una reacción que se utiliza en metalurgia es la de un mol del compuesto gaseoso
P4O10 con seis moles de óxido de calcio de tal manera que se obtiene un solo
compuesto, que es un fosfato. ¿Cuántos moles de este fosfato se obtienen?

A) 7 B) 3.5 C) 3 D) 2

24) El vinagre que usamos en nuestras ensaladas es una solución acuosa diluida de
ácido acético, un compuesto cuya fórmula es CH3COOH. La masa molar del ácido
acético es:

A) 61 g/mol B) 60 g/mol C) 58 g/mol D) 57 g/mol

25) Para neutralizar una muestra de 4.50 mL de un vinagre cuya densidad es 1.01 g/mL
se requiere agregar 38.0 mL de una solución 0.100 N de NaOH. ¿Cuál es el % en
masa del ácido acético en esta muestra de vinagre?

A) 5.01 % B) 0.501 % C) 3.81 % D) 0.381 %

26) El pH de una solución de NaOH 0.100 N es:

A) 0.1 B) 1 C) 8 D) 13

27) Para los compuestos NH4Cl y Na2CO3, su comportamiento ácido-básico en agua es,
respectivamente:

A) ácido, base B) base, ácido C) ácido, neutro D) base, neutro

368
28) Si en la molécula de benceno se sustituye un hidrógeno por un grupo OH se
produce el fenol, una sustancia cuya estructura es:

OH

Se pueden disolver hasta 83 g de fenol en un litro de agua. ¿Cuántos litros de agua se


necesitarían para disolver un mol de fenol?

A) 11.3 B) 0.0113 C) 0.1133 D) 1.13

29) En una fábrica se produce cobre metálico a partir de una solución que contiene
iones Cu2+, utilizando una reacción electroquímica. El enunciado correcto es:

A) El Cu metálico se obtiene en el ánodo por reducción de los iones Cu2+


B) El Cu metálico se obtiene en el ánodo por oxidación de los iones Cu2+
C) El Cu metálico se obtiene en el cátodo por reducción de los iones Cu2+
D) El Cu metálico se obtiene en el cátodo por oxidación de los iones Cu2+

30) Cuando un gramo de agua es adicionado a una mezcla de ácido y agua, la nueva
mezcla es 25% en masa de ácido. Cuando 2 gramos de ácido son adicionados a la
nueva mezcla, la mezcla ahora contiene 50% en masa de ácido. El % en masa de
ácido en la mezcla original era:

A) Menos de 25% B) Entre 25% y 30%


C) Entre 31% y 36% D) Más de 36%

31) En un recipiente de 22.4 L de capacidad, a una T de 298 K, se coloca la misma


cantidad en gramos de nitrógeno, oxígeno y CO gaseosos. La fracción mol de
nitrógeno en esta mezcla es:

A) Menor a 0.3 B) Entre 0.3 y 0.4


C) Entre 0.4 y 0.5 D) Mayor a 0.5

32) Si en un recipiente, de 10.0 L de capacidad y a una T de 273 K, se introdujeran un


mol de metano, un mol de etano y un mol de propano, la densidad de esta mezcla
gaseosa sería, en g/cm3:

A) 9 x 10-3 B) 8 x 10-3 C) 0.9 D) 0.8

33) En el más simple de los alquinos, el etino o acetileno, el % en masa de hidrógeno


es:

A) 50% B) 25% C) 14.3 % D) 7.7 %

369
34) La corriente que circula en una celda electroquímica se puede medir en amperes
(A). Un ampere equivale a una carga de un coulomb (C) que pasa cada segundo por
un punto del circuito. La carga de un electrón es 1.6 x 10-19 C. Si en una celda
electroquímica circula una corriente de 500 mA, significa que el número de
electrones que pasan por un punto del circuito cada hora son:

A) 3.125 x 1018 B) 1.125 x1022


C) 1.125 x 1018 D) 3.125 x 1022

35) De acuerdo a la expresión: “X” → 20682 Pb + 


Podemos concluir que el átomo radiactivo “X” es:
(Las partículas alfa son núcleos de helio)

A) 20280 Hg B) 21081 Tl C) 20284 Po D) 21084 Po

********************************************************************************
FIN DEL EXAMEN

370
XXII Olimpiada Nacional de Química. 2do Examen Nivel A y B.
Total: 3 problemas cada nivel.
Tiempo asignado: 90 minutos. Chihuahua 2013

1. El aire que respiramos es una mezcla de gases que constituyen a la atmósfera terrestre y es
esencial para la vida en el planeta. Su composición varía dependiendo de la localidad de que se
trate y regularmente se monitorea para establecer las contingencias ambientales.
Una composición aproximada del aire es 78% en volumen de nitrógeno, 21% de oxígeno y 1%
de una mezcla “X” de otros gases entre los que se encuentran principalmente el vapor de agua y
el dióxido de carbono. La masa molar aparente del aire que se puede utilizar en cálculos es de
28.82 g/mol. Considerando que sólo estos gases están presentes en una muestra de 2.5 litros de
aire proveniente de la Ciudad de Toluca donde la presión barométrica media es de 74.46 kPa y
que se comportan idealmente a 20 °C, contesta las preguntas siguientes:
(Utiliza los cuadros para tus cálculos y marca la respuesta correcta). Respuestas sin
Cálculos no son válidas.
R = 8.314 J/mol K = 8.314 Pa m3/mol K T = t + 273.15 1 atm = 101325 Pa

1.1. La presión parcial del nitrógeno en la muestra de aire es:

a) 16.38 kPa b) 32.76 kPa c) 49.14 kPa d) 58.08 kPa

1.2. La cantidad de sustancia, en moles, de oxígeno que hay en la muestra de aire a una temperatura
de 20 °C es:

a) 0.0160 mol b) 0.0172 mol c) 0.0596 mol d) 0.0639 mol

371
1.3. El volumen parcial del nitrógeno en la muestra de aire es:

a) 0.525 L b) 1.05 L c) 1.58 L d) 1.95 L

1.4. La masa molar aparente de la mezcla “X” es:

a) 26.0 g/mol b) 29.0 g/mol c) 30.0 g/mol d) 30.84 g/mol

1.5. La masa en gramos de nitrógeno en la muestra es:

a) 0.224 g b) 0.449 g c) 1.668 g d) 1.906 g

1.6. La densidad de la muestra a 20 °C es:

a) 427.7 g/m3 b) 488.8 g/m3 c) 880.5 g/m3 d) 977.63 g/m3

372
2. Una mezcla de gases que se comportan de acuerdo al modelo ideal tiene una masa total de 3.50 g
y ocupa un volumen de 7.5 L, a 27 oC y una presión de un bar. Los gases son nitrógeno e
hidrógeno.

2.1. Calcula el % en masa de estos gases en la mezcla:

La ley de los gases ideales es PV = nRT

donde:
(R = 0.082 L atm / mol K); (R = 8.314 J / mol K); (1 J = 1 kg m2 s-2);
(1 atm = 101.325 kPa); (1 Pa = 1 N m-2); (1 N = 1 kg m s-2); (1 bar = 105 Pa)

373
3. EN LA SIGUIENTE SECCIÓN MARCA CON UNA X EL INCISO QUE CONSIDERES
CORRECTO.

3.1. Al hablar de partículas elementales en reposo es cierto que:


a) La masa del protón es aproximadamente cien veces la del electrón.
b) La masa del protón es igual a la del electrón.
c) La masa del electrón es cero.
d) La masa del protón es ligeramente inferior a la del neutrón.

3.2. El cloro tiene dos isótopos naturales cuyas masas son 35 y 37 unidades. ¿Cuál será la
contribución de los isótopos si la masa atómica del cloro es igual a 35,5 umas?
a) Mayor proporción del 35Cl que de 37Cl.
b) Tendrán la misma proporción.
c) Mayor proporción del 37Cl que de 35Cl.
d) No se puede determinar con los datos aportados.

3.3. Si a partir de 1,3 g de un metal X se obtienen 1,9 g de X2O3, ¿cuál será la masa atómica de X?
a) 40 b) 52 c) 104 d) 64

3.4. Los átomos de un elemento X tienen en su núcleo 20 protones. Los estados de oxidación más
comunes de este elemento deben ser:
a) 0 y +2 b) 1 c) 0 y +1 d) 0, +1 y +2

3.5. Elije qué tres formas moleculares están constituidas exclusivamente por átomos de hidrógeno:
a) Hidrógeno, deuterio y ozono.
b) Hidrógeno, tritio y agua pesada.
c) Hidrógeno, tritio y deuterio.
d) Hidrógeno, hidronio y deuterio.

3.6. Al ir de izquierda a derecha en el tercer periodo de la tabla periódica, los óxidos y los cloruros
cambian sus propiedades de iónicas a covalentes. Este cambio se debe a que:
a) Aumenta el volumen atómico.
b) Desciende la primera energía de ionización.
c) Incrementa la electronegatividad.
d) Disminuye el número de electrones de valencia

3.7. Un compuesto de fósforo y azufre utilizado en las cabezas de cerillos contiene 56,29 % de P y
43,71 % de S. La masa molar correspondiente a la fórmula empírica de este compuesto es:
a) 188,1 g/mol b) 220,1 g/mol c) 93,94 g/mol d) 251,0 g/mol

3.8. El isótopo 42K tiene un tiempo de semidesintegración de 12 horas. ¿Cuál es la fracción de la


concentración inicial de dicho isótopo que queda después de 48 horas?
a) 1/16 b) 1/8 c) ¼ d) 1/2

374
3.9. En el siguiente diagrama de la tabla periódica se indican algunos elementos cuyas letras no se
corresponden con las de sus símbolos.

Con respecto a estos elementos, indica cuál de las siguientes afirmaciones es correcta:
a) Q y J forman el compuesto de mayor carácter iónico.
b) X y J forman el compuesto de mayor carácter iónico.
c) R y T forman el compuesto de mayor carácter covalente.
d) R y J forman el compuesto de mayor carácter covalente.

3.10. Se irradió a temperatura constante una mezcla que contenía cloro e hidrógeno en un frasco
cerrado. Después de cierto tiempo el contenido de cloro disminuyó en un 20 % comparado con
la mezcla inicial y la composición de la mezcla resultante fue la que se indica a continuación:
60 % en volumen de cloro, 10 % en volumen de hidrogeno y 30% en volumen de HCl. ¿Cuál
era la composición inicial de la mezcla original?
a) 75 % de Cl2 y 25 % de H2 b) 30% Cl2 y 70 % de H2
c) 45 % de Cl2 y 65 % de H2 d) 25 % de Cl2 y 75 % de H2

3.11. Se disuelven 12,8 g de carbonato sódico en la cantidad de agua suficiente para preparar 325
mL de disolución. La concentración de esta disolución en mol·L−1 es:
a) 3,25 b) 0,121 c) 0,0393 d) 0,372

3.12. Cuántas moléculas de agua de cristalización pierde el sulfato de aluminio sabiendo que al
calentarlo pierde un 48,68% de su masa.
a) 12 b) 24 c) 18 d) 6

3.13. Una muestra de 0,01 moles del cloruro de un elemento X reaccionan completamente con 200
cm3 de una disolución 0,1 mol/L de nitrato de plata. ¿Cuál es la identidad de dicho elemento?
a) K b) Ca c) Al d) Si

3.14. Se tiene una disolución de PbI2 saturada en la cual la concentración de iones yoduro es
2.42X10-3 mol/L. El producto de solubilidad del PbI2 es igual a:
a) 5,7 x 10-8 b) 2,3 x 10-6 c) 7,1 x 10-9 d) 2,4 x 10-6

3.15. ¿Cuál de las siguientes sales dan lugar a una disolución ácida?
a) CH3COONa b) NH4Cl c) Na2CO3 d) NaHCO3

3.16. ¿Cuántos litros de agua destilada deben añadirse a 1 L de disolución acuosa de HCl con pH = 1
para obtener una disolución con pH = 2?
a) 10 L b) 0,9 L c) 2 L d) 9 L

3.17. ¿Qué concentración mínima de HCl deberá tener una disolución de HCl para que al poner en
contacto 50 mL de ésta con 500 mg CaCO3 se logre la disolución completa del sólido?
a) 0,1 mol/L b) 0,2 mol/L c) 0,05 mol/L d) no se disuelve

375
3.18. ¿Cuál de las siguientes sustancias se pueden reducir con Ag?
E0 V/ENH E0 V/ENH E0 V/ENH
2+ 3+ 3+ 2+
Cu /Cu 0,34 Cr /Cr –0,74 V /V –0,255
3+ 2+ 2+ 2+ 3+
Fe /Fe 0,77 Cr /Cr –0,90 VO /V 0,36
+ 2+ 2+ 3+ 2+
VO /VO 1,00 Zn /Zn –0,76 V /V –0,255
2
2+ 3+ 2+ 3+ +
VO /V 0,36 TiO /Ti 0,10 Ag /Ag 0,80

2+ 2+ +
a) Cu b) VO c) VO d) ninguna
2

3.19. A 60 °C el agua destilada tiene pH = 6,51 y por lo tanto:


a) La concentración de OH− no es igual a la de H3O+.
b) El valor de Kw a 60 °C es igual a 10-13,02.
c) A 60 °C también se cumple que pH + pOH = 14.
d) Es imposible, el agua neutra debe tener pH = 7.

3.20. ¿Cuál de las siguientes líneas gráficas representa mejor el volumen del ácido añadido a un
cierto volumen de álcali frente al pH de la disolución resultante?

a) b) c) d)

FIN DEL EXAMEN NIVEL B

DE AQUÍ EN ADELANTE SÓLO CONSTESTAN LOS DE NIVEL A.

3.21. La configuración electrónica del Li en el estado fundamental es 1s2 2s1 y por tanto:
a) El Li es un elemento del grupo 2.
b) El átomo de Li no tiene propiedades magnéticas.
c) Esta configuración también podría ser 1s2 2p1.
d) El átomo de Li es paramagnético.

3.22. Con respecto a la teoría de enlace, indique cuál de las siguientes afirmaciones es cierta:
a) La molécula de CO2 es polar debido a que presenta estructuras resonantes.
b) La geometría de la molécula de PCl3 es bipiramidal regular.
c) El momento dipolar del BeF2 es cero por ser una molécula simétrica.
d) La polaridad del CCl4 es debida a la diferencia de electronegatividad del carbono y del cloro.

3.23. Si el 238U experimenta emisión α, ¿cuál es el otro núclido que se produce?


a) 234Th b) 234U c) 234Pa d) 236Np

376
3.24. ¿Cuál de las siguientes mezclas es una disolución reguladora con un pH menor de 7?
(Ka ácido acético = 1,7·10–5 y Kb NH3 = 1,7·10–5)
a) 10 mL de ácido acético 0,1 mol/L + 10 mL de NaOH 0,1 mol/L
b) 10 mL de ácido acético 0,1 mol/L + 5,0 mL de NaOH 0,1 mol/L
c) 10 mL de NH3 0,1 mol/L + 5,0 mL de HCl 0,1 mol/L
d) 10 mL de NH3 0,1 mol/L + 10 mL de HCl 0,1 mol/L

3.25. Los potenciales normales de electrodo para el Al3+/Al y Cr3+/Cr son -1,66 V y -0,74 V,
respectivamente. ¿Cuál de las siguientes afirmaciones es cierta para la siguiente reacción en
condiciones estándar? Al + Cr3+ → Al3+ + Cr
a) ∆E° = 2,40 V y la reacción es espontánea.
b) ∆ E° = 0,92 V y la reacción es espontánea.
c) ∆E° = 0,92V y la reacción es no espontánea.
e) ∆E° = 2,40V y la reacción es no espontánea.

4.1. Escriba el producto o la materia prima de las siguientes reacciones:


A 1) O3 C D
1) EtMgBr
C10H12 2) Me-S-Me C9H10O C11H16O
2) H2O

B CH3-NH Me Me
AlCl3 H , cat. CH3 N
- H 2O
Me

A B

C D

377
4.2. Proponga reactivos para realizar las siguientes reacciones:

O OH O

H A CH3 B CH3

CH2
CH3 OH

OH

A B C D E

FIN DEL EXAMEN NIVEL A

378
XXII OLIMPIADA NACIONAL DE QUÍMICA
TERCER EXAMEN Tiempo asignado: 180 minutos Chihuahua 2013
NIVEL A: 7 problemas; NIVEL B: 5 problemas

NIVEL B NIVEL A
Problema 1 Del 1.1 al 1.9 Del 1.1 al 1.6
Problema 2 Del 2.1 al 2.6 Del 2.1 al 2.7
Problema 3 Todo el problema Todo el problema
Problema 4 Todo el problema Todo el problema
Problema 5 Todo el problema ----------------------
Problema 6 ---------------------- Todo el problema
Problema 7 ---------------------- Todo el problema
Problema 8 ---------------------- Todo el problema

1. Soluciones Ideales: La Acetona.


La acetona o propanona (CH3-CO-CH3) es un compuesto químico que se evapora fácilmente, es
inflamable y soluble en agua. Se usa en la fabricación de plásticos, fibras, medicamentos y otros
productos químicos, así como disolvente de otras sustancias químicas. Durante el proceso de
evaporación, a presión constante de una atmósfera, la acetona absorbe 501.7 J/g, a su temperatura
de ebullición normal de 56.1 °C, en tanto que para el proceso de fusión a –94.45 °C se requieren
98.62 J/g. En un trabajo de la escuela un grupo de alumnos encontraron, para la acetona, que se
reporta en la literatura una constante ebulloscópica de 1.71 K kg/mol en tanto que no pudieron
encontrar el valor de la constante crioscópica, por lo que procedieron a calcularla con la
información que se tiene.
(Utiliza los cuadros para tus cálculos y marca la respuesta correcta).
R = 8.314 J/mol K = 8.314 Pa m3/mol K T = t + 273.15 1 atm = 101325 Pa
Respuestas sin Cálculos no son válidas.

1.1. El valor de la constante crioscópica de la acetona en K kg/mol es:

a) 0.752 b) 2.228 c) 2.692 d) 3.156

379
1.2. Se preparó una solución que contiene 150 g de acetona y 15 g de un soluto cuya masa molar es
180.16 g/mol. Suponiendo que la constante crioscópica de la acetona tiene un valor de 2.96 K
kg/mol, la temperatura de fusión normal de esta solución es:

a) 177.06 K b) 271.5 °C c) 180.34 K d) 274.79 °C

1.3. El soluto empleado contiene 60% en masa de carbono, 4.44% de hidrógeno y 35.55% de
oxígeno. La fórmula de este compuesto es:

a) C6 H12O6 b) C9H8O4 c) C10H12O3 d) C11H16O2

1.4. La temperatura de ebullición cambia con la concentración. La molalidad de una solución que
hierve a 56.9 °C es:

a) 0.270 b) 0.297 c) 0.351 d) 0.468

380
1.5. La temperatura de ebullición cambia con la presión barométrica local. En la ciudad de
Chihuahua la presión barométrica media es de 853 hPa (hectopascales: 1 hPa = 100 Pa). De
acuerdo con la ecuación de Clausius Clapeyron, la temperatura de ebullición de la acetona pura
en la ciudad de Chihuahua es:

a) 251.96 K b) 324.00 K c) 329.09 K d) 334.67 K

1.6. La ecuación de Antoine permite calcular la presión de vapor de un líquido volátil. Para la
acetona las constantes de la ecuación de Antoine son:
A = 4.4245 B = 1312.3 C = – 32.445
Considerando la ecuación de Antoine, la presión de vapor de la acetona a 25 °C es:

a) 0.306 bar b) 0.451 bar c) 0.598 bar d) 1.570 bar

1.7. A cierta temperatura la acetona tiene una presión de vapor de 13.33 kPa. De acuerdo con la
ecuación de Antoine la temperatura es:

a) 349.5 K b) 337.6 K c) 313.6 K d) 280.0 K

381
Cuando se mezclan dos compuestos volátiles la presión de vapor de la mezcla es la suma de las
presiones que ejercen cada uno de ellos de acuerdo con la ley de Raoult:

1.8. A cierta temperatura la presión de la acetona pura es 35.5 en tanto que la de


otro compuesto volátil es 40.3 . Se preparó una mezcla de 2.5 mol de acetona con 3.5
mol del segundo compuesto. La presión de vapor total de esta mezcla es:

a) 39.96 kPa b) 38.3 kPa c) 37.9 kPa d) 37.5 kPa

1.9. Cuando se mezclan 116.0 g de acetona con 222.0 g del segundo compuesto la presión de vapor
total de la mezcla es 38.38 kPa. La masa molar del segundo compuesto es:

a) 38.7 g/mol b) 44.4 g/mol c) 74.0 g/mol d) 111.0 g/mol

Información Adicional:
1 atm = 101325 Pa 1 bar = 105 Pa 1 hPa = 102 Pa 1 kPa = 103 Pa
R = 8.314 J/mol K = 8.314 Pa m3/mol K

∆ ∆

382
∆ 1 1 La temperatura T en Kelvin
ln .

La presión P en bar y la
.
temperatura T en Kelvin

2. Un estudiante olímpico de la Universidad de Chihuahua fue encargado de determinar la


composición de una muestra que contenía una mezcla de NaCl y KCl. Para lograrlo tomó 25,0 g
de la muestra y, una vez disuelta en agua, le añadió 840,0 mL de una disolución de AgNO3 0,5
mol/L. Observó la formación de un precipitado blanco que filtró. En el líquido filtrado sumergió
un alambre de cobre que pesaba 100,0 g y, después de cierto tiempo, comprobó (adicionando
unas gotas de HCl) que en la disolución ya no había iones plata pero que el alambre pesaba
101,52 g debido a un depósito de plata. Con estos datos resolvió el problema y determinó el
porcentaje de cada una de las sales en la mezcla.

2.1. Escribe la ecuación balanceada de la reacción que corresponde a la formación del precipitado.

2.2. Indica la cantidad de nitrato de plata añadida a la disolución de la mezcla de sales.

g AgNO3

2.3. Escribe la ecuación balanceada de la reacción que corresponde al aumento del peso del alambre
de cobre.

2.4. Calcula la masa de plata metálica que se depositó en el alambre de cobre.

g Ag

2.5. Calcula los moles de Cu que se disolvieron.

mol de Cu

383
2.6. Calcula el % de NaCl y KCl en la muestra original.

2.7. Calcula los moles de nitrato de plata que se gastaron para formar el precipitado de plata
metálica del halogenuro.

moles de AgNO3

3. La concentración total de un ácido débil disuelto en una disolución amortiguadora de pH=8,8 es


igual a 2 x 10-2 mol/L. La forma básica del ácido tiene color y, en consecuencia, puede absorber
luz de la zona visible del espectro electromagnético. El valor de la absortividad molar (ε) de esta
especie es 2,1 x 104 cm2 mol-1. Al medir una capa de 1 cm de la disolución que contenía el ácido
débil se observó que se absorbía el 60% de la luz incidente Po.
Datos: Ley de Lambert-Beer: log (Po/P) = A = εlc
en donde Po y P corresponden, respectivamente, a la potencia del haz de luz incidente y del
emergente después de atravesar el material absorbente; A es la magnitud que se mide y que se
denomina absorbancia, ε es la constante de proporcionalidad llamada absortividad molar y l es
la longitud de la capa de disolución que atraviesa el haz de luz.
De esta experiencia indica:

3.1. ¿Cuál es el valor de A medido?

3.2. ¿Cuál es la concentración de la forma básica del ácido débil?

3.3. ¿Cuál es el valor del pKa del ácido?

384
4. En el laboratorio de Química Inorgánica se encontraron 10 frascos sin etiquetar que contenían
disoluciones acuosas de la siguientes sustancias: NH4Cl, Ag2SO4, Pb(NO3)2, NaOH, NH4I, KCl,
Na2SO4, AgNO3, KI, Ba(OH)2.
Un estudiante que se preparaba para la olimpiada nacional pudo identificarlas todas mediante
reacciones a la gota entre ellas mismas. Para lograrlo, colocó un poco de cada disolución en un
frasco gotero y los número del 1 al 10. En la siguiente tabla se muestran los resultados de las
pruebas positivas obtenidos por el estudiante:

Abreviaturas usadas: ppbl = precipitado blanco, ppam= precipitado amarillo,


gol = desprendimiento de gas con olor característico.
TUBO 1 2 3 4 5 6 7 8 9 10
1 ppbl gol ppbl
2 ppbl ppbl
3 ppbl ppbl
4 ppbl ppbl ppam ppam ppbl
5 ppam ppam ppam
6 ppam ppam gol ppam
7 ppbl
8 ppam ppam ppbl
9 gol ppbl gol ppbl
10 ppbl ppbl ppbl ppam ppam

4.1. En la siguiente lista indica cuál es la disolución que corresponde a cada compuesto y escribe la
ecuación balanceada de la reacción correspondiente a cada identificación.
Frasco Fórmula del
Reacción de identificación
No. compuesto
1
2
3
4
5
6
7
8
9
10

385
5. En este problema tendrás que acomodar nueve de los primeros 29 elementos de la Tabla Periódica
en el diagrama que se muestra a continuación. Toma en cuenta los datos que se te proporcionan y
escribe en cada recuadro el símbolo del elemento que le corresponde:

1 2 3

4 5 6

7 8 9

Los primeros 29 elementos son:

H He Li Be B C N O F Ne

Na Mg Al Si P S Cl Ar K Ca

Sc Ti V Cr Mn Fe Co Ni Cu

Muy importante: cuando termines de acomodar los elementos notarás que el número
atómico siempre va en aumento desde la casilla 1 hasta la 9.

Datos:
a) Tienes que incluir dos gases nobles.

b) Si multiplicas los números atómicos de dos de los elementos incluidos, el resultado es mayor a
800.

c) Sólo debes incluir un metal alcalino y colocarlo en la columna central.

d) En la casilla central debes colocar el elemento cuyo óxido tiene una masa molar aproximada
de 101.96 g/mol.

e) En un kilogramo de este elemento, que se ubica en una esquina, hay casi 21 moles.

f) En otra esquina está un elemento cuya molécula es diatómica. Si pones la misma cantidad en
gramos de esta molécula y de monóxido de carbono, su fracción mol será 0.5.

g) No debes incluir ningún elemento que esté presente en la molécula de benceno.

h) De la molécula de la sal común, sólo debes incluir uno de los elementos que la constituyen y
colocarlo en la columna derecha.

386
6. Para la reacción entre gases: A (g)  2 B (g), a una temperatura dada, se tiene que las presiones
en el equilibrio son: PB = 1.7 bar y PA = 0.65 bar. El volumen inicial es de 5.0 L. Manteniendo T
constante, se duplica el volumen del recipiente.

6.1. Calcula el valor de Kp en la situación inicial antes de duplicar el volumen.

6.2. Calcula las nuevas PB y PA cuando el sistema alcanza nuevamente el equilibrio.

PB = ________________bar PA = ________________bar

7. El níquel cristaliza en una red cúbica centrada en las caras y su densidad es 8,94g/cm3 a 20 °C.
¿Cuál es el volumen y la longitud de la arista de la celda unitaria?

Datos: V del cubo = a3, donde a es igual a la arista del cubo.

Cálculos del volumen del cubo:

Cálculo de la arista del cubo:

387
8.1. Escriba un descriptor estereoquímico apropiado para los siguientes compuestos:

Me
CH3 CH2CH3 CO2H CO2H

H NH2
H
CH3
H H HO CH3 Me

A B C D

8.2. Escriba los reactivos y materias primas apropiados para realizar la siguiente secuencia sintética:

I II III
A
B C
C6H11Br

A B C

I II III

388
8.3. Escriba el producto principal de las reacciones siguientes:

CH3 O
I)
N
HNO3
A
H2SO4
Mononitración

O
II)

O 1) 4 MeMgBr
B
O 2) H2O

O
III)

Br2
C
FeBr3

A B C

FIN DEL TERCER EXAMEN.

389
390
XXII OLIMPIADA NACIONAL DE QUÍMICA
EXAMEN INTERNACIONAL
5 PROBLEMAS Tiempo: 4 HORAS Chihuahua 2013

1. El compuesto A es una sal estable que contiene a un metal de transición M. La química de este
metal es notable ya que, al igual que algunos otros elementos de transición, forma muchos
compuestos que con frecuencia son complejos por su valencia variable. En el siguiente esquema se
muestran catorce reacciones partiendo desde A hasta M que dan lugar a diferentes compuestos en
los que interviene este elemento.

1  2 H+ / H2O 3 Zn / H+
B (s) C (ac)

4 H2O 5 H+ / H2O
A (s) D (ac)

6 SO2 / H+
7 E (ac) 7 F (ac)
F (ac) G (ac) E (ac)
H+ H +

11 H2O 12 H2O 13 H2O

8 Cl2 9  10 H2
M (s) I (l) J (s) K (s)

14 
I+K

A continuación se muestra una lista de los posibles compuestos o iones que en el esquema se
identifican con letras:

M, M2+, M2O5, MO3-, MO2+, M2+, MCl2, M3+, MCl3, MCl4, MO22+, MO2+

Con esto y con la información que te indicamos a continuación, deberás identificar a cuál
corresponde cada uno de ellos.

 Además del elemento M, la sal A de este elemento contiene 11.97% de N, 3.45% de H y


41.03% de O (Porcentajes en masa).

391
 Sobre la flecha están indicados los reactivos necesarios para cada transformación; todas las
sustancias indicadas con una letra contienen al metal pero este elemento no se encuentra en
los subproductos. El símbolo Δ significa calor.

1.1. Calcula la relación de moles de N, H y O en la molécula de la sal.

1.2. Se sabe que el número de moles de nitrógeno y de M en la molécula es el mismo. Con esta
información realiza los cálculos que te permitan identificar M y, en consecuencia identificar
A.

En la siguiente tabla se muestran los potenciales estándar del sistema redox de algunos de los
reactivos utilizados en el esquema.

SO3 Zn2+ H+ Cl2


-0.936 -0.762 0 1.39
E0
SO2 Zn H2 Cl-

1.3. De las especies químicas anteriores la más reductora es: _______

1.4. De las especies químicas anteriores la más oxidante es: _______

1.5. Indica sobre la línea correspondiente la especie química identificada para cada letra

M = _______ A = ______ B = ______ C = ______ D = _______ E = _______

F = _______ G = ______ I = _______ J = _______ K = ______

1.6. Escribe la ecuación de las reacciones 1 a 14.


1

392
4

10

11

12

13

14

********************************************************************************

2. El Nitrógeno.
El nitrógeno, elemento químico que en condiciones normales se encuentra en estado gaseoso,
tiene una gran participación en nuestras vidas y en nuestro entorno. Es el componente principal
de la atmósfera con un 78% en volumen del aire que respiramos. (Considera la masa molar del
aire Maire = 28.82 g/mol)
El nitrógeno cuenta con grandes aplicaciones comerciales a través de sus compuestos que
involucran la fabricación de fertilizantes (NH3), otros con importantes efectos benéficos para
salud como es el óxido nítrico (NO) y desafortunadamente algunos de sus compuestos son fuente
de uno de los contaminantes más severos de la atmósfera (anhídrido nítrico, N2O5).
Considerado como un gas inerte por su baja reactividad, se emplea industrialmente para crear
atmósferas protectoras en alimentos empacados. Otra aplicación importante se encuentra en su
uso como gas criogénico para congelar rápidamente a temperaturas del orden de 78 K y
recientemente para el inflado de llantas de automóviles. El nitrógeno se obtiene por destilación
del aire líquido a muy bajas temperaturas.
En 1908, Robert Bosch y Fritz Haber desarrollaron el método para fabricar el amoníaco, primer
paso para la producción de fertilizantes, de acuerdo con la reacción:
½ N2(g) + 3/2 H2(g)  NH3(g)

Consulta el Anexo de Datos para la información necesaria para resolver las preguntas.
(Utiliza los cuadros para tus cálculos, en todos los casos marca la respuesta correcta.
Respuestas sin Cálculos no son válidas
R = 8.314 J/mol K = 8.314 Pa m3/mol K T = t + 273.15 1 atm = 101325 Pa

393
2.1. La cantidad de nitrógeno que se puede obtener de 1 m3 de aire a 25 °C y un bar de presión es:

a) 237.2 g b) 643.8 g c) 881.1 g d) 906.4 g

2.2. El porciento en masa del nitrógeno en el aire a 25 °C y 1 bar de presión es:

a) 79.8% b) 75.8% c) 51.2% d) 18.1%

2.3. La temperatura que alcanza una rosa cuando se le sumerge en nitrógeno líquido en la Cd. de
México (Pbarométrica = 0.7799 bar) es:

a) 63.3 K b) 75.6 K c) 77.34 d) 126.2 K

2.4. La cantidad de NH3 que se puede obtener en la reacción de Robert Bosch y Fritz Haber, de 1 kg
de nitrógeno suponiendo reacción completa es:

a) 303.6 g b) 404.8 g c) 1214.3 g d) 1821.4 g

394
Considera la reacción de disociación del amoniaco a 25 °C y 1 atmósfera:
NH3(g)  ½ N2(g) + 3/2 H2(g)

2.5. La constante de equilibrio de disociación del NH3 a 25 °C es:

a) 1.2858 x 10–3 b) 1.0071 c) 6.6561 d) 777.74

2.6. A cierta presión el grado de disociación es  = 0.05; la cantidad de NH3 que se puede obtener
de 1 kg de nitrógeno en la reacción en el equilibrio es:

a) 576.8 g b) 865.2 g c) 950 g d) 1153.6 g

Anexo de Datos para el Nitrógeno:


Toeb = 77.34 K Ttriple = 63.14 K Hfus = 0.3604 kJ/mol
Tofus = 63.3 K Ptriple = 0.1252 bar Hvap = 5.57 kJ/mol
Tc = 126.19 K Pc = 33.978 bar

. (P en bar y T en Kelvin)
A = 3.7362 B = 264.651 C = –6.788

Propiedades Termodinámicas
Gas H°f/kJ mol–1 G°f/kJ mol–1 S°/kJ mol–1 K–1
Oxígeno 0 0 205.037
Hidrógeno 0 0 130.57
Amoniaco –45.94 –16.5 192.67

********************************************************************************

395
3.1. Escriba los productos de las siguientes secuencias de reacciones:

I) A
1) O3
2) CH3SCH3
3) LiAlH4
4) H2O

II)
CH3 B
1) Br2, h
2) EtO Na

3) R-CO3H
4) CH3OH, H

OH
III)

1) HBr
C
2) Mg
O
3)

4) H2O
5) PCC
6) H2N-NH2, KOH
OH
, calor
HO

396
3.2. Escriba el producto final y los correspondientes intermediarios que se obtienen para las
siguientes secuencias sintéticas:
O
I)
CH3O OCH3

O O

H , calor,
HO OH - H2O

1) LiAlH4

2) H2O

H , calor

+
HO OH

397
II)
A
O

MeO 1) 4 MeMgBr
OMe
2) H2O
O

HCl

C B

AlCl3

********************************************************************************

4. En Química es común utilizar curvas de calibración para cuantificar analitos. En este caso se
utilizan instrumentos que proporcionen una señal (S) que sea proporcional a la especie química
cuya concentración deseamos conocer (S = mC) y se preparan una serie de disoluciones estándar
de concentración conocida de manera que se obtenga una recta, con una pendiente conocida, que
permita conocer el valor de la constante de proporcionalidad m. Con este dato es posible
posteriormente medir una disolución del mismo analito y conocer su concentración (y en
consecuencia su cantidad). Para lograrlo es necesario que el valor de la constante m sea la misma
y esto no siempre es fácil de reproducir cuando en la muestra analizada existen efectos
interferentes debidos a otras sustancias presentes en la muestra (efecto matriz) distintas del
analito a cuantificar. Tal es el caso de algunos productos naturales en que los efectos del medio
son difíciles de reproducir en los estándares o de eliminar en la muestra analizada. En estas
ocasiones se aplica el método llamado de adiciones patrón que consiste en preparar cada
estándar con un mismo volumen conocido de muestra.
La representación gráfica de la variación de la señal medida en función de la concentración tiene
el aspecto de una recta con ordenada al origen diferente de cero (S = b + mC). La concentración
de la disolución problema se puede calcular a partir de la pendiente (m) y de la ordenada al
origen (b) o bien por extrapolación gráfica a la abscisa que corresponde al origen de ordenadas
(considerando el valor absoluto).
Este fue el procedimiento utilizado para determinar el contenido de ácido ascórbico (vitamina C)
en cuatro muestras de diferentes jugos de naranja. Para obtener las gráficas correspondientes se
procedió de la forma que se indica a continuación:
En una celda adecuada para efectuar la medición, se colocaron 10.0 mL de una disolución de
pH=10 a la cual se agregó un volumen medido de muestra de jugo. Este procedimiento se realizó
con cada uno de cuatro jugos determinados; el volumen de muestra añadido en cada caso se
indica en la tabla “A”. A continuación se fueron añadiendo, alícuotas de 5 microlitros de una

398
disolución patrón de ácido ascórbico (vitamina C) de concentración 0.01 mol L-1 (como la masa
molar del ascórbico es 176.13 g/mol, cada adición corresponde a 8.8 µg agregados). La densidad
promedio del jugo de naranja es 1.042 g mL-1.
Se midió la intensidad de corriente producida al efectuar la medida de las disoluciones obtenidas
(manteniendo un potencial constante) y, para simplificar, se representó este valor directamente en
función de los microgramos de ácido ascórbico añadidos. Los resultados obtenidos se muestran
en la figura que se muestra a continuación. En la misma Tabla “A” se encuentra también la
información obtenida de estos datos experimentales, lo cual te permitirá calcular el contenido de
vitamina C en cada muestra de jugo y completar la Tabla “B” para indicar las
correspondientes ppm.1

I (nA) Muestra I:
500
Determinación de ecuación de recta de regresión
vitamina C
en jugos de naranja 450
y = 5.43x + 199.8
por el método de R² = 0.9999
adiciones patrón 400

350

Muestra II 300

Muestra III
250

Muestra IV
200
Muestra I

150

100

50

0
‐60 ‐40 ‐20 0 20 40 60
microgramos de patrón agregados

Figura 1

Tabla “A”
                                                            
1
 Recuerda que “ppm” representa “partes por millón”, es decir,  una parte  en  un millón de partes  (por 
ejemplo 1 microgramo en 1 mililitro de agua pura ó 1 miligramo en 1 gramo de una disolución o de una 
mezcla sólida). 

399
Muestra L agregados Pendiente Ordenada Abscisa
de muestra al origen al origen
I 100 5.43 199.8 -36.8
II 50 5.40 181.1 -33.5
III 50 5.38 141.5 -26.3
IV 400 5.39 110.8 -20.6

Por otra parte se utilizó el mismo método de adiciones patrón para efectuar determinaciones de
hidroximetilfurfuraldehído (HMF) (producto de transformación de azúcares reductores por
calentamiento); el contenido de HMF es un parámetro importante para determinar si un producto
es fresco o si ha sido adulterado por adición de algún azúcar.
Los valores de pH de cada jugo y los resultados obtenidos en esta determinación se encuentran
también indicados en la Tabla B. Con estos resultados y con los que obtengas del ácido
ascórbico podrás determinar la calidad de las diferentes muestras de jugo de naranja y, por
comparación con los valores requeridos en la norma mexicana, indicar cuál es el mejor de dichos
jugos.
Tabla “B”
Muestra pH Azúcares HMF Vitamina
Reductores ppm C
% µg en ppm
disolución en jugo de
medida naranja
I 4.0 12 20
II 4.0 4.8 < 0.03
III 3.5 4.1 < 0.03
IV 3.6 8.2 37
Norma mexicana 3 a 4 3.5 a 12 < 20 >300

De acuerdo con tus resultados, el mejor jugo es:

La peor calidad la tiene la muestra:

Una muestra fue adulterada con un azúcar


reductor (glucosa); lo más probable es que se
trate de la muestra:

Un estudiante olímpico interesado en conocer algo más de la oxidación de la vitamina C, decidió


estudiar la reacción de ésta con ferricianuro de potasio. Para lograrlo utilizó 10.0 mL de la
disolución patrón de ácido ascórbico y observó que, para llegar al punto de equivalencia, se
consumieron 9.1 mL de una disolución patrón de ferricianuro de potasio 0.022 mol L-1. Indica los
coeficientes estequiométricos para balancear la reacción de valoración:

____ Vit C + ____ [Fe(CN)6]3- → ____ Vit Cred + ____ [Fe(CN)6]4-

********************************************************************************

400
5. Puedes calcular la constante de Avogadro utilizando un método electroquímico. Supón que
construyes una celda electroquímica que consta de dos electrodos de cobre inmersos en una
solución acuosa de H2SO4 0.5 M. Cada electrodo tiene una masa inicial de 14 g. Haces pasar una
corriente constante de 600 mA durante 30 minutos y observas que la masa de uno de los
electrodos ha disminuido y ahora es de 13.6446 g. La masa del otro electrodo no ha variado y
durante el proceso notaste que en su superficie se formaban burbujas.

5.1. Escribe las ecuaciones de las reacciones que ocurren en el ánodo y en el cátodo:

Ánodo ____________________________________

Cátodo ____________________________________

5.2. Calcula el número de electrones que han pasado por el circuito en este proceso de electrólisis:

Número de electrones =

5.3. Calcula la masa de un átomo de cobre:

Masa de un átomo de cobre =

5.4. Si la masa molar del cobre es 63.546 g mol-1, calcula la constante de Avogadro experimental:

Constante de Avogadro experimental =

5.5. Calcula cuántos moles de gas se obtienen en este proceso y su masa en gramos:

Cantidad de sustancia: __________ mol Masa: ___________ g

Información importante: la unidad de corriente es el ampere (A) y la unidad de carga es el


coulomb (C).
A = C/s
donde “s” representa un segundo. La carga de un electrón es 1.602x10-19 C.

FIN DEL EXAMEN INTERNACIONAL

401
402
XXII Olimpiada Nacional de Química
Examen Experimental de Química Analítica Chihuahua 2013

DETERMINACIÓN DE LA PUREZA DE UNA MUESTRA DE ÁCIDO BÓRICO

El ácido bórico o ácido trioxobórico (III) es un ácido débil que es utilizado como antiséptico,
insecticida, retardante de la flama y precursor de otros compuestos químicos. El equilibrio de
disociación del ácido bórico en agua es:

H3BO3  BO2- + H+ + H2O Ka = 5,81 x 10-10

El valor de esta constante es muy pequeño y el error que se comete cuando se quiere titular el ácido
bórico directamente con una base fuerte no es aceptable. Sin embargo, se conoce que el borato foma
complejos con algunos compuestos orgánicos polihidroxilados, tales como el manitol, cuya
estructura se muestra a continuación:
OH OH

OH
HO

OH OH Simbolizado como Ma

La formación de un complejo Ma2BO2- (con una constante de formación igual a 103.3) libera iones
H+ en concentración igual a la del ácido bórico inicial y hace posible valorar cuantitativamente el
ácido bórico con una base fuerte. En este caso, la reacción de valoración del ácido bórico con una
base fuerte es:
H3BO3 + 2Ma + OH-  Ma2BO2- + 2H2O

Al laboratorio de Química Analítica de esta Universidad han llegado varias muestras de ácido bórico
cuya pureza se debe determinar con urgencia y esta mañana deberás ayudarnos a entregar los
resultados rápidamente. Para ello se te proporciona una muestra de ácido bórico que fue previamente
pesada y disuelta en 1,00 litro de una disolución que contenía manitol en exceso. Cuentas además
con una disolución de NaOH de concentración exactamente conocida y con fenolftaleína como
indicador.

Dato: La cantidad de ácido bórico que previamente se pesó y disolvió en la disolución de


manitol se indica en el frasco que contiene la muestra que tienes en tu lugar de trabajo.

¡Advertencia!
Mientras te encuentres en el laboratorio debes usar los lentes de seguridad o tus propios lentes y usar
la perilla adecuada siempre que tengas que medir algún volumen con la pipeta. La primera vez que
no trabajes con lentes o pipetees con la boca será motivo de una amonestación del supervisor
experimental. Repetir una falta de esta naturaleza involucra una penalización de 5 puntos sobre la
puntuación total de este problema. La tercera falta es considerada grave e incompatible con el
trabajo experimental ya que viola las normas vitales de seguridad; serás expulsado del laboratorio y
obtendrás cero en toda la prueba experimental. Si requieres exceso de algún reactivo se te
proporcionará con una penalización de 2 puntos.

403
HOJA DE RESPUESTAS

Nivel B: Deberá responder los incisos a, b, c y d.


Nivel A: Deberá responder todos los incisos.

a) Escribe la ecuación balanceada que representa la formación del complejo entre el ácido bórico y
el manitol.

b) Realiza por triplicado la titulación de la disolución de ácido bórico (disuelto en la de manitol) y


completa la tabla con los resultados obtenidos.
Titulación Volumen de disolución de Volumen de NaOH utilizado en la
ácido bórico neutralización
1
2
3

c) Indica el volumen de titulante que utilizarás en los cálculos: _________________

d) Calcula el porcentaje de pureza de la muestra de ácido bórico proporcionada.

e) Calcula el valor de la constante de la reacción de nuetralización del ácido bórico con una base
fuerte en presencia de manitol.

404
XXII Olimpiada Nacional de Química
Examen Experimental de Química Orgánica Chihuahua 2013

REACCIONES DE ALDEHÍDOS Y CETONAS


CONDENSACIÓN DE CLAISEN-SCHMIDT
OBTENCIÓN DE DIBENZALACETONA

OBJETIVOS.
Que el alumno:
a) Efectúe en el laboratorio una reacción de condensación aldólica cruzada.
b) Realice una reacción de adición nucleofílica-eliminación.

ANTECEDENTES.
En este experimento se va a obtener la dibenzalacetona mediante una condensación aldólica cruzada
y posterior deshidratación del aldol obtenido como intermediario.

REACCIÓN.
O

O
H NaOH
+
2 H2O / EtOH +
CH3 CH3
2 H2O

MATERIAL
Vaso de precipitados de 250 mL 1 Espátula 1
Matraces Erlenmeyer de 125 mL 2 Baño con resistencia eléctrica 1
Probeta de 25 mL 1 Recipiente para hielo 1
Pipeta de 10 mL 1 Papel filtro 2
Matraz Kitasato con manguera 1 Frasco para cromatografía 1
Pinza de tres dedos con nuez 1 Frasco vial 1
Embudo Büchner con empaque de hule 1 Cámara de I2 1
Placa para cromatografía en capa fina 1 Papel pH

REACTIVOS
Benzaldehído Acetona
Hidróxido de sodio Etanol
Acetato de etilo Hexano
Ácido clorhídrico (1:1)

405
PROCEDIMIENTO.

En un matraz Erlenmeyer de 125 mL coloque 1.1 g de hidróxido de sodio, 12 mL de agua y 10 mL


de etanol. En seguida agregue poco a poco y con agitación 1.5 mL de benzaldehído y 0.5 mL de
acetona. Continúe agitando la mezcla de reacción durante 15 min más, manteniendo la temperatura
de la mezcla de reacción entre 20-25 0C; use (de ser necesario) un baño de agua fría o tibia según sea
el caso.

Filtre el precipitado, lave con agua fría por lo menos 8 veces, hasta que las aguas madres del lavado
tengan un pH = 7. Seque por filtración al vacío y purifique el producto por recristalización de etanol
(10 mL). El producto obtenido bajo estas condiciones tiene una pureza entre 96 y 98%.

Si al recristalizar el producto la solución se torna de color rojo-naranja, es debido a que el pH es


ligeramente alcalino, por lo que debe agregar ácido clorhídrico (1:1) hasta pH = 7.

Para comprobar la pureza, realice una cromatografía en capa fina (ccf), para ello use una mínima
cantidad del producto disuelto en AcOEt. El eluyente a usar es Hexano:AcOEt, 3:1. Use como
revelador una cámara de I2.

Dibuje la placa, indicando el punto de aplicación, el frente del disolvente y determine el Rf. Además
debe remarcar la mancha que se revela con el I2 y se debe entregar la placa dentro de la bolsa de
plástico, engrápela a su examen.

RESULTADOS.

Dibuje en el cuadro la placa resultante de la cromatografía en capa fina realizada en el experimento.

Rf =

Coloque el resto del producto obtenido dentro de la otra bolsa y engrápela a su examen.

406
PREGUNTAS.

1. Explique brevemente por qué se debe adicionar primero el benzaldehído y después la acetona a la
mezcla de reacción.
______________________________________________________________________________
______________________________________________________________________________
______________________________________________________________________________
__________________________________________________________________________

2. ¿Por qué se obtiene un solo producto y no una mezcla de productos en este experimento?
______________________________________________________________________________
______________________________________________________________________________
______________________________________________________________________________
__________________________________________________________________________

3. Indique por qué se pierde fácilmente agua (reacción de eliminación) en un medio alcalino en el
producto de adición, para obtener un producto α,β-insaturado.
______________________________________________________________________________
______________________________________________________________________________
______________________________________________________________________________
__________________________________________________________________________

4. ¿Qué tratamiento se debe dar a los residuos acuosos de la reacción para poder desecharla en el
drenaje?
______________________________________________________________________________
______________________________________________________________________________
______________________________________________________________________________
__________________________________________________________________________

BIBLIOGRAFÍA.

1) C. R. Conard, M. A. Dolliver; Organic Syntheses Coll. Vol. 2, 167, 1943.

2) R. J. W. Cremlyn, R. H. Still; Named and Miscellaneous Reactions in Practical Organic


Chemistry, Heinemann, London, 1967.

3) J. McMurry; Química Orgánica, 8a Ed., Cengage Learning, México, 2012.

407
408
XXIII Olimpiada Nacional de Química
1er Examen Nivel A y B.
Total: 30 preguntas. Tiempo asignado: 75 minutos. Distrito Federal 2014

Anota en el recuadro correspondiente la letra del inciso que contesta correctamente cada pregunta.
No olvides que si consideras que ningún inciso es el correcto deberás anotar una letra X.
ESCRIBE CLARAMENTE LAS LETRAS MAYÚSCULAS PARA EVITAR CONFUSIONES AL
CALIFICAR. SI NO ESCRIBES TUS RESPUESTAS CON TINTA, TU EXAMEN PODRÍA SER
ANULADO.

En Vietnam, la fábrica VINACHEM, vende el fertilizante Lam Thao, cuya composición aproximada
en porciento en masa es: P2O5 (17%), CaO (34%), MgO (20%) y el resto es SiO2.

1) De los cuatro compuestos presentes en este fertilizante, el que tiene una molécula
con el menor porcentaje en masa de oxígeno es:

A) P2O5 B) CaO C) MgO D) SiO2

2) El Lam Thao se vende en costales de 50 kg. ¿Cuántos moles de P2O5 hay en un


costal de este fertilizante?

A) Menos de 25 moles B) Entre 25 y 50 moles


C) Entre 50 y 75 moles D) Más de 75 moles

3) La fórmula molecular del P2O5 es en realidad P4O10; ¿cuál es el % en masa de


fósforo en la molécula de P4O10?

A) Menos de 35% B) Entre 35 y 45%


C) Entre 45 y 55% D) Más de 55%

4) En Química Orgánica se utiliza una disolución de P4O10 en DMSO


(dimetilsulfóxido) para oxidar alcoholes. El grupo funcional que caracteriza a los
alcoholes es:

A) -OH B) -COOH C) -CHO D) -C-O-C-

5) La fórmula del DMSO es (CH3)2SO, su densidad es de 1.1 g/cm3 y es miscible con el


agua cuya densidad es de 1.0 g/cm3. ¿Cuántos litros de DMSO se requieren para
tener los mismos moles de agua que hay en un litro de agua?

A) Menos de 4.1 litros B) Entre 4.1 y 4.5 litros


C) Entre 4.5 y 4.9 litros D) Más de 4.9 litros

409
6) La fórmula del benceno es C6H6 y si se sustituye un hidrógeno por un grupo CH3 se
obtiene el tolueno:
CH3

¿Cuántos moles de tolueno tenemos en 92 gramos de este compuesto?

A) 1 mol B) 0.5 moles C) 2 moles D) 1.18 moles

7) La fórmula del dibenzoilmetano es la que aparece a continuación. Su masa molar es,


en gramos/mol:
O O

A) 188 B) 222 C) 224 D) 226

8) El aminoácido más simple es la glicina. Su fórmula condensada es C2H5NO2 y tiene


una densidad de 1.1607 g/cm3. Por lo tanto en un decímetro cúbico (dm3), ¿cuántos
moles de glicina tenemos?

A) Menos de 14 moles B) Entre 14 y 15 moles


C) Entre 15 y 16 moles D) Más de 16 moles

9) En gramos, una parte por millón (ppm) equivale a un gramo en un millón de gramos.
Si se pueden disolver 2.50 g de glicina en un kilogramo de agua, ¿cuál es la
concentración de glicina en ppm?

A) Menos de 1000 ppm B) Entre 1000 y 1500 ppm


C) Entre 1500 y 2000 ppm D) Más de 2000 ppm

10) La ley de los gases ideales es PV=nRT


donde: (R = 0.082 L atm/mol K), (R = 8.314 J/K mol), (1 J = 1 kg m2 s-2),
(1 atm = 101.325 kPa), (1 Pa = 1 N m-2), (1 N = 1 kg m s-2).
¿En qué condiciones de P y T, un mol de gas ideal ocupa un volumen de 8.0 L?

A) 1 atm y –224.35 oC B) 50.66 kPa y 48.8 K


C) 0.25 atm y –224.35 oC D) 101.324 kPa y 48.8 K

410
11) En un recipiente de 8.0 L de capacidad hay la misma cantidad en gramos de N2 y
O2 gaseosos. La fracción mol de N2 es:

A) No es posible saberlo si no se B) Menor a 0.5


especifican P y T
C) 0.5 D) Mayor a 0.5

12) Si se queman completamente 12 g de carbón que contienen un 0.2% en masa de


azufre, ¿cuál es la máxima cantidad en moles de SO2 que se pueden obtener?

A) Menos de 1x10-3 moles B) Entre 1x10-3 y 1x10-2 moles


C) Entre 0.01 y 0.1 moles D) Más de 0.1 moles

13) Por descomposición del carbonato de calcio se obtienen óxido de calcio y CO2.
¿Cuántos gramos del carbonato se requieren para obtener 22.4 litros de CO2 a una P
de una atmósfera y T de 10 oC?
A) Menos de 95 gramos B) Entre 95 y 105 gramos
C) Entre 105 y 115 gramos D) Más de 115 gramos

14) La constante de Avogadro es 6.0221x1023 y la carga de un electrón es 1.60x10-19


coulombs (C). ¿Cuál es la carga de un mol de electrones?

A) Menos de 80000 C B) Entre 80000 y 90000 C


C) Entre 90000 y 98000 C D) Más de 98000 C

15) Cuando un gramo de agua es adicionado a una mezcla de ácido y agua, la nueva
mezcla es 25% en masa de ácido. Cuando 2 gramos de ácido son adicionados a la
nueva mezcla, la mezcla ahora contiene 50% en masa de ácido. El porciento en
masa de ácido en la mezcla original era:
A) Menos de 35% B) Entre 35 y 38%
C) Entre 38 y 42% D) Más de 42%

16) Un estudiante toma 20.0 mL de una disolución acuosa de HCl 3.0 mol L-1 y le
agrega agua destilada hasta tener 250.0 mL de disolución. El pH de la disolución
preparada es:

A) Menor a 1 B) Entre 1 y 1.5


C) Entre 1.5 y 2.5 D) Mayor a 2.5

17) Un ión Li+ tiene un volumen experimental de 9.98x10-31m3. ¿Cuál es


aproximadamente el diámetro en angstroms (Å) de este ión si lo consideramos
esférico y el volumen de una esfera es igual a 4/3  r3, donde r es el radio de la
esfera.
A) Menor a 0.5 B) Entre 0.5 y 1.0
C) Entre 1.0 y 1.5 D) Mayor a 1.5

411
18) Si se disuelven 14 g de permanganato de sodio en agua suficiente para tener 250
mL de disolución, la concentración molar de este compuesto es:
A) Menor a 0.38 mol L-1 B) Entre 0.38 y 0.42 mol L-1
C) Entre 0.42 y 0. 44 mol L-1 D) Mayor a 0.44 mol L-1

19) El estado de oxidación del manganeso en el permanganato de potasio (KMnO4) es:

A) Mayor a +V B) +V C) +III D) Menor de +III

20) El permanganato de potasio reacciona con hidróxido de potasio y se obtiene el


manganato de potasio (K2MnO4), agua y oxígeno gaseoso. Al balancear la ecuación
se encuentra que por cada mol de permanganato que reacciona, se obtienen de
oxígeno gaseoso:
A) Menos de un mol B) Un mol
C) Dos moles D) Más de dos moles

21) En la reacción de la pregunta anterior, se puede afirmar que los elementos que se
oxidan y reducen son, respectivamente:

A) K, O B) Mn, O C) O, K D) O, Mn

22) Cuando se deposita plata metálica a partir de una disolución que contiene iones
Ag+, utilizando una reacción electroquímica. El enunciado correcto es :

A) La plata metálica se deposita en el ánodo por oxidación de los iones Ag+.


B) La plata metálica se deposita en el ánodo por reducción de los iones Ag+.
C) La plata metálica se deposita en el cátodo por oxidación de los iones Ag+.
D) La plata metálica se deposita en el cátodo por reducción de los iones Ag+.

23) El gas LP consta básicamente de propano (C3H8) y butano (C4H10). Si suponemos


que contiene el 75% en masa de propano, ¿cuántos moles totales habría,
aproximadamente, en un tanque que contiene 25 kg de la mezcla?
A) Menos de 525 moles B) Entre 525 y 540 moles
C) Entre 540 y 550 moles D) Más de 550 moles

24) Si se quema totalmente una mezcla de 100 moles de propano y 50 moles de


butano, de tal manera que sólo se produce CO2 y agua, ¿cuántos moles de CO2 se
producen?

A) 700 moles B) 500 moles C) 300 moles D) 150 moles

412
25) Se disuelve una cierta cantidad “x” de KCl en agua destilada para preparar 500 mL
de la disolución “1”. Se disuelve la mitad de “x” de KCl en agua destilada para
preparar 200 mL de la disolución “2”. Al mezclar volúmenes iguales de las
disoluciones 1 y 2 se obtiene una nueva disolución cuya concentración de KCl es
0.422 mol L-1. ¿Cuántos gramos de KCl hay en la cantidad “x”?
A) Menos de 12 gramos B) Entre 12 y 15 gramos
C) Entre 15.1 y 20 gramos D) Más de 20 gramos

26) La corriente que circula en una celda electroquímica se puede medir en amperes
(A). Un ampere equivale a la carga de un coulomb (C) que pasa cada segundo por
un punto del circuito. Si una corriente de 2 mA circula en una celda electroquímica,
el número de electrones que pasan por un punto del circuito cada segundo son:

A) Menos de 1.2 x 10 16 B) Entre 1.2 x 10 16 y 1.3 x 10 20


C) Entre 1.2 x 10 20 y 1.3 x 10 23 D) Más de 1.2 x 10 23

27) De acuerdo a la expresión “X” → 23490 Th + , podemos concluir que el átomo


radiactivo “X” es: (Las partículas alfa son núcleos de helio)

A) 23491 Pa B) 23191 Pa C) 24494 Pu D) 23892 U

28) Considera que el HCl y el NaOH son electrolitos fuertes. Si preparas una
disolución acuosa al mezclar 40.0 mL de una disolución acuosa de NaOH 0.10 mol
L-1 con 10 mL de otra disolución acuosa de HCl 0.45 mol L-1, ¿cuál será el pH de la
nueva disolución?

A) 1.0 B) 1.5 C) 0.5 D) 2.0

29) ¿Cuál es la solubilidad del hidróxido de cobre (expresada en mol L–1) del Cu(OH)2
y el pH de una disolución saturada de Cu(OH)2 en agua pura?
Datos: Ks del Cu(OH)2 = 2.210–20.

A) 1.810–7 mol L–1 y pH = 7.6 B) 2.8 10–7mol L–1 y pH = 7.3


C) 2.810–7 mol L–1 y pH = 7.6 D) 1.810–7 mol L–1 y pH = 7.3

30) La presencia del ión Fe3+ en una disolución acuosa se detecta por aparición de una
coloración rojo “sangre” cuando se añaden unas gotas de una disolución de algún
tiocianato alcalino. Esta identificación se debe a:

A) La reacción de reducción del Fe3+ por el SCN–.


B) La reacción de complejación del Fe3+ con el SCN–.
C) Un cambio de pH al añadir el tiocianato.
D) La precipitación del Fe(SCN)3.

FIN DEL PRIMER EXAMEN

413
414
XXIII Olimpiada Nacional de Química. 2do Examen Nivel A y B.
Total: 12 problemas cada nivel.
Tiempo asignado: 90 minutos. Distrito Federal 2014

ENCIERRA CON UN CÍRCULO LA RESPUESTA CORRECTA

(Niveles A y B)
1. Una botella de reactivo contiene una disolución acuosa de H2SO4 diluido al 10% en masa (con una
densidad de 1,066 g/cm3). Se toman 500 mL de esa disolución y se diluyen con 500 mL de agua
destilada. ¿Cuál es la molaridad, la concentración de iones H3O+ y el pH de esta última disolución?

Dato: 2ª constante de acidez (Ka2) del ion HSO4− = 1,2·10−2.

Concentración del Concentración de iones pH


ácido H3O+
A 0,54 mol/L 1,08 mol/l 0,03
B 1,08 mol/L 2,16 mol/L 0,41
C 1,08 mol/L 1,08 mol/L 0,03
D 0,54 mol/L 0,61 mol/L 0,21

2. Cuando se mezclan 50 mL de una disolución 10−4 mol/L de ioduro sódico con otros 50 mL de
disolución 10−3 mol/L de nitrato de plomo es cierto que: (Dato: pKs PbI2 = 8,2)

A) Precipita cuantitativamente el Pb2+ como PbI2.

B) No se cumple el producto iónico necesario para que se forme precipitado.

C) Las concentraciones iónicas son: Na+= I− = 5·10−5; y Pb2+ = NO3− = 5·10−4 mol/L.

D) Precipita parcialmente PbI2 y queda Pb(NO3)2 disuelto.

3. ¿Cuál será el pH de la disolución reguladora de pH que se obtiene al disolver en agua destilada


0,100 moles de un ácido débil HA (pKa = 5,0) con 0,05 moles de su base conjugada?

A) 5,0 B) 5,3 C) 4,7 D) 2,5

4. Se tiene una disolución que contiene iones Ag+, Ca2+ , Mn2+ y Cu2+, todos en igual concentración
0,001 mol/L. Si a esta disolución se le añade EDTA sólido para obtener una concentración total
0,002 mol/L de este complejante se obtendrá que:

Datos: Los valores de las constantes de estabilidad (pKc) con EDTA son:
AgY− = 7,3; CaY2− = 10,7; MnY2− = 14,0 y Cu2+ = 18,8.

A) Se obtiene AgY− y CaY2− en concentración 0,001 mol/L cada uno.

B) Únicamente se compleja cuantitativamente el ión Cu2+ y parcialmente el Mn2+.

415
C) Se forman todos los complejos pero ninguna reacción es cuantitativa.

D) Se obtiene CuY2− y MnY2− en concentración 0,001 mol/L cada uno.

********************************************************************************

(Niveles A y B)
5. Existen diversas formas de indicar la concentración de una solución; entre estas se encuentran la
molaridad, la molalidad, el porciento en masa, la fracción mol, las ppm (partes por millón) y una
muy utilizada que es el porciento en masa sobre volumen generalmente expresado en gramos por
100 mL de solución. Con frecuencia tenemos que transformar de una forma de expresión de la
concentración a otra, siendo necesario para algunas el conocimiento de la densidad de la
solución.

Por ejemplo, para el cálculo de la disminución de la temperatura de congelación del agua Tfus,
así como el del aumento de la temperatura de ebullición (Teb), se requiere del conocimiento de
la molalidad de la solución: Tfus = Kc m
Teb = Kb m; donde “m” es la molalidad de la solución, Kc es la constante crioscópica y Kb es la
constante ebulloscópica; en ambas constantes, las unidades son K kg mol – 1.

Por otro lado, para el cálculo de la presión osmótica (), basta con conocer la concentración en
moles por litro de solución.  = RTC, donde R es la constante universal de los gases, T la
temperatura absoluta en K y C es la concentración en mol/L.

Una solución de sacarosa (C12H22O11) contiene 135 g de este compuesto en 315 g de agua a una
temperatura de 20 °C. A esta temperatura la densidad de la solución es de 1.127 g/cm3.

Para el agua Kc = 1.86 K kg mol – 1 y Kb = 0.51 K kg mol – 1. Considere que la densidad del agua
es de 1.0 g/cm3. Contesta las siguientes preguntas:
(Utiliza los cuadros para tus cálculos; encierra con un círculo la respuesta correcta; sin
cálculos la respuesta no se toma en cuenta)

R = 8.314 J/mol K = 8.314 Pa m3/mol K; T = t + 273.15 1 atm = 101325 Pa

5.1. El porciento en masa de sacarosa en la solución es:

A) 13.5 % B) 30.0 % C) 42.9 % D)75.0 %

416
5.2. La molaridad “M” de la solución es:

A) 0.988 mol/L B) 1.25 mol/L C) 2.53 mol/L D) 2.92 mol/L

5.3. La molalidad “m” de la solución es:

A) 0.300 mol/kg B) 0.877 mol/kg C) 0.988 mol/kg D) 1.25 mol/kg

5.4. Las partes por millón (ppm) de sacarosa en agua son:

A) 3.00 x 103 ppm B) 4.29 x 105 ppm C) 3.00 x 106 ppm D) 4.29 x 106 ppm

5.5. La fracción mol de sacarosa en la solución es:

A) 0.0221 B) 0.0224 C) 0.0229 D) 0.0232

417
5.6. Una solución acuosa de “B” que tiene 60% en masa de sacarosa, a 20 °C tiene una densidad de
1.286 g/cm3. La temperatura de fusión de esta solución es:

A) – 1.16 °C B) – 4.36 °C C) – 8.16 °C D) – 9.38 °C

5.7. La presión osmótica de la solución “B” es:

A) 3.339 x 10 5 Pa B) 5.498 x 10 6 Pa C) 7.071 x 10 5 Pa D) 8.945 x 10 6 Pa

********************************************************************************

(Niveles A y B)
6. Indica cuál es la opción correcta con respecto a las siguientes aseveraciones:

I. Isótopos son átomos de un mismo elemento con diferente número de electrones.


II. La masa atómica relativa de un elemento viene dada por su número total de electrones.
III. La masa atómica relativa de un elemento es aproximadamente la suma de la masa de
protones y electrones.
IV. La masa atómica relativa de un elemento es que presenta que presenta igual a la suma de
protones y neutrones.

A) Sólo I y II son falsas. B) I y IV son ciertas.


C) Sólo la IV es cierta. D) Ninguna es cierta.

7. En el diagrama de la tabla periódica que se presenta a continuación se indican algunos elementos


cuyas letras no corresponden a las de sus símbolos. Con respecto a estos elementos, el compuesto
que presenta el mayor carácter iónico está formado por:

418
A) A y E B) C y E C) D y F D) D y E

8. El isótopo 42K tiene un periodo de semi-desintegración de 12 horas. ¿Cuál es la fracción de la


concentración inicial de dicho isótopo, que queda después de 48 horas?

A) 1/16 B) 1/8 C) 1/4 D) 1/2

9. Indica cuál es la opción correcta:

A) Los hipocloritos son sales del ácido hipocloroso y ejemplos de estas sales son NaClO2,
KClO2, CaClO2.

B) Los cloratos son sales del ácido clórico y ejemplos de estas sales son NaClO3, KClO3,
Ca(ClO3)2.

C) Los percloratos son sales del ácido perclórico y ejemplos de estas sales son NaClO, KClO,
Ca(ClO)2.

D) Los cloritos son sales del ácido cloroso y ejemplos de estas sales son NaClO2, KClO2,
CaClO2.

10. Señala la opción en la que se encuentren los coeficientes estequiométricos de los compuestos
que permiten balancear correctamente la siguiente ecuación:

PH3 + KMnO4 + H2SO4 → K2SO4 + MnSO4 + P4 + H2O

A) Fosfina 4; fósforo 1

B) Fosfina 5; permanganato de potasio 6

C) Fosfina 20; permanganato de potasio12

D) Fosfina 20; sulfato de manganeso 6

(Sólo nivel B)
11. Si con dos litros de pintura, aplicaras con una brocha una capa de pintura de espesor constante e
igual a 100 micras de espesor, ¿cuál sería la superficie en metros cuadrados que podrías pintar?
Una micra = 10−6 metros

A) 10 m2 B) 12 m2 C) 20 m2 D) 24 m2

419
(Sólo Nivel B)
12. Un kilogramo de osmio metálico ocupa un volumen de 44.5 cm3. ¿Cuál es la densidad de un
mol de este elemento?

A) Menos de 20 g cm−3 B) Entre 20 g cm−3 y 180 g cm−3

C) Entre 180 g cm−3 y 4000 g cm−3 D) Más de 4000 g cm−3

********************************************************************************

(Sólo nivel A)
11. Indica las estructuras del producto de cada una de las siguientes reacciones:
HBr, Et2O
A

HBr, Et2O
B
O
O Ph
Ph O
O

Br2, CCl4
C
Br2, H2O
D
H2SO4, H2O
E
CALOR

1) BH3
F
2) NaOH, H2O2

1) O3, CH2Cl2, -78 oC


G + H
2) (CH3)2S
RESPUESTAS
A B C D

E F G+H

420
(Sólo nivel A)
12. Escribe las estructuras de los intermediarios y del producto final de la siguiente secuencia
sintética:

O 1) SOCl2 O
CALOR H2N NH2 Cl
OH A B C
2) i-Pr2CuLi KOH AlCl3 ( C12H16O)
OH
HO
CALOR

RESPUESTAS

A B C

FIN DEL EXAMEN

421
422
XXIII OLIMPIADA NACIONAL DE QUÍMICA
TERCER EXAMEN Tiempo asignado: 180 minutos Distrito Federal 2014
NIVEL A: 9 problemas; NIVEL B: 6 problemas

1. Se le encargó a un estudiante de la Facultad de Química de la UNAM que analizará una aleación


de plomo y plata; para lograrlo disolvió una muestra de 0,2500 g de la aleación en 50 mL de
ácido nítrico de concentración 5 mol/L y trató la disolución resultante con yodato potásico en
exceso. El precipitado seco, que obtuvo después de filtrar y lavar, tuvo una masa de 0,6605 g. En
el líquido filtrado realizó una prueba cualitativa para asegurar que ambos cationes habían
precipitado cuantitativamente.

Datos:Valores de Eo V/ENH HNO3/NO = 0,96; Ag+/Ago = 0,80; Pb2+/Pb = -0,126

Masas molares de AgIO3 = 282,9 g/mol; Pb(IO3)2 = 557 g/mol

pKs AgIO3 = 7,5; pKs Pb(IO3)2 = 7,5

1.1. Escribe las ecuaciones de las reacciones que ocurren al disolver la aleación en HNO3.

1.2. Indica la ecuación que representa el tratamiento de la disolución con KIO3.

1.3. Indica qué prueba cualitativa realizó el estudiante para demostrar que ambos cationes habían
precipitado cuantitativamente.

1.4. Indica qué porcentaje plomo y plata contenía la aleación.


Cálculos:

423
2. Mónica es una estudiante de la Olimpiada Nacional de Química que necesitaba determinar el
contenido de Na2CO3 y NaHCO3 en una muestra pero que sólo contaba con poco material de
vidrio, papel pH y una balanza. Logró su objetivo con buenos resultados con el siguiente
procedimiento.
Midió 0,500 g de la mezcla a la que añadió ácido clorhídrico; observó el desprendimiento de un
gas y, cuando la reacción cesó, evaporó hasta sequedad para eliminar agua y el exceso del ácido
clorhídrico. Volvió a medir el residuo sólido obteniendo una masa de 0,450 g. Este sólido lo
disolvió en 100 mL de agua y midió el pH de la disolución obtenida.

2.1. Escribe las reacciones balanceadas entre el carbonato de sodio y el bicarbonato ácido de sodio
con el ácido clorhídrico.

2.2. Indica que pH tenía la disolución del residuo sólido.

2.3. Calcula el porcentaje en masa de cada uno de los componentes de la mezcla.

2.4. ¿Qué pH hubiera tenido una disolución inicial de los 0,500 g de la mezcla en 100 mL si el
porcentaje de cada uno de ellos hubiera sido de 50% en masa? Datos: pKa (H2CO3) = 6,4 y 10,3.

Un profesor le indicó que el procedimiento usual para determinar la composición de una mezcla
de Na2CO3 y NaHCO3 es registrar los valores del pH, que se obtienen con un pHmetro, al
efectuar una titulación con HCl de concentración conocida; añadió que este procedimiento le
permitiría determinarlos en una mezcla que contuviera otro compuesto sin propiedades ácido-
base. Le informó que se podrían registrar dos volúmenes que corresponderían a: a) el primero a la
neutralización parcial del carbonato de sodio y b) el segundo a la neutralización total del
carbonato y el bicarbonato de sodio.

424
2.5. ¿Qué volúmenes de HCl 0,100 mol/L se hubieran gastado si se hubiera partido de 100 mg de
una mezcla que contuviera 45% de Na2CO3, 35% de NaHCO3 y 20% de un compuesto neutro?
a) En la neutralización parcial del carbonato de sodio:

b) En la neutralización total del carbonato y el bicarbonato de sodio:

2.6. Para completar la información el profesor le comentó que si no tenía pHmetro podría realizar
dos titulaciones con dos indicadores diferentes y le dio una lista de varios para que ella decidiera
cuáles serían los mejores para encontrar los dos volúmenes mencionados.
Para el primer volumen:

Para el segundo volumen:

Datos de intervalo de vire de los indicadores:


Fenolftaleína: 8 a 10; Rojo de metilo: 4,4 a 6,2; Rojo fenol 6,4 a 8,0; Azul nilo 10,1 a 11,1.

■■■■■■■■■■■■■■■■■■■■■■■■■■■■■■■■■■■■■■■■■■■■■■■■■■■■■■■■■■■■■■■■■■

3. Debido a los cambios climáticos la temperatura ambiente, en época de invierno, ha descendido


notablemente en muchos lugares alcanzando cifras récord por debajo de la temperatura de
congelación del agua (0 °C). Esto ha provocado el congelamiento del agua en tuberías, en
depósitos de agua, lagos, ríos y junto con éstos las cataratas del Niágara.

En algunos lugares se cuenta con calefacción para contender a las bajas temperaturas y en otros
se ha recurrido al calentamiento (muy riesgoso) de las habitaciones utilizando carbón, el cual
tiene un calor de combustión de  393505.2 J/mol.

Un método ingenioso para evitar que la temperatura de los graneros descienda por debajo de 0 °C
es distribuir cubetas de 10 litros con agua de tal manera que al bajar la temperatura al agua se
enfría pero al llegar a su temperatura de fusión se mantiene constante mientras se congela. La
capacidad térmica molar del agua es de 75.3 J/mol, en tanto que su calor latente de fusión (fusión)
es de 6025 J/mol.

Para combatir el frío se recomienda la ingestión de alimentos de alto contenido energético, por
ejemplo los que contienen azúcares y grasas como en los chocolates, dulces, galletas y pasteles.
En la actualidad los alimentos empacados deben indicar el contenido energético en el empaque.

425
Por ejemplo una barra de “granola” de 21 g provee 397.5 kJ al consumirla, en tanto que una
barra de 100 g de chocolate (85% cacao) provee 2664 kJ.

(Utiliza los cuadros para tus cálculos y marca con X la respuesta correcta).
R = 8.314 J/mol K = 8.314 Pa m3/mol K; T = t + 273.15; 1 atm = 101325 Pa
(1 Cal = 1000 calorías = 1 kcal; 1 cal = 4.184 J)

Considera la presión barométrica de una atmósfera. La capacidad térmica del aire es 1.012 J/g °C
y su masa molar es de 29 g/mol.

Con base en la información anterior contesta las preguntas siguientes.

3.1. La cantidad de aire en kilogramos que hay en una habitación (que mide 4 m x 4m x 3 m) a –5
°C es:

A) 2.182 kg B) 63.27 kg C) 75.23 kg D) 196.2 kg

3.2. La cantidad de calor que se requiere por kilogramo de aire para calentar la habitación de una
temperatura de – 5 °C a 20 °C es:

A) 14822 J B) 19763 J C) 25300 J D) 24703 J

3.3. La cantidad de carbón que se requiere para calentar un metro cúbico de aire de una temperatura
de – 5 °C a 20 °C es:

A) 1.0169 g B) 1.1307 g C) 1.1864 g D) 1.6948 g

426
3.4. Un granero pierde por las noches alrededor de 16 736 kJ. Considerando que la temperatura del
agua sea de 20 °C, ¿cuántas cubetas de agua de 10 L se requieren para que la temperatura no
descienda por debajo de 0 °C? (Considere que la densidad del agua es 1.0 g/cm3 y que la
capacidad térmica específica del agua es de 4.184 J/g °C).

A) 0.5 cubetas B) 4 cubetas C)5 cubetas D) 20 cubetas

3.5. (Sólo Nivel B). La ingesta diaria recomendada para mantener el peso es de 2000 Cal (1 Cal = 1
kcal). Una galleta de “granola” de 25 g cubierta de chocolate, contiene 25% de chocolate. Si una
persona se alimentara sólo con estas galletas, ¿cuántas galletas debería comer al día para
mantener su peso?

A) 48 galletas B) 12 galletas C) 21 galletas D) 16 galletas

■■■■■■■■■■■■■■■■■■■■■■■■■■■■■■■■■■■■■■■■■■■■■■■■■■■■■■■■■■■■■■■■■■

427
4. Todos los óxidos de nitrógeno son tóxicos, pero sólo uno de ellos se produce a partir de sus
elementos durante una tormenta eléctrica o por la combustión del combustible de un motor. Para
reducir la cantidad de contaminación emitida al aire los automóviles deben contener un
catalizador; un coche sin catalizador emite en la atmósfera, en promedio, 1 mol de dicho óxido
por cada10 km recorridos.

4.1. Escribe todas las posibles fórmulas de los óxidos de nitrógeno e indica el número de oxidación
del nitrógeno en cada uno de ellos.

4.2. ¿Cuál es la masa molar del óxido que se produce considerando que cada molécula del óxido
formado tiene una masa de 4,99•10-23g?

4.3. ¿Cuál es la cantidad promedio (en gramos) de este óxido, que emite en la atmósfera un coche
sin catalizador al recorrer toda la Avenida de los Insurgentes en la Ciudad de México (28,8 km)?
Cálculos:

4.4. ¿Esta cantidad excede la que estará permitida por la Norma Euro-6 estándar a partir de
septiembre de 2014 (0,08 g/ km)? Justifica tu respuesta.
Cálculos:

Justificación:

5. Un estudiante de la Olimpiada Nacional de Química realizó el siguiente experimento:


Tomó 185 mL de una disolución de HCl (pureza 15% m/m y densidad 1,081 g/mL) y les añadió
un trozo de carbonato de calcio. Observó que la disolución parecía hervir por el desprendimiento
de gas; cuando la reacción terminó (todo el sólido había desaparecido) se dió cuenta de que aún
quedaba ácido clorhídrico presente en la disolución, el cual neutralizó con una disolución de
hidróxido de potasio, encontrando que la masa de ácido clorhídrico constituía el 5,5% de esta
disolución resultante.

428
5.1. Escribe la reacción que se produce entre el ácido clorhídrico y el carbonato de calcio.

5.2. Calcula la masa de ácido clorhídrico presente en la disolución inicial.

5.3. Calcula la masa de la disolución al terminar la reacción.

5.4. Calcula la masa del trozo de carbonato de calcio añadido. En caso de que no hayas podido
calcular la masa de la disolución en el inciso anterior (5.3) considera que este valor fuera 212 g.

429
5.5. ¿Cuántos mililitros de una disolución de KOH 4,5 mol L-1 se requirieron para neutralizar el
ácido clorhídrico (al 5,5% m/m) que quedaba en la disolución inicial?

■■■■■■■■■■■■■■■■■■■■■■■■■■■■■■■■■■■■■■■■■■■■■■■■■■■■■■■■■■■■■■■■■■

6. (Sólo Nivel B). En este problema tendrás que acomodar nueve de los primeros 29 elementos de la
Tabla Periódica en el diagrama que se muestra a continuación. Toma en cuenta los datos que se te
proporcionan y escribe en cada recuadro el símbolo del elemento que le corresponde:

1 2 3

4 5 6

7 8 9

Los primeros 29 elementos son:

H He Li Be B C N O F Ne

Na Mg Al Si P S Cl Ar K Ca

Sc Ti V Cr Mn Fe Co Ni Cu

Muy importante: cuando termines de acomodar los elementos notarás que el número
atómico siempre va en aumento desde la casilla 1 hasta la 9. Debes escribir el nombre y el
símbolo que corresponde a cada casilla.

Datos:
1.- De la molécula de la sal común, sólo debes incluir uno de los elementos que la constituyen y
colocarlo en la columna derecha.

430
2.- Si multiplicas los números atómicos de dos de los elementos incluidos, el resultado es mayor
a 800.
3.- Sólo debes incluir un metal alcalino y colocarlo en la columna central.
4.- En la casilla central debes colocar el elemento cuyo óxido tiene una masa molar aproximada
de 101.96 g/mol.
5.- En un kilogramo de este elemento, que se ubica en una esquina, hay casi 21 moles.
6.- En otra esquina está un elemento cuya molécula es diatómica. Si pones la misma cantidad en
gramos de esta molécula y de monóxido de carbono, su fracción mol será 0.5.
7.- No debes incluir ningún elemento que esté presente en la molécula de benceno.
8.- Además, tienes que incluir dos gases nobles.

FIN DEL TERCER EXAMEN NIVEL B

■■■■■■■■■■■■■■■■■■■■■■■■■■■■■■■■■■■■■■■■■■■■■■■■■■■■■■■■■■■■■■■■■■

6. (Sólo Nivel A). El antraceno y muchos otros compuestos aromáticos policíclicos se han
sintetizado por medio de una reacción de ciclación llamada reacción de Bradsher o
ciclodeshidratación aromática. Este método, propuesto por C. K. Bradsher, de la Universidad de
Duke, se ilustra a través de las siguientes reacciones. Escribe las estructuras de los intermediarios
A al C.
O

OH Cl

A B C
+
H2SO4,  (C13H12)
AlCl3,  (C20H16O) (C20H16O)

REACCIÒN
DE BRADSHER HBr, AcOH, CALOR

9-FENILANTRACENO
RESPUESTAS
A B C

431
7. (Sólo Nivel A). Escribe las estructuras de los intermediarios y del producto final de la siguiente
secuencia sintética:

1) KOH
H2O, EtOH o
HO Br CALOR 120 C
EtO2C CO2Et A B C
EtO:-+Na (C10H18O5) 2) HCl, H2O
EtOH
SO3H
(cat.)

CALOR
1) LiAlH4
E D
2) H2O (C5H8O2)

RESPUESTAS

A B C D

432
8. (Sólo Nivel A). Completa el siguiente esquema, escribiendo las estructuras de los productos A al
G:

O O
O
H +
H
H H
1) O3, CH2Cl2, - 78 oC O 1) O3, CH2Cl2, - 78 oC
2) (CH3)2S 2) (CH3)2S
A B
(C7H10) (C7H10)

NC CN NC CN
, CALOR , CALOR
NC CN NC CN

C NO HAY REACCIÓN
(C13H10N4)

B + HCl D + E + F + G

RESPUESTAS

A B C D

E F G

433
9. (Sólo Nivel A). Escribe las estructuras de los intermediarios y del producto final de la siguiente
secuencia sintética:

H3CO
OH H3CO
a, b c, d 1) e
A
(C7H8O) 2) f OH
O
POCl3
N

1) O3, CH2Cl2
C + D B
(C14H12O2) (C8H8O) 2) (CH3)2S (C22H20O)
Br2, AlCl3
CALOR

E
(C14H11BrO2)

RESPUESTAS
a b c d e

f A B

C D E

FIN DEL TERCER EXAMEN NIVEL A

434
 

XXIII OLIMPIADA NACIONAL DE QUÍMICA


EXAMEN INTERNACIONAL
16 PROBLEMAS Tiempo: 4 HORAS Distrito Federal 2014

Siempre que se te pidan cálculos, indícalos claramente dentro de los espacios asignados;
respuestas sin cálculos pueden no ser tomadas en cuenta.

********************************************************************************

Problema 1. DESTILACIÓN.

La separación de dos sustancias volátiles por destilación está basada en la diferencia de presiones de
vapor de los compuestos puros. Esto hace que los vapores generados a partir de una solución de los
dos componentes volátiles tenga una composición (fracción mol) en el vapor, mayor en el
componente más volátil que la que tenía el líquido originalmente. Debido a las interacciones entre
las moléculas de ambos compuestos, la presión de vapor de cada componente se ve afectada por la
presencia del otro. Si la solución se comporta idealmente, la presión de vapor de cada componente
(Pi) se puede calcular a partir de la ley de Raoult: Pi = Xi Pi*, donde Pi es la presión de vapor del
componente (i), Pi* la presión de vapor del componente puro y Xi su fracción mol en el líquido. La
presión total sobre la solución (P) está dada por la suma de las presiones de ambos compuestos:

P = PA + PB = XA PA* + XB PB*; como XA + XB = 1 sustituyendo XA se tiene

P = PA* + (PB*  PA*) XB; (línea de composición del líquido, Fig. 1)

La composición en el vapor (Yi) se puede calcular a partir de: Yi = Pi/P = Xi Pi*/P. En el vapor
también la suma de fracciones mol es igual a 1: YA + YB = 1

La presión total sobre la solución (P) se puede calcular también a partir de la fracción mol en el
vapor:

1/P = YA/PA* + YB/PB* que representa a la línea de composición del vapor (Fig. 1)

435
 

Al reducir la presión sobre una solución se


alcanza el punto de burbuja donde se genera
la primera muestra de vapor el cual tiene una
composición mayor (YB) del componente más
volátil que la que había en el líquido.
T = constante
P Si se reduce aún más la presión se alcanza el
Punto punto de rocío donde queda la última gota de
Líquido de líquido con una composición menor (XB) del
burbuja
componente más volátil.
Composición de En este punto como se ha evaporado
Punto
líquido L +V de rocío prácticamente todo el líquido, la composición
del vapor es prácticamente igual a la que tenía
el líquido originalmente.
Composición
de vapor
Si la Presión total se encuentra en un punto
XB YB intermedio entre la que corresponde al punto
Vapor de burbuja y la del punto de rocío, sólo se
evapora parcialmente la mezcla y las
0 XB, YB 1 composiciones del líquido y del vapor
cambian dependiendo de la presión total a la
que se encuentre la solución.

“A” y “B” son compuestos volátiles. Se tiene una mezcla de 3 mol de “A” y 7 mol de “B”. Las
presiones de vapor de los compuestos puros son PA* = 150 mmHg y PB* = 350 mmHg. La solución
se comporta idealmente. Calcula y marca con X la respuesta correcta en todos los incisos de este
problema.

1.1. La presión a la que se forma la primera muestra de vapor (punto de burbuja).


Cálculos:

a) 210 mmHg b) 245 mmHg c) 270 mmHg d) 290 mmHg

1.2. La composición del vapor formado (YB).


Cálculos:

a) 0.155 b) 0.184 c) 0.375 d) 0.845

436
 

1.3. La presión a la que desaparece la última gota de líquido (punto de rocío).


Cálculos:

a) 180 mmHg b) 210 mmHg c) 250 mmHg d) 240 mmHg

1.4. La composición de la última gota de líquido XB.


Cálculos:

a) 0.68 b) 0.50 c) 0.42 d) 0.21

Si la presión sobre el líquido se reduce a 270 mmHg, el líquido sólo se evapora parcialmente,
teniendo un número de moles en el líquido n(líq) y un número de moles en el vapor n(vap). Si sólo se
ha evaporado 5.625 mol de la mezcla, calcula:

1.5. La composición del líquido a esta presión.


Cálculos:

a) 0.80 b) 0.77 c) 0.60 d) 0.34

1.6. La composición del vapor a esta presión.


Cálculos:

a) 0.78 b) 0.60 c) 0.54 d) 0.37

437
 
1.7. El número de moles de “B” en el líquido.
Cálculos:

a) 1.175 mol b) 2.625 mol c) 4.375 mol d) 5.125 mol

1.8. El número de moles de “B” en el vapor.


Cálculos:

a) 1.175 mol b) 2.625 mol c) 4.375 mol d) 5.125 mol

Problema 2.

En experimentos de fisicoquímica de superficies es común poner las muestras a estudiar, ya sea en


atmósferas inertes donde se usa nitrógeno gaseoso o bien en condiciones de ultra alto vacío donde
las presiones en el interior del recipiente son del orden de 10 -7 pascales.
Una atmósfera equivale a 760 mmHg o a 1.01325 x105 pascales. 1 bar = 105 pascales. 1 m3 = 1000
L. Número de Avogadro: 6.023 x 1023.

Supondremos que los gases se comportan de acuerdo al modelo ideal PV = nRT;


R = 8.314 J/mol K = 8.314 Pa m3/mol K

2.1. Si en un recipiente de un litro de capacidad colocamos un gramo de nitrógeno gaseoso a una


temperatura de 25 0C, calcula la presión en milímetros de mercurio en el interior del recipiente y
el número de moles de nitrógeno contenido en el recipiente.
Cálculos:

P en mmHg: ____________ Moles de N2: _______________________

438
 
2.2. Si ahora deseamos extraer parte del N2 contenido en el recipiente anterior para llevar la presión
interna hasta un valor de ultra alto vacío de 10-7 pascales, calcula los moles de N2 que quedan en
el interior del recipiente. Considera además que el recipiente se enfría a una temperatura de
– 50 0C.
Cálculos:

Moles de N2 dentro del recipiente: ___________

2.3. En otro experimento, se tiene un recipiente de un litro de capacidad a -50 0C y lleno de N2 a una
presión de 10-4 mmHg. Manteniendo la presión y la temperatura constantes, se introduce una
pieza metálica con una superficie de 2.0 cm2. En este metal, las moléculas de N2 se pueden
adsorber, esto es, como pegarse a la superficie; si una molécula se adsorbe, podemos suponer que
ya no ejerce presión dentro del recipiente y por lo tanto la presión debe disminuir cuando se
alcanza un equilibrio entre la pieza metálica introducida en el recipiente y el gas contenido en el
recipiente. El área que ocupa una molécula de N2 adsorbida es de 0.162 nm2 (nanómetros
cuadrados). Suponiendo que la mitad de las moléculas de N2 inicialmente presentes logran
adsorberse, calcula el porcentaje del área del metal, en cm2, que recubrirían suponiendo que se
forma una película de una molécula de espesor, esto es, una monocapa. Considera despreciable el
volumen de la pieza metálica.
Cálculos:

Marca con una X la respuesta correcta:


A) Se alcanza a recubrir la totalidad de la superficie metálica.
B) Sólo se cubre entre 50 y 60% de la superficie metálica.
C) Sólo se cubre entre 25 y 35% de la superficie metálica.
D) Se recubre menos de un 20% de la superficie metálica.
E) Ninguna de estas respuestas concuerda con mi resultado.

********************************************************************************

439
 
Problema 3. Complete la siguiente secuencia sintética:

O
OH POCl3 NH3 POCl3 1) LiAlH4
A B C D
CALOR 130o C 2) H2O (C7H9N)

RESPUESTAS:

A B C D

Problema 4. Complete la siguiente secuencia sintética:

O O
1) NH2OH 1) LiAlH4 1)
A C
B (C9H19N)
2) H2SO4 (C6H11NO) 2) H2O AcOH
120o C 2) H2, Pd(C)

RESPUESTAS:

A B C

Problema 5. La ambretólida se obtiene a partir del hibisco y tiene un olor a almizcle. La


ambretólida sintética se obtiene a partir del compuesto A. Escriba las estructuras, ignorando la
estereoquímica, para los compuestos B al G. (En el paso 4, recuerde que el zinc se utiliza para
convertir un dibromoalcano vecinal en un alqueno).

440
 

O O O
HCl, H2O
B HBr C
HO OH
A CALOR (C16H32O5) (C16H29Br3O2)

EtOH
H2SO4 (cat.)
CALOR

F AcO:-+Na E Zn, EtOH D


(C20H36O4) AcOH (C18H33BrO2) CALOR (C18H33Br3O2)

1) KOH, EtOH, CALOR


2) HCl, H2O

CALOR
G
(C16H30O3) O O
(C16H28O2)

RESPUESTAS:

Problema 6. Estimados participantes de la Olimpiada Nacional de Química 2014, bienvenidos al


Distrito Federal, territorio Chilango. El diccionario de la Real Academia Española en su edición
2009, así como de la Academia Mexicana de la Lengua, definen a la palabra chilango como un
adjetivo de uso coloquial que también se puede usar como sustantivo de natural para referirse a algo,
a alguien perteneciente o relativo al Distrito Federal. Se dice también que el término fue utilizado
por primera vez en Veracruz. En tiempos pasados, la mayoría de los delincuentes condenados eran
enviados al Distrito Federal para concentrarlos y posteriormente enviarlos a la cárcel de San Juan de
Ulúa en Veracruz. Al llegar al puerto, los presos eran atados de manos y formados en hilera en

441
 
forma similar, decían ellos, a una chilanga, conocida en ese entonces, como un atado de chiles. De
ahí que el chilango se asocie con aquellos "delincuentes" provenientes del Distrito Federal. Nos
hemos referido al atado de chiles, y dentro de ellos se encuentra la capsaicina, la cual es un
componente activo de los pimientos picantes (Capsicum). Es irritante para los mamíferos; produce
una fuerte sensación de ardor (pungencia) en la boca. La siguiente secuencia sintética fue usada en
1955 para llevar a cabo la síntesis de la capsaicina. Escriba las estructuras de los intermediarios y de
la capsaicina:

PBr3 A 1) NaCH(CO2Et)2 B
OH
(C8H15Br) 2) KOH, H2O, CALOR (C11H18O4)
3) HCl, H2O
CALOR
H3CO
(160 - 180 oC)
HO CH2NH2 SOCl2
CAPSAICINA D C
(C18H27NO3) (C10H17ClO) CALOR (C10H18O2)
N

RESPUESTAS:

********************************************************************************

442
 
Problema 7. Un nuevo elemento químico ha sido descubierto por la sonda espacial “voyager” en
una de las lunas de Júpiter. De este elemento fue posible caracterizar varios de sus isótopos, algunos
de los cuales son radioactivos:

Número de masa: 428 432 433 438 438

Tiempo de vida media: 10 ms estable 210 horas 3x1014 años 15 min

Abundancia: ---- 75% ----- 25% -----

Masa relativa: 427.9874 431.9798 432.9583 437.9221 437.9572

7.1. ¿Cuál es la masa atómica que le corresponde a este nuevo elemento?


Cálculos:

Masa atómica del nuevo elemento: ________________

7.2. El isótopo de número de masa 438 sufre un decaimiento muy inusual pues éste se fragmenta en
tres partes: E → 140Ba + 152Cs + 146Xe
Con esta información menciona, ¿cuál es el número atómico que presenta este nuevo elemento?
Cálculos:

Número atómico del nuevo elemento: ________________

Este elemento al hacerlo reaccionar con oxígeno a altas temperaturas (por encima de los 700 °C) se
obtiene un óxido de fórmula E2O5. No se detectó la presencia de ningún otro óxido cuyo contenido
de oxígeno fuese mayor.

7.3. ¿A qué familia periódica de los elementos representativos debe pertenecer este elemento?
______________

7.4. ¿Cuáles son los estados de oxidación que esperarías presente este elemento? _______________

7.5. De entre éstos, ¿cuál esperarías que fuese el más común? _________

7.6. ¿Qué comportamiento químico esperarías observar en este elemento?


El de un metal _____ El de un no metal _____ (MARCA EL CORRECTO)

443
 

Problema 8. De entre los siguientes procesos selecciona aquel que sea más exotérmico (pon una X):

a) Na (g) → Na+ (g) + e-


b) P (g) + e- → P- (g)
c) O- (g) + e- → O2- (g)
d) Cl (g) + e- → Cl- (g)
e) Mg+ (g) → Mg2+ (g) + e-

Problema 9. Una sustancia desconocida que contiene los siguientes elementos químicos: Ni, C, N,
H y Cl presenta las siguientes características:

La combustión completa de 250 mg de esta sustancia produce 153.75 mL de dióxido de carbono


(este volumen se midió en la Ciudad de México donde la temperatura y la presión eran
respectivamente de 25 °C y 585 mmHg).

Durante la combustión también se genera vapor de agua el cual se colectó en una trampa de
hidróxido de sodio. El incremento en la masa de la trampa fue de 174.24 mg.

Al disolver 0.500 g de esta sustancia en agua se observa que ésta conduce la electricidad de una
forma eficiente. Al hacer reaccionar esta disolución con un exceso de nitrato de plata fue posible
obtener 462.48 mg de cloruro de plata.

Finalmente, el análisis por absorción atómica mostró que el contenido de níquel es de 18.93%.

Con esta información Completa la siguiente información:

Moles de C contenidos en 250 mg de sustancia: ________________. %C: __________.

Moles de H contenidos en 250 mg de sustancia: ________________. %H: __________.

Moles de Cl contenidos en 500 mg de sustancia: _______________. %Cl: __________.

Fórmula empírica de esta sustancia: _______________________.

Problema 10. Las estructuras de Lewis son una importante herramienta en la predicción de la
geometría molecular y consecuentemente, cuando se generan correctamente permiten predecir la
presencia de momentos dipolares. Genera las estructuras de Lewis para las siguientes especies: SF4,
POCl3 y NO3-.

444
 

En alguna(s) de estas estructuras, el átomo central presenta lo que se conoce como “expansión del
octeto de Lewis”. Indica con una pequeña X, en cuál(es) se observa este fenómeno.

Problema 11. Para cada óxido, clasifícalo como ácido (A), básico (B) o anfótero (AN): (Coloca la
letra que corresponde a un lado de la fórmula)

Li2O SO3 MgO P2O5 Al2O3

Cl2O CrO3 Mn2O7 Fe2O3 CO2

Problema 12. El comportamiento magnético de dos distintos compuestos de coordinación de


cobalto(II) es significativamente distinto pues mientras en uno de ellos el paramagnetismo es muy
intenso en el otro este fenómeno es sustancialmente menor. Asigna en cada caso la geometría del
centro de coordinación, el diagrama de desdoblamiento del campo cristalino e indica el número de
electrones desapareados que presenta cada centro metálico. Los compuestos de coordinación son el
hexacianocobaltato de potasio y el tetraclorocobaltato de potasio: K4[Co(CN)6] y K2[CoCl4].

K4[Co(CN)6] K2[CoCl4]

Problema 13. El número de Avogadro es una magnitud relacionada al sistema internacional de


unidades, pues el número de partículas que contiene un mol lo definimos empleando la referencia de
la masa del 12C. ¿Cuál sería la magnitud del número de Avogadro si en lugar de gramos se
emplearan onzas para medir la masa de las sustancias? (1 onza = 28.3495 gramos)

Cálculos:

Nuevo número de Avogadro (base onzas): _____________________________

445
 
Problema 14. La siguiente figura ilustra la celda unitaria de la estructura del cloruro de sodio.
¿Cuántos iones sodio y cuántos iones cloro están contenidos en esta celda? (Considera que, por
ejemplo, si un átomo es compartido por dos celdas este átomo sólo contribuye con ½ a la cuenta de
átomos en esa celda unitaria).

Número de iones sodio _______

Número de iones cloruro _______

********************************************************************************

446
 
Problema 15. La extracción es un procedimiento de purificación frecuentemente utilizado en
síntesis orgánica y se basa en las diferencias de solubilidad de un soluto en agua y en otro disolvente
inmiscible (generalmente un disolvente orgánico poco polar como éter o hexano). Durante la
extracción una disolución que contiene el soluto se pone en contacto con otro líquido y, después de
agitar vigorosamente se deja que las dos capas de líquidos se separen. Como consecuencia el soluto
se distribuye y se obtienen concentraciones diferentes en cada disolvente. El coeficiente de partición
o distribución KD de una sustancia, también llamado coeficiente de distribución (D), o coeficiente de
partición (P), es el cociente o razón entre las concentraciones de esa sustancia en la fase orgánica y
la acuosa. El coeficiente de distribución del ácido oxálico en éter es igual a:

KD = 1,78

es decir que el ácido oxálico es 1,78 veces más soluble en éter que en agua.

15.1. Calcula la masa de ácido oxálico que se disuelve en la capa orgánica cuando se añaden 300
mL de éter a 300 mL de una disolución de 10 g de ácido oxálico en agua. Calcula el
rendimiento de la extracción (% de ácido extraído al éter).
Cálculos:

Masa: ____________ Rendimiento: _________

15.2. Calcula el rendimiento de la extracción cuando se realiza la extracción de los 10 g de ácido


oxálico (disueltos en los 300 mL de agua) mediante tres extracciones sucesivas con éter de
100 mL cada una.

Rendimiento: _________

447
 
15.3. ¿Qué conclusión se deduce de este cálculo?

Problema 16. La resonancia magnética nuclear (RMN) es una poderosa técnica que es ampliamente
utilizada para elucidar la estructura de compuestos orgánicos. Los espectros que se obtienen son
relativamente fáciles de interpretar y dependen de los átomos de algunos elementos (como H y C)
que dan señales diferentes cuyo número depende del tipo de átomos y de los átomos vecinos.
El número de señales presentes en un espectro de RMN se corresponde con el número de tipos de
hidrógeno distintos existentes en la molécula, entendiendo por “tipos de hidrógeno” aquellos en los
que no hay diferencias debido a la naturaleza de este elemento (H) ni tampoco por los alrededores
que puede percibir este hidrógeno por su situación concreta en una molécula.
Por ejemplo, en el propano CH3CH2CH3 existen dos tipos de hidrógenos diferentes que darán lugar
a un espectro con dos señales en posiciones diferentes (los de los grupos CH3 y los del grupo CH2).
Adicionalmente es frecuente encontrar que la señal contenga múltiples picos, con una proporción
definida, que representen a un solo tipo de hidrógeno; este efecto de multiplicidad se debe al efecto
magnético que ejercen los núcleos de hidrógeno contiguos. En los espectros de resonancia de
hidrógeno el número de picos del hidrógeno unido a un carbón es igual al número de hidrógenos
unidos al carbono vecino más uno. Así, en el propano la señal de los hidrógenos del grupo CH2
tendrá una señal con 6+1 picos debido a los 6 hidrógenos de los grupos CH3; en cambio la señal que
corresponde a los protones de los grupos CH3 sólo tendrá tres picos (2+1).
En la siguiente molécula se tienen cuatro tipos de hidrógenos diferentes y, en consecuencia, el
espectro muestra cuatro señales con multiplicidad diferente en el espectro:
(2 picos para los seis protones de los carbonos 1; 7 picos para el protón del carbono 2; cuatro para
los dos protones del carbono 3; y tres picos para el del carbono 4).
La altura total de cada señal (intensidad) corresponde al número de protones.

448
 
16.1. Marca con un número (como en el ejemplo anterior), los carbonos que contienen protones
equivalentes para que puedas predecir cuántas señales se tendrán en los correspondientes
espectros de RMN y cuántos picos contendrán cada una de ellas.

Tipo de hidrógeno Número de picos

CH2 CH3

H2C CH

H3C CH2 CH3

Tipo de hidrógeno Número de picos

H3C C CH

O N C O

C CH2 HC C

H3C O CH3

FIN DEL EXAMEN

449
 

450
XXIII Olimpiada Nacional de Química
Examen Experimental de Química Orgánica Distrito Federal 2014

Síntesis de difenicarbinol por reducción de benzofenona

ANTECEDENTES
El enlace doble C=O de un aldehído o una cetona se encuentra polarizado debido a la diferencia de
electronegatividad (Pauling) entre los átomos de oxígeno (3.5) y carbono (2.5). Por esta razón una
de las reacciones más comunes de estos compuestos es la adición nucleofílica, la cual implica la
adición de un nucleófilo al carbono electrofílico del grupo carbonilo. Puesto que el nucleófilo utiliza
su par de electrones para formar el nuevo enlace con el carbono, dos electrones del enlace doble
carbono-oxígeno deben desplazarse hacia el átomo de oxígeno electronegativo, donde pueden ser
estabilizados como un anión alcóxido.


O Nu Nu
:O HO
H 2O
 + Nu: + HO:

Reactivo Producto de adición


Carbono trigonal Intermediario alcóxido nucleofílica
Carbono tetraédrico Carbono tetraédrico
hibridación sp2

El borohidruro de sodio (NaBH4) y el hidruro doble de litio y aluminio (LiAlH4) son los reactivos
más comunes para reducir un aldehído o una cetona a los alcoholes correspondientes. Estos
reactivos son llamados hidruros complejos debido a que no tienen una estructura simple como la del
hidruro de sodio (NaH) y la del hidruro de litio (LiH), pero se caracterizan por su capacidad de
donar un ión hidruro (H:−).

H H
Na H B H Li H Al H
H H

El borohidruro de sodio fue descubierto en la década de 1940 por H. Schlesinger. Es un agente


reductor moderado, muy utilizado a escala de laboratorio, siendo un compuesto inorgánico que se
presenta en forma de polvo blanco. Es insoluble en éter y soluble en alcohol y dimetoxietano. Se
utiliza en el laboratorio para la conversión de cetonas y aldehídos a alcoholes. A diferencia del
LiAlH4, el borohidruro de sodio no reduce ésteres, amidas, ácidos carboxílicos, haluros de alquilo,
nitrilos, nitroderivados, oximas, epóxidos y tosilatos.

En este experimento se va a llevar a cabo una reducción de una cetona con borohidruro de sodio,
siguiendo dos principios de la química verde (química sostenible). Uno de estos es la eficiencia
energética y el otro implica el usar una sustancia auxiliar inocua (agua como disolvente).

451
REACCIÓN A EFECTUAR:
1) NaBH4, H2O
O HO H
CH3OH
o
25 C

2) HCl, H2O

PROCEDIMIENTO
En un vaso de precipitados de 100 mL, disuelve 0.5 g de benzofenona (ya se encuentran pesados
dentro del vaso de precipitados y la cantidad exacta se encuentra en la etiqueta) en 5 mL de metanol
(se encuentra en la piseta etiquetada sobre la mesa de trabajo área común). Coloca una barra de
agitación magnética dentro del vaso, y a su vez pon el vaso sobre la placa del agitador magnético, y
mantén la disolución bajo una agitación vigorosa.

La disolución de borohidruro de sodio en agua se encuentra en el frasco gotero etiquetado como


disolución de NaBH4/H2O (0.25 g de borohidruro de sodio en 4.5 mL de agua). Empleando una
pipeta de plástico, adiciona la disolución acuosa del borohidruro de sodio a la disolución de la
benzofenona, una gota cada vez. (PRECACUCIÓN: debes tener el cuidado de dejar que
desaparezca cualquier turbidez en la disolución, antes de adicionar otra gota de la disolución
del borohidruro de sodio).

Cuando termines de adicionar toda la disolución del borohidruro de sodio, debes mantener la
agitación magnética vigorosa por 20 minutos más. Debes de observar la formación de cristales en
suspensión (o bien una capa aceitosa). Agrega 5 mL de agua destilada (se encuentra en la piseta
etiquetada sobre la mesa de trabajo área común) a la mezcla de reacción.

Debes de descomponer el exceso del borohidruro de sodio, por la adición lenta de una disolución de
HCl 10% v/v que se encuentra en el frasco gotero. Se necesitan más o menos 48 gotas. Te sugerimos
que adiciones 40 gotas y tomes el pH de la disolución, y en caso de ser necesario adiciona más
disolución de HCl, lentamente, para llegar a un pH = 5.

El producto crudo lo debes aislar por


filtración al vacío, empleando el embudo
Büchner y el matraz Kitazato. Debes
armar el equipo como se muestra en la
figura 1.

Lava los cristales con dos porciones de


15 mL, cada una, de agua destilada.
Elimina la mayor cantidad de agua de los
cristales, dejándolos en el embudo
Büchner y con la llave de vacío abierta.

Figura 1. Equipo para filtrar al vacío

Purifica el producto por medio de una recristalización con hexano (se encuentra en la piseta
etiquetada sobre la mesa de trabajo área común y se requieren 15 mL), utiliza la cafetera para

452
calentar la disolución a ebullición (PRECAUCIÓN: debes de poner agua de la llave a la cafetera
antes de conectarla a la corriente eléctrica). El producto puro lo debes aislar por medio de una
filtración al vacío. Baja los cristales con un poco más de hexano frío (se encuentra dentro del
recipiente con hielo). Deja evaporar el hexano de los cristales puros, dejando el producto dentro del
embudo Büchner por unos 5 minutos, sin cerrar la llave de vacío.

Determina la masa del producto que obtuviste. Pesa la bolsa de plástico y después coloca el
producto y vuelve a pesar. Uno de los maestros te debe firmar las dos pesadas.

Peso de la bolsa: Firma del profesor:

Peso de la bolsa + producto Firma del profesor:

PRUEBAS DE CARACTERIZACIÓN

Ya que pesaste el producto deberás hacer las siguientes pruebas:

a) Pruebas a la gota.
Coloca en los dos viales que se te proporcionan, la materia prima (benzofenona) y tu producto
recristalizado. Adiciona unas gotas de disolvente (metanol) y sobre la disolución anterior, agrega en
cada vial, por separado, unas gotas de disolución de 2,4-dinitrofenilhidrazina. Anota tus
observaciones en la siguiente tabla (formación o no de precipitado). La disolución de la 2,4-
dinitrofenilhidrazina se encuentra dentro del frasco gotero etiquetado como Solución de 2,4-DNF.

Benzofenona Difenilcarbinol
Observaciones

Tomando en cuenta que la estructura de la 2,4-dinitrofenilhidrazina es la siguiente:

453
Escribe la ecuación química de la reacción que efectuaste y da la estructura del (o los) producto (s)
formado (s):

b) Análisis por cromatografía en capa fina

INTRODUCCIÓN TEÓRICA

La cromatografía en capa fina es una técnica de adsorción sólido-líquido, utilizada en química


orgánica para realizar un análisis cualitativo eficiente y rápido de compuestos desconocidos o para
determinar la composición de una mezcla de compuestos.

Esta técnica cromatográfica consiste en la utilización de una fase estacionaria (alúmina o sílica-gel)
y de una fase móvil, que son los disolventes orgánicos de diferente polaridad.

El proceso de separación se basa en que la fase móvil asciende a través de la fase estacionaria, la
elución de la muestra problema está en función de su polaridad, es decir, de la afinidad que presente
con alguna de las dos fases.

La fase estacionaria consiste de una película delgada (100 m) de un material disperso sobre una
superficie plana (de vidrio, aluminio o celulosa).

Las ventajas del método son la rapidez para llevar a cabo el análisis (un análisis normal toma de 2 a
10 minutos) y se pueden detectar cantidades de material de 2 a 20 g.

La secuencia de pasos que se siguen para llevar a cabo el análisis por cromatografía en capa fina
son:

1) Se traza una línea paralela a la base de la cromatoplaca (aproximadamente a 0.5 cm de la base de


la misma) sobre la superficie del adsorbente con un lápiz (NOTA: no recargues mucho la punta del
lápiz, ya que se puede llegar a romper la superficie de la fase estacionaria). Las dos muestras que se
va a analizar (se requiere la punta de la espátula, esto es aproximadamente 1 mg), se colocan cada
una por separado en un vial, y se adicionan unas gotas del disolvente hasta observar la disolución de
la muestra (en esta caso con metanol). Con un tubo capilar (una aguja hipodérmica sin punta

454
funciona muy bien) se aplica una pequeña fracción de la solución sobre la placa. Una vez que hayas
aplicado la disolucipon de una muestra, antes de aplicar la disolución de la siguiente muestra, lava la
aguja con acetato de etilo, eliminando el exceso del disolvente con el pedazo de papel filtro que se te
proporciona.

0.5 cm
parte superior límite de ascensión
del disolvente

parte inferior X X punto de aplicación


0.5 cm

2) La cromatografía se lleva a cabo colocando la cromatoplaca sobre la cual se aplicó la muestra,


dentro de un frasco de vidrio con tapa, el cual ya debe contener la fase móvil (eluyente) (unos 5.0
mL de una mezcla de hexano-acetato de etilo, 80:20). El frasco debe estar cerrado con la finalidad
de mantener una atmósfera saturada con el disolvente de la fase móvil y debe contener un pedazo de
papel filtro para asegurar dicha atmósfera.

Efectuadas estas operaciones, la fase móvil asciende rápidamente por capilaridad sobre el
adsorbente de la fase estacionaria hasta la marca superior (0.5 cm antes de la parte superior).
Inmediatamente se saca la placa de la cámara de elución y se deja evaporar el disolvente. La muestra
se puede encontrar en cualquier punto.

3) Una medida física de la polaridad de la muestra es el Factor de Retención (Rf), el cual se


determina dividiendo la distancia que recorrió la muestra entre la distancia que recorrió el
disolvente, el Rf siempre es menor a 1. Bajo las mismas condiciones de análisis cromatográfico, el
Rf es constante y permite identificar cualitativamente un compuesto o bien determinar si se
encuentra presente en una mezcla problema:

455
linea hasta la que recorrió
el eluyente

b a
Rf =
b
a

x punto de aplicación
de la muestra

Tú vas a efectuar un análisis comparativo entre la materia prima y el producto puro, como se indica
en la siguiente imagen.

Una vez que dejes eluir la mezcla de disolventes, deja evaporar el eluyente. Revela la cromatoplaca
con vapores de yodo, los cuales se encuentran dentro del frasco etiquetado apropiadamente como I2.
Determina el Rf tanto de la materia prima como del producto puro. Debes entregar la cromatoplaca,
colócala dentro la bolsa de plástico y pide al profesor que engrape las dos bolsas, una con la
cromatoplaca y la otra con el producto que obtuviste.

Cálculo del Rf de la materia prima y del producto puro:

456
CÁLCULO DEL RENDIMIENTO

Tabla de propiedades físicas de reactivos y disolventes


Reactivos y Fórmula Masa Punto de Densidad
disolventes Molecular Molecular fusión / punto (g/mL)
(g/mol) de ebullición
(°C)
Benzofenona C13H10O 182.22 pf = 47 - 51
Metanol CH3OH 32.04 pe = 65 0.791
Borohidruro NaBH4 37.83 pf > 300
de sodio
Difenilcarbinol C13H12O 184.23 pf = 65 - 67

a) Reactivo limitante.
De acuerdo con los datos que se te proporcionan en la tabla anterior, ¿cuál es el reactivo limitante?
Indica con claridad tus cálculos:

O HO H
Fórmula química: C13H10O Fórmula química: C13H12O
Masa molar: 182.218 g/mol Masa molar: 184.234 g/mol

Fórmula química: BH4Na


NaBH4
Mas molar: 37.833 g/mol

b) Rendimiento.
De acuerdo con la cantidad de producto que obtuviste, calcula el rendimiento (%) con el que
obtuviste el difenilcarbinol.

457
458
XXIII Olimpiada Nacional de Química
Examen Experimental de Fisicoquímica Distrito Federal 2014

Determinación de la Constante Crioscópica (Kc) del Agua

Introducción:

Una de las propiedades coligativas que ha tenido gran aplicación es la “Disminución de la


Temperatura de Fusión, ΔTfus”, que es la diferencia de temperaturas de fusión, del disolvente puro
menos la de la solución ΔTfus = Tfus disolvente puro – Tfus solución.

Esto ha permitido determinar la pureza de los disolventes así como la determinación de la masa
molar de solutos. Para determinar la temperatura de fusión de un líquido se recurre al análisis
térmico que consiste en enfriar una muestra introduciéndola en un baño frío y registrar la
temperatura a intervalos regulares de tiempo. Posteriormente se traza la gráfica de “temperatura vs
tiempo” (Figura 1), en la que se aprecia una meseta que indica la temperatura a la cual se está
congelando la solución (tfus).

Figura 1.

En ocasiones la temperatura del líquido disminuye por debajo de la temperatura de fusión


permaneciendo en estado líquido, dando lugar a lo que se conoce como Equilibrio metaestable, el
cual se rompe fácilmente con una pequeña perturbación (agitación), lo que hace que la solución se
congele instantáneamente liberando el calor latente de fusión e incrementando la temperatura hasta
alcanzar la temperatura de fusión de la solución (Figura 2).

459
Figura 2.

La disminución de la temperatura de una solución se puede calcular fácilmente a partir de la


ecuación: ΔTfus = Kcm, donde “Kc” es la constante crioscópica (característica de cada disolvente) y
“m” es la molalidad de la solución (ΔTfus = Tfus disolvente puro – Tfus solución).

La constante crioscópica se puede determinar experimentalmente registrando las temperaturas de


fusión de soluciones de diferente concentración, calculando el ΔTfus, y construyendo la gráfica de
ΔTfus vs m (Figura 3), el valor de la pendiente de la gráfica corresponde al de la constante
crioscópica.

Figura 3.

********************************************************************************

460
Puesto que la molalidad de una solución se puede calcular como: m = (w2)/(M2w1), donde: w1 =
masa (en gramos) de disolvente; w2 = masa (en gramos) de soluto; M2 = masa molar del soluto en
kg/mol.

Al sustituir “m” en la ecuación del ΔTfus = Kcm, se tiene que: ΔTfus = Kc(w2/M2w1)

Conocidas las masas de soluto y disolvente así como la constante crioscópica Kc, basta con
determinar experimentalmente el descenso de la temperatura de fusión para calcular la masa molar
del soluto M2. Por ejemplo:

“Se preparó una solución con 30 g de un compuesto en 120 g de agua pura. La temperatura de
fusión de la solución resultó ser de (-2.6 oC). Para el agua Kc = 1.86 K kg mol-1 y su temperatura de
fusión normal es de 0 oC. La masa molar del soluto es:” (marca con una X la respuesta correcta)

Cálculos:

a) 149 g/mol b) 179 g/mol c) 214.56 g/mol d) 298 g/mol

********************************************************************************

Determinación de la Constante Crioscópica (Kc) del Agua


Procedimiento experimental: (Lea cuidadosamente todo el procedimiento).

1. Prepare un baño de hielo con sal: En el vaso de unicel introduzca un tubo de ensaye vacío y
prepare el baño de sal alternando capas de hielo con capas de sal rodeando al tubo de ensaye (si
no introduce desde el principio un tubo de ensaye, el hielo con sal se tornará muy duro y le
impedirá introducir los tubos con las soluciones). Una vez preparado el baño retire el tubo de
ensaye vacío. Registre la temperatura del baño de hielo con sal.

Temperatura del baño: _______oC

2. Introduzca e Tubo 3 en el baño de hielo con sal con la solución acuosa 3 molal y con el
termómetro registe la temperatura de la solución cada 60 segundos, durante 10 minutos. Registre
sus datos en la Tabla 1.

3. Repita el paso (2) con las otras soluciones.

4. Repita el paso (2) con agua destilada.

461
Tabla 1. Registro de datos experimentales
Agua destilada Tubo 1. Solución 1 m Tubo 2. Solución 2 m Tubo 3. Solución 3 m
Tiempo / Temp. / Tiempo / Temp. / Tiempo / Temp. / Tiempo / Temp. /
o o o o
s C s C s C s C

5. Trace para cada evento la gráfica de la curva de enfriamiento (temperatura vs tiempo). Utilice las
hojas cuadriculadas para trazar las gráficas.

6. Determine a partir de la gráfica la temperatura de congelación de las soluciones y la del agua


destilada. Señale en la gráfica la temperatura de congelación. Registre sus datos en la Tabla 2.

Tabla 2. Temperaturas de congelación experimentales


Concentración / m Temperatura / oC ΔTfus / oC
0 (agua pura)
1
2
3

7. Trace la gráfica “ΔTfus versus m (concentración)” y obtenga la constante crioscópica del agua a
partir de la gráfica. (Utilice una hoja cuadriculada para trazar las gráfica).

Cálculos: (Indique cómo calcula la pendiente)

Kc = __________ kg/mol

462
Determinación de la Constante Crioscópica (Kc) del Agua
Utiliza esta hoja para trazar las gráficas

463
Determinación de la Constante Crioscópica (Kc) del Agua
Utiliza esta hoja para trazar las gráficas

464
XXIII Olimpiada Nacional de Química
Examen Experimental de Química Analítica Nivel A Distrito Federal 2014

Código en el laboratorio: __________________

DETERMINACIÓN DE LA PUREZA DE UNA MUESTRA COMERCIAL DE


CLORURO DE POTASIO

El cloruro de potasio es un compuesto inorgánico con aplicación en áreas muy diversas (química,
medicina, procesamientos de alimentos y hasta en la ejecución de la pena capital de un condenado a
muerte por inyección letal).
Una forma de determinar la pureza de la sal en una muestra comercial de cloruro de potasio consiste
en precipitar los cloruros con un exceso de nitrato de plata; el precipitado se separa por filtración y
se lava con agua destilada. Los iones plata sobrantes se titulan por el método de Volhard que
consiste en utilizar tiocianato de sodio o potasio como reactivo titulante; la titulación se efectúa en
medio ácido y el punto final de la valoración se pone en evidencia por la formación del complejo
Fe(SCN)2+ de color rojo.
Tu labor esta mañana consiste en determinar la pureza de una muestra de KCl que se encuentra en el
laboratorio de Química Analítica pero de las cuales se desconoce su pureza.

PROCEDIMIENTO
1. Pesar con exactitud una cantidad de la muestra de cloruro de potasio, disolverla en agua destilada
y añadir una disolución de nitrato de plata de concentración conocida.
2. Filtrar cuantitativamente el precipitado de cloruro de plata obtenido, lavar éste con agua destilada
y recoger el líquido filtrado y las aguas de lavado en un matraz volumétrico. Adicionar ácido
nítrico y completar con agua destilada hasta el aforo del matraz.

“Por el escaso tiempo disponible esta mañana hemos adelantado estos dos primeros pasos del
procedimiento con diversas porciones de la muestra. Para la precipitación del cloruro de plata
utilizamos 25,00 mL de una disolución de nitrato de plata de concentración 0,300mol/L; el líquido
filtrado y las aguas de lavado se recogieron en un matraz volumétrico de 100,0 mL al que se
añadieron 5 mL de ácido nítrico 8 mol/L antes de completar el aforo con agua destilada. De esta
disolución te proporcionamos un volumen aproximado de 40mL. TU TRABAJO SE INICIARÁ
DESDE EL PASO NÚMERO 3. La masa de KCl pesada para cada problema está indicada en la
etiqueta de la disolución problema”.

3. Tomar una alícuota de 10,0mL de esta disolución problema y añadir 1 mL de una disolución de
alumbre férrico. Al titular con una disolución patrón de tiocianato de potasio de concentración
conocida (la concentración exacta estará indicada en la etiqueta que contiene el reactivo) se
observará la formación de un precipitado de color blanco de tiocianato de plata; continuar
añadiendo este reactivo titulante hasta la aparición de una coloración roja que persista. En la
cercanía del punto de equivalencia debe agitarse enérgicamente.
4. Efectuar esta titulación por triplicado.

465
NOTAS
1. Si requieres volumen adicional de cualquier disolución se te podrá proporcionar pero esto implica
una penalización en la calificación final.
2. Deberás verter los desechos de las titulaciones efectuadas en el frasco de residuos proporcionado.
3. Evita que te caiga disolución de nitrato en la piel pues ésta mancha y tarda días en quitarse. Si
esto ocurriera lávate las manos rápidamente.

Código en el laboratorio_______________

HOJA DE RESPUESTAS

I. Registra el número de la disolución de nitrato de plata proporcionada sobrante de la precipitación


y la masa de la muestra de la muestra de cloruro de potasio pesada inicialmente:

Disolución No. _______________ Masa de cloruro de potasio ___________________

II. Registra en la siguiente tabla los datos de los volúmenes utilizados en las tres titulaciones.

No. de Volumen de la alícuota de Volumen de la disolución de tiocianato de potasio


titulación disolución filtrada gastado
1
2
3

Volumen de tiocianato que utilizarás en tus cálculos: ___________________________________

III. Escribe todas las ecuaciones químicas de las dos reacciones de precipitación y la de formación
del complejo involucradas en el método de VOLHARD.

IV. En relación con el método de Volhard previa precipitación del cloruro de plata, indica cuál
aseveración es cierta:
A) La constante de formación del complejo FeSCN2+ es menor que la del AgSCN.
B) La constante de formación del complejo FeSCN2+ es mayor que la del AgSCN.
C) Por solubilidad, el FeSCN2+ precipita antes que el AgSCN.
D) El producto de solubilidad del FeSCN2+ es menor que el del AgSCN.

466
V. Indica cuál es la concentración de los iones Ag+ en la disolución filtrada después de la
precipitación.

VI. Calcula la pureza de la muestra de cloruro de potasio.

VII. Se conoce que el complejo de Fe3+ con tiocianato puede ser medido con un fotómetro en una
longitud de onda de 450 nm, que sigue la ley de Lambert y Beer (A=εIC) y que la reacción de
formación del complejo es cuantitativa.

Si se supone que la misma coloración que el ojo puede detectar en el método de Volhard diera
una lectura de A = 0,300 en un equipo con una celda cuya longitud de onda de paso óptico (I)
fuera de 1 cm y que el valor de la constante de absortividad molar (ε) fuera igual a 18000 cm-1
mol-1, indica cuál es la concentración de tiocianato que permite determinar el punto final de iones
Ag+ con este indicador y qué error representaría este valor con respecto a la concentración
detectada.

Concentración de tiocianato:

Concentración de tiocianato en exceso


% error 100
Concentración de iones plata en la disolución titulada

467
468
XXIII Olimpiada Nacional de Química
Examen Experimental de Química Analítica Nivel B Distrito Federal 2014

Código en el laboratorio: __________________

DETERMINACIÓN DE LA PUREZA DE UNA MUESTRA COMERCIAL DE


NITRATO DE PLATA

El nitrato de plata es un compuesto inorgánico con numerosas aplicaciones en áreas muy diversas
(médicas, biológicas, químicas e industriales). En el laboratorio de Química Analítica hemos
recibido una donación con dos frascos de nitrato de plata comercial cuya pureza no está
especificada. Tu labor esta mañana será ayudarnos a determinar mediante la titulación de una
disolución de esta sal utilizando tiocianato de potasio como reactivo titulante.

PROCEDIMIENTO

1. Pesar con exactitud una cantidad de la muestra de nitrato de plata, disolverla en agua destilada y
añadir 50 mL de una disolución 8 mol/l de ácido nítrico antes de completar con agua destilada
hasta el aforo en un matraz volumétrico de 1,00 litro.
2. Tomar una alícuota de 10,0mL de esta disolución de la muestra y añadir 1 mL de una disolución
de alumbre férrico. Al titular con una disolución patrón de tiocianato de potasio de concentración
conocida se observará la formación de un precipitado de color blanco de tiocianato de plata;
continuar añadiendo este reactivo titulante hasta la aparición de una coloración roja que persista.
En la cercanía del punto de equivalencia debe agitarse enérgicamente.
3. Efectuar esta titulación por triplicado.

NOTAS

1. Para ahorrar tiempo hemos realizado el primer paso del procedimiento y te proporcionamos
aproximadamente 40 mL de una de las disoluciones preparadas.
2. Si requieres volumen adicional de nitrato de plata se te podrá proporcionar pero esto implica una
penalización en la calificación final.
3. Deberás verter los desechos de las titulaciones efectuadas en el frasco de residuos proporcionado.
4. Evita que te caiga disolución de nitrato en la piel pues ésta mancha y tarda días en quitarse. Si
esto ocurriera lávate las manos rápidamente.

469
Código en el laboratorio_______________

HOJA DE RESPUESTAS

I. Registra el número de la disolución de nitrato de plata que se te proporcionó y la masa de nitrato


de plata con la que se preparó esta disolución:

Disolución No. _______________ Masa de nitrato de plata ___________________

II. Registra en la siguiente tabla los datos de los volúmenes utilizados en las tres titulaciones.
No. de Volumen de la alícuota de nitrato de Volumen de la disolución de tiocianato de
titulación plata potasio gastado
1
2
3

III. Volumen de tiocianato que utilizarás en tus cálculos: __________________________________

IV. Escribe todas las ecuaciones químicas de las reacciones de precipitación y formación del
complejo involucradas en el método de VOLHARD.

V. Indica cuál es la concentración de los iones Ag+ en la disolución que se te proporcionó.

VI. Calcula la pureza de la muestra de nitrato de plata.

470
XXIV Olimpiada Nacional de Química

1er EXAMEN NIVEL A y B


Total: 30 preguntas Tiempo asignado: 75 minutos Guadalajara 2015

Anota en el recuadro correspondiente la letra del inciso que contesta correctamente cada pregunta.
No olvides que si consideras que ningún inciso es el correcto deberás anotar una letra X.

USA LETRAS MAYÚSCULAS Y ESCRIBE CLARAMENTE PARA EVITAR CONFUSIONES


AL CALIFICAR. SI NO ESCRIBES TUS RESPUESTAS CON TINTA, TU EXAMEN PODRÍA
SER ANULADO.

1) Un compuesto poco conocido es el carburo de aluminio. En su molécula, el % en


masa de carbono es 25%. La fórmula de este compuesto es:
A Al4C3 B Al2CO
C Al4H2CO3 D Al6C9

2) La densidad del carburo de aluminio es de 2.36 g/cm3. Si con este compuesto


llenamos completamente un recipiente cuya masa es de 300 g y su volumen de 1.0
litro, la masa del compuesto dentro del recipiente es:
A 300 g B 2360 g
C 302.36 g D 2660 g

3) Un compuesto más conocido del aluminio, es el cloruro de este compuesto. En un


kilogramo de este material, ¿cuántos moles tenemos de cloruro de aluminio?
A Menos de seis moles B Entre 6 moles y 7 moles
C Entre 7 moles y 8 moles D Más de 8 moles

4) La molécula de cloruro de aluminio se puede encontrar como un hexahidrato. En


cambio la molécula de sulfato de magnesio suele estar asociada a siete moléculas de
agua; la masa molar de este heptahidrato es, aproximadamente en g mol-1:
A 246.4 B 120.4
C 138.4 D 182.38

El cloruro de cobre(II), CuCl2, en forma anhidra es café pero puede absorber humedad
para formar un dihidrato de color azul verdoso. La densidad del compuesto anhidro es
3.39 g/cm3 y la del compuesto dihidratado es 2.51 g/cm3. La densidad del agua es 1.0
g/cm3.
5) ¿Cuántos gramos de agua absorberán 100 g del compuesto anhidro hasta convertirse
totalmente en el compuesto dihidratado?
A Menos de 14 g B Entre 14 y 24 g
C Entre 24 y 34 g D Más de 34 g

471
El volumen de una esfera se calcula como V = 4/3 r3 donde r es el radio de la esfera.
6) Si los cien gramos de CuCl2 anhidro estuvieran en forma de una canica esférica,
¿Cuál sería el radio de esta canica en cm?
A 0.20 B 2.12
C 1.43 D 1.91

7) Supón que tienes una canica esférica de CuCl2 anhidro con un volumen de 30.0 cm3 y
que al absorber humedad para formar el compuesto dihidratado se mantuviera la
forma esférica. ¿Cuál sería el nuevo volumen de la esfera, en cm3?
A Menos de 45 B Entre 45 y 50
C Entre 50 y 55 D Más de 55

8) En el bórax, la molécula está asociada a diez moléculas de agua: Na2B4O7·10H2O, por


lo que el % en masa de boro en esta molécula es:
A Menor a 12% B Entre 12% y 20%
C Entre 20% y 30% D Mayor a 30%

La molécula de clorofila contiene un ion Mg2+. De hecho hay varios tipos de clorofila,
pero las más comunes son: Clorofila a (C55H72O5N4Mg) y Clorofila b (C55H70O6N4Mg).
9) La masa molar de la clorofila a es, aproximadamente en g mol-1:

A 907 B 909
C 891 D 893

10) La clorofila a es muy soluble en etanol, una molécula orgánica que tiene el grupo
funcional típico de los alcoholes que es:
A -O- B -CHO
C -OH D -COOH

11) En la molécula de clorofila b, el % en masa del magnesio es:

A Menor al 3% B Entre 3% y 5%
C Entre 5% y 7% D Mayor al 7%

12) En gramos, una ppm (partes por millón) equivale a un gramo en un millón de
gramos. Al disolver 9.00 mg de clorofila a en medio litro de agua, ¿cuál es la
concentración de clorofila a en ppm?
A Menos de 10 ppm B Entre 10 ppm y 100 ppm
C Entre 100 ppm y 200 ppm D Más de 200 ppm

13) Una aleación de samario y cobalto tiene fuertes propiedades magnéticas. En esta
aleación hay un átomo de samario por cinco de cobalto, por lo que el % en masa de
samario es:
A Menos de 17% B Entre 17% y 24%
C Entre 24% y 31% D Más de 31%

472
14) En el grupo de los lantánidos, además del Samario se encuentra el Cerio. En el
compuesto (NH4)2Ce(NO3)6,el estado de oxidación del cerio es:
A +4 B + 10
C +2 D +6

15) La ley de los gases ideales es PV = nRT.


(R = 0.082 L atm / mol K) (R = 8.314 J / mol K) (1 J = 1 kg m2 s-2)
(1 atm = 101.325 kPa) (1 Pa = 1 N m-2) (1 N = 1 kg m s-2 )
¿En qué condiciones de P y T, un mol de gas ideal ocupa un volumen de 20.0 L?
A 100 kPa y 298 K B 1.2 atm y 273 K
C 500 kPa y 1203 K D 0.98 atm y 298 K

16) En un recipiente de 8.0 L de capacidad hay la misma cantidad en gramos de N2 y H2


gaseosos. La fracción mol de N2 es:
A No es posible saberlo si no se B Menor a 0.5
especifican P y T
C 0.5 D Mayor a 0.5

17) Si se hacen reaccionar 2 moles de N2 con tres moles de H2 para formar amoniaco
(NH3), la máxima cantidad de moles de NH3 que se puede obtener es:
A Dos B Tres
C Cuatro D Seis

18) Al valorar un ácido con una disolución de NaOH se comprobó que el pH de la


disolución, en el punto de equivalencia era 8.4. De ello se dedujo que:
A Hubo error al medir porque el pH B El ácido era fuerte.
debe ser 7 en el punto de
equivalencia.
C La concentración del ácido era baja. D El ácido era débil.

19) Por descomposición de carbonato de calcio se obtiene óxido de calcio y CO2.


¿Cuántos gramos del carbonato se requieren para obtener 20.0 litros de CO2 a una P
de 100 kPa y T de 25 oC?
A Menos de 75 g B Entre 75 g y 95 g
C Entre 95 g y 105 g D Más de 105 g

20) Si se adicionan dos gramos de agua a una mezcla de ácido y agua, la nueva mezcla
tiene un 20% en masa de ácido. Cuando 6 gramos de ácido son adicionados a la
nueva mezcla, la mezcla ahora contiene 50% en masa de ácido. El % en masa de
ácido en la mezcla original era:
A Menos de 25% B Entre 25% y 30%
C Entre 30% y 35% D Más de 35%

473
21) Para producir ácidos carboxílicos en el laboratorio en ocasiones se utiliza una
disolución acuosa de permanganato de potasio (KMnO4). El estado de oxidación del
manganeso en este compuesto es:
A +6 B +5
C +3 D +1

22) Si se hace reaccionar el KMnO4 con ácido sulfúrico concentrado se obtiene un


compuesto que puede explotar al contacto, el Mn2O7. En la reacción propuesta se
obtiene además agua y KHSO4. Al balancear esta reacción se encuentra que por cada
mol de permanganato, reaccionan de H2SO4:
A 0.5 moles B 1.0 mol
C 1.5 moles D 2.0 moles

23) ¿Cuál de estas sustancias tiene una densidad menor en estado sólido que en estado
líquido?
A Agua B Benceno
C Dióxido de carbono D Ácido benzoico

24) Si la reacción de la pregunta 22, se lleva a cabo en disolución acuosa del ácido, la
reacción no balanceada es:

KMnO4(ac) + H2SO4(ac) → MnSO4(ac) + K2SO4(ac) + H2O(l) + O3(g)

Por cada mol de permanganato que reacciona se obtienen de ozono:


A Menos de un mol B Un mol
C Dos moles D Más de dos moles

25) Cuando se deposita cobre metálico a partir de una disolución que contiene iones
Cu2+, utilizando una reacción electroquímica, el enunciado correcto es:
A El cobre metálico se deposita en el ánodo por oxidación de los iones Cu2+
B El cobre metálico se deposita en el ánodo por reducción de los iones Cu2+
C El cobre metálico se deposita en el cátodo por reducción de los iones Cu2+
D El cobre metálico se deposita en el cátodo por oxidación de los iones Cu2+

26) El gas LP consta básicamente de propano y butano. Si suponemos que contiene el


75% en masa de propano, ¿cuál es la fracción mol de propano en esta mezcla?
A 0.250 B 0.798
C 0.750 D 0.202

27) Considera que el HCl y el NaOH son electrolitos fuertes. Si preparas una disolución
acuosa al mezclar 80.0 mL de una disolución acuosa de HCl 0.20 mol L-1 con 20 mL
de otra disolución acuosa de NaOH 0.50 mol L-1, ¿cuál será el pH de la nueva
disolución?
A 1.22 B 1.0
C 11.78 D 2.22

474
28) La corriente que circula en una celda electroquímica se puede medir en amperes (A).
Un ampere equivale a una carga de un coulomb (C) que pasa cada segundo por un
punto del circuito. La carga de un electrón es 1.6 x 10-19 C.
Si en una celda electroquímica circula una corriente de 5 mA, significa que el número
de electrones que pasan por un punto del circuito cada segundo son:
A 3.125 x 10-16 B 3.2 x10 7
16
C 3.125 x 10 D 3.2 x 10-17

29) ¿Cuantos gramos de metanol se necesitan, aproximadamente, para tener la misma


cantidad en moles que hay en 100 g de etanol (CH3CH2OH)?
A 46.0 g B 71.87 g
C 69.5 g D 143.75 g

30) Si en la molécula de benceno se sustituye un hidrógeno por un grupo OH se obtiene


fenol. Para quemar un mol de fenol y producir CO2 y H2O, ¿cuántos moles de
oxígeno gaseoso se requieren?
A 7 B 7.5
C 15 D 12

FIN DEL PRIMER EXAMEN VERIFICA QUE TU EXAMEN CONSTA DE


30 PREGUNTAS

No olvides que si consideras que ningún inciso es el correcto deberás anotar una letra X.
RECUERDA QUE DEBES ESCRIBIR TODAS TUS RESPUESTAS CON TINTA Y
USANDO LETRAS MAYÚSCULAS.

475
476
XXIV Olimpiada Nacional de Química. 2do Examen Nivel A y B.

Total: 26 problemas Nivel B y 28 problemas Nivel A


Tiempo asignado: 90 minutos Guadalajara 2015

1. De acuerdo con el principio de Arquímedes, “Un cuerpo total o parcialmente sumergido en un


fluido en reposo, recibe un empuje de abajo hacia arriba igual al peso del volumen del fluido que
desaloja”. Esto aplica para los globos “de gas”, lo que les permite elevarse en el aire.
La capacidad de ascensión de un globo es la diferencia entre el peso del aire desplazado y el peso
del globo con gas.
Un globo de “San Valentín” con una capacidad de 33.5 L se llena con Helio a una presión
absoluta de 1.5 atm y a una temperatura de 20 °C. Si el globo se suelta a nivel del mar (Pbarométrica
= 760 mmHg), contesta las siguientes preguntas. (Considera la masa del globo despreciable)

Encierra con un círculo la respuesta correcta (con tinta) y anota tus cálculos en los cuadros
correspondientes.

1.1. La masa de aire desplazada por el globo es: (considera la masa molar del aire como 29 g/mol).
Considera comportamiento ideal del gas. R = 0.082 L atm / mol K.

A) 5.57 g B) 40.41 g C) 44.60 g D) 592.4 g

1.2. La masa de Helio dentro del globo es:

A) 8.36 g B) 40.41 g C) 60.62 g D) 122.56 g

477
1.3. La capacidad de ascensión del globo es:

A) 2.79 g B) 4.55 g C) 11.14 g D) 32.05 g

2. Si el globo se llena con 60% en mol de Helio y 40% en mol de aire a 1.0 atm y 25°C:
2.1. La densidad de la mezcla Helio y aire es:

A) 8.53 g/L B) 1.012 g/L C) 0.971 g/L D) 0.573 g/L E) 0.675 g/L

22.2. La masa molar aparente de la mezcla de Helio y aire es:

A) 7.25 g/mol B) 14.0 g/mol C) 16.5 g/mol D) 33.0 g/mol

478
3. Se desea elevar un aparato de monitoreo ambiental que pesa 4.00 kg. ¿Qué volumen mínimo
tendrá un globo con Helio capaz de elevar el aparato a 1 atm y 20 °C?

A) 199 L B) 262.4 L C) 2913 L D) 3846 L

EN LAS SIGUIENTES PREGUNTAS, ENCIERRA CON UN CÍRCULO LA LETRA DEL


INCISO CORRECTO:

4. En la reacción entre dos elementos Q y M, cuyos número atómicos son: Q (Z = 16) y M (Z = 19),
el compuesto que se formará con mayor probabilidad será:

A) Un compuesto iónico de fórmula MQ.


B) Un compuesto iónico de fórmula MQ2.
C) Un compuesto iónico de fórmula M2Q.
D) Un compuesto covalente de fórmula M2Q.

5. El isótopo 42K tiene un tiempo de semidesintegración de 12 horas. ¿Cuál es la fracción de la


concentración inicial de dicho isótopo que queda después de 48 horas?

A) 1/16 B) 1/8 C) 1/4 D) 1/2

6. El cloro tiene dos isótopos naturales cuyas masas son 35 y 37 unidades. ¿Cuál será la contribución
de los isótopos si la masa atómica del cloro es igual a 35,54 unidades?

A) Mayor proporción del cloro 35 que de cloro 37.


B) Tendrán la misma proporción.
C) Mayor proporción del cloro 37 que de cloro35.
D) No se puede determinar con los datos aportados.

7. Dadas las siguientes afirmaciones, indica cuál es la respuesta correcta:

1 Por regla general, el radio atómico en un periodo disminuye de izquierda a derecha.


2 Por regla general, el radio atómico en un grupo aumenta de arriba hacia abajo.
3 Por regla general, para todo elemento la segunda energía de ionización es mayor que la
primera.
4 Por regla general, el radio de A− es mayor que el de A.

479
A) Sólo la 1 y 3 son ciertas. B) Sólo la 2 y 3 son ciertas.
C) Sólo la 1 y 4 son ciertas. D) Todas son ciertas.

8. Al ir de izquierda a derecha en el tercer periodo de la tabla periódica, los óxidos y los cloruros
cambian sus propiedades de iónicas a covalentes. Este cambio se debe a que:

A) Aumenta el volumen atómico.


B) Desciende la primera energía de ionización.
C) Incrementa la electronegatividad.
D) Disminuye el número de electrones de valencia.

9. La forma geométrica de la molécula PCl3 es:

A) Plana triangular B) Bipirámide triangular


C) Pirámide cuadrada D) Pirámide triangular

10. Señala la proposición correcta:

A) La molécula de agua es lineal.


B) El volumen molar del hielo es menor que el del agua líquida.
C) En agua sólo se disuelven compuestos iónicos.
D) La molécula de agua puede actuar como ácido y como base de Brönsted-Lowry.
E) En la molécula de agua, el oxígeno presenta hibridación sp2.

11. A partir de la posición del oxígeno en la tabla periódica y de su configuración electrónica se


puede afirmar que:

A) Es el elemento más electronegativo de la tabla.


B) Sus valencias covalentes son 2, 4 y 6.
C) Sus átomos y moléculas son paramagnéticos.
D) Forma el mismo tipo de compuestos que el resto de los elementos de su grupo.

12. Si se tiene un litro de disolución de ácido acético, HAc de concentración 0,01 mol/L y otra de
HCl de igual concentración, que se quieren neutralizar con NaOH de igual concentración, es
cierto que:

A) Antes de neutralizarse, ambas disoluciones tienen el mismo pH.


B) El HCl requiere un volumen mayor que el ácido acético.
C) Los dos ácidos necesitan igual volumen de la disolución de sosa.
D) Se necesitan más datos para saber qué ácido necesitará más sosa para su neutralización.

13. La fenolftaleína es un indicador ácido-base cuya color es rosa por encima de pH=9,2. ¿En cuál
de las siguientes disoluciones, todas en concentración 1 mol/L se observaría este color al añadir
fenolftaleína? -OH indica fenol. Datos: Valores de pKa: CH3COOH/CH3COONa = 4,8;
NH4+/NH3 = 9,2; -OH/-O-= 10,1

A) Na2SO4 B) CH3COOH C) NH3 D) -OH

480
14. En la siguiente reacción redox balanceada: 3CuS + 8HNO3  3CuSO4 + 8NO + 4H2O

A) el CuS es el agente oxidante


B) el azufre se reduce y el nitrógeno se oxida
C) el azufre se oxida y el nitrógeno se reduce
D) hay un intercambio de 8 electrones

15. ¿Cuál es el pH de una disolución saturada de Ca(OH)2? Valor de Kps = 5,5 x10-6

A) 11,37 B) 8,72 C) 12,05 D) 7,03

16. En el ión complejo CrCl2(NH34+ el número de oxidación del cromo y su número de


coordinación son, respectivamente:

A) 0 y 6 B) III+ y 6 C) 0 y 7 D) III- y 4

17. Si de una disolución saturada de un sólido prácticamente insoluble en agua se evapora la mitad
del agua manteniendo la temperatura constante, ¿cuál será la molaridad de la disolución?

A) Igual a la inicial B) La mitad de la inicial


C) El doble de la inicial D) Cuatro veces la inicial

18. Para obtener 15,54 g de un metal a partir de la reducción de una solución acuosa de sus iones
divalentes se requiere una carga total de electricidad de 45741,6 C. ¿Cuál es el metal?
Datos: 1 Faraday = 96500 C mol-1

A) Cr B) Ni C) Zn D) Co

19. Entre las sustancias R, Q y P se establece el siguiente equilibrio químico: R(g) + Q(g) ⇆ P(g).
Si se mezclan R y Q en un matraz de 10 L de capacidad y se deja que reaccionen hasta que se
alcance el equilibrio a unas determinadas condiciones, la mezcla gaseosa contiene igual número de
moles de R, Q y P. En tales condiciones la constante de equilibrio Kc es 4 mol·dm–3. ¿Cuántos
moles de P se han formado?

A) 0,25 B) 0,40 C) 2,0 D) 2,5

20. Un anillo de plata que pesa 7,275 g se disuelve en ácido nítrico y se añade un exceso de cloruro
de sodio para precipitar toda la plata como AgCl. Si el peso de AgCl(s) es 9,000 g, ¿cuál era el
porcentaje de plata en el anillo?

A) 6,28% B) 75,26% C) 93,08 D) 67,74%

21. ¿Cuál es el estado de oxidación del azufre en el ditionito de sodio (Na2S2O4)?

A) VIII+ B) VI+ C) III+ D) IV+

481
22. Los potenciales normales de electrodo para las siguientes reacciones son:
Ag+ (ac) + e– → Ag (s) E° = 0,80 V
Fe3+ (ac) + e– → Fe2+ (ac) E° = 0,77 V
Cu2+ (ac) + 2 e– → Cu (s) E° = 0,34 V

Por lo tanto, el agente reductor más fuerte es:

A) Ag (s) B) Cu (s) C) Fe2+ (ac) D) Cu2+ (ac)

23. ¿Cuántos moles de Cl2 (g) se pueden producir por electrólisis de una disolución acuosa
concentrada de NaCl, si se utiliza una corriente de 2,00 A de intensidad durante 8,0 horas?

A) 0,298 B) 0,149 C) 0,894 D) 0,596

24. ¿Cuál de los siguientes iones será reducido por el ion Cr2+ (ac) en condiciones estándar?
Potenciales normales de electrodo:

E° Pb2+ /Pb Ca2+/Ca Al3+/Al Fe2+/Fe Zn2+/Zn Cr3+/Cr2+


(V) -0,13 -2,87 -1,67 -0,44 -0,76 -0,42

A) Ca2+ (ac) B) Pb2+ (ac) C) Al3+ (ac) D) Fe2+ (ac)

25. Una disolución acuosa tiene 6,00% en masa de metanol (CH3OH) y su densidad es 0,988 g/mL. La
molaridad del metanol en esta disolución es:

A) 0,189 mol/L B) 1,05 mol/L C) 0,05 mol/L D) 1,85 mol/L

LA PREGUNTA 26 SÓLO DEBEN CONTESTARLA LOS DE NIVEL B.


NIVEL A SEGUIR CONTESTANDO DE LA 27 a 29.

26. Escribe las fórmulas químicas de los compuestos que se indican en cada recuadro (Puedes
escribir la fórmula condensada o la desarrollada). Coloca una X en el compuesto que tenga la menor
masa molar. Coloca dos XX en el compuesto que tenga la mayor masa molar.

Agua oxigenada o peróxido de hidrógeno Sulfato de hierro(III)

Tetraborato de sodio decahidratado Permanganato de potasio


482
Propano Butano

Ciclohexano Isopropanol

Tolueno Acetona o Propanona

LAS SIGUIENTES PREGUNTAS SÓLO DEBEN CONTESTARLAS LOS DE NIVEL A.

27. ¿Cuál isómero del dimetilbenceno produce únicamente 2 productos en una reacción de
mononitración (SEAr)?
HNO3 I II
DIMETILBENCENO +
H2SO4 C8H9NO2 C8H9NO2

Indica las estructuras de los compuestos (I) y (II) y la del isómero del dimetilbenceno.
RESPUESTAS

ISÓMERO DIMETILBENCENO COMPUESTO I COMPUESTO II

483
28. Indica todos los pasos, condiciones y reactivos, que son necesarios para efectuar cada una de las
siguientes transformaciones:

28.a.

RESPUESTA

484
28.b.

RESPUESTA

485
29. Considera la siguiente secuencia sintética:

I CH3 Ag2O CALOR


A B C
N (EXCESO)
CH3
I CH3
(EXCESO)

CALOR Ag2O
F E D

¿Cuál de las siguientes opciones es la correcta? Marca con una X la respuesta correcta.

A)

B)

C)

D)

486
XXIV Olimpiada Nacional de Química
TERCER EXAMEN
Total: 4 problemas Nivel B; 5 problemas Nivel A Guadalajara 2015
Tiempo asignado: 180 minutos

■■■■■■■■■■■■■■■■■■■■■■■■■■■■■■■■■■■■■■■■■■■■■■■■■■■■■■■■■■■■■■■■■■

1. El tetróxido de dinitrógeno (N2O4), también llamado óxido de nitrógeno IV es un dímero del


dióxido de nitrógeno (NO2). Es un poderoso oxidante, altamente tóxico y corrosivo. El N2O4
recibió mucha atención como comburente de cohetes debido a que se inflama espontáneamente al
entrar en contacto con la hidracina. El N2O4 se descompone para producir el NO2, alcanzando una
condición de equilibrio que depende de las condiciones de presión y temperatura.
N2O4(g) ⇆ 2 NO2(g)

La tabla siguiente proporciona las propiedades termodinámicas del (NO2) y del (N2O4).
Gas H°f,25°C/(kJ/mol) S°f,25°C/(J/mol K)
NO2 33.9 240
N2O4 9.7 304.3

Marca con un “X” la respuesta correcta.


1.1. El rH° a 25 °C en kJ/mol es:

a) 14.5 kJ/mol b) 24.2 kJ/mol c) 48.4 kJ/mol d) 58.1 kJ/mol

1.2. El rS° a 25 °C en J/mol K es:

a) –64.3 b) +128.6 c) +175.7 d) –368.6

1.3. El rG° a 25 °C en kJ/mol es:

a) 5.71 kJ/mol b) 46.45 kJ/mol c) 92.9 kJ/mol d) 144.74 kJ/mol

487
1.4. A 30 °C el rG° = 4503 J/mol, la constante de equilibrio Kp a esta temperatura es:

a) 0.1467 b) 0.1675 c) 0.1855 d) 0.1974

1.5. A 25 °C la constante de equilibrio Kp es 0.11367; si la presión total de equilibrio es de 1.00


bar, la presión parcial del N2O4 en bares es:

a) 0.285 bar b) 0.333 bar c) 0.570 bar d) 0.715 bar

■■■■■■■■■■■■■■■■■■■■■■■■■■■■■■■■■■■■■■■■■■■■■■■■■■■■■■■■■■■■■■■■■■
Escribe en el cuadro la letra MAYÚSCULA del inciso correcto:

2. Una forma de categorizar a los combustibles fósiles con relación al calentamiento global es a
través de la energía emitida por gramo de CO2 expelido a la atmósfera. Mientras mayor sea la
energía liberada por gramo de CO2 emitido, mejor será el combustible.

La combustión de C8H18 (gasolina) libera 1309 kcal/mol, en tanto que la combustión del gas
metano CH4 libera 213 kcal/mol y la combustión de carbón libera 94.05 kcal/mol.

2.1.- ¿Cuántos gramos de oxígeno se requieren para la combustión de un gramo de gasolina?


Respuesta
A) menos de 3 gramos B) entre 3 y 4 gramos
C) entre 4.1 y 5 gramos D) más de 5 gramos

2.2. ¿Cuántas kilocalorías se liberan por gramo de CO2 expelido a la atmósfera en el caso de la
gasolina (C8H18)?
Respuesta
A) menos de 2.5 kcal B) entre 2.5 y 3.5 kcal
C) entre 3.6 y 4 kcal D) más de 4 kcal

2.3.- ¿Cuántas kilocalorías se liberan por gramo de CO2 expelido a la atmósfera en el caso del gas
natural (CH4)?
Respuesta
A) menos de 2.5 kcal B) entre 2.5 y 3.5 kcal
C) entre 3.6 y 4 kcal D) más de 4 kcal

488
El calentamiento de aire de una habitación requiere 1000 kcal para elevar su temperatura en 10ºC.

2.4. ¿Qué volumen de metano en condiciones STP (1 atm y 0 ºC) se requiere quemar para
calentar la habitación?
Respuesta
A) menos de 100 L B) entre 100 y 150 L
C) entre 151 y 200 L D) más de 200 L

2.5. ¿Cuántos gramos de CO2 se generan?


Respuesta
A) menos de 150 g B) entre 150 y 200 g
C) entre 201 y 250 g D) más de 250 g

Suponiendo que la cantidad de energía en forma de calor necesaria para calentar la habitación,
proviene de electricidad con 80% de eficiencia y que la electricidad fue producida a partir de la
combustión de carbón con 30% de eficiencia:

2.6. ¿Cuántos gramos de carbón se requieren quemar para calentar la habitación?


Respuesta
A) menos de 400 g B) entre 400 y 450 g
C) entre 451 y 500 g D) más de 500 g

2.7. ¿Cuántos gramos de CO2 se generan en este proceso?


Respuesta
A) menos de 1500 g B) entre 1500 y 2000 g
C) entre 2001 y 2500 g D) más de 2500 g

■■■■■■■■■■■■■■■■■■■■■■■■■■■■■■■■■■■■■■■■■■■■■■■■■■■■■■■■■■■■■■■■■■

3. El ácido sulfhídrico (H2S) es un gas incoloro inflamable, de sabor algo dulce y olor a huevos
podridos, que en altas concentraciones puede ser venenoso; ocurre en forma natural y como
producto de actividades humanas. Las fuentes industriales de ácido sulfhídrico incluyen a las
refinerías de petróleo, plantas de gas natural, plantas petroquímicas, plantas de hornos de coque,
plantas que procesan alimentos y curtidurías. El máximo nivel de ácido sulfhídrico permitido en
el aire es 0.01 mg/L.
Para determinar el contenido de ácido sulfhídrico en el aire de una planta química se usó un
método consistente en hacerlo reaccionar con el yodo generado coulombimétricamente por
electrólisis de una disolución acuosa de KI.
El procedimiento seguido se describe a continuación:
 La disolución de KI se electrolizó (con electrodos de platino inatacables) durante 2 minutos
con una intensidad de 2 mA.
 Se burbujearon 2 litros de aire en la disolución de KI electrolizada y se observó la
decoloración total de la disolución de yodo.

489
 Se añadió 1 mL de una disolución de almidón.
 Se volvió a electrolizar la disolución durante 35 s con la misma intensidad de corriente de 2
mA hasta observar la aparición de color azul.

Datos: Faraday = 96500 C/mol Masa molar de H2S = 34.1 g mol-1


Rendimiento de la corriente para lageneración de yodo = 100 %
Eo (V/ENH): H2S/S = 0.14; I2/I- = 0.54

RESPUESTAS

3.1. Marca con una x el electrodo en el cual se genera el yodo:

Cátodo Ánodo

3.2.¿ Cuál es la reacción que se produce en ese electrodo durante la electrólisis de la disolución de
KI?

3.3. ¿Qué reacción ocurre en el electrodo contrario?

3.4. ¿Cuál es la reacción que se produce entre el H2S y el yodo generado?

3.5. ¿Cuál es el propósito de añadir almidón?

3.6. ¿Qué cantidad de electricidad total se consume para la generación del yodo necesario para
llegar al punto de equivalencia?
Cálculos:

490
3.7. ¿Cuál es el contenido total de H2S (en mg/L) en la planta de aire? Considera condiciones
normales de temperatura y presión.
Cálculos:

3.8. En relación con los límites de contaminación permitidos el contenido de H2S en el aire de la
planta es:

Inferior al máximo permitido Superior al máximo permitido

■■■■■■■■■■■■■■■■■■■■■■■■■■■■■■■■■■■■■■■■■■■■■■■■■■■■■■■■■■■■■■■■■■
4. Al calentar un metal R con un no metal L se produce un compuesto binario A, que contiene
63.36% del elemento R. Al hacer reaccionar A con HCl, se produce una sal C (de fórmula RCln,
que contiene 74.47% de cloro) y un gas binario B (de fórmula LZ4, que contiene 12.47% del
elemento Z). El compuesto B arde en el aire y como resultado se obtienen agua y un óxido D
(LqOw) que contiene 46.76% del elemento L que es el principal componente de la arena.

Tienes que:
 Determinar la identidad de los elementos R y L.
 Encontrar las fórmulas de los compuestos A, B, C y D.
 Escribir las ecuaciones balanceadas de las reacciones químicas que se mencionan.
*Nota. Todas tus respuestas deben ser escritas en los cuadros correspondientes y, siempre que
proceda, estar justificadas con cálculos.

4.1. ¿Cuál es la fracción en masa de R en la sal C?


Cálculos:

La fracción en masa de R es _____________________________

4.2. ¿Cuál es la relación estequiométrica de los elementos R y Cl en la sal C y, por ende, cuál es
la masa molar del elemento R y cuál es el metal R?
Cálculos:

La relación es ______________________

La masa molar es ___________________

El metal es ________________________

491
4.3. Con base en los resultados de la pregunta (4.2) escribe la fórmula del compuesto C.

La fórmula del compuesto C es ______________________________

4.4. Calcula la fracción masa de cada uno de los elementos del óxido D (LqOw) que se produce
cuando B se quema en el aire.
Cálculos:

Las fracciones son: ___________ para __________ y _________para__________.

4.5. ¿Cuáles son los valores de los subíndices q y w en el óxido D y, por ende, cuál es el valor de
la masa molar de L, y cuál es la identidad del no metal L?
Cálculos:

El valor de “q” es ________ y el de “w” es ________

La masa molar de L es _________________________

El no metal L es ______________________________

4.6. Como consecuencia de los resultados obtenidos en la pregunta 4.5, establece la fórmula del
compuesto D.

La fórmula de D es __________________________________

4.7. Calcula la masa molar del elemento Z y, en consecuencia, identifica al elemento Z y


establece la fórmula del compuesto B.
Cálculos:

La masa molar de Z es ____________________

El elemento Z es _________________________

La fórmula del compuesto B es _____________

492
4.8. Conocida la identidad de los componentes del compuesto A, encuentra la relación
estequiométrica que presentan y, como consecuencia, establece la fórmula de A.
Cálculos:

La relación estequiométrica es ___________________________

La fórmula del compuesto A es __________________________

4.9. Escribe las ecuaciones balanceadas de las tres reacciones que se mencionan en el problema:
ECUACIÓN 1:

ECUACIÓN 2:

ECUACIÓN 3:

FIN DEL EXAMEN NIVEL B.

■■■■■■■■■■■■■■■■■■■■■■■■■■■■■■■■■■■■■■■■■■■■■■■■■■■■■■■■■■■■■■■■■■

493
DE AQUÍ EN ADELANTE SÓLO RESPONDEN LOS DE NIVEL A.

5.1.a. Dibuje la estructura de los intermediarios y la del producto final en la siguiente secuencia
sintética:

O O
C H2 (exceso) 1) KOH, H2O
OCH3 H B
A C
NO2 NaOH / H2O Pd (C) C6H11NO2 2) HCl, H2O (+,-)-Prolina

RESPUESTAS

A B C

5.1.b. Dibuje la estructura del enantiómero R de la Prolina:

RESPUESTA
ESTRUCTURA ENANTIÓMERO R DE LA PROLINA

494
5.2. Complete la siguiente secuencia sintética:

H
CH3 (a) (b) (c)
Br OH O

(d) (f)

OH Cl
(e)
O O

RESPUESTAS

(a) (b) (c)

(d) (e) (f)

495
5.3. ¿Cómo llevaría a cabo cada una de las siguientes transformaciones?

5.3.a. Conversión de 4-nitrocumeno en 4-isopropilfenol:

NO2 OH

RESPUESTA:

496
5.3.b. Conversión de 4-nitrocumeno en 3-nitrocumeno:

NO2
NO2

RESPUESTA:

FIN DEL EXAMEN NIVEL A.

497
498
XXIV Olimpiada Nacional de Química
Examen Internacional
Total de preguntas: 41 Tiempo asignado: 4 horas Guadalajara 2015

Siempre que se te pidan cálculos, indícalos claramente dentro de los espacios asignados;
respuestas sin cálculos pueden no ser tomadas en cuenta.

I. QUÍMICA ANALÍTICA.

Titulación fotométrica de As(III) y Sb(III) en agua.


La ley de Lambert-Bouguer-Bernard-Beer (comúnmente conocida como ley de Lambert-Beer) es
una ecuación matemática que expresa la absorción de luz por la materia. Esta ley afirma que la
cantidad de luz que sale de una muestra disminuye por la cantidad de material de absorción en su
trayectoria (concentración), por la distancia que la luz debe atravesar a través de la muestra
(depende del recipiente o celda que contiene la disolución) y por la probabilidad de que fotones de
particular longitud de onda sean absorbidos por el material. Esta relación se expresa como:

A = εlc
en donde
A = Absorbancia
ε = Absortividad molar
l = longitud de la celda
c = concentración molar

Esta relación suele utilizarse para determinar la concentración de un determinado analito a partir del
registro de la absorbancia de estándares de concentración conocida. Otro procedimiento analítico
que se basa en esta misma relación matemática es la llamada titulación fotométrica.

La valoración fotométrica es un procedimiento analítico que permite detectar el punto final de una
titulación utilizando medidas de absorbancia a una longitud de onda donde uno de los reactivos o de
los productos de una reacción absorba. Experimentalmente, las medidas de absorbancia se realizan
después de cada adición del titulante y la gráfica obtenida consta de dos segmentos de rectas, uno
anterior y otro posterior al punto estequiométrico, las cuales se extrapolan; el punto final se localiza
en la intersección de ambas.

Si la reacción no es cuantitativa la gráfica presenta una curvatura en la cercanía del punto de


equivalencia; sin embargo, en los puntos alejados de la equivalencia hay un exceso de reactivos que
desplazan el equilibrio y hacen que la reacción sea más cuantitativa. Por lo tanto se pueden tomar
estos puntos para trazar los segmentos de rectas y extrapolar para determinar el punto final de la
titulación. Este método se puede utilizar para determinar el contenido de arsénico y antimonio en
una muestra que contenga a ambos.

El arsénico y el antimonio son metales que pueden estar presentes en aguas subterráneas que son
utilizadas para consumo y esto significa un grave problema de salud al no cumplir con los límites
máximos permitidos por la Organización Mundial de la Salud (10 µg/L para arsénico y 5 µg/L para
antimonio en agua).

499 
En algunos Estados de la República Mexicana este problema es preocupante por lo que algunos
profesores y estudiantes de la Universidad de Guadalajara decidieron estudiar la posibilidad de
determinar su contenido por medio de una valoración fotométrica realizada en medio KBr usando
KBrO3 como reactivo titulante. Con este reactivo se genera Br2 que puede oxidar al As(III+) y al
Sb(III+).

El medio utilizado es HCl de elevada concentración para asegurarse que estos elementos se
encuentren en forma de cloruros de As(III) y Sb(III). La longitud elegida para la titulación es
350 nm, donde únicamente absorben SbOCl y Br2.

Datos:
Eo (V/ENH a pH = 0): BrO3-/Br2 = 1,48; Br2/Br- = 1,06

(AsO2)+/(AsO)+ = 0,55 V (simplificado puede escribirse As(V)/As(III)

(SbO2)+/(SbO)+ = 0,72 V (puede escribirse Sb(V)/Sb(III)

PROCEDIMIENTO

En el medio de HCl de elevada concentración y con un exceso de KBr se tituló, con una disolución
patrón de KBrO3 1x10-4 mol/L, una alícuota de 100,00 mL de agua procedente de un pozo de agua para
consumo. Las medidas de absorbancia obtenidas (con una celda de 1 cm de longitud de paso óptico)
después de cada adición y en la longitud de onda de 350 nm, se muestran en la gráfica siguiente. En ella
se observan dos puntos de inflexión para los volúmenes de 4,9 y 9,1 mL.

Titulación fotométrica de As(III) y Sb(III)
0.45
A 0.40
B 0.35
S 0.30
O
0.25
R
0.20
B
0.15
A
0.10
N
0.05
C
I 0.00
0 1 2 3 4 5 6 7 8 9 10 11 12 13 14
A
V (mL de KBrO3)

500
PREGUNTAS
I.1. Escribe la ecuación balanceada que representa la generación del bromo.

I.2. Completa las ecuaciones balanceadas para las reacciones del bromo con AsO+ y SbO+.

AsO+ + Br2 →
SbO+ + Br2 →

I.3. Indica cuál es la especie química que se titula primero.

I.4. ¿Cuál es la concentración (en mol/L) de As(V) en la muestra de agua? Indica tus cálculos.

I.5. ¿Cuál es la concentración (en mol/L) de Sb(V) en la muestra de agua? Indica tus cálculos.

I.6. Calcula la absortividad molar de la especie SbO2+ en la longitud de onda de 350 nm.

I.7. ¿Cuál es la concentración de Br2 cuando el volumen total de KBrO3 añadido es igual a 13 mL?
No consideres el efecto de dilución por adición de titulante.

I.8. Indica cuáles son las concentraciones de As(III) y Sb(III) en la muestra de agua (en µg/L) y, en
consecuencia, si esta agua es apta para su consumo.

501 
I.9. Uno de los estudiantes que trabaja en este proyecto quiso calcular la absortividad molar del
bromo en la longitud de onda de medida; para ello tomó 5,00 mL de una disolución de KBrO3
0,001 mol/L a la que añadió KBr, HCl y agua hasta completar un volumen de 10,00 mL. Midió
una absorbancia de 0,6 a esta disolución en una celda de longitud de paso óptico igual a 2 cm.
¿Cuál es el valor de la absortividad molar del bromo?

II. FISICOQUÍMICA.
El ciclo de Carnot es un ciclo termodinámico que se puede utilizar para representar una máquina
térmica o un refrigerador dependiendo del sentido en el que se recorra el ciclo.
Como máquina, el ciclo se recorre en la secuencia indicada en el diagrama a través de las siguientes
etapas:

 Expansión isotérmica reversible (III)


(absorbe Qabs de la fuente caliente Tc)
P   
I  Expansión adiabática reversible (IIIII)
(proceso adiabático Q = 0, el gas se enfría
II
hasta Tf)

 Compresión isotérmica reversible (IIIIV)


(el gas cede Qced a la fuente fría Tf)
IV
III  Compresión adiabática reversible (IVI)
(proceso adiabático Q = 0, el gas se calienta
hasta Tc)

La eficiencia “” de la máquina térmica se puede evaluar a través de las temperaturas de la fuente
caliente y la fuente fría:  = (Tc – Tf)/Tc, o bien, a través de la relación del trabajo efectuado durante
el ciclo sobre los alrededores entre el calor absorbido de la fuente caliente,  = – Wc/Qabs, donde el
trabajo Wc es la suma del calor absorbido (Qabs) más el calor cedido (Qced) a los alrededores.
En los procesos isotérmicos se cumple que PiVi = PfVf, en tanto que en los adiabáticos reversibles
se cumplen: PiVi = PfVf; TiVi(-1) = TfVf(-1); TiPi(1-) = TfPf(1-).

502
Problema II.1: Un gas diatómico ( = 7/5) realiza el ciclo de Carnot de la figura. Inicialmente se
encuentra a una presión de 8 atm y ocupa un volumen de 10 L a una temperatura de 400 K. En la
primera etapa se expande isotérmicamente hasta un volumen de 20.0 L y a continuación se expande
adiabáticamente hasta que alcanza una temperatura de 300 K.
En la segunda parte del ciclo, se comprime isotérmicamente a 300 K y posteriormente se comprime
adiabáticamente hasta alcanzar el estado inicial.

Encierra con un círculo la respuesta correcta de las siguientes preguntas


II.1.1. La eficiencia del ciclo es:

a) 0.250 b) 0.333 c) 0.750 d) 1.33

II.1.2. Una máquina similar a la anterior tiene una eficiencia de 0.45, si el calor absorbido por esta
máquina de Carnot es de 45.78 kJ, la cantidad de calor cedido (Qced) a los alrededores es:

a) – 20.6 kJ b) –25.18 kJ c) –45.78 kJ d) +20.6 kJ

II.1.3. Si una máquina opera con una fuente caliente a 400 K, y tiene una eficiencia de 0.35, ¿cuál
debe ser la temperatura de la fuente fría?

a) 91 K b) 140 K c) 169 K d) 260 K

Considera el enunciado original del Problema.


II.1.4. La presión en el punto II es:

a) 2 atm b) 4 atm c) 6 atm d) 8 atm

503 
II.1.5. El volumen en el punto III es:

a) 26.7 L b) 29.9 L c) 41.1 L d) 54.7 L

II.1.6. La presión en el punto III es:

a) 0.487 atm b) 1.46 atm c) 2.59 atm d) 2.67 atm

II.1.7. El volumen en el punto IV es:

a)20.5 L b) 31.5 L c) 38.0 L d) 40.7 L

II.1.8. La presión en el punto IV es:

a) 2.35 atm b) 2.46 atm c) 2.92 atm d) 3.15 atm

504
Problema II.2: La hidrogenación del eteno (C2H4) para formar etano (C2H6) generalmente requiere
un catalizador que reduce la energía necesaria para llegar al estado de transición. En esta gráfica se
muestran los cambios de energía, en kJ/mol, asociados al avance de la reacción, para el proceso sin
catalizador y con catalizador.


E
N
E
R
G 30
Í
A

kJ/mol

20

ETENO

10 ETANO

AVANCE DE LA REACCION 

Analiza esta gráfica y contesta las siguientes preguntas:

II.2.1. La energía de activación para el proceso sin catalizador es _____________ kJ /mol

II.2.2. La energía de activación para el proceso con catalizador es ____________ kJ /mol

II.2.3. La reacción de hidrogenación del eteno es __________________________________


(Debes elegir si es ENDOTÉRMICA o EXOTÉRMICA)

505 
II.2.4. Para la hidrogenación de 100 g de eteno, calcula la energía liberada o consumida.
Cálculos:

La energía es: ____________________ kJ/mol

Para que tu respuesta sea correcta en las preguntas II.2.1, II.2.2 y II.2.4, deberás agregar un signo
positivo o negativo según sea el caso. Recuerda que el uso de estos signos es importante para indicar
si se trata de un proceso en el que se libera energía o en el que se consume energía.

II.2.5. Si en un reactor de hidrogenación, de volumen constante e igual a 20.0 litros, se colocan 100
g de eteno y 100 g de hidrógeno gaseoso a una temperatura inicial de 20 oC. Para un tiempo en
que no se ha iniciado la reacción, calcula la presión, en bar, dentro del reactor considerando
comportamiento ideal. R = 8.314 x 10-2 L bar/K mol.
Cálculos:

P inicial _________ bar

II.2.6. Calcula la cantidad de sustancia en moles gaseosas dentro del reactor al completarse la
reacción.

Moles gaseosas al concluir la reacción: _____________

506
III. QUÍMICA INORGÁNICA.
A continuación se enlistan los primeros 4 potenciales de ionización de dos distintos elementos
químicos (en kJ/mol).

III.1. De entre la siguiente lista de elementos químicos propón a cuál corresponde cada uno de éstos.
1er potencial de 2do potencial de 3er potencial de 4to potencial de
kJ/mol
ionización ionización ionización ionización
Elemento 1 737 1450 7732 10540
Elemento 2 717 1509 3284 4940
Na, Al, Mg, Ne, B, Li, Mn.

ELEMENTO 1:________________ ELEMENTO 2:________________

Considera las siguientes especies todas ellas en estado gaseoso: Cl-, Ca2+, S2-, K+, Ar.

III.2. ¿Cuál de ellas esperarías que tuviese la mayor energía de ionización? R: _______________

III.3. ¿Cuál de ellas presentará la menor afinidad electrónica? R: ______________

III.4. Considerando que entre los halogenuros de los metales alcalinos la constante de Madelung no
varía significativamente, de entre de esas sustancias en cuál se espera observar la mayor energía
de red cristalina. R: _______________

III.5. ¿Cuál de las siguientes reacciones esperarías fuese más exotérmica?


a) El segundo potencial de ionización del Ca.
b) La primera afinidad electrónica del nitrógeno.
c) La segunda afinidad electrónica del oxígeno.
d) El primer potencial de ionización del F-(g).
e) La primera afinidad electrónica del Na.
R: ______________

III.6. Cuando el 215At emite un positrón y dos partículas α, ¿cuál núclido genera?
R: ______________

III.7. De entre los siguientes diagramas que representan a la distribución de cierto número de
electrones en los orbitales 4s y 3d, selecciona de entre aquellos:

a) Describen a un ion metálico (d4 o d5) en su estado basal. R: _______________

b) Violan exclusivamente la regla de Hund. R: _______________

c) Violan tanto el principio de exclusión como la regla de Hund. R: ______________

d) No cumplen exclusivamente el principio de exclusión. R: ______________

e) Describe al correctamente al ion [Cr(OH)(H2O)5]+. R: _______________


(Considera que el oxígeno es un donador de campo débil).

507 
(A) (B) (C) (D)

(E) (F) (G) (H)

(I) (J) (K) (L)

III.8. Empleando el modelo de repulsión de los pares electrónicos de la capa de valencia selecciona
de entre los siguientes compuestos nitrogenados:
a) NO2- ( ONO) b) NO3- ( ONO) c) H3CNO2- ( ONO)
d) NO2 ( ONO) e) (CH3)2NOH ( CNO) f) C5H5N (piridina) (
CNC)
g) NH4+ ( HNH)

a) ¿En cuál de ellos el ángulo seleccionado es el mayor? R: _____________

b) ¿En cuál de ellos el ángulo seleccionado es el menor? R: _____________

III.9. El decaimiento radioactivo del uranio-238 da origen a una serie de elementos químicos que
terminan en plomo-206 (el cual es estable y ya no decae). Considerando que en esta serie
radioactiva sólo se producen emisiones de partículas α y β-, ¿cuál(es) de los siguientes núclidos
forman parte de esa familia radioactiva?

a) ଶଷସ
ଽଵܺ
ଽଶ
b) ଶଷସܺ c) ଶଶ଻
଼଼ܺ d) ଼ସܺ

e) ସଽܺ f) ସଶܺ g) ଶଷସ


ଽ଴ܺ

R: _____________

508
III.10. Estados de oxidación:
¿Cuál(es) de los siguientes compuestos no esperarías que fuesen posibles para el antimonio?
a) SbO b) NaSbO4 c) Na2SbO4 d) Sb2O5

e) SbF3 g) H2SbO3 h) SbOCl3

R: _____________

III.11. De acuerdo a la serie espectroquímica: CO > CN > SCN> PR3 > OH > H > H2O > X. ¿En
cuál(es) de los siguientes compuestos de coordinación esperarías que tuviesen el mayor momento
magnético?

a) K3[Ru(Cl)6] b) K4[RuCl6] c) [Cr(CO)4Cl2]Cl d) [Ni(H2O)6]2+

e) K4[Fe(CN)6] R: _____________

III.12. Considerando que el efecto trans sigue el siguiente orden: CO > CN- > NO2- > Cl- > NH3

¿Cuál(es) de los siguientes métodos te permitiría(n) obtener principalmente el isómero trans de


[PtX2(NO2)2]?

a) K2[Pt(CN)4] + 2 equivalentes de NO2-


b) cis-K2[PtCl2(CN)2] + 2 equivalentes de NO2-
c) K2[PtCl4] + 2 equivalentes de NO2-
d) trans-K2[PtCl2(CN)2] + 2 equivalentes de NO2-
R: _____________

III.13. De la siguientes especies ¿cuál(es) esperarías que fuese(n) paramagnética(s)?

Na(g), CH4O-, NO2-, O2-, CH3-, O3, NO2+, Cr2+(g), BH3(g). R: _____________

III.14. La estructura cristalina del LiF es isomorfa a la de NaCl, pero como era de esperarse, la celda
unitaria de LiF es más pequeña con a = 4.03 Å, mientras que en NaCl a = 5.65 Å. Ambos sólidos
presentan una estructura cúbica en la que a=b=c y α=β=90°. Calcula la densidad que se esperaría
observar para el LiF (en g/cm3).

R: Densidad del LiF(s) _____________

509 
IV. QUÍMICA ORGÁNICA.
IV.1. Indica todos los pasos, condiciones y reactivos, que son necesarios para efectuar la siguiente
transformación:

510
IV.2. Escribe los productos e intermediarios de la siguiente secuencia sintética:

511 
IV.3. Considera la siguiente secuencia sintética:

Las estructuras de los compuestos que se forman en la ruta sintética anterior son:

¿Cuál de las siguientes opciones muestra en forma correcta las estructuras (1) a (6) en la
secuencia sintética para las estructuras A a la F?

A B C D E F
a) (1) (2) (3) (4) (5) (6)
b) (2) (3) (4) (6) (5) (1)
c) (3) (2) (6) (4) (5) (1)
d) (2) (3) (6) (4) (5) (1)

512
IV.4. El Cembreno, o en algunos casos Neocembreno, es un diterpeno natural que se encuentra en
la resina del pino. El Cembreno, C20H32, presenta una fuerte absorción en el UV a 245 nm.
Cuando el Cembreno se hidrogena parcialmente con el catalizador de Lindlar (Pd/Ba(OAc)2), se
obtiene el Dihidrocembreno, C20H34, el cual ya no presenta absorción en el UV.
Cuando se lleva a cabo la hidrogenación exhaustiva del Cembreno, empleando el catalizador de
Adams (Pt), se obtiene el Octahidrocembreno, C20H40.
Cuando el Cembreno se hizo reaccionar con ozono y posteriormente con sulfuro de dimetilo, se
obtuvieron los siguientes productos:

Propón una estructura razonable para el Dihidrocembreno, el Cembreno y el


Octahidrocembreno. En el caso de la estructura del Cembreno, esta debe ser consistente con la
regla isoprénica.

Fin del Examen Internacional Total de preguntas: 41 Tiempo asignado: 4 horas

513 
514
XXIV Olimpiada Nacional de Química
Examen Experimental de Química Analítica NIVEL A y B Guadalajara 2015

Código en el laboratorio: __________________

IDENTIFICACIÓN DE LA SAL DE UN CATIÓN ALCALINO Y


DETERMINACIÓN DE SU CONSTANTE DE SOLUBILIDAD

El laboratorio de Química Analítica de esta Universidad de Guadalajara recibió un donativo con


once frascos que contienen diferentes sales de cationes alcalino térreos de calidad R. A.
Desafortunadamente las etiquetas están deterioradas y no ha sido posible identificarlas en forma
individual aunque si se conoce cuáles son las sustancias donadas.
Tu labor esta mañana será ayudarnos a identificar una de ellas y calcular el valor de su constante de
solubilidad. Para esta tarea se te proporcionan dos tubos de ensaye que contienen un poco de la sal.
Para calcular su constante de solubilidad se requiere tener una disolución saturada de dicha sal; para
ello se requiere permitir alcanzar el equilibrio de solubilidad a una temperatura controlada y tomar la
disolución sobrenadante.

Las sales donadas son:


Mg(OH)2, Sr(OH)2, Ca(OH)2, Ba(OH)2, MgC2O4, CaC2O4, BaC2O4, SrC2O4, CaCO3, BaCO3 y
MgCO3.

MATERIALES Y REACTIVOS

Individuales

Bureta con soporte y pinzas


1 pipeta volumétrica (de 5 ó 10 mL)
3 matraces Erlenmeyer
2 tubos de ensaye con la sal a identificar (colocados en vasa o gradilla)
1 frasco con disolución saturada de la sal
1 frasco con disolución de ácido c1orhídrico de concentración conocida

Colectivos

Gotero con disolución de permanganato de potasio en medio ácido


Gotero con disolución de ácido c1orhídrico
Mechero de alcohol
Frascos lavadores con agua destilada
Lápices con punta

PROCEDIMIENTO
Reconocimiento del catión de la sal

Como es de tu conocimiento, muchos de los cationes de los metales alcalinos y alcalino térreos
pueden ser identificados por el color que proporcionan a la flama de un mechero, este color se
manifiesta cuando sus electrones de valencia son excitados por el calor de una f1ama y emiten la

515
energía absorbida para regresar a su estado basal; esta radiación corresponde a la región del espectro
electromagnético de la zona del visible. Cabe mencionar que este fenómeno se observa para muchos
cationes pero no siempre puede ser observado pues en algunos casos la emisión no corresponde a la
zona visible o el calor proporcionado no es suficiente para la excitación.

Esta prueba te permitirá decidir cuál es el catión que contiene la sal que deberás identificar pero,
antes de comenzar el procedimiento, deberás recordar cuáles son los resultados que esperarías para
cada una de ellas.

Completa la siguiente tabla.

Catión Color que proporciona a la flama


Ca2+
Ba2+
Sr2+
Mg2+

IDENTIFICACIÓN DEL ANIÓN DE LA SAL

Para identificar el anión de la sal cuentas con las disoluciones de dos reactivos.
Antes de iniciar el procedimiento completa la siguiente información.

1. El permanganato de potasio en medio ácido te podrá indicar si el anión de la sal es: ___________
porque se observará _________________________________ como consecuencia de la siguiente
reacción balanceada: __________________________________________________________

2. El ácido clorhídrico te permitirá indicar si el anión de la sal es: __________________ porque se


observará ________________________ como consecuencia de siguiente reacción balanceada:
_______________________________________________________

PROCEDIMIENTO

1. Tomar un poco de la sal sólida con la punta de un lápiz humedecida en ácido clorhídrico y
acércala a la flama del mechero de alcohol. Observa y anota tus observaciones.

2. Coloca en el matraz Erlenmeyer una alícuota (5,00 ó 10,0 mL) de disolución saturada de la
muestra, añade unas gotas de fenolftaleína y titula con la disolución valorada de HCl de
concentración conocida.

3. Efectúa esta titulación por triplicado.

NOTAS
1. Para ahorrar tiempo hemos realizado el paso del procedimiento de equilibrio heterogéneo y te
proporcionamos aproximadamente 40 mL de una de las disoluciones saturadas.
2. Si requieres un volumen adicional de cualquiera de las dos disoluciones se te podrá proporcionar
pero esto implica una penalización en la calificación final.
3. Deberás verter los desechos de las titulaciones efectuadas en el frasco de residuos proporcionado.

516
XXIV Olimpiada Nacional de Química
Examen Experimental de Química Analítica NIVEL A y B Guadalajara 2015

Código en el laboratorio: __________________

HOJA DE RESPUESTAS

I. Como resultado del ensayo a la flama el catión de la muestra proporcionada es: ____________

II. Las pruebas efectuadas con las disoluciones de ácido clorhídrico y permanganato en medio ácido
te permiten indicar que el anión en tu muestra es: ____________________________________

III. Escribe la ecuación balanceada de la reacción de neutralización involucrada en la titulación:


____________________________________________________________________________

IV. Registra en la siguiente tabla los datos de los volúmenes utilizados en las tres titulaciones:

Titulación Volumen de la alícuota de la Volumen de la disolución de ácido


No. disolución saturada de tu sal clorhídrico
1
2
3

V. Volumen de ácido clorhídrico que utilizarás en tus cálculos: _____________________

VI. ¿Cuál es la concentración del anión de la muestra problema?

VII. ¿Cuál es la solubilidad de la sal?

VIII. Calcula cuál es el producto de solubilidad de la muestra.

517
518
XXIV Olimpiada Nacional de Química 2015
EXAMEN EXPERIMENTAL DE QUÍMICA ORGÁNICA Guadalajara 2015

OBTENCIÓN DEL ÁCIDO ACETILSALICÍLICO POR MEDIO DE UN


PROCESO DE QUÍMICA VERDE

OBJETIVO.
Que el alumno:
a) Efectúe la síntesis de un derivado de un ácido carboxílico como lo es un éster.
b) Sintetizar ácido acetilsalicílico a nivel microescala.
c) Sintetizar el ácido acetilsalicílico por un proceso de química verde.

ANTECEDENTES.
El Ácido Acetilsalicílico es la droga maravillosa por excelencia. Se utiliza ampliamente como
analgésico (para disminuir el dolor) y como antipirético (para bajar la fiebre). También reduce la
inflamación y aún es capaz de prevenir ataques cardiacos. Debido a que es fácil de preparar, la
aspirina es uno de los fármacos disponibles menos costosos. Es producida en grandes cantidades, de
hecho cada año la producción a nivel mundial es ≈40 000 toneladas de aspirina, y sólo en los
Estados Unidos más de 50 millones de gentes consumen de 10 a 20 mil millones de tabletas de
aspirina (circ.ahajournals.org/content/123/7/768.full).

El nombre comercial de Aspirina deriva del vocablo Spirea, que en botánica designa una familia de
plantas y de ahí la sílaba "spir". La letra "A" indica el proceso de acetilación al que se somete el
ácido salicílico para convertirse en ácido acetilsalicílico. La sílaba final "in" era una terminación
empleada con frecuencia para los medicamentos, en aquella época.

519
OBJETIVO.

En esta práctica se va a llevar a cabo una reacción en la que se ilustra un proceso de química verde,
a través del cual se lleva a cabo una reacción de acilación de un fenol, utilizando como sustrato el
ácido salicílico para obtener el éster correspondiente, el ácido acetilsalicílico, empleando anhídrido
acético como agente acilante y en presencia de bases de metales alcalinos (e.g. hidróxido de sodio,
hidróxido de potasio o bien carbonato de sodio). La reacción se lleva a cabo a temperatura ambiente.

QUÍMICA VERDE.
El objetivo de la química verde es desarrollar tecnologías químicas benignas al medio ambiente,
utilizando en forma eficiente las materias primas (de preferencia renovables), eliminando la
generación de desechos y evitando el uso de reactivos y disolventes tóxicos y/o peligrosos en la
manufactura y aplicación de productos químicos.

Para determinar qué tan benéfica al medio ambiente es una reacción o proceso, se pueden calcular
varios parámetros, dentro de los cuales está el porcentaje de economía atómica, que se define de la
siguiente manera:1

 Masa Molar Producto 


Porcentaje de economía atómica    x 100
 Masa Molar Reactivos 

En este experimento se lleva a cabo un proceso de química verde, en el cual se efectúa la siguiente
reacción:

O
OH O 1) NaOH
O O (o KOH O O
OH + o Na2CO3) O
O 25 C 0
+
OH
OH
Ácido Salicílico 2) HCl
Anhídrido Acético
Ácido Acético
Ácido Acetilsalicílico

PROCEDIMIENTO:2,3
En el frasco etiquetado como Ácido se encuentra ya pesado el ácido salicílico. Es importante que
veas y escribas en este examen la cantidad que se pesó y que se encuentra escrita en donde está la
clave que se te asignó para poder realizar este experimento. Adiciona 1.2 mL de anhídrido acético
(los cuales se encuentran en el frasco etiquetado como anhídrido). Con una espátula mezcla bien los
dos reactivos. Una vez que obtengas una mezcla homogénea, adiciona 0.5 g de NaOH (esta cantidad
ya está pesada y se encuentra dentro del frasco etiquetado NaOH) y agita nuevamente la mezcla con
la espátula por 10 minutos. Al acabar de agitar, ya debe de estar formado el producto (sólido color
blanco). Adiciona lentamente 6 mL de agua destilada y posteriormente una disolución de ácido
clorhídrico al 50% (se encuentra dentro del frasco etiquetado HCl 50 %) hasta que el pH de la
disolución sea ácido (papel tornasol color rojo). La mezcla se deja enfriar en un baño de hielo. El
producto crudo se aísla por medio de una filtración al vacío. Los cristales del ácido acetilsalicílico
impuro se pueden bajar con agua fría (Es importante que el agua esté bien fría y que sea poca la
cantidad que se utilice, ya que el ácido acetilsalicílico es ligeramente soluble en agua).

520
El producto crudo lo debes de aislar por
filtración al vacío, empleando el
embudo Büchner y el matraz Kitasato.
Debes armar el equipo como se muestra
en la figura 1.

Figura 1. Equipo para filtrar al vacío

Purifica el producto crudo por medio de una recristalización, para lo cual debes disolver el producto
crudo con 5 mL de acetato de etilo caliente (se encuentran ya medidos en el frasco etiquetado
AcOEt). Una vez disuelto, adiciona 5 mL de hexano (se encuentran dentro del frasco etiquetado
hexano) y enfría la mezcla dentro de un baño de hielo. Aísla los cristales por medio de una filtración
al vacío.

Determina la masa del producto que obtuviste. Mide la masa de la bolsa de plástico que se te
proporcionó y después coloca el producto y vuelve a medir la masa del contenido. Uno de los
maestros te debe firmar las dos medidas:

Masa de la bolsa: Firma del profesor:

Masa de la bolsa + producto Firma del profesor:

ANÁLISIS POR CROMATOGRAFÍA EN CAPA FINA


INTRODUCCIÓN TEÓRICA
La cromatografía en capa fina es una técnica de adsorción sólido-líquido, utilizada en química
orgánica para realizar un análisis cualitativo eficiente y rápido de compuestos desconocidos o para
determinar la composición de una mezcla de compuestos.
Esta técnica cromatográfica consiste en la utilización de una fase estacionaria (alúmina o sílica-gel)
y de una fase móvil, que son los disolventes orgánicos de diferente polaridad.
El proceso de separación se basa en que la fase móvil asciende a través de la fase estacionaria, la
elución de la muestra problema está en función de su polaridad, es decir de la afinidad que presente
con alguna de las dos fases.
La fase estacionaria consiste de una película delgada (100 m) de un material disperso sobre una
superficie plana (de vidrio, aluminio o celulosa).
Las ventajas del método son la rapidez para llevar a cabo el análisis (un análisis normal toma de 2 a
10 minutos) y se pueden detectar cantidades de material de 2 a 20 g.

521
La secuencia de pasos que se siguen para llevar a cabo el análisis por cromatografía en capa fina
son:

1) Se traza una línea paralela a la base de la cromatoplaca (aproximadamente a 0.5 cm de la base de


la misma) sobre la superficie del adsorbente con un lápiz (NOTA: no recargues mucho la punta del
lápiz, ya que se puede llegar a romper la superficie de la fase estacionaria). La muestra que se va a
analizar (1 mg o menos) se coloca en un tubo de ensayo, y se adicionan unas gotas del disolvente
hasta observar la disolución de la muestra. Con una aguja de calibre pequeño, que funciona como un
tubo capilar, se aplica una pequeña fracción de la solución sobre la placa. Debes limpiar el capilar
con acetato de etilo antes aplicar una muestra.

0.5 cm
parte superior límite de ascensión
del disolvente

parte inferior X X punto de aplicación


0.5 cm

2) La cromatografía se lleva a cabo colocando la cromatoplaca sobre la cual se aplicó la muestra,


dentro de un frasco de vidrio con tapa, el cual ya debe contener la fase móvil (unos 5.0 mL). El
frasco debe estar cerrado con la finalidad de mantener una atmósfera saturada con el disolvente de la
fase móvil y debe contener un pedazo de papel filtro para asegurar dicha atmósfera.

Efectuadas estas operaciones, la fase móvil asciende rápidamente por capilaridad sobre el
adsorbente de la fase estacionaria hasta la marca superior (0.5 cm antes de la parte superior).
Inmediatamente se saca la placa de la cámara de elución y se deja evaporar el disolvente. La muestra
se puede encontrar en cualquier punto.

522
3) Una medida física de la polaridad de la muestra es el Factor de Retención (Rf), el cual se
determina dividiendo la distancia que recorrió la muestra entre la distancia que recorrió el
disolvente, el Rf siempre es menor a 1. Bajo las mismas condiciones de análisis cromatográfico, el
Rf es constante y permite identificar cualitativamente un compuesto o bien determinar si se
encuentra presente en una mezcla problema:

línea hasta la que recorrió


el eluyente

b a
Rf =
b
a

x punto de aplicación
de la muestra

Tú vas a efectuar un análisis comparativo entre una pastilla de Aspirina marca BayerMR (se
encuentra dentro del Vial 2) y el producto puro que acabas de obtener en este experimento y el cual
debes poner en el Vial 1. Aplica las dos muestras disueltas cada una en acetato de etilo y aplícalas
en la cromatoplaca como se indica en la siguiente imagen.

Una vez que dejes eluir el eluyente (una mezcla acetona:hexano (50:50) que ya se encuentra dentro
de la cámara de elución), saca la cromatoplaca de la cámara con unas pinzas y deja evaporar el
eluyente. Revela la cromatoplaca con una lámpara de luz Ultravioleta. Un maestro te va a ayudar a
ver las manchas. Con un lápiz marca el contorno en presencia del maestro y el cual debe firmar
dándote el visto bueno.

Determina el Rf tanto de la materia prima como del producto puro. Debes entregar la cromatoplaca,
colócala dentro de la otra bolsa de plástico y pide al profesor que engrape la bolsa, junto con la bolsa
que contiene el producto que obtuviste.

523
Cálculo del Rf de la materia prima y del producto puro:

Visto bueno del profesor en cuanto al Firma del profesor:


contorno de las dos muestras

CALCULO DEL RENDIMIENTO

Reactivos y Fórmula Masa Molar Densidad


disolventes Molecular (g/mol) (g/mL)
Ácido C7H6O3 138.1220
salicílico
Anhídrido C4H6O3 102.0890 1.08
acético
Hidróxido de HNaO 39.9968
sodio
Ácido C9H8O4 180.1590
acetilsalicílico

524
A) REACTIVO LIMITANTE.
De acuerdo con los datos que se te proporcionan en la tabla anterior, ¿cuál es el reactivo limitante?
Indica con claridad tus cálculos:

Reactivo limitante:

B) RENDIMIENTO.
De acuerdo con la cantidad de producto que obtuviste, calcula el rendimiento con el que obtuviste el
ácido acetilsalicílico.

525
ASPECTOS TEÓRICOS DEL EXPERIMENTO

1. Escribe el mecanismo de la reacción que llevaste a cabo, indicando claramente la función del
hidróxido de sodio:

2. a) ¿Se podría haber utilizado ácido sulfúrico en lugar de hidróxido de sodio? R: ______________

b) En el caso de que tu respuesta sea si, escribe el mecanismo de la reacción de esterificación del
ácido salicílico y el anhídrido acético en presencia de ácido sulfúrico.

526
BIBLIOGRAFÍA

1) Curzons, A. D.; Constable, D. J. C.; Mortimer, D. N.; Cunningham, V. L.; Green Chemistry 3, 1-
6, 2001.
2) Handel-Vega, E.; Loupy, A. P. D.; Collazo, J. M.; Pat. WO1998IB02083 19981218.
3) Zhong, G.-Q.; Hecheng Huaxue 11, 160-162, 2003.

527
528
 

RESPUESTAS

529
 

530
XV Olimpiada Nacional de Química. 1er Examen Nivel A y B.
RESPUESTAS

1) 3 6) 20 11) 7 16) 47 21) 82 26) 12


2) 9 7) 13 12) 16 17) 78 22) 11 27) X
3) 12 8) 19 13) 80 18) 14 23) 63 28) 3
4) 21 9) 19 14) 11 19) 23 24) 10 29) 1
5) 7 10) 1 15) 54 20) 8 25) 26 30) 78

XV Olimpiada Nacional de Química. 2o Examen Nivel A y B.


RESPUESTAS

m3
1.013x10 5 Pa(20x10 -16 )
PV' s mol
1.1) n' Gas   3
 8.1732 x10  4
RT Pa m s
8.314 (298.15 K)
mol K
mol
n Gas  n' Gas (t)  8.1732 x10  4 (3600 s)  2.9424 mol
s
nO2  5(2.9424 mol)  14.712 mol

nO2 14.712 mol


1.2) n O 2  0.20n Aire  n Aire    73.56 mol
0.20 0.20

1.3) n N 2  0.80(73.56 mol)  58.847 mol


28 g
W N 2  n N 2 M N 2  58.847 mol x  1647.7 g
1 mol

1.4) Por cada mol de gas son 4 mol de H2O, entonces:


n H 2O  4n Gas  4(2.9424 mol)  11.770 mol

1.5) Al final de la combustión con aire se tiene por cada mol de gas:
n Total  n CO 2  n H 2O  n N 2  3 mol  4 mol  20 mol  27 mol
3 mol
 CO 2   0.1111
27 mol

1.6) C3H8 (g) + 5 O2 (g)  3 CO2 (g) + 4 H2O (g)


(+ 20 N2)
m Total  m CO 2  m H 2O  m N 2  (3x44)  (4x18)  (20x28)  132  72  560  764 g
560 g
% mN  x100  73.3 %
2
764 g

2. a) 3. e) 4. e) 5. d) 6. c) 7. c) 8. b)

9. a)
531
10. F > O > Br > B > Li > Cs

11. c) 12. e)

13.
5
W

1
P O L O N I O

2
A Z U F R E

3
C A R B O N O

4
S I L I C I O

14. d) 15. d) 16. c) 17. d)

18.

O O O

C C C
O O O O O O

19. d) 20. d) 21. c)

532
22.
REACCIÓN (a) REACCIÓN (b) REACCIÓN (c) REACCIÓN (d)

S A o S E A

REACCIÓN (e) REACCIÓN (f) REACCIÓN (g)

S o A S T

XV Olimpiada Nacional de Química. 3er Examen Nivel A.


RESPUESTAS

1. MM(CuSO4) = 159.55 g mol-1, MM(H2O) = 18 g mol-1

a) 2.574 g CuSO4xH2O – 1.647 g CuSO4 = 0.927 g H2O

1.647 g CuSO4 = 0.0103 mol CuSO4

0.927 g H2O = 0.0515 mol H2O

mol CuSO 4 0.0103 mol 1


Relación   Fórmula: CuSO45H2O
mol H 2 O 0.0515 mol 5

b) 2.574 g CuSO4xH2O – 1.833 g CuSO4yH2O = 0.741 g H2O

1.647 g CuSO4 = 0.0103 mol CuSO4

0.741 g H2O = 0.0515 mol H2O

mol CuSO4 0.0103 mol 1


Relación   Fórmula: CuSO4H2O
mol H 2 O 0.0412 mol 4

0.812 g Cu X O
c)  78.84 g Cu X O / mol CuSO 4
0.0103 mol CuSO 4

78.84 g CuXO – 16.00 g O = 62.84 g Cu  CuO

63.55 g Cu 16.00 g O
 79.89 % Cu  20.11 % O
79.55 g CuO 79.55 g CuO

533
0.1131 g MSO 4 233.3 g BaSO 4 1 mol BaSO 4
2. x x  120.32 g/mol MSO 4
0.2193 g BaSO4 1 mol BaSO4 1 mol MSO 4

120.32 g/mol MSO4 – 96 g/mol SO42- = 24.32 g/mol  Magnesio (Mg)

3. 0
E celda 
0.0592
n
log Cu 2  


log Cu 2  añadido 
0.009 x 2
0.0592
 0.30  [Cu2+] = 2 M, y el volumen final se considera
que no cambia

2 mol
x 100 mL x 134.5 g/mol  26.90 g de CuCl 2 añadido
1000 mL

4. a) Kps = [Pb2+][CrO42-] = s2 = 1.77x10-14

s  1.77 x10 14  1.33 x10 7 mol/L

b) Kps = 1.77x10-14 = [Pb2+][CrO42-] = (x)(0.1+x)

Esto da una ecuación de segundo grado, pero un método más sencillo es efectuar la
aproximación de que x es muy pequeña con respecto a 0.1, y por lo tanto (0.55+x)  0.55.
Esta suposición es correcta porque la solubilidad es muy pequeña y será aún más baja en
presencia de un ion común.
1.77 x10 14
Kps = 1.77x10-14 = (x)(0.1)  x  1.77x 10 -13 mol / L
0.1

c) Kps = 10-9.7 = [Ba2+][CrO42-] = (x)(0.1+x)

-9.7 10 9.7
Kps = 10 = (x)(0.1)  x  10 -8.7  2 x10 9 mol/L
0.1

d) 1.77x10-13 mol/L de Pb2+ y 2x10-9 mol/L de CrO42-

32.08 36.01
5. Na   1.395  x  2y  z O  2.25  4y  3z
23 16

19.51
Cl   0.549  x
35.5

Resolviendo las ecuaciones: x = 0.549; y = 0.144; z = 0.558

0.549 x 58.5 g NaCl = 32.11 %; 0.144 x 142 g Na2SO4 = 20.45 %;

0.558 x 85 g NaNO3 = 47.43 %


534
Ka 
[H  ][Base] Ka
  
Base  10 -6.06.3 
Base
6.
Ácido  H   
Ácido  Ácido  2  [Base] = 2[Ácido]

x = 2[C-x]  x = ⅔C  C= 40 mL x 0.2 mol/L = 8x10-3 mol

100 mL
x = 5.3x10-3 mol  5x10 -3 mol x  26.7 mL
0.2 mol

7. Compuesto A Compuesto B

Br

HO CH3 HO CH3

Br Br

8. a) Inciso b b) 24 = 16 estereoisómeros

9. Respuesta A: 1) BH3 (u otro borano) / THF; 2) H2O2, NaOH.

Respuesta B: HBr en presencia de peróxidos.

Respuesta C: H2SO4 + H2O (H+ /H2O)

Respuesta D: KMnO4, KOH, Frío


(OsO4, KOH)
(Perácido, NaOH, H2O)
(Perácido, H2O, HCl)

10. a) n
PV

 
9.5x105 Pa 0.030m3 
 11.694 mol
RT Pa m 3
8.314 293.15 K 
mol K

 3

11.694 mol 8.314 Pa m 293.15 K 
nRT  mol K 
b) V  5
 0.285 m 3
PV 10 Pa

9.5 bar 0.030 m 3 


o por la Ley de Boyle: V  0.285 m 3
1.0 bar

c) Al inicio Vi = 0.285 m3, Ti = 293.15 K


Al final Vf = 0.03 m3, Tf = ?

535
TfVf-1 = TiVi-1   = P/V = 1.4
 -1 1.4 1
V   0.285 m 3 
Tf  Ti  i   293.15 K  3 
  721.4 K
 Vf   0.03 m 

 Pa m 3 
11.694 mol 8.314
 318.15 K 
nRT  mol K 
d) P  3
 1.0311x10 6 Pa  10.311 bar
V 0.030 m

e) Como se enfría dentro del tanque es QV.

QV = nVT = (11.694 mol)(5/2)(8.314 J mol-1 K-1)(20.45 K)

QV = - 6076.5 J

f) Para los alrededores S = Q / T

S = 6076.5 J / 293.15 K = 20.73 J K-1

Tf 5 J  293.15 K 
g) S  n C V ln  11.694 mol x x 8.314 ln    19.89 J K
-1

Ti 2 mol K  318.15 K 

11. Inciso e.

12. (a) 0.80 MJ/mol (b) 1.31 MJ/mol (c) 1.40 MJ/mol (d) 1.08 MJ/mol

13.
A = P4 B = PCl3 C = PCl5 X=4 Z=4

14. ( d ) O3 ( e ) NO- ( g ) B(OH)3 ( h ) NH3

( f ) PF5 ( a ) PF6- ( e ) NO ( c ) XeF4

( i ) IF5 ( a ) XeF6

15. ( c ) NH4+ ( e ) PF6- ( b ) BF3

( a ) BeCl2 ( b ) NO3- ( c ) ClO4-

536
16.
*

*

2p 2p

*

2s 2s

17. Inciso d.

18. Nombre del compuesto __Cloruro de hexaaminrutenio (II)_____________________


Número de coordinación del metal _6____ Estado de oxidación del metal _2+_____
Geometría de la molécula __Octaédrica________

19. Inciso a.

20. Inciso e.

XV Olimpiada Nacional de Química. 3er Examen Nivel B.


RESPUESTAS

Trabajo
1. a) Potencia   Trabajo = (800 J/s)(1800 s) = 1.44x106 J
Tiempo

537
b) n
PV

 
9.5x10 5 Pa 0.030 m 3 
 11.694 mol
RT  J 
 8.314 293.15 K 
 mol K 

 3

11.694 mol 8.314 Pa m 318.15 K 
nRT  mol K 
c) P   1.031x10 6 Pa  10.31 bar
V 0.030 m 3

 5  J 
d) V  nC V T  11.694 mol   8.314 45 K  20 K   6076.5 J
 2  mol K 

e) Qced = V = nCV(293.15 K -318.15 K) = - 6076.5 J

P 
- R ln 2   8.314 J  ln 760 
2. a) H Vap   P1  mol K  92.51 
 42226.6
J
1 1 1 1 mol
 
T2 T1 373.15 K 323.15 K

J
8.314
1 1 R P 1 mol K ln 400 mmHg  2.7951 x 10 -3 K -1
b)  - ln 2  
T2 T1 H Vap P1 323.15 K J 92.51 mmHg
40656
mol

T2 = 357.76 K = 84.62 0C

B B 1718
c) log P  A -  t C   273.52  68.06 0 C
tC A - log P 8.321  log 500

3. MM(CuSO4) = 159.55 g mol-1, MM(H2O) = 18 g mol-1

a) 2.574 g CuSO4xH2O – 1.647 g CuSO4 = 0.927 g H2O

1.647 g CuSO4 = 0.0103 mol CuSO4

0.927 g H2O = 0.0515 mol H2O

mol CuSO 4 0.0103 mol 1


Relación   Fórmula: CuSO45H2O
mol H 2 O 0.0515 mol 5

b) 2.574 g CuSO4xH2O – 1.833 g CuSO4yH2O = 0.741 g H2O

1.647 g CuSO4 = 0.0103 mol CuSO4

0.741 g H2O = 0.0515 mol H2O

538
mol CuSO4 0.0103 mol 1
Relación   Fórmula: CuSO4H2O
mol H 2 O 0.0412 mol 4

0.812 g Cu X O
c)  78.84 g Cu X O / mol CuSO 4
0.0103 mol CuSO 4

78.84 g CuXO – 16.00 g O = 62.84 g Cu  CuO

63.55 g Cu 16.00 g O
 79.89 % Cu  20.11 % O
79.55 g CuO 79.55 g CuO

4. a) Ca(OH)2 + 2 HCl  CaCl2 + 2 H2O (CaO + 2 HCl  CaCl2 + 2 H2O)

g 37 g 1000 mL
6000 L x 1.19 x x
mL 100 g 1L
b)  7.24x10 4 mol HCl
g
36.5
mol

1 mol Ca(OH) 2 74.1 g Ca(OH) 2 1 kg 1 bolsa


7.24x10 4 mol HCl x x x x  134 bolsas
2 mol HCl 1 mol Ca(OH) 2 1000 g 20 kg

c) No fueron suficientes las 100 bolsas para neutralizar el HCl por lo que el río quedo
contaminado.

0.1131 g MSO 4 233.3 g BaSO 4 1 mol BaSO 4


5. x x  120.32 g/mol MSO 4
0.2193 g BaSO4 1 mol BaSO4 1 mol MSO 4

120.32 g/mol MSO4 – 96 g/mol SO42- = 24.32 g/mol  Magnesio (Mg)

0.03795 mL disol.NaOH 0.1 mol NaOH 1 mol HCl mol HCl


6. x x  0.1518
0.025 L disol.HCl 1 L disol.NaOH 1 mol NaOH L disol.HCl

mol HCl 1000 mL mol


[HCl]  0.1518 x  6.072 obtenida por la titulación con NaOH
L disol.HCl 25 mL L

18 g HCl 1.1 g disol.HCl 1 mol HCl 1000 mL mol


[HCl] Comercial  x x x  5.42
100 g disol.HCl 1 mL disol.HCl 36.5 g HCl 1L L

El ácido está más concentrado de lo que indica la etiqueta-

539
32.08 36.01
7. Na   1.395  x  2y  z O  2.25  4y  3z
23 16

19.51
Cl   0.549  x
35.5

Resolviendo las ecuaciones: x = 0.549; y = 0.144; z = 0.558

0.549 x 58.5 g NaCl = 32.11 %; 0.144 x 142 g Na2SO4 = 20.45 %;

0.558 x 85 g NaNO3 = 47.43 %

8.
NH4Cl As NaOH

Hg H2O NaCl

Ác. Acético NaCN NaHCO3

9. Inciso e.

10.

Ru-1 > Ru0 > Ru2+ > Ru3+ > Ru4+ > Ru6+

11. Inciso d.

12. Inciso b.

13. Inciso d.

14.
A= B= C= X= Z=
P4 PCl3 PCl5 4 4

15. Inciso c.

16. Inciso c.

17. Inciso a.

540
XV Olimpiada Nacional de Química. Examen Internacional.
RESPUESTAS

1.
1 2 3

Ar Ir Li

4 5 6

Pt O Mg

7 8 9

I Dy Au

2.
Br
-PINENO COMPUESTO A COMPUESTO B Br

Br OH

C10H16
C10H17BrO

OH
COMPUESTO C COMPUESTO D COMPUESTO E
Br
O
OH

C10H16O C10H18O2

C10H15Br
Br

541
3. a)
O
H H

CH3O:- +Na
H
CH3OH OCH3 + Na+ -:Br
O
CH3O:- +Na
Br

Adición en
el centro
SN2

H O:- +Na
H H
Br OCH3
H
OCH3 H H
O:- +Na
Br

b) Respuesta iii.

4. a)

COMPUESTO A COMPUESTO B
C N C N
HO H H OH
H OH H OH
CH2OH CH2OH

COMPUESTO C COMPUESTO D
CO2H
CO2H
HO H
H OH
H OH H OH
CH2OH CH2OH

542
b)

ÁCIDO (+)-TARTÁRICO ÁCIDO (-)-TARTÁRICO


CO2H CO2H
H OH HO H
HO H H OH
CO2H CO2H

ÁCIDO (meso)-TARTÁRICO

CO2H
H OH
H OH
CO2H

5. Respuesta d.

6. a)
TC  TF 700 K - 500 K
   0.2857
TC 700 K

b) En la expansión isotérmica T = 0 y por lo tanto U = 0, Q = -W.

Vf  J   55 L 
W  nRT ln  (2.75mol) 8.314 700 K  ln   16190 J
Vi  mol K   20 L 

Q= + 16190 J

c) Del calor absorbido una parte se convierte en trabajo y la otra va a la fuente fría.

WC = -  Qabsorbido = -0.2857 x 16190 J = -4625.5 J

La diferencia con el calor absorbido es el cedido a la fuente fría, por lo tanto:

Qcedido = -Wciclo-Qabsorbido = -(-4625.5 J)-(16190 J) = -11564.5 J

1 1
γ 1 γ 1 γ 1 T  γ 1  T  γ 1  700 K  0.4
d) Tf Vf  Ti Vi  Vf   i Vi  Vf   i  Vi    (55 L)  127.55 L
 Tf   Tf   500 K 

543
Vf Q rev  J  55 L J
e.1) S I  nR ln   2.75 mol 8.314 ln  23.129
Vi T  mol K  20 L K

 16190 J J
O bien: S I   23.129
700 K K

e.2) En la expansión adiabática S = 0 para un proceso adiabático reversible.

e.3) En la compresión isotérmica como Sciclo = S = 0 y S = 0 para procesos adiabáticos


reversibles (expansión y compresión).

Sciclo = SI + SII + SIII + SIV = 0  SII = SIV = 0

Por lo tanto: SIII = SI = -23.129 J/K

Vf  J  46.38 L J
O bien: S  nR ln  2.75 mol 8.314 ln  23.129
Vi  mol K  127.55 L K
Pero primero hay que calcular el volumen Vf.

e.4) SIV = 0

e.5) Sciclo = 0

f) S = -23.129 J/K

g) S = +23.129 J/K

7.

0.6

0.5
Absorbancia (u. a.)

0.4

0.3

0.2

0.1

0.0 1.5 3.0 4.5 6.0 7.5 9.0 10.5 12.0 13.5 15.0
-5
Concentración de R (x 10 mol/L)

a) Sólo el complejo.

544
b) 7.5 / 2.5 = 3, por lo tanto: CoR3

A 0.535
c)    21400 L mol -1cm -1
lC
1 cm 2.5 x 10 -5 mol 
 L 

Absorbancia leída en el punto de intersección 0.500 mol


C exp    2.3 x 10 5
L 21400 L
21400 1 cm
mol cm

100 mol 100


% de disociación  C exp x  2.3 x 10 -5  8%
C teórica L mol
-5
2.5 x 10
L

% de disociación
  0.08
100

d)
CoR3  Co + 3R
Co-Co Co 3Co

M L Co(3Co) 3 27 4 2.5 x 10 -5 


3 3

Kc    8.64 x 10 16
ML 3  Co(1 -  ) 1-

8.
2 AgCl  2 Ag + Cl2
2(143.4 g) 71 g

3 Cl2 + 3 H2O  ClO3- + 5 Cl- + 6 H+


3(71 g) 5(35.5 g)

Cl- + Ag+  AgCl


35.5 g 143.4 g

10 % 71 g Cl 2 5(35.5 g Cl - ) 143.4 g AgCl


100 g x x x x  8.3 g AgCl
100 % 2(143.4 g AgCl) 3(71 g Cl 2 ) 35.5 g Cl 

MTotal AgCl = 100 g -10 g + 8.3 g = 98.3 g AgCl

100 g - 98.3 g
% Error  x100 %  1.7 % por defecto
100 g

545
9.

B
Br
Cl

rB-Br  1.7102  0.4792  1.776 rB-Cl  1.4132  0.8032  1.625

rB-I  02  1.2702  1.270

y 0.479 0.803
θ  tan -1 θ Br  tan -1  15 0 38 ' θ Cl  tan -1  29 0 36 '
x 1.710 1.413

Ángulo Br-B-Cl = 1800 – 15038’ – 29036’ = 134046’

Carga total = 0 = 0.274 + X – 0.068 – 0.102  X = -0.104

1.776
a) La distancia de enlace B-Br
1.625
b) La distancia de enlace B-Cl
1.270
c) La distancia de enlace B-I
134046’
d) El ángulo de enlace Br-B-Cl
- 0.104
h) La carga X del átomo de I

546
XVI Olimpiada Nacional de Química. 1er Examen Nivel A y B.
RESPUESTAS

1) C 6) A 11) D 16) C 21) D 26) B


2) B 7) B 12) X 17) B 22) C 27) D
3) C 8) B 13) B 18) C 23) A 28) B o D
4) A 9) B 14) C 19) C 24) A 29) B
5) D 10) A 15) D 20) B 25) C 30) B

XVI Olimpiada Nacional de Química. 2o Examen Nivel A y B.


RESPUESTAS

1. a) Considerando 100 g de muestra se obtiene el por ciento de cada gas.


30 g 70 g
n prop   0.6818 mol C 3 H 8 n buta   1.2069 mol
g g
44 58
mol mol

0.6818 mol
nT = 0.6818 mol + 1.2069 mol = 1.8887 mol y prop   0.36099
1.8887 mol
1.2069 mol
y buta   0.63901
1.8887 mol

 650 mmHg 

760 mmHg
x 1 atm 101325
Pa 
atm 

 30 m
3

P V
nT  T T     1048.8 mol
RT  Pa m 3
 8.314 29.15 K 
 mol K 

n prop  y prop n T  0.36099 x 1048.8 mol  378.6 mol propano

b) n buta  y buta n T  0.63901 x 1048.8 mol  670.2 mol butano

0.6818 mol
c) y prop   0.36099
1.8887 mol

g
d) m prop  n prop M prop  378.6 mol x 44  16658.4 g  16.6584 kg
mol

g
e) m buta  n buta M buta  670.2 mol x 58  38871.6 g  38.8716 kg
mol

f) 90 0C = 363.15 K

547
 3

1048.8mol  8.314 Pa m 363.15 K 
n T RT  mol K 
P   105552 Pa
V 30 m 3

g) Pbuta = ybutaPT = 0.63901 x 105552 Pa = 67449 Pa

m T m prop  m buta 16.6584 kg  38.8716 kg kg


h) δ mezcla    3
 1.851 3
VT VT 30 m m

i) La densidad es la misma que a 25 0C, ya que no cambian ni la masa ni el volumen en el


sistema.
kg
δ mezcla  1.851 3
m

j) Si resisten los empaques, la presión en estas condiciones es de P = 105552 Pa = 1.0417 atm y los
empaques resisten hasta 3 atm.

120 g 1500 g
2. n SO 2   1.875 mol C 3 H 8 n CO2   34.091 mol
g g
64 44
mol mol

1.875 mol
nT = 1.875 mol + 34.091 mol = 35.966 mol y SO 2   0.052
35.966 mol

3.
HA ⇄ H+ + A-
Co
Co-Co) Co Co

Ka 
H A   10 10   10
 - 2.4 2.4
3.78
 pKa = 3.78
HA 0.1  10 2.4

4. a) PbO2 + 2 e- + H2SO4  PbSO4 + 2 OH- Cátodo (-)

Pb + H2SO4  PbSO4 + 2 H+ + 2 e- Ánodo (+)

b) Q = 10 A h x 60 min/h x 60 s/min = 36000 A s = 36000 C = nFM

1 mol e - 1 mol PbO 2 293.2 g


m  36000C x x 
x  54.70 g
96484 C 2 mol e 1 mol PbO 2

548
H   10 OH   10   10   10
-14 -14
  -6
-pH
 10 -8 mol L-1 mol L-1
5.
H  10
 -8

Cd   0.003 mol L


2 -1
SO   0.003 mol L
4
2 -1


Ks CdSO4  Cd 2 SO 4  2
  Cd   SO  
2
4
2
Ks  10 1..59  0.16 mol L-1


Ks Cd(OH)2  Cd  2 OH      2 
2 2
    3
Ks 3 10 -13.8
4

4
 10 4.8 mol L-1

Las concentración de Cd2+ es menor a la del Ks del CdSO4 y la concentración de OH- es


menor a la del Ks del Cd(OH)2, por lo que queda disuelto el Cd2+.

6. Inciso d.

7. Inciso a.

8. Mn2+ + 4 H2O  MnO42- + 8 H+ + 4 e-


MnO4- + 1 e-  MnO42-

Mn2+ + 4 MnO- + 4 H2O  5 MnO42- + 8 H+


Mn2+ + 4 MnO- + 8 OH-  5 MnO42- + 4 H2O

9. Inciso d.

10. Inciso b.

11.
RESPUESTA a RESPUESTA b

H2O / H+ HBr / Peróxidos

RESPUESTA c RESPUESTA d
1) BH3 (o R2BH) KMnO4, KOH, Frío
(o OsO4, NaOH)
2) NaOH, H2O2
[o 1) Ác. Perbenzoico,
2) NaOH]

549
12.
H RESPUESTA a
Et
H
KOH, EtOH
a) H Me
H Cl
H CALOR
H
H H Me
Et

RESPUESTA b
Br
H CH3
KOH, EtOH
H H
b) CALOR
H 3C
CH 3 H3C Ph

RESPUESTA c
-+
O O O: K
S
c) O
CALOR
CH 3

XVI Olimpiada Nacional de Química. 3er Examen Niveles A y B.


RESPUESTAS

1. HClconc   1.19 g HClconc x 37 g HCl


x
1 mol HCl 1000 mL
x  12.06 mol L-1
1 mL 100 g HCl conc 36.5 g HCl 1L

1 mL 12.06 mol
HCl diluido   x  0.024 mol L-1
500 mL 1L

0.24 mol HCl 1 mol NaOH


20 mL HCl x x  4.82 x 10 -4 mol NaOH
1000 mL 1 mol HCl

1000 mL
4.82 x 10 -4 mol NaOH x  4.8 mL de NaOH 0.1 M
0.1 mol

2. a) 2 AgNO3 + M  2 Ag + M(NO3)2 + 9.70 % de la masa

Cu(NO3)2 + M  Cu + M(NO3)2 - 5.51 % de la masa

b) 100 g + 9.70 g = 109.7 g = 100g + [(107.9 g/mol)(2x)] - xMMMetal

100 g – 5.51 g = 94.49 g = 100 g + [(63.5 g/mol)(x)] – xMMMetal

550
x = moles que reaccionan de M, y MMMetal = masa molar del metal; por lo que hay un
sistema de 2 ecuaciones con 2 incógnitas, y resolviendo queda que la masa molar es de
118.7, por lo que el metal es Estaño (Sn).

c) El valor del potencial estándar del sistema M2+/M es menor que el del sistema Cu2+/Cu, y que el
del sistema Ag+/Ag es mayor que el del sistema M2+/M.

3. a) Para que precipite el bromuro la concentración mínima de iones plata es:


Ks  3.3 x 10 -13 Ag  Br -   
Ag   
1 x 10 -3

3.3 x 10 -13
 3.3 x 10 -10
mol
L
Para que precipite el cloruro la concentración mínima de iones plata es:

Ks  1.8 x 10 -10 Ag  Cl -   
Ag   
1 x 10 -3

1.8 x 10 -10
 1.8 x 10 -7
mol
L

b) Br 

libre 
3.3 x 10 -13
1.8 x 10 -7
 1.8 x 10 -6
mol
L

(0.001 - 1.8 x 10 -6 )
0.001
x 100  99.82 %

Casi todo el bromuro está precipitado.

4. a)

S S S

O S O O S O O S O
2
O O O

b) S(IV) y S(0) [o S(VI) y S(II-) también es válido], carga formal es cero para ambos azufres.

c) sp3 (o también sp3d2).

5.
Jeringa 1 2 3 4 5

Gas He H2 O2 Cl2 N2

6. a) CO2, NH3, HCl.

b) CO2 + H2O  H2CO3


NH3 + H2O  NH4+ + OH-
HCl + H2O  H3O+ + Cl-

c) La de CO2 y la de HCl.

d) La de NH3.

551
e) N 2   0.018 g x 1 mol  6.43 x 10 -4 M
1L 28 g

O 2   0.039 g x 1 mol  1.22 x 10 -3 M


1L 32 g

CO 2   1.45 g x 1 mol  3.29 x 10 -2 M


1L 44 g

NH 3   470 g x 0.89 kg x 1 mol  24.6 M


1 kg 1L 17 g

HCl  695 g x 1.2 kg x 1 mol  22.85 M


1 kg 1L 36.5 g

7. a) masa = 1 tonelada = 1000 kg = 106 g

 10 6 g
Q comb  x 393.505 kJ/mol  - 32.792 x 10 6 J  3.2792 x 1010 J
12 g/mol

b)
500 g H2O
500 g H2O líquida  500 g H2O líquida 
vapor
25 0C Qsensible 100 0C Qlatente 100 0C

J
Q sensible  500 g x 4.184 0
x (100 0 C - 25 0 C)  1569000 J
g C

J
Q latente  500 g x 2260  1.13 x 10 6 J
g

Q = Qsensible + Qlatente = 156900 J + 1.13 x106 J = 1,286,900 J = 1.2869 x 106 J

c) Qaprovechado = 87 % Qliberado = (0.87)(-3.2792 x 1010 J) = -2.8529 x 1010 J

Para convertir 1 g se requiere:

J J
Q sensible  1 g x 4.184 0
x (100 0 C - 25 0 C)  313.8 J  Q'  313.8
g C g

Q’total = vap + Q’sensible = 2260 J/g + 313.8 J/g = 2573.8 J/g

552
Q aprovechado 2.8529 x 1010 J
masa vapor    11.084 x 10 6 g  1.1084 x 10 4 kg
Q' total 2573.8 J/g

1000 g Pa m 3
x 8.314 x373.15 K
nRT 18 g/mol mol K
d) V   1.701 m 3
P 101325 Pa

e) W = - PopV = -105 Pa(1.701 m3 – 0.001 m3) = -1.7 x 105 Pa m3

f) W = -PopV = -Poph  A = r2 ; r = 10 cm = 0.1 m

A = 3.1416(0.1 m)2 = 0.031416 m2

W = -12 x 105 Pa(0.031416 m2)(0.1 m) = -3769.92 Pa m3 = -3769.92 J

Tf 1.7 K
8. a) Tb  K b m  m   0.913978 mol/kg
Kf 1.86 K kg/mol

Tb  K b m  0.51 K kg/mol 0.913978 mol/kg   0.466 K


W2
M2 1000 g/kg  K f W2
b) Tf  K f m  K f 
W1 M 2 W1
1000 g/kg

1000 g/kg  K f W2 1000 g/kg 1.86 K kg/mol258 g   180.03 g/mol


M2  
Tf W1 1.7 K 1568 g 
c) Pvap = P*vap puro x

1568 g 258 g
n1   87.11 mol n2   1.43 mol
18 g /mol 180 g /mol

nT = n1 + n2 = 88.54 mol  x1 = 0.9838

Pvap = P*vap puro x = (585 mmHg)(0.9838) = 575.55 mmHg

40 g
d) C  3.33 mol
12 g /mol
6.67 g
H  6.67 mol  (CH2O)n = (30)n
1 g /mol
53.33 g
O  3.33 mol
16 g /mol

553
180
e) n1  6  La fórmula es (CH2O)6 = C6H12O6
30

9. a) C6H6 (l) + 15/2 O2 (g)  6 CO2 (g) + 3 H2O (l)

 
 15 
ΔH comb  6H CO 2 (g)  3H H 2O(l)  ΔH C6 H 6 (l)  ΔH O 2 (g) 
2
 

ΔH C 6 H 6 (l)  6H CO 2 (g)  3H H 2 O(l)  ΔH comb

 cal   cal   cal  cal


ΔH C6 H 6 (l)  6  94050   3  68320     780980   11720
 mol   mol   mol  mol

b) ΔH C 6 H 6 (l)  6H CO 2 (g)  3H H 2O(l)  ΔH C 6 H 6 (g)

 cal   cal   cal  cal


ΔH C6 H 6 (l)  6  94050   3  68320     787200   17940
 mol   mol   mol  mol

cal cal cal


c) ΔH vap  ΔH C6 H 6 (g) - ΔH C6 H 6 (l)  17940  11720  6220 para el benceno
mol mol mol

cal  cal  cal


d) ΔH vap  ΔH H 2O(g) - ΔH H 2O(l)  57800   - 68320   10520 para el agua
mol  mol  mol

m 1g  cal  cal
e) Q  nΔ Comb  ΔH Comb   - 780980   10012.56
M 78 g/mol  mol  g

f) Por cada mol de benceno (78 g) se producen 3 moles de agua (54 g)

54 g 54g
m H 2O  m C6 H 6  1g  0.6923 g
78 g 78g

10. a)
NH2 N N NH2
N2

HSO4-
+ + H2SO4
O2N

NH2 O 2N NH2

554
b)
N N OH
N2 OH
HSO4-
+ + H2SO4
O2N H3C
O 2N H3 C

c)
N2

HSO4-
+ KI O2N I + KHSO4 + N2

O2N

d)
N2

HSO4-
+ HBF4 O2N F + + N2
BF3 + H2SO4
O2N

11.
Compuesto A Compuesto B Compuesto C
NH2
O

N N
HO O
CO2H
Novocaína
NH2

C N
O O

12. Alcohol a:
OH

OH

555
Alcohol b:
OH

OH

Alcohol c:

OH
OH

13. Compuesto A:

Compuesto B:
O
O
O

Compuesto C:
O O

Compuesto D:
O
O

556
Compuesto E:

XVI Olimpiada Nacional de Química. Examen Internacional.


RESPUESTAS

1a) ΔH 0250 C  [(45.94)  (0)] kJ/mol  -45.94 kJ/mol

1 3
1b) ΔS 0250 C  [192.67  (191.502) - (130.57)] kJ/mol K  -98.936 kJ/mol K
2 2

1c) ΔG 0250 C  ΔH 0250 C - TΔΔ0250 C  45940 J/mol  298.15K (-98.936 J/mol K)  -16442.23J/mol

ΔG 0  16442.23 J/mol
1d) ln Kp    6.633 Kp = 759.8 a 25 0C.
RT (8.314 J/mol K)(298.15 K)

- H 0  1 1  -(-45940)  1 1 
     
R  T2 T1 
1e) K 2  K 1e  759.8e 8.314 J/mol  348.15 K 298.15 K 
 759.8e  2.6616  759.8(0.069833)  53.06

PNH3
1f) Kp  1/2
; n T  n N 2  n H 2  n NH 3
P N2 PH3/22

140
n N2   5 mol (inicio)
28

En el equilibrio: n N 2  5 - 4.911  0.089 mol


n H 2  15 - 14.733  0.267 mol
n NH 3  ( 4.911)(3)  9.822 mol
nT = 10.178 mol

PT VT (13.485 atm)(20 L)
T   323.15 K  50 0 C
nR (10.178 mol)(0.082 L atm/mol K)

n N 2 RT (0.089 mol)(0.082 L atm/mol K)(323.15 K)


1g) PN 2    0.11792 atm
V 20 L

PH 2  3PN 2  0.35375 atm

557
(9.822 mol)(0.082 L atm/mol K)(323.15 K)
PNH 3   13.0133 atm
20 L

PNH3 13.0133
Kp    180.1
P1/2
N2 P 3/2
H2 0.117920.5 0.353751.5
2a)
Forma oxidada Forma reducida E, V/ENH
X H2O2 (H+) H2O +1.77

2b) ánodo: 2 Fe(II)  2 Fe(III) + 2 e-

cátodo: H2O2 + 2 H+ + 2 e-  2 H2O

reacción: 2 Fe(II) + H2O2 + 2 H+  2 Fe(III) + 2 H2O

nE 0celda 2 (1.77-0.77)

E 0
celda E 0
cátodo E 0
ánodo K  10 0.0592
 10 0.0592
 10 33.9

2c) Fe3+ H2O  FeOH2+ + H+

[FeOH 2 ][H  ]
Ka  3
 6.3 x 10 3
[Fe ]

6.3 x 10 3
 2
[H ]  pH = 1.6
0.1

2d) Kps = [Fe3+][OH-]3 = 6.3 x 10-38 [OH-] = (6.3 x 10-38/0.1)1/3 = 12.07

pH = 14 – 12.07 = 1.93

55.85 g/mol
2e) [Fe 3 ]  0.0001 g x  1.79 x 10 -5 mol/L
0.100 L

Kps = [Fe3+][OH-]3 = 6.3 x 10-38 [OH-] = (6.3 x 10-38/1.79 x 10-5)1/3 = 10.82

pH = 14 – 10.82 = 3.18

2f)
1 mol Fe 2 O 3 2 mol Fe
0.4289 g Fe 2 O 3 x x  5.37 x 10 3 mol Fe(III) en la alícuota de 50 mL
g Fe 2 O 3 1 mol Fe 2 O 3
159.7
1 mol Fe 2 O 3

(5.37 x 10-3 mol Fe(III))(100 mL/50 mL) = 1.07 x 10-2 mol Fe(III) en la muestra inicial.

558
2g) 5 H2C2O4 + 2 MnO4- + 6 H+  2 Mn2+ + 10 CO2 + 8 H2O

1 mol H 2 C 2 O 4 2 mol KMnO 4


2h) 0.04960 g H 2 C 2 O 4 x x  2.2 x 10 -4 mol KMnO 4
90 g H 2 C 2 O 4 1 mol H 2 C 2 O 4

2.2 x 10 -4 mol KMnO 4


 2.12 x 10 -2 mol/L
0.0104 L

2i) 5 Fe2+ + MnO4- + 8 H+  5 Fe3+ + Mn2+ + 4 H2O


- 2.2 x 10 -2 mol MnO 4 5 mol Fe 2
0.00960 L MnO 4 x 
x 
 0.0010176 mol Fe 2 en los
1 L MnO 4 1 mol MnO 4
10.0 mL de disolución, en los 100 mL de disolución había 0.010176 mol de Fe2+.

2j) 0.010176 Fe(II) / 0.0107 Fetotal = 0.951 Fe(II) y por lo tanto 0.049 Fe(III).

2k) 0.0107 mol Fetotal x (55.85 g/mol / 1.876 g) x 100 % = 31.85 %.

3a) 1.3028 g – 1.1424 g = 0.1694 g I = 1.33 mmol I

3b) 1.1424 g – 1.0622 g = 0.0802 g Cu = 1.29 mmol Cu

3c) 1.33 mmol I / 1.29 mmol Cu = 1, por lo tanto la fórmula del compuesto formado es CuI.

3d) 2 Cu + I2  2 CuI

3e) 1 x 10-3 mol Cu(NO3)2 producen 1 x 10-3 mol CuI,


por lo que se obtienen (1 x 10-3 mol CuI)(190.5 g/mol) = 0.1905 g CuI.

3f) 2 Cu(NO3)2 + 4 KI  2 CuI + I2 + 4 KNO3

3g) Especie: I2.

Reactivo: Na2S2O3.

Indicador: Almidón.

4a)
A B C D
Compuesto Fe2O3 FeSO4 FeCl3 Fe(OH)3

559
Letra que Proceso
corresponde
g Calentar la sustancia al aire a elevadas temperaturas.

a Calentar la sustancia a elevadas temperaturas en un flujo de


hidrógeno.
d Calentar la sustancia a elevadas temperaturas en un flujo de cloro.
c Reacción con ácido sulfúrico diluido.

h Reacción con ácido clorhídrico diluido.

f Reacción con una disolución acuosa de hidróxido de sodio.

e Reacción con peróxido de hidrógeno en disolución ácida seguida


por una reacción con una disolución acuosa de hidróxido de sodio.
(Dos reacciones).

b Reacción en disolución acuosa con magnesio en polvo.

4b) (Nota: Las reacciones son con Fe(III) pero se toman por buenas si se hacen con Fe(II).
a 2 Fe2O3 + 6 H2  4 Fe + 6 H2O

b FeSO4 + Mg  MgSO4 + Fe

c Fe + H2SO4  FeSO4 + H2

d 2 Fe + 3 Cl2  2 FeCl3

e 2 FeSO4 + H2O2 + H2SO4  Fe2(SO4)3 + 2 H2O


Fe2(SO4)3 + 6 NaOH  2 Fe(OH)3 + 3 Na2SO4

f FeCl3 + 3 NaOH  Fe(OH)3 + 3 NaCl

g 2 Fe(OH)3  Fe2O3 + 3 H2O

h Fe(OH)3 + 3 HCl  FeCl3 + 3 H2O

5) Respuesta b.

560
6a)
CHO CHO
HO OH
H OH H OH H OH
H O
H O Emulsina HO H HO H
H
+
HO O -Glicosidasa
H OH OH HO H H OH
HO
H OH
H H H OH H OH
H H
CH2OH CH2OH

D-Galactosa D-Glucosa

6b)
HO OH
H OH
H O
H H O
HO O
H OH HO OH
H OH
H H
H H

6c)
HO OH
H OH
H O
H H O
HO O
H OH HO OH
H OH
H H
H H

CH3I Ag2O
CHO CHO
MeO OMe
H OMe H OMe H OMe
H O
H H O MeO H MeO H
MeO O
H +
MeO MeO OMe MeO H H OH
H MeO
H H
H OH H OH
H H
CH2OMe CH2OMe

2,3,4,6-tetra-O-metil- 2,3,6-tri-O-metil-
D-galactosa D-glucosa

561
6d)
HO OH
H OH
H O
H H O
HO O
H OH HO OH
H OH
H H
H H

1) Br2 / H2O
HO
2) CH3I / Ag2O CHO C O
MeO OMe
H OMe H OMe H OMe
H O
H O HCl, H2O
H MeO H MeO H
MeO O
H +
MeO MeO Calor MeO H H OH
H MeO
H H O
H OH H OH
H
CH2OMe CH2OMe

2,3,4,6-tetra-O-metil- 2,3,6-tri-O-metil
D-galactosa glucónico

7.
COMPUESTO A COMPUESTO B COMPUESTO C

Br Br Br

Br OH

COMPUESTO D COMPUESTO E -PINENO


OH
O
OH

562
8.
COMPUESTO A COMPUESTO B COMPUESTO C

HO O

COMPUESTO D COMPUESTO E COMPUESTO F


OH OH
O

COMPUESTO G

Br
Br

563
564
XVII Olimpiada Nacional de Química. 1er Examen Nivel A y B.
RESPUESTAS

1) B 6) A 11) A 16) A 21) B 26) A 31) B


2) C 7) D 12) X 17) C 22) D 27) D 32) A
3) D 8) A 13) D 18) C 23) B 28) D 33) B
4) B 9) B 14) D 19) A 24) C 29) B
5) B 10) A 15) C 20) C 25) B 30) X

XVII Olimpiada Nacional de Química. 2do Examen Niveles A y B.


RESPUESTAS

Fisicoquímica:
1. Respuesta inciso B.

m 1Cp 1 (Teq  T1 )  m 2 Cp 2 (Teq  T2 )  0  m 1Cp 1Teq  m 2 Cp 2 Teq  m 1Cp 1T1  m 2 Cp 2 T2

m1Cp1T1  m 2 Cp 2 T2 n 1 C p1T1  m 2 C p 2 T2
Teq   o bien Teq 
m1Cp1  m 2 Cp 2 m1 C p1  m 2 C p 2

m1 500 g m2 100 g
n1    7.8678 mol n2    8.3264 mol
M1 63.55 g/mol M 2 12.01 g/mol

(7.8678 mol)(5.85 cal/mol K)(80 0 C)  (8.3264 mol)(2.07 cal/mol K)(20 0 C)


Teq   63.65 0 C
[(7.8678 mol)(5.85 cal/mol K)]  [(8.3264 mol)(2.07 cal/mol K)]

2. Respuesta inciso B

40 % en masa de O2  32 g/mol  n = 1.250 mol


60 % en masa de Ar  39.95 g /mol  n = 1.502 mol  YAr = 0.5458

PAr = YArP = 0.5458(4.15 bar)

3. Respuesta inciso D

CH3OH(g) ⇄ CO(g) + 2 H2(g)


1 bar 0 bar 0 bar
0.25 bar 0.75 bar 1.5 bar

P B PC2 (0.75)(1.5) 2
Kp    6.75
PA 0.25

4. Respuesta inciso C

565
K1 = 2.035x10–2; T1 = 420 ºC = 693.15 K
K2 = 1.866x10– 1; T2 = 450 ºC = 723.15 K

J  1.866x10 -1 
- 8.314 ln 
mol K  2.035x10 -2   307816 J/mol  307.816 kJ/mol
ΔH 0 
1 1

723.15 K 693.15 K

5. Respuesta inciso B

J  1.866x10 -1 
(1.01325 bar)(100 L) - 8.314 ln 
PV mol K  2.035x10 -2   55.49 K
T 
nR (22 mol)(0.083 L bar/mol K)

6. Respuesta inciso C

(2 mol H2)(0.80%) = 1.6 mol H2 que ha reaccionado.

2 mol NH 3
N2(g) + 3 H2(g) ⇄ 2 NH3(g), entonces  1.6 mol H 2 x  1.06 mol NH 3
3 mol H 2

7. Respuesta inciso D

Ln[H2]t = Kt – Ln[H2]0  Ln[H2]t = (0.2 s-1)(4 s) – Ln[2] = 0.1068

[H]t = e0.1068 = 1.113

********************************************************************************

Química Analítica:
8. Horizontales

1. __disolvente__

3. __anfolito__

7. __dura__

9. __cloro__

10. __Faraday__

Verticales

2. __electrólisis__

566
4. __neutralizar__

5. __ácido__

6. __oxigenada__

8. __oxidante__

d i s o l v 2e n t e
1

l
e
c
a n f o l i t o
3 4

e r
u 5 á ó
6
t c l o
7
d u r a i i x
a d s i 8
o
9
c l o r o i g x
i s e i
z n d
10
M i c h a e l F a r a d a y
r d n
a t
e

9.
Reacción Tipo de reacción
ánodo
Li  Li+ + e- Oxidación

cátodo
I2 + 2e-  2 I- Reducción

10. Porque el litio reacciona (se oxida) al contacto con el agua.

********************************************************************************

567
Química Inorgánica:
11.
1 2 3
H He C

4 5 6
N O Na

7 8 9
S Cl Ar

12.

B O M Ba

Y Te E

S Pe Ra

Química Orgánica:
13.
Estructura de A Estructura de B

HO
H
H
O

(E o Z)

14. D

568
15.
Producto A Producto B Producto C
O

HO HO CH3
OH H3N

O NH3 Cl: OH
Cl:
O

********************************************************************************

XVII Olimpiada Nacional de Química. 3er Examen Nivel A.


RESPUESTAS

1.
a) Estructura de la Colina b) Estructura de la Acetilcolina

:OH :OH
H3C OH H 3C O CH3
N N

H3 C CH3 H3C CH3


O

2.
Condiciones a Compuesto B

O O O

H
H3C CH3 N CH3
ó
Base de
O + fuerza media

H3C Cl

569
Compuesto D Compuesto E

O NH2

H
O2 N NO2
N CH3

O2N NO2

O S O

OH

3.

Estructura del compuesto A Estructura del compuesto B

H H H

H H H

4.
a) Tipo de reacción que se lleva a cabo b) Estructura del segundo reactivo

H
Diels-Alder

Cl

PV (1.0 atm)(10 L)
5. PV = nRT  n   0.41 mol
RT (0.082 L atm/mol K)(298.15 K)

570
n 0.41 mol
M   0.54 M  pH = -log[H+] = -log 0.54 = 0.26
V 0.750 L

6. M2+ (ac) + 2 e-  M (s)

1 mol e _ 15.54 g
14475 C x  0.15 mol e _  2 mol e _ x  207.7 g  Plomo
96485 C 0.15 mol e _

7. Cl- + Ag+  AgCl  (precipitado blanco)

2.08 g AgCl 1 mol AgCl 1 mol KCl 74.45 g


x x x x100  27.0 % KCl
4 g analito 143.3 AgCl 1 mol AgCl 1 mol KCl
% KNO3 = 100 -27 = 73 % KNO3

1 mol 1 mL
8. por cada 100 g HCl  37.2 g x  1.03 mol  x100g  88.5 mL  0.09 L
36.46 g 1.13 g

1 mol 1 mL
Por cada 100 g NaCl  41.2 g x  0.7 mol  x100g  70.4 mL  0.07 L
58.44 g 1.42 g

1.03 mol 0.7 mol


M HCl   11.44 M M NaCl   10 M
0.09 L 0.07 L

VHCl + VNaCl = 6 L

1.92 mol 1L
HCl  6 L x x  1L
1L 11.44 mol

8.33 mol 1L
NaCl  6 L x x  5L
1L 10 mol

9.
1 mL sol. CeCl3 0.1 mol CeCl3 3 mol ác. oxál. 126 g ác. oxál.
1 L sol. ác. oxál. x x x x  18.9 g ác. oxálico
1 mL sol. ác. oxál. 1 L sol. CeCl3 2 mol CeCl3 1 mol ác. oxál.

10.
COMPUESTO No. DE COMPUESTO No. DE
REACCIÓN REACCIÓN
DICROMATO DE 8ó6 ÓXIDO DE SILICIO 13
AMONIO (IV)
CARBONATO DE 1 ó 12 CARBONATO DE 2ó7
COBRE (II) SODIO HIDRATADO
BICARBONATO DE 7 ó 12 NITRATO DE SODIO 13
SODIO

571
CLORURO DE 5 ó 13 CARBONATO DE 12 ó 9
HIERRO (III) ZINC
NITRATO DE 11 ÓXIDO DE ZINC 9 ó 13
PLOMO (II)
ÓXIDO DE MAGNESIO 13 ÓXIDO DE 3ó4
MERCURIO (II)

11. a) 10 C8H8 + 121 O2  72 CO2 + 8 CO + 90 H2O

121 mol O 2 100 mol aire


b) 1 mol C 8 H 8 x x  57.6 mol aire  Inciso C.
121 mol C 8 H 8 21 mol O 2

0.400 g C 8 H 8 1 mol C 8 H 8 121 mol O 2 100 mol aire


c) x x x  0.20 mol aire s -1
1s 114.23 g C 8 H 8 10 mol C 8 H 8 21 mol O 2

(0.20 mol s -1 )(8.314 Pa m 3 mol -1 K -1 )(373 K) 100 mol aire


V x  6.19 x 10 -3 m 3 s -1 aire
101000 Pa 21 mol O 2
Inciso B

12. ln [A]t = -Kt + ln [A]0  ln [0.5] = -K(5730 años) + ln [1.0]

K = -1.21 x 10-4 años-1

ln [0.25] = -Kt + ln [1.0]  t = 11460 años

Inciso E.

XVII Olimpiada Nacional de Química. 3er Examen Nivel B.


RESPUESTAS

1. P* = 56.18 Torr, P = 55.24 Torr, MMEtOH = 46.068 g mol-1


P 55.24 Torr
X1    0.983268
P * 56.18 Torr

n1 n1 n
X1   n1  n 2   n 2  1 - n1
n1  n 2 X1 X1

520.8 g
n1   11.305 mol
46.068 g mol -1

11.305 mol
n2   11.305 mol  0.192374 mol
0.983268

572
w2 12.5 g
n2   M2   64.978 g mol -1
M2 0.192374 mol

Respuesta inciso B.

Tf (353.35 - 351.85)K


2. Tf  K f m  m   0.21429 mol kg -1
Kf 7.00 K kg mol-1

1000 g
n1   7.80214 mol
128.17 g mol -1

n  n 1  n 2  8.01643 mol

7.80214 mol
y1   0.97327  97.327 % de pureza Respuesta Inciso C.
8.01643 mol

3. Para el agua Kc = 1.86 K kg mol-1 y Kb = 0.51 K kg mol-1

3.1 w2 = 15.0 g, M2 = 342,296 g mol-1, w1 = 75 g = 0.075 kg

15.0 g
n2   0.043822 mol
342.296 g mol -1

0.043822 mol
m  0.58429 mol kg -1
0.075 kg

Tf  K f m  1.86 K kg mol-1 (0.58429 mol kg -1 )  1.0867

Tf = -1.0867 0C Respuesta inciso D.

3.2 Basta con un análisis de los datos para contestar que es mayor que la solución de sacarosa.

Teb = Kb m ; M2 = 180.156 g mol-1

w2 30 g
180.156 g mol -1
M2
m   1.1101 mol kg -1
w1 150 g
M1 1000 g kg -1

Teb  0.51 K kg mol -1 (1.1101 mol kg -1 )  0.566 K para la glucosa

Teb  0.51 K kg mol -1 (0.58429 mol kg -1 )  0.298 K para la sacarosa

Teb de la glucosa es mayor que el Teb de la sacarosa  Respuesta inciso A.

573
1 cm Hg 1 atm
4.   27 cm H 2 O x x  0.0.26 atm
13.59 cm H 2 O 76 cm Hg

 0.026 atm
C  -1 -1
 1.06 x 10 -3 mol L-1
RT 0.082 L atm mol K (298 K)

0.713 g 1L
x -3
 6726 g mol -1  Respuesta inciso D.|
0.100 L 1.06 x 10 mol

PV (1.0 atm)(10 L)
5. PV = nRT  n   0.41 mol
RT (0.082 L atm/mol K)(298.15 K)

n 0.41 mol
M   0.54 M  pH = -log[H+] = -log 0.54 = 0.26
V 0.750 L

6. M2+ (ac) + 2 e-  M (s)

1 mol e _ 15.54 g
14475 C x  0.15 mol e _  2 mol e _ x  207.7 g  Plomo
96485 C 0.15 mol e _

7. Cl- + Ag+  AgCl  (precipitado blanco)

2.08 g AgCl 1 mol AgCl 1 mol KCl 74.45 g


x x x x100  27.0 % KCl
4 g analito 143.3 AgCl 1 mol AgCl 1 mol KCl

% KNO3 = 100 -27 = 73 % KNO3

1 mol 1 mL
8. por cada 100 g HCl  37.2 g x  1.03 mol  x100g  88.5 mL  0.09 L
36.46 g 1.13 g

1 mol 1 mL
Por cada 100 g NaCl  41.2 g x  0.7 mol  x100g  70.4 mL  0.07 L
58.44 g 1.42 g

1.03 mol 0.7 mol


M HCl   11.44 M M NaCl   10 M
0.09 L 0.07 L

VHCl + VNaCl = 6 L

1.92 mol 1L
HCl  6 L x x  1L
1L 11.44 mol

8.33 mol 1L
NaCl  6 L x x  5L
1L 10 mol

574
9.
1 mL sol. CeCl 3 0.1 mol CeCl 3 3 mol ác. oxál. 126 g ác. oxál.
1 L sol. ác. oxál. x x x x  18.9 g ác. oxálico
1 mL sol. ác. oxál. 1 L sol. CeCl 3 2 mol CeCl 3 1 mol ác. oxál.

10.
COMPUESTO No. DE COMPUESTO No. DE
REACCIÓN REACCIÓN
DICROMATO DE 8ó6 ÓXIDO DE SILICIO 13
AMONIO (IV)
CARBONATO DE 1 ó 12 CARBONATO DE 2ó7
COBRE (II) SODIO HIDRATADO
BICARBONATO DE 7 ó 12 NITRATO DE SODIO 13
SODIO
CLORURO DE 5 ó 13 CARBONATO DE 12 ó 9
HIERRO (III) ZINC
NITRATO DE 11 ÓXIDO DE ZINC 9 ó 13
PLOMO (II)
ÓXIDO DE MAGNESIO 13 ÓXIDO DE 3ó4
MERCURIO (II)

XVII Olimpiada Nacional de Química. Examen Internacional.


RESPUESTAS

1.
1.1.

Tiempo en que [S*] tiene el valor máximo = ___0.82______ minutos

575
1.2.
t [S*]

1.3. C.

1.4. S*  S 0 e
K E t

1.5. t = ___1.39_______ minutos

1.6. t = ___1.03_______ minutos

2.
A B C
O O OH
H

D E F
OH

O
O

N+ H

N+ O O

O O
ACETAMINOFENO
OH

NH

576
3.
A B C D

CH3CH2CH2-C N CH3CH2CH2-C O CH3CH2CH2-C OH CH3CH2CH2-C O

CH3 CH3 OH

E F G H

CH3CH2CH2-C O CH3CH2CH2-C O CH3CH2CH2-C O O

Cl OCH3 H CH3CH2CH2-C
O
H

I J K

CH3CH2CH2-CH2 NH2 CH3CH2CH2-C O CH3CH2CH2CH2

NH2 CH3CH2CH2-CH N

4.
A B C

O O
O O
HO O

HO OH HO

577
D E F

O O

C
HO
HO
O

H
O
O
H

5.
INCISO:

6.
Reacción inciso a:
CHO

OH

HO
MALTASA + DISACÁRIDO A
OLIGOSACÁRIDO
HO

OH

CH2OH
GALACTOSA

578
Reacción inciso b:
CH2OH

HO
EMULSINA + DISACÁRIDO B
OLIGOSACÁRIDO
OH

OH

CH2OH
FRUCTOSA

Reacción inciso c:

N N
H
OH H
HO

N
HO H HO
O OH
OH H
O
HO O O
OH H
OH
HO

579
Reacción inciso d:
OLIGOSACÁRIDO

1) CH3I (exceso), Ag2O


2) HCl (20%)

CHO
CH2OCH3 CHO
OCH3
O OCH3
CH3O +
+ CH3O CH3O
OH
OH CH3O
OH
OH OH
CH2OCH3
CH2OCH3 CH2OCH3
2,3,6-tri-O-metil-D-glucosa 2,3,6-tri-O-metil-D-fructosa 2,3,4,6-tetra-O-metil-D-galactosa

ESTRUCTURA DEL OLIGOSACÁRIDO:


HO OH
O

OH
HO H HO
O OH
OH
O
HO O O
OH H
OH
HO

7. Ca5(PO4)3OH (s) ⇄ _5_Ca2+ (ac) + _3_PO43- (ac) + _1_OH- (ac) Kps = 6.8x10-37

7.1.

Cálculos:
Equilibrio 5x 3x x

Kps = [Ca2+]5[PO43-]3[OH-] = 6.8x10-37 = (5x)5(3x)3(x) = 84375x9


6.8x10-37
x9  2.72x105
84375
Solubilidad = 2.72x10-5 mol/L

580
7.2. Ca5(PO4)3OH (s) + H+ (ac) ⇄ _5_Ca2+ (ac) + _3_PO43- (ac) + H2O (l)

Keq 
Ca  PO  x OH   K
2 5
4
3 3 
ps

6.8x10-37
 6.8x10 23
H  OH  K
 
w 1x10 -14

Valor de K: 6.8x10-23

7.3. Ca5(PO4)3F (s) ⇄ _5_Ca2+ (ac) + _3_PO43- (ac) + F- (ac) Kps = 2.5x10-60

Cálculos:
Equilibrio 5x 3x x

Kps = [Ca2+]5[PO43-]3[F-] = 2.5x10-60 = (5x)5(3x)3(x) = 84375x9

2.5x10-60
x 9  6.7x108
84375

Solubilidad = 6.7x10-8 mol/L

7.4.
Indica tus cálculos:

Keq 

OH 

K Ca
ps5 ( PO 4 ) 3 OH


6.8x10-37
 2.72x1023
F  K Ca
ps
5 ( PO 4 ) 3 F
2.5x10 - 60

Valor de K = 2.72x1023

8.
a)
Cálculos:

 0.01 mol  4
  1.08x10 mol NaOH  1.08x10 mol HA
-4
10.8 mL 
 1000 mL 

1.08x10-4 mol HA
x100 mL  1.08x10-3 mol HA en la muestra R
10 mL alicuota HA

0.0496 g
 45.9 g/mol HA
1.08x10 - 4 mol

Masa molar = 49.9 g/mol

581
b)
1.08x10-4 mol HA
 1.08x10- 2  HA en R
0.010 mL alicuota HA

Concentración de HA = 1.08x10-2 mol/L

c)
A- y H+

d)
(647-10)S cm-1 = 637 S cm-1

 = 637 S cm-1

e)
   
κ  i Ci λ i0  637 μS cm 1  H  (350 S cm 2 eq 1 )  A  (54 S cm 2 eq 1 )

   

H  A 
637 μS cm 1 10-6 S 1x106 cm3

2
404 S cm eq 1
x
1 μS
x
1m 3
x
1m3
1000 L
 1.577x10 3 eq L1

1.577x10-3 eq L-1 = 1.577x10-3 mol L-1

f)
1.577x103 mol L1
x 100  14.6%
1.08x10 2 mol L1

g)
2
x 2C0 x 2C0 (0.146)2 (1.08x102 )
Ka     2.7x10 4
C0 (1 - x) 1  x 1  0.146

Valor de Ka = 2.7 x 10-4

9.
H+

M3+ M2O5 SO42- MO3 MO2 Fe3+ NO3-

E0

0.0 0.1 0.17 0.7 0.77 0.93


0.6
M MO2 Fe2+ NO
H2SO3 M2O5 M3+
H2

582
a) Al elemento M se le añade HCl 1 M.

2 M + 6 HCl ⇄ 2 MCl3 + 3 H2 NO

b) MCl3 se pone en disolución acidificada de sulfato de hierro (II).

M3+ + 3 Fe2+ ⇄ M + 3 Fe3+


NO
3 H2O + 2 M3+ + SO42- ⇄ 2 MO2 + SO32- + 6 H+

c) MO2 y M se ponen en contacto en disolución ácida.

12 H+ + 3 MO2 + M ⇄ 4 M3+ + 6 H2O


SI
6 H+ + 4 MO2 ⇄ 2 M3+ + M2O5 + 3 H2O

d) M3+ y MO3 se ponen en contacto en disolución acuosa.

3 H2O + 2 M3+ + 2 MO3 ⇄ 2 MO2 + M2O5 + 6 H+ NO

e) El elemento M es tratado con HNO3 concentrado.

H2O + M3+ + NO3- ⇄ MO3 + NO + 2 H+ SI

********************************************************************************

583
584
XVIII Olimpiada Nacional de Química. 1er Examen Nivel A y B.
RESPUESTAS

1) B 6) C 11) C 16) A 21) D


2) B 7) B 12) B 17) D 22) B
3) D 8) A 13) C 18) C 23) C
4) C 9) C 14) B 19) D 24) A
5) A 10) C 15) A 20) B 25) D

XVIII Olimpiada Nacional de Química. 2do Examen Nivel A y B.


RESPUESTAS

1. a) SN1 b) SN1 c) SN2

2. c) C

3. c) 2

4.
REACTIVOS COMPUESTO A RESPUESTA
OBTENER
a) HBr / Peróxidos CH3 ( c )
H3C C CH2CH3
Br
b) H2 / Pd(C) H3 C
C
CH2CH3 ( d )
O
c) HBr Br H ( a )
H2C C CH2CH3
CH3
d) 1) O3, 2) Me2S CH3 ( b )
H3C C CH2CH3
H
e) H2O, HCl Cl Cl ( g )
H2C C CH2CH3
CH3
f) H2CrO4 CH3 ( e )
H3C C CH2CH3
OH
g) Cl2 Br OH ( i )
H2C C CH2CH3
CH3
h) H2N-NH2, KOH

i) Br2 / H2O

585
5. 5 a) D 5 b) C 5 c) D 5 d) D

6. c ó e

7. b

8. e

9. a

10. b

11. d

12. b

13. d

14. b

15. d

16. b

17. a

18. a) Li

19. b) [Xe]4f145d106s26p2

20. a) HCO3-, HSO3-, NO3-, NO2-

21. a) Contiene 28 electrones, 30 protones y 35 neutrones

22. e) 4 enlaces sencillos y 12 electrones de no-enlace

23. a) +5, +6, +7, +5, -1

24. d) covalente polar

XVIII Olimpiada Nacional de Química. 3er Examen Niveles A y B.


RESPUESTAS

FISICOQUÍMICA
1. B) entre 3 y 4 gramos.

586
2. C) entre 3.6 y 4 kcal.

3. D) más de 4 kcal.

4. B) entre 100 y 150 L.

5. C) entre 201 y 250 g.

6. D) más de 500 g.

7. B) entre 1500 y 2000 g.

8. A) menos de 370 m3.

9. C) entre 36 y 45 mol.

10. B) entre 0.2 y 0.3 g.

QUÍMICA ANALÍTICA (Sólo Nivel B)


11. [H+][OH-] = 10-14 [Na+]+[H+] = [OH-]
- +
[OH ] = 2[Na ] [H+] = [Na+]
[H+] 2[H+]= 10-14 [H+] = 10-14 / 2 = 10-7.15 [OH-] = 10-6.84 M
+
pH = -log[H ] = 7.15

12. La fórmula es: C6H2(NO2)3

OH OH OH OH
HNO3 NO2 O2N NO2

H2SO4
NO2 NO2 NO2

139 g/mol 184 g/mol 229 g/mol

a) 0.010 L x 0.10 mol/L = 0.001 mol KOH = 0.001 mol ácido A, por ser la estequiometría de 1:1.
OH
O2N NO2
0.229 g / 0.001 mol = 229 g/mol 

NO2

Volumen de KOH (mL) Cálculo y valor de pH medido


10,00 pH = 7, neutralización de un ácido fuerte con una base
fuerte

20,00 pH = 12.7

587
20.0 mL – 10.0 mL = 10.0 mL  0.001 mol KOH

OH   0.001
- mol
0.020 L
 5x10 2
M  pOH  1.30  pH  14 - 1.30  12.7

13. Reacciones:
Ba(NO3)2 + H2SO4  BaSO4 + 2 HNO3

MgCl2 + 2 AgNO3  2 AgCl + Mg(NO3)2

1.12 g / 233.4 g/mol = 4.80x10-3 mol de Ba (II)


4.80x10-3 mol de Ba (II) x 261.3 g/mol = 1.25 g Ba(NO3)2

0.0263 L x 0.1 mol/L AgNO3 = 2.63x10-3 mol de Ag (I)


2.63x10-3 mol de Ag (I) x 1 mol MgCl2 / 2 mol AgNO3 = 1.315 x 10-3 mol MgCl2
1.315x10-3 mol MgCl2 x 250 mL / 25 mL = 1.25 g MgCl2

Por lo tanto el porcentaje en masa de cada sal es del 50 %.

QUÍMICA ANALÍTICA (Sólo Nivel A)


14. [H+][OH-] = 10-14 [Na+]+[H+] = [OH-]
- +
[OH ] = 2[Na ] [H+] = [Na+]
+ + -14
[H ] 2[H ]= 10 [H+] = 10-14 / 2 = 10-7.15 [OH-] = 10-6.84 M
pH = -log[H+] = 7.15

15.
OH OH OH OH
HNO3 NO2 O2N NO2

H2SO4

NO2 NO2 NO2

139 g/mol 184 g/mol 229 g/mol

a) 0.01105 L x 0.09 mol/L = 9.945x10-4 mol KOH = 9.945x10-4 mol ácido A, por ser la
estequiometría de 1:1.
OH
NO2
-4
0.183 g / 9.945x10 mol = 184 g/mol 

NO2

b) 2,4-Dinitrofenol.

5.60 g 1000 mL 1 mol


c) x x  0.304 mol/L
100 mL 1L 184 g

588
d) 3.6

e)
Volumen de KOH (mL) Cálculo y valor de pH medido
0 pH = 2.83

11.05 pH = 8.3

12.16 pH = 11.65

K a  10 3.6 
H A   H 
   2

x2
 x  1.46x10 3  [H  ]  pH  2.83
HA HA 0.01  x

12.95  3.6
pH NaOH  12.95  pH Disol.   8.3
2

12.16 mL -11.05 mL = 1.11 mL  9.99x10-5 mol KOH

OH   0.01216
-5
9.99x10 mol
-
 4.5x10 3
M  pOH  2.35  pH  14 - 2.35  11.65
L  0.010 L

16. Reacciones:
Ba(NO3)2 + H2SO4  BaSO4 + 2 HNO3
BaCl2 + H2SO4  BaSO4 + 2 HCl

MgCl2 + 2 AgNO3  2 AgCl + Mg(NO3)2


BaCl2 + 2 AgNO3  Ba(NO3)2 + 2 AgCl

1.17 g / 233.4 g/mol = 5x10-3 mol de Ba (II) totales


5x10-3 mol = X mol Ba(NO3)2 + Y mol BaCl2 Eq. 1

0.1 mol Ag(I) 2 mol Mg(II) y Ba(II) 250 mL


0.025 L x x x  0.125 mol Mg(II) y Ba(II)
1L 4 mol Ag(I) 25 mL

0.125 mol = Z mol Mg(NO3)2 + Y mol BaCl2 Eq. 2

2.13 g = [(X mol Ba(NO3)2)(261.3 g/mol)] + [(Y mol BaCl2)(208.2 g/mol)]


+ [(Z mol MgCl2)(95.2 g/mol)] Eq. 3

Al resolver el sistema de 3 ecuaciones con 3 incognitas se obtiene:

Ba(NO3)2 = 0.659 g; BaCl2 = 0.515 g; MgCl2 = 0.956

589
QUÍMICA INORGÁNICA (Sólo Nivel B)

17.
1 2 3
S i B e c q u e r e l
i m
4 5 6
t I b a s e s
7 9
n O i a n e
10 11
e t g l i n e a l
u a l e n
12
t n a c h o
r e t t a d
13
a t o e r l o
14
A(K) r o u
15 16 17
r r l r O j a
18
a l c a l i n o s
19
d t g
20 21
ó i s o t o p í a
n P s

QUÍMICA INORGÁNICA (Niveles A y B)

18. c) N d) O

19. c) Si

20. e) Al3+

21. a) LiF, Mn(NO3)2, Hg2Cl2, N2O5, HNO3

22. a) 19

23. c) CH2

24. a) 32

25. d) BaF2

26. d) Cl2

27. a) 2 enlaces dobles y 8 electrones de no enlace

28. a) SF3+

29. c ) Únicamente iv.

30. e) S

590
31. b) NF5 y c) NO3

32. b) el Cr2O72- es el agente oxidante

QUÍMICA ORGÁNICA (Sólo Nivel A)

33.
ESTRUCTURA ESTRUCTURA ESTRUCTURA
COMPUESTO A COMPUESTO B INTERMEDIARIO C

EtO NO2 EtO NH2 EtO N2 Cl

ESTRUCTURA ESTRUCTURA
COMPUESTO D COMPUESTO E

EtO N N OH EtO N N OEt

34.
ESTRUCTURA ESTRUCTURA ESTRUCTURA ESTRUCTURA
COMPUESTO A COMPUESTO B COMPUESTO C COMPUESTO D
O NO2
NH2
O
O

ESTRUCTURA ESTRUCTURA ESTRUCTURA


COMPUESTO E COMPUESTO F COMPUESTO G
O O
OEt N
HN
O
O

591
ESTRUCTURA ESTRUCTURA
COMPUESTO D COMPUESTO E
O
NH2
HN

Estas respuestas se consideran correctas también

35.
ESTRUCTURA ESTRUCTURA
COMPUESTO A COMPUESTO B
O
O
NO2
HN

ESTRUCTURA ESTRUCTURA
COMPUESTO C COMPUESTO D
CH3
F OH

Br CH3

36.
RESPUESTA

A, D, F, I, J

592
XVIII Olimpiada Nacional de Química. Examen Internacional.
RESPUESTAS

Química Orgánica
1.
ESTRUCTURA ESTRUCTURA ESTRUCTURA
COMPUESTO A COMPUESTO B COMPUESTO C
CO2H CO2H OH
H OH H OH H OH
H OH H OH H OH
CO2H OH OH

COMPUESTO A COMPUESTO B COMPUESTO C


¿PRESENTA ¿PRESENTA ¿PRESENTA
ACTIVIDAD ACTIVIDAD ACTIVIDAD
ÓPTICA? ÓPTICA? ÓPTICA?

NO SI NO

2.
ESTRUCTURA ESTRUCTURA ESTRUCTURA
COMPUESTO A COMPUESTO C COMPUESTO D
O O O O

H OH OCH3

ESTRUCTURA ESTRUCTURA
COMPUESTO E COMPUESTO F
O O
OCH3
O

OH

593
3.
ESTRUCTURA ESTRUCTURA ESTRUCTURA
COMPUESTO A COMPUESTO B COMPUESTO C

O
O CH3-OH
* OH
* O HO
CH3O
O
O

ESTRUCTURA ESTRUCTURA ESTRUCTURA


COMPUESTO D COMPUESTO E COMPUESTO F

O
*
O
OH O
O

4.
ESTRUCTURA ESTRUCTURA ESTRUCTURA
COMPUESTO A COMPUESTO B COMPUESTO C

OH Cl
MgBr

ESTRUCTURA ESTRUCTURA
COMPUESTO D COMPUESTO E

H
H

ESTRUCTURA
COMPUESTO F

594
5.
REACTIVO REACTIVO ESTRUCTURA ESTRUCTURA
OXIMERCURACIÓN DESMERCURACIÓN COMPUESTO A COMPUESTO B

CH3 CH3 CH3 H


Hg(OAc)2 NaBH4 CH3 H
CH3 H HO H
HO H H CH3 H H
H2O

ESTRUCTURA ESTRUCTURA ESTRUCTURA


COMPUESTO C COMPUESTO D COMPUESTO E
CH3 CH3 H CH3 CH3 H CH3 CH3 H
CH3 H Br H
H H OH CH3 H
CH3 H H Br
H H

Fisicoquímica
6.
6.1. D) más de 1.5 g/L

6.2. C) entre 15.1 y 30 mg/L

6.3. B) entre 0.2 y 0.3 h – 1

6.4. B) entre 1.0 y 1.2 g/L

6.5. C) entre 4 y 5 horas

xxxxxxxxxxxxxxxxxxxxxxxxxxxxxxxxxxxxxxxxxxxxxxxxxxxxxxxxxxxxxxxxxxxxxxxxxxxxxxxx

7.
7.1. Adimensional

7.2. Tnormal X _129.95_ oC T585 X _123.95_ oC

7.3.

T normal mezcla _115.85_ oC

595
Química Analítica
8.
a) H+ + OH- → H2O

HA + OH- → H2O + A-

b) (6 mL x 0.1 mol/L)/10 mL = 0.06 M

c) (14 mL - 6 mL) x 0.1 mol/L/10 mL = 0.08 M

d)
Volumen de NaOH Electrolito(s)
0 H+ y Cl-
6 Na+ y Cl-
14 Na+, Cl- y A-
28 Na+, Cl-y A- y OH-

e)
Electrolito Valor de o (en S eq-1 cm2)

HCl 25.6x10-3 / 6x10-5 = 426.6

NaCl 7.6x10-3 / 6x10-5 = 126.7

NaA oNaA = 114.06

NaOH oNaOH = 240

Química Inorgánica

9. a)
A: _CuSO4•3H2O_____________ B: __ CuSO4•H2O_____________

C: _ CuSO4___________________

b) Durante el calentamiento de E ocurre una reacción redox. Identifica a E y F, y escribe la


ecuación química para esta reacción.

E: __CuO______________________ F: _Cu2O___________________

Reacción: __4 CuO → 2 Cu2O + O2___________________________________

c) El compuesto D se forma cuando la mitad de C se ha descompuesto para formar E. ¿Cuál es la


fórmula empírica de D?

D: ___ Cu2SO5____[Cu2O(SO4)]_____________

596
XIX Olimpiada Nacional de Química. 1er Examen Nivel A y B.
RESPUESTAS

1) 2 6) 1 11) D 16) X 21) X 26) A 31) C 36) B


2) 10 7) 7 12) B 17) B 22) D 27) A 32) A 37) D
3) 11 8) 6 13) B 18) A 23) A 28) B 33) D 38) C
4) 17 9) 8 14) B 19) C 24) A 29) B 34) C 39) A
5) 7 10) 9 15) C 20) X 25) D 30) A 35) B 40) D

XIX Olimpiada Nacional de Química. 2do Examen Nivel A y B.


RESPUESTAS

QUÍMICA ANALÍTICA.
1. b) En I, II y V

2. a) acetato de sodio

3. c) el elemento que se reduce es el platino

4. d) Mayor que 7

5. a) Un ácido monoprótico de masa molar 88 g mol-1

6. c) 71,40 mL

7. c) 10 mL de disolución 0,30 mol L-1

8. d) Cloruro de amonio

9. b) 7,1x10–7 mol·L–1

10. d) 12,34

FISICOQUÍMICA.
11. 11.1. a) 3.4 g

11.2. d) 14.2

12. e) mayor que 2500 %

13. c) N2O

14. c) entre 60.7 y 61.6

15. d) 0.750

597
QUÍMICA INORGÁNICA.
16.
1 2 3 4 5 6 7 8 9 10 11 12 13 14 15 16 17 18

L At

Ab La P

Er I O Di

C Ae Se Le

Me Nt

Al

¿Cuál es el mensaje oculto enviado por los habitantes del planeta PAJOYKOT?

_La tabla periódica es elemental______________________________________

QUÍMICA ORGÁNICA.

17.
RESPUESTAS
a) J b) F c) D
CH3 CH3 CH3
Cl OH CH3
CH3 CH3 OH

d) P e) B
CH3
CH3
Br
Br
CH3
CH3

598
18.
RESPUESTA

d) D, B
CH3
H
H
D H

H CH3
H CH3 E2
H
CH3 H H
H
Br H
H H
Br

SN2

H
CH3 OH
H
HO CH3
H H
H H
B

XIX Olimpiada Nacional de Química. 3er Examen Nivel A.


RESPUESTAS

QUÍMICA ANALÍTICA
1. Compuesto D: CaC2O4

Compuesto E: CaCO3

Compuesto F: CaO

La sustancia que se pierde entre 140 y 200 oC es: CaO

La sustancia que se pierde entre 140 y 200 oC es: H2O

La sustancia que se pierde entre 450 y 500 oC es: CO

La sustancia que se pierde entre 600 y 700 oC es: CO2

599
Ecuaciones correspondientes efectuadas durante el calentamiento del oxalato de calcio
hidratado.

CaC2O4 H2O (s)  CaC2O4 (s) + H2O (g)

CaC2O4 (s)  CaCO3 (s) + CO (g)

CaCO3 (s)  CaO (s) + CO2 (g)

2.
a) De su primera observación dedujo que se trataba de
un ácido: ______________diprótico_______
monoprótico, diprótico, triprótico

b) La concentración del ácido en la disolución es igual a:


0.5 mol L-1

c) El (los) valor(es) del pka del ácido es (son) igual(es) a:


1.9, 6.3

d) La masa molar del ácido es igual a:


116 g/mol

e) Lo que le permitió deducir que la fórmula condensada


del ácido era igual a: C4H4O4

f) Lo que le permitió deducir que la fórmula


semidesarrollada del ácido podría ser: HO2CCH=CHCO2H

g)

HO2C CO2H HO2C H

C C C C

H H H CO2H

3. Balancear la siguiente reacción redox:


_3_ (NH4)2PtCl6  _3_ Pt + _2_ N2 + _2_ NH4Cl + _16_ HCl

600
FISICOQUÍMICA.
1. a) 0.024

2. d) 34.5 %

3. c) 1.33 %

4. d) 10.76 g

5. c) 373.81 K

6. b) 60.0 g/mol

7. d) 3.16 x 106

8. a) A>C>B

b) 0.333

c)
A B C

OH

C8H18 CH3(CH2)4OH

d) 2 C6H11OH + 17 O2  12 CO2 + 12 H2O

e) 2 C8H18 + 21 O2  8 CO2 + 8 CO + 18 H2O

f) C8H18 + 10.0556 O2  3.1111 CO2 + 4.8889 CO + 9 H2O


18 C8H18 + 181 O2  56 CO2 + 88 CO + 162 H2O

g) B) Una mayor fracción mol de C

h) B) Una mayor fracción mol de A

QUÍMICA INORGÁNICA.
1. a) BN

b) 1s2 2s2 2p1

c) 1s2 2s2 2p3

601
d) B) X es un análogo del grafito, formado por láminas infinitas de anillos hexagonales. Los
ángulos de enlace son de 1200 y la hibridación de los átomos es sp2. Y es un análogo del
diamante, con estructura tetraédrica, ángulos de enlace de 1090 e hibridación sp3.

QUÍMICA ORGÁNICA (Sólo Nivel B)

1.
1 2 3
Etano Acetona Metano
CH3CH3 O CH4

H3C CH3
4 5 6
Benceno Metanol Ác. butanoico
CH3OH
CH3CH2CH2COOH

7 8 9
Ác. acético Ác. benzoico Etanol
CH3COOH CH3CH2OH
COOH

QUÍMICA ORGÁNICA (Sólo Nivel A)

1.
a b c d
OH- / calor ⇔ HCl CH3OH ⇔ H+

e f C D
O CO2H CO2CH3


Base
X
X = Halógeno,
OC(O)R NO2 NO2

2.
A B C D
O ⇔ ⇔
NHNH O CH3MgX

602
E F G H
OH O NH2 O
⇔ ⇔

OH NH2 O

I J
O ⇔
H2N-OH

3.
A B C
HO2C CO2H HO2C CO2H

HO2C CO2H

D E
O
O O

CO2H

ECUACIONES
COMPUESTO B

CO2 +

HO2C CO2H C CO2H


HO OH

COMPUESTO C
O
O O
HO2C CO2H
+ H2O

603
4.
a)

b) c)

Sesquiterpeno Si, unión de unidades de isopreno


cabeza con cola

XIX Olimpiada Nacional de Química. Examen Internacional.


RESPUESTAS

Química Analítica
PROBLEMA I.
to
1. El elemento X pertenece al 4 grupo.

2. (0,100 mL  0,029 mL)(0,1M)(107,9 g/mol)  0,7661 g de Ag

3. SO2 (g) + H2O (l)  H2SO3 (ac)

H2SO3 (ac) + Ba(OH)2 (ac)  BaSO3 (s) + H2O (l)

4. (1,156 g / 217,3 g/mol)(32 g/mol) = 0,1702 g de S.

5. 1,00 g argirodita – 0,7661 g Ag – 0,1702 g S = 0,0637 g de X

6. (0,1702 g S / 32 g/mol)- (0,7661 g Ag / 107,9 g/mol x 2) = 0,0017687 mol de sulfuro restante


para el elemento X, (XS2).

(0,0017687 mol/ 2) = 0,00088 mol de X.

(0,0637 g / 0,00088 mol) = 72.39 g/mol  Germanio (Ge)

7. Ge: 0,00089 mol / 0,00089 mol = 1

Ag: 0,00709 mol / 0,00089 mol = ,97

S: 0,00532 mol / 0,00089 mol = 5,98  Ag8GeS6

8. GeO2 (s) + 4 HCl (ac)  GeCl4 (ac) + 2 H2O (l)

GeO2 (s) + 2 NaOH (ac)  Na2GeO3 (ac) + H2O (l)

604
PROBLEMA II.
Titulación de sulfato de sodio
40

G (S)
36.3
35
30.3
30
26.0
25
22.0

20 18.9
16.2
15 13.5 13.3
13.5
13.4 13.2
10

0
0 1 2 3 4 5 6 7 8 9 10
Volumen agragado de cloruro de bario 0.001 mol/L (mL)

1. Na2SO4 (ac) + BaCl2 (ac)  BaSO4 (s) + 2 NaCl (ac)

+ -
2. Na y Cl

3. En el punto de equivalencia la conductancia tendría que ser de 13.1 S si es que no se


solubilizara nada del BaSO4. (Se obtiene trazando las rectas y es el valor en donde intersectan,
en 5 mL de BaCl2).

4. La conductancia debida al BaSO4 disuelto se puede obtener en el punto de equivalencia, en 5 mL


de BaCl2 agregado, y es la diferencia entre el punto experimental y el punto teórico (punto de
intersección de las rectas).

16.2 S -13.1 S = 3.1 S

Ci 
3.1x10 S1 cm 
6 1
x
1000 cm 3
 1.04x10 5 mol
5.
2eq mol -1 83 S eq cm  66 S eq
1 2 1
cm 2  1L L

2+ 2- -5 2 -10
6. Kps = [Ba ][SO4 ] = (1.04x10 mol/L) = 1.08x10

Fisicoquímica.
PROBLEMA III.
5 3 g
PVM (232x10 Pa)(0.012 m )(29 mol )
1. m    3312.6 g
RT m3
(8.314 Pa
mol K
)(293.15 K)

605
2. 1 es para O2; 2 es para N2.

M ap  y 1 M 1  y 2 M 2  y X M X  0.20(32 mol
g
)  0.79(28 mol
g
)  0.01M X  29 mol
g

g
0.48 mol
MX   48 mol
g

0.01

g
P X VM X (0.01)(232x10 5 Pa)(0.012 m 3 )(48 mol )
3. m X    54.83 g
RT m3
(8.314 Pa
mol K
)(293.15 K)

4. 3 es para CO2; 4 es para SO2.

y X M X  y 3 M3  y 4 M 4  0.48  y 3  y 4  0.01

Resolviendo las ecuaciones simultáneas  y3 = yCO2 = 0.008; y4 = ySO2 = 0.002

PCO 2 V M CO 2 g
(0.008)(232x10 5 Pa)(0.012 m 3 )(44 mol )
5. m CO 2   3
 40.2 g
RT (8.314 Pa m
mol K
)(293.15 K)

6.
Gas n (mol) M (g/mol) Masa (g) Masa total (g)
O2 0.1 32 3.2 % O2 =(mO2/mTotal)x100
He 0.7 4 2.8 % O2 =(3.2 g/11.6 g)x100
N2 0.2 28 5.6 % O2 = 27.6 %
11.6

7. PMO = 33 pies[(ppO2/FO2)-1)] = 33 pies [(1.6/0.32)-1] = 132 pies

8. Despejando de la ecuación PMO

ppO2 = FO2[(MOD/33 pies)+1] = 0.15[(275 pies/33 pies)+1] = 1.4

De la tabla de límites recomendados para una ppO2 = 1.4, el tiempo recomendado es de 150 min.

PROBLEMA IV.
1. Cátodo.

2+
2. Cu + 2 e-  Cu Cu+ + e-  Cu

2 H3O+ + 2 e-  H2 + 2 H2O 2 H+ + 2 e-  H2
606
3. 1 L = 1840 g x 0.96 = 1776.4 g H2SO4 1776.4 g / 98 g mol-1 = 18.1 M

0.001 C
4. -19 -
 6.25x1015 e 
1.60x10 C / e
- 23
5. (1 mol e )(6.022x10 e- / mol e-)(1.60x10-19C / e-) = 9.64x104 C

9.64 C / 36000 s = 2.67 A


-3
6. 0.140 g / 63.546 g/mol = 2.20x10 mol Cu

Carga = (1.0 A)(600 s) = 600 C

1e- 1 mol e  1 mol Cu


600 C x x 
x  3.11x10 -3 mol Cu
-19
1.60x10 C 6.022x10 e 23
2 mol e 

El gas producido es hidrógeno;

3.11x10-3 mol Cu - 2.20x10-3 mol Cu = 9.1x10-4 mol H2

Química Orgánica.
PROBLEMA V.
1.
a b c d

O O O
Br2 R-CO3H OH
1 equivalente
2 equivalentes

607
PROBLEMA VI.
1.
a) b) c) d)
K2Cr2O7
O H2N-NH2
KOH H2SO4
Cl AlCl3

(o anhídrido (o Zn-Hg / H+) ( o KMnO4 u otro


acético) oxidante)

e) f) g h)

SOCl2 AlCl3 NaNO2


HCl
(o PCl3 o PCl5) (u otro ácido
de Lewis) (o HNO2)

i) j)

CuCN
MgBr

PROBLEMA VII.
1.
A B C
O OH
O OH CH2OH
H OH H OH
H OH
HO H HO H
HO H
CH2OH
CH2OH HO O

D E
O H O H
H OH HO H
H OH H OH
HO H HO H
CH2OH CH2OH

608
a)

Si es ópticamente
activo

b)
A B C
O OH
O OH CH2OH
S H OH R H OH
R H OH
S HO H R HO H
S HO H
CH2OH
CH2OH HO O

D E
O H O H
R H OH S HO H
R H OH R H OH
S HO H S HO H
CH2OH CH2OH

c)

Epímeros

Diastereoisómeros

609
PROBLEMA VIII.
1.
A B C

CH3 OH CH3 H
CH3
H
CH3 CH3
CH3

D E
H
H
CH3 O
O CH3 O
O
H
CH3 H CH3

Química Inorgánica.
PROBLEMA IX.
- -
1. Ánodo: Ag + Cl  AgCl + e

Cátodo: ½Hg2Cl2 + e-  Hg + Cl-

Global: Ag + ½ Hg2Cl2  Hg + AgCl


0 0
2. G = -nFE = -(1)(96485)(0,0455) = -4,39 kJ/mol; el signo negativo dice que la reacción es
espontánea.
0 ΔE 0  ΔE 0 
3. H = G + TS = -nFE + nF = -nF(E-T   ) = 5,33 kJ/mol
ΔT  ΔT 
0
4. E (Ag∣AgCl(s), Cl-) = 0,222 V

K ps
E0 (Ag∣AgCl(s), Cl-) = 0,799  0.06 log
Cl 

0
5. Potencial normal del electrodo de calomel E = 0,0455 V + 0,222 V = 0,2675 V

log Kps = 2(0,2675 V – 0,789) / 0,06 = -17,38

Kps = 10-17,38 = 4,13x10-18

610
XX Olimpiada Nacional de Química. 1er Examen Nivel A y B.
RESPUESTAS

1) D 6) C 11) C 16) A 21) B 26) A


2) C 7) D 12) B 17) B 22) D 27) X (M2N5)
3) B 8) C 13) C 18) D 23) B 28) A
4) A 9) D 14) B 19) D 24) D 29) C
5) C 10) C 15) C 20) B 25) C 30) D

XX Olimpiada Nacional de Química. 2do Examen Nivel A y B.


RESPUESTAS

1. a) Dv3+ < Gr1– < Tñ2– < Mr3– < Rm4–

b) DvGr3, DvGr

Mayor Igual Menor


c) El elemento Dn con respecto al Qr tendrá un número de
electrones de valencia... ( ) ( X ) ( )

d) El elemento Tñ con respecto al Dv tendrá un carácter


metálico... ( ) ( ) ( X )

e) El elemento Rm con respecto al Gr tendrá un radio


atómico... ( X ) ( ) ( )

f) El elemento Dl con respecto al St tendrá un número de


electrones de valencia... ( ) ( ) ( X )

g) El hidrácido del elemento Jn con respecto al del Ov


tendrá un carácter ácido... ( ) ( ) ( X )

h) El elemento Py respecto al elemento Sd tendrá una


electronegatividad… ( X ) ( ) ( )

i) El elemento Jn respecto al elemento Mu tendrá una


primera energía de ionización... ( X ) ( ) ( )

2.
238 235
U: A = 238-206 = 32; Z = 92-82 = 10 U: A = 235-207 = 28; Z = 92-82 = 10

4α = 32 4α = 28
2α-β = 10 2α-β = 10
α=8 α=7
2(8)-β = 10 2(7)-β = 10
β = 16-10 = 6 β = 14-10 = 46
β=6 β=4

611
3. B) Ca

4. D) CuSO4·5 H2O

5. D) Provocar una respiración intensa para reducir los niveles de CO2 en la sangre.

6. C) 0.029 g

7. D) Fe (s) + 2 HCl (ac) → FeCl2 (ac) + H2 (g) 1,50

8. C) Únicamente se compleja el Fe3+.

9.
a)
Qagua = magua Cpagua Tagua = (300 g)(4.184 J/g K)(40 K) = 50208 J

Qcombustión = metanol Hcombustión

metanol = Qcombustión / Hcombustión = (-50208 J)/(-29800 J/g) = 1.6848 g

V = m/d = (1.6848 g)/(0.789 g/mL) = 2.135 mL

b)
PV = nRT; n = m/M ⇒ m = PVM/RT

mbutano = [(105 Pa)(0.025 m3)(58 g/mol)]/[(8.314 Pa m3/mol K)(293.25 K)] = 59.49 g

Qagua = -Qbutano = (59.49 g)(49.2 kJ/g) = 2927 kJ

magua = Qagua / [(Cpagua)(Tagua)] = 2927x103 J/[(4.184 J/g K)(40 K)] = 17489.2 g

c)
mmetano = 90 g(1 mol/16 g) = 5.625 mol metano = 10 g(1 mol/30 g) = 0.333 mol

ymetano = nmetano/ntotales = (5.625 mol/5.958 mol) = 0.9441

% mol = 94.41%

d)
maparente = mtotal/ntotal = 100 g /5.958 mol = 16.784 g/mol

612
e)
ntotal = PV/RT = (5.5x105 Pa)(0.05 m3)/[(8.314 Pa m3/mol K)(303.15 K)] = 10.91 mol

mtotal = (ntotal)(maparente) = (10.91 mol)(16.784 g/mol) = 183.11 g

Qlibera = (mtotal)(Hcombustión) = (183.11 g)(54 kJ/g) = 9888 kJ

10.
NH4Cl As NaOH
Hg H2O NaCl
CH3CO2H NaCN NaHCO3

La sustancia adecuada es: __NaHCO3____

11.
I II III

OH
H
(E)-2-penteno
cis-3-hexeno
(R)-2-hexanol

IV V

(Z)-2-buteno

H Cl
(S)-3-clorooctano

613
12.
I II III IV V

a) OsO4
Br2 / h M+ -:B 1) O3 Br2 / FeBr3
b) KMnO4, H2O,
(o NBS) 2) Zn / H+ frío

c) 1) RCO3H
2) H2O,ácido

XX Olimpiada Nacional de Química. 3er Examen Nivel A y B.


RESPUESTAS

Problema No. 1
La disolución 1: Fe(NO3)2
La disolución 2: Mg(NO3)2
La disolución 3: AgNO3
La disolución 4: Pb(NO3)2
La disolución 5: Cu(NO3)2

Ecuaciones balanceadas: 2 AgNO3 + M → M(NO3)2 + 2 Ag

M1(NO3)2 + M2 → M2(NO3)2 + M1

Identificación de sales por sus propiedades físicas: La de hierro y cobre por su color característico,
verde y azul respectivamente.

Problema No. 2
Fracción de la concentración inicial de dicho isótopo: El tiempo transcurrido corresponde a 4 vidas
medias (48/12 = 4). Por lo tanto, la fracción de la concentración inicial de 42K que queda es (1/2)4 =
1/16.

Problema No. 3
[H  ][A  ]
a) HA ⇄ H+ + A- ⇒ Ka 
[HA]

b) HA + OH- → H2O + A-

c) 0,0667 M x 0.003 L = 2x10-4 mol

614
Ka 
10   2,45 2

y Ka 
10   2,65 2

d)
C A  10  2,45  C B  10  2,65 

10   2,65 2
10  0,0002mol
 3,88

0,013L


  
e)
C B  10  2,65
  0,010 L * C B   0,0002mol 
  
 0,013 L   0,013 L 

1,83x10-6 CB = 7,266x10-8

CB = 3,97x10-2 mol/L

Ka 
10   2,65 2
 1,32x10 4
f)
3.9x10 -2
 10  2,65

Si se considera CB = 0,042 mol/L, Ka = 1,20x10-4

CA 
10   2,45 2
 10 2,45
g)
1,32 x10  4

CA = __0,099__mol/L

Si Ka = 1,20x10-4, CA = 1,08x10-1 mol/L

100 C B
h) VA   40,10 mL
CA

V inicial de A = 40,10 mL

Si CB = 0,042 mol/L, VA= 38,9 mL

i) V para neutralizar el ácido HA = 60 mL

j) 0,060 L x 0,667 mol/L = 4x10-3 mol, 0,784 g / 4x10-3 mol = 196 g/mol

k) C6H12O7

l)
H H OH H OH

HOCH2 C

O
OH OH H OH , Ácido glucónico

615
Problema No. 4
a) P = Phidrostática + Pbárica

Phidrostática = (1.029 g/cm3)(2000 cm)(981 cm/s2) = 2018898 g/cm s2

(2018898 g/cm s2)(1 dina cm/1 g cm2 s-2)(1bar/106 dina cm-2) = 2.0189 bar

Pbárica = 1 atm = 1.013 bar

P = 2.0189 bar + 1.013 bar = 3.0319 bar

b) gas % masa masa del gas moles del gas fracción mol

O2 17 17 g 17 g/32 g/mol = 0.53125 mol nO2/ntotal = 0.152

N2 83 83 g 83 g/28 g/mol = 2.96429 mol nN2/ntotal = 0.848

ntotal = 3.49554 mol

Fracción mol: 0.152

c) Por la relación estequimétrica de la ecuación 1:1, es el 7 %.

d) Vestación = 3x3x6m3 = 54 m3

n= PV/RT = [(3.5x105 Pa)(54 m3)]/[(8.314 Pa m3/mol K)(288.15 K)] = 7889.23 mol

nCO2 = (0.07)(7889.2) = 552.2 mol

Problema No. 5
1)
A B C D E

Mg MgSO4 Mg(OH)2 MgSO4 MgCO3

F G H I
CO2 MgO Mg3N2 NH3

PV = nRT; P = 1 atm; V = 0.4972 L; R = 0.082 L atm/mol K; T = 303.15 K

n = 0.020 mol H2; Relación estequiométrica Metal-Hidrógeno gas (1:1);

0.4862 g/0.020 mol = 24.31 g/mol, por lo tanto se trata del magnesio (Mg).

2.1) Mg (s) + H2SO4 (ac) → MgSO4 (ac) + H2 (g)

616
2.2) MgSO4 (ac) + 2 NaOH (ac) → Mg(OH)2 (s) + Na2SO4 (ac)

2.3) Mg(OH)2 (s) + 2 HCl → MgCl2 (ac) + 2 H2O (l)

2.4) MgCl2 (ac) + Na2CO3 (ac) → MgCO3 (s) + 2 NaCl (ac)

2.5) MgCO3 (s) → MgO (s) + CO2 (g)

2.6) MgO (s) + H2O (l) → Mg(OH)2 (s)

2.7) 3 Mg (s) + N2 (g) → Mg3N2 (s)

2.8) Mg3N2 (s) + 6 H2O (l) → 3 Mg(OH)2 (s) + 2 NH3 (g)

3) Hidrógeno gaseoso, H2 ↑

Problema No. 6
1)
Compuesto A B C D E F G

Fórmula NH3 CO2 NaCl H2O NH4Cl NaHCO3 Na2CO3

2)
CaCO3 → CaO + CO2 (g)

CO2 + NH3 + H2O + NaCl → NaHCO3 + NH4Cl

2 NaHCO3 +  → Na2CO3 + H2O + CO2

3) a) Ninguna.
b) El CO2 y el NH3 debían de reutilizarse para que el proceso fuera viable desde el punto de
vista económico. El NH4Cl se hacía reaccionar con CaO, obtenido al calentar la caliza
(CaCO3), obteniéndose otra vez amoniaco (NH3) para reciclarlo en el proceso.

2 NH4Cl + CaO → CaCl2 + 2 NH3 + H2O

Problema No. 7
En A: 400 kg = 200 kg de agua ≈ 11111.11 mol de agua

200 kg de acetona ≈ 3448.276 mol de acetona

A moles totales = 14559.4 mol

(0.5)(B mol totales) + (0.8)(C mol totales) = 11111.11 mol H2O

(0.5)(B mol totales) + (0.2)(C mol totales) = 3448.276 mol Acetona

617
Resolviendo el sistema de 2 ecuaciones con 2 incógnitas:

Moles totales presentes en cada corriente:

A: 14559.4 mol; B: 1787.996 mol; C: 12771.39 mol

Problema No. 8
I II III IV

OH
O O O
NO2
NO2 NH2 CN

Problema No. 9
I II III

Br CO2H

Problema No. 10
I II III IV

E o trans R Z o cis S

Problema No. 11
1) P = nRT/V

n = 2000 g/44 g/mol= 45.4545 mol; T = 25 oC+ 273.15 = 298.15 K; R = 8.314 Pa m3/mol K;
V = 4.84x10-3 m3

618
P = nRT/V = 2328 bar

2) m = PVM(RT)

P = 67.89x105 Pa; V = 4.84x10-3 m3; M = 44 g/mol; R = 8.314 Pa m3/mol K; T = 298.15 K

m = PVM(RT) = 583.3 g

3) m = PVM(RT)

P = 67.89x105 Pa; V = 8.5x10-3 m3; M = 44 g/mol; R = 8.314 Pa m3/mol K; T = 298.15 K

mTotal = 1024.3 g de CO2 en estado líquido; 1100 g – 1024.3 g = 75.7 g

4) Hvap = -RLn(P2/P1) / (1/T2 – 1/T1) = 10604.4 Pa m3 = 10.6 kJ

T1 = 25 oC+ 273.15 = 298.15 K

T2 = 31.1 oC + 273.15 = 347.12 K

5) Hsub = -RLn(P2/P1) / (1/T2 – 1/T1) = 27669.5 Pa m3 = 27.67 kJ

T1 = -78.2 oC+ 273.15 = 194.95 K

T2 = -56.6 oC + 273.15 = 216.55 K

Problema No. 12
 a 80 oC (353.15 K): 32.93 dinas cm-1 (Haciendo una regresión lineal  = 75.23462 – 0.11978T, T
en K)

Aplicando logaritmo a toda la ecuación: log  = log o + n log(1-T/Tc)

Resolviendo dos ecuaciones con 2 incógnitas para 45oC (318 K) y 20oC (293.15 K)

Valor de las constantes para el líquido “X“: o = 77; n = 1.14

XX Olimpiada Nacional de Química. Examen Internacional.


RESPUESTAS

Fisicoquímica.
1.
T = 15 ºC + 273.15 = 288.15 K
log10P = 4.018 - [1204/(288.15-53.23)] = - 1.10715
P = 10-1.107 = 0.07816 bar = 7816 Pa

619
2.
T = 110 ºC + 273.15 = 383.15 K
log10P = 4.078 - [1344/(383.15-53.77)] = - 0.002393
P = 10-0.002393 = 0.99450 bar = 99450 Pa

3.1.
PTotal = PBenceno + PTolueno = XBencenoP*Benceno + XToluneoP*Tolueno
PTotal = (5.50 mol/9.00 mol)(36.1 kPa) + (3.50 mol/9.00 mol)(12.24 kPa)
PTotal = 22.06 kPa + 4.76 kPa = 26.82 kPa

3.2.
YTolueno = PTolueno/PTotal = XToluenoP*Tolueno/(XBencenoP*Benceno + XToluenoP*Tolueno)
YTolueno = (3.5 mol/9 mol)(12.24 kPa)/[(5.5 mol/9 mol)(36.1 kPa) + (3.5 mol/9 mol)(12.24 kPa)
YTolueno = 4.76 kPa/26.82 kPa= 0.1775

3.3.1. La fracción mol del tolueno: 0.60

3.3.2. La fracción mol del benceno: mayor que 0.75

3.3.3. La presión total es aproximadamente: 11850 Pa

3.3.4. La presión total es aproximadamente: 7050 Pa

3.3.5. La fracción mol del benceno: 0.50

3.3.6. La fracción mol del tolueno: 0.28

Química Orgánica.
4.
I II
CO2Me OH

MeO OMe
MeO OMe
OMe
OMe

620
III IV
CN
NH2

MeO OMe MeO OMe

OMe OMe

5.
I II III
Br
O CH2

IV V

CN CO2H

6.
I II III

O
O

CH3 H

621
Química Inorgánica.
7a)
XeF2 XeF4

F
F F
Xe Xe
F F
F
Lineal Cuadrada

Hibridación: sp2d Hibridación: sp3d2

7b) XeF2 (2+ o II); XeF4 (4+ o IV)

7c.1) XeF4 + H2O → XeOF2 + 2 HF

7c.2) 2 XeF4 + 3 H2O → Xe + XeO3 + 6 HF + F2

7c.3) XeF6 + H2O → XeOF4 + 2 HF

7c.4) XeF6 + 3 H2O → XeO3 + 6 HF

7d)
XeO3 XeOF2 XeOF4

F O

Xe F F
O O Xe Xe
O F F
O
F

Hibridación: sp3 Hibridación: sp3d Hibridación: sp3d2

7e) XeO3 (6+ o VI); XeOF2 (4+ o IV); XeOF4 (6+ o VI)

7f) disHoXe-F = [(106 + 158) kJ/mol]2 = 132 kJ/mol

7g) fHoXeF4 = 2disHoF2 – 4 disHoXe-F = 2(158 kj/mol) – 4(132 kJ/mol) = 212 kJ/mol

8. (2.21 g/dm3)(8.314 kPa dm3/K mol)(373.15 K/1.39x102 kPa) = 49.32 g/mol

XNO2(46 g/mol) + YN2O4(92.02 g/mol) = 49.32 g/mol

Resolviendo la ecuación con base en X + Y = 1

XNO2 = 0.928 YN2O4 = 0.072

622
PNO2 = (0.928)(1.4 bar) = 1.2992 bar PN2O4 = (0.072)(1.4 bar) = 0.1008 bar

Kp 
NO   1.2825 atm 
2
2 2

 16.53
N O  0.0995 atm
2 4

9a)
N 7.5
log log
N0 15  5730 años
t   t 1/2  5730
log 2 log 2

9b)
N 3
log log
N0 15
t   t 1/2  5730  13305 años ; si es más antigua.
log 2 log 2

9c)
ln 2 ln 2
 t  2000 años
N  N0e t1/2
 (15 dpm)e 5730 años
 11.77 dpm

9d)
14
7 N  01n  14
6 C  11H

 t  N 
9e) t  -  1/2   ln 
 ln 2  N 0 

Química Analítica.
10.a. Fe(II), Ni(II), Co(II)

10.b. 500 micromol

10.c. Fe3+

10.d. Fe3+ + 3 OH- ⇄ Fe(OH)3 (s)

10.e. 460 micromol

10.f. 3.1947

10.g. 2129.92

10.h. 0.13519

10.i. 90.12721

623
10.j. 0.300

10.k. 7.0997

10.l. 85.63

10.m. 94.85

10.n. 4.7

624
XXI Olimpiada Nacional de Química. 1er Examen Nivel A y B.
RESPUESTAS

1) C 6) B 11) A 16) D 21) C 26) D 31) B


2) C 7) C 12) B 17) C 22) B 27) C 32) C
3) D 8) B 13) C 18) B 23) A 28) X 33) D
4) A 9) B 14) A 19) C 24) B 29) A 34) A
5) A 10) C 15) D 20) D 25) A 30) B 35) C

XXI Olimpiada Nacional de Química. 2do Examen Nivel A y B.


RESPUESTAS

1. C

2. C

3. D

4. C

5. A

6. C

7. C

8. D

9. C

10. A

11. C

12.1. 0.468
MM (propano, C3H8) = 44 g/mol MM (butano, C4H10) = 58 g/mol

m propano  (20000 g)(0.40)  8000 g m butano  (20000 g)(0.60)  12000 g

n propano  (8000 g)/(44 g/mol)  181.82 mol


n butano  (12000 g)/(58 g/mol)  206.90 mol

y propano  (181.82 mol)/(388.72 mol)  0.4677  0.468

12.2. 2.4 x 10-2 m3

625
V = nRT/P = (1 mol)(8.314 Pa m3/mol K)(293.15 K)/105 Pa = 0.02437 m3

12.3. 7.24 mol

mtotal = 700g; mbutano = (700 g)(0.60) = 420 g; nbutano = (420 g)/(58 g/mol) = 7.241 mol

12.4. 77 g

ybutano = (1-0.4677) = 0.5333

nbutano = (ybutano)(ntotal) = (0.5333)(2.5 mol) = 1.33 mol

mbutano = (nbutano)(MMbutano) = (1.33 mol)(58 g/mol) = 77.1 g

12.5. Entre 8 y 11 mol

n = PV/RT = [(105 Pa)(0.250 m3)]/[(8.314 Pa m3/mol K)(293.15 K)] = 10.25 mol

13. C

14. D

15. A

16. B

17. C

18.1.
O

S S O

18.2.
Estado de oxidación Carga formal
S (central) Cuatro Cero
S Cero Cero

18.3. sp3

626
19.

1 2 3

H He C

4 5 6

N O Na

7 8 9

S Cl Ar

XXI Olimpiada Nacional de Química. 3er Examen Nivel A y B.


RESPUESTAS

FISICOQUÍMICA
Problema No. 1.
1.1.  = (Tc – Tf)/Tc = (873.15 K – 573.15 K)/(873.15 K) = 0.3436

1.2. Qabsorbido = Wrealizado + Qcedido = 25 kJ + 15 kJ = 40 kJ

 = Wrealizado/Qabsorbido = 25 kJ/40 kJ = 0.625

1.3. Wrealizado = (25 kJ)(3600 s) = 90 000 kJ

1.4. Qabsorbido = Wrealizado/ = [(30 kJ/s)(15 min)(60 s/1 min)]/0.475 = 56 842 kJ

1.5.  = (Tc – Tf)/Tc

Tc = Tf/(1 - ) = 393.15 K / (1 – 0.75) = 1 172.6 K

Problema No. 2.
CH4 (g) → 2 H2 (g) + C (grafito)

ΔH0 = 74.85 kJ/mol

ΔS0 = 80.67 kJ/mol

ΔH  TS 74.85x10 3 J/mol  298 K 80.67 J/mol


ln K    20.51
 RT  8.314 J/mol K 298 K 

K = 1.24x10-9

627
Problema No. 3.
3.1.  = Tf /(Tc – Tf)

Tf = (Tc)/(1+) = [(9.5)(293.15 K)]/(1+9.5) = 265.23 K

t = T – 273.15 = 265.23 K – 273.15 = –7.92 0C

3.2. Qabsorbido = [(1235 g)(6025 J/mol)] / 18 g/mol = 4.13x105 J

W = Qabsorbido/ = 4.13x105 J / 9.5 = 4.35x104 J

3.3.  = Tf /(Tc – Tf) = 253.15 K / (303.15 K – 253.15 K) = 5.063

QUÍMICA ANALÍTICA
Problema No. 4.
4.1. A

Problema No. 5.

Respuesta: C

Problema No. 6.

6.1. Masa molar de XSO4 = [(0,2260 g)(233,3 g/mol)] / 0,4382 g = 120,32 g/mol

Masa de X = 120,32 g – 32 g – 64 g = 24.3 g, por lo tanto el elemento X es magnesio (Mg)

6.2. MgSO4 (ac) + BaCl2 (ac) → BaSO4 (s) + MgCl2 (ac)

Problema No. 7.
7.1. 1era. titulación:

Na2CO3 + 2 HCl → 2 NaCl + CO2 + H2O

Na2C2O4 + 2 HCl → H2C2O4 + 2 NaCl

HCl + NaOH → NaCl + H2O

H2C2O4 + 2 NaOH → Na2C2O4 + 2 H2O

2da. titulación:

Na2C2O4 → Na2CO3 + CO

Na2CO3 + 2 HCl → 2 NaCl + CO2 + H2O

HCl + NaOH → NaCl + H2O

628
7.2. 1era. titulación: se determina la cantidad de X.

[H+]inicial = (20 mL)(0,2 mM) = 4 mmol HCl

[H+] = 4 mmol – [(8,3 mL)(0,1016 mM)] = 3,1628 mmol HCl

Por lo tanto, X mg = (3,1628 mmol / 2)(106 mg/mmol) = 167 mg X

% Na2CO3 = (167 mg / 737,1 mg)(100) = 22,65 %

2da. Titulación: Se determina la cantidad de X y de Y.


[H+]inicial = (50 mL)(0,2 mM) = 10 mmol HCl

[H+] = {104 mmol – [(14,7 mL)(0,1016 mM)]} / 2 = 4,25 mmol X + Y

4,25 mmol – [(614,8 mg)(0,2265) / 106 mg/mmol] = 2,88 mmol Y

Por lo tanto, Y mg = (2,88 mmol)(134 mg/mmol) = 385,9 mg Y

% Na2C2O4 = (385,9 mg / 641,8 mg)(100) = 60,13 %

% NaCl = 100 –(22,65 + 60,13) = 17,22 %.

Problema No. 8.
AgBr (s) ⇆ Ag+ (ac) + Br- (ac)

Ag+ (ac) + 2 NH3 (ac) ⇆ Ag(NH3)2+ (ac)

AgBr (s) + 2 NH3 (ac) ⇆ Ag(NH3)2+ (ac) + Br- (ac)

Kr 
Ag(NH ) Br   5,01x10
3 2
  13
 1.26x10 5  10 4.9
NH 3  2
10 7.4

s  (104.9 )(0,1 M) 2  3,55x104 M

629
QUÍMICA INORGÁNICA
Problema No. 9.
1
s u l f a t o
2
u r a n i o
3
p r e c i p i t a c i ó n
4
j o u l e
5
c a l c i o
6
s i l i c i o
7
a l c a n o s
8
i n d i c a d o r
9
g r a f i t o
10
g l u c o s a
11
g a s

Problema No. 10.


CATIÓN IDENTIFICADO
+
1. K (ion potasio)

2. Fe3+ (ion hierro (III))

3. Mn2+ (ion manganeso (II))

4. Cu2+ (ion cobre (II))

5. Ni2+ (ion niquel (II))

6. Co2+ (ion cobalto (II))

7. Zn2+ (ion zinc)

630
QUÍMICA ORGÁNICA
Problema No. 11.
1 2 3
Etano Acetona Metano

CH3CH3 CH3COCH3 CH4

4 5 6
Benceno Metanol Ácido butanoico

C6H6 CH3OH CH3CH2CH2CO2H

7 8 9
Ácido acético Ácido benzoico Etanol

CH3CO2H C6H5CO2H CH3CH2OH

Problema No. 12.


Inciso B:


+ CN
CN
CN
CN
Dieno Dienófilo Biciclo[2.2.1]hept-5-eno-2,3-dicarbonitrilo

Problema No. 13.


A)
C K

I II

Problema No. 14.


NO2
N2Cl I
2 KI
1) 4 Fe, 12 HCl

2) 2 NaNO2,
4 HCl
NO2
N2Cl I

631
Problema No. 15.
OCH3 OCH3 OCH3

NO2
HNO3

NO2

OCH3 OCH3 CH3 OCH3


O
H2, Pd / C CH3

NO2 NH2 CH3 N

OCH3
OCH3
CH3

H2, Pd / C

CH3 NH
CH3 N

CH3
CH3

XXI Olimpiada Nacional de Química. Examen Internacional.


RESPUESTAS

FISICOQUÍMICA
1.1. D) CH4 (g) + 2 H2O (g)  CO2 (g) + 4H2 (g).

1.2. A) +164.8 kJ/mol

CH4 + H2O → CO + 3 H2 ∆Hr0 = 206 kJ/mol


CO + H2O → CO2 + H2 ∆Hr0 = - 41.2 kJ/mol
∆Hr0 = 164.8 kJ/mol

1.3. C) Disminuir la presión y aumentar la temperatura.

1.4. B) Disminuir la temperatura.

632
1.5. C) Disminuir la presión y aumentar la temperatura.

1.6. D) 214.8 J/mol K.

S r0  (3SH0  S 0CO )  (SH0 O  S 0CH ) = (3[130.6]+197.9)-(188.7+186.2) = 214.8 J/mol K


2 2 4

1.7. C) 142.1 kJ/mol.

Gr0  Hr0  TS r0 = 206150 J/mol –[(298.5 K)(214.8 J/mol K) = 142.1 kJ/mol

1.8. A) 8.05 x 10 –10


 ΔG 0r 69637 J/mol

∆Gr0 = -RTlnK ke RT


e (400 K)(8.314 J/mol K)
 8.05x1010

2.1. [N2O5]3.2 min = 0.22 - 0.23 mol L-1.

t = 3.2 min = 192 s


4
s 1 )(192 s)
[A]  [A]0 e  kt  [0.25 mol L1 ]e  (5.1x10  0.227 mol L1

2.2. t = 16.5 – 16.7 minutos

1 [A] 1 0.15
t   ln  4
ln  1001.62 s  16.69 min
k [A]0 5.1x10 0.25

2.3. t= 31.6 minutos

1 0.38[A]0
[A] = 0.38[A]0 t 4
ln  1897.22 s  31.62 min
5.1x10 [A]0

QUÍMICA ANALÍTICA
3a. 3 Zn2+(ac) + 2 Fe(CN)63-(ac) → Zn3[Fe(CN)6)2↓

3b.
I

3c. (3 x 0,020 x 0,04))/(0,010/2) = 0,12 mol/L.

633
3d. (0,01 x 0,12 x 2/3 x 96500) = 77.2 C.

3e. (0,01 x 0,12 x 2 x 96500) = 231.6 C.

QUÍMICA ORGÁNICA
4.
Base
O O O O
+

EtONa / EtOH

O 1) (CH3)2CuLi
O
2) CH3I
NH2NH2

 / HO

A (C8H14)
H2, 1 eq.

Pd-C; 1 atm
(Ph)3P=CH2

H2 / PtO2
O

B (C8H14)

H2 / PtO2

1) MeLi
2) H
+ 3) 
C (C8H14)
H2 / PtO2

H 1) O3
2) SMe2
O

634
5.

SOCl2

CO2H
COCl
AlCl3

Zn, H

6.
HC O CO2H

H OH HNO3 H OH

H OH H OH

OH CO2H

7.
HC O CO2H

H OH Br2
H OH

H OH H2O
H OH

OH OH

8.

HC O HC
3 NH NH2 N-NH-Ph
H OH N-NH-Ph

H OH H OH
H
OH OH

QUÍMICA INORGÁNICA
9.1. O2-, CO32-, PO43-

9.2a. Fe3+ = [Ar]3d5

635
9.2b. Fe(OH)3

9.2c. Fe(SCN)2+ + Y4- ⇆ FeY1- + SCN1-

9.2d. Fe4[Fe(CN)6]3

9.3a. IIA

9.3b. CaCO3

9.3c. Coral, mármol, cáscara de huevo, etc.

9.4a. 2+

9.4b. Zn2+ = [Ar]3d10

9.4c. Zn2+ + 2 OH1- ⇆ Zn(OH)2↓

9.4d. Zn2+ + 4 OH1- ⇆ [Zn(OH)4]2-

9.4e. Zn2+ + H2S ⇆ ZnS↓ + 2 H1+

9.5. Mn2+

9.6. Paramagnéticos: Fe3+ y Mn2+


Diamagnéticos: Ca2+ y Zn2+

9.7. MnO41-

9.8. 2 MnO41- + SO32- + 2 OH1- ⇆ 2 MnO42- + SO42- + H2O

9.9. 2 MnO42- + 3 SO32- + H2O ⇆ 2 MnO2 + 2 OH1- + 3 SO42-

9.10. MnO2 + 4 HCl ⇆ Cl2 + 2 H2O + Mn2+ + 2 Cl1-

636
XXII Olimpiada Nacional de Química. 1er Examen Nivel A y B.
RESPUESTAS

1) D 6) C 11) D 16) C 21) B 26) D 31) B


2) A 7) D 12) A 17) B 22) C 27) A 32) A
3) C 8) B 13) C 18) C 23) D 28) D 33) D
4) A 9) B 14) D 19) A 24) B 29) C 34) B
5) B 10) X 15) B 20) D 25) A 30) C 35) D

XXII Olimpiada Nacional de Química. 2do Examen Nivel A y B.


RESPUESTAS

1.1. PN 2  0.78(74.46 kPa)  58.08 kPa .

1.2. Para el O2:


(YO PTot VTot ) 0.21(74.46x103 Pa)(2.5x103 m3 )
nO  2
  0.01604 mol
2
RT Pa m3
(8.314 )(293.15 K)
mol K

1.3. Volumen parcial:

VN  YN (VTot )  0.78(2.5 L)  1.95 L


2 2

1.4. M Aire  YN 2 M N 2  YO 2 M O 2  YX M X

28.82 = 0.78(28 g/mol) + 0.21(32 g/mol) + 0.01MX

MX = (28.82-28.56) / 0.01 = 26 g/mol

1.5.
(YN PTot VTot M N ) 0.78(74.46x103 Pa)(2.5x103 m3 )(28 g/mol)
mN  2 2
  1.558 g
2
RT Pa m3
(8.314 )(293.15 K)
mol K

1.6.
(PTot M Aire ) (74.46x103 Pa)(28.82 g/mol)
Aire   3
 880.48 g/m3
RT Pa m
(8.314 )(293.15 K)
mol K

2.1. m Tot  m N 2  m H 2  3.50 g  ( 28 g/mol)(n N 2 )  ( 2 g/mol)(n H 2 )

637
PV (0.9869 atm)(7.5 L)
n N  nH    0.301 mol
2 2
RT (0.082 L atm )(300.15 K)
mol K

Se resuelve el sistema de 2 ecuaciones con dos incógnitas ( n N 2 y n H 2 )

n N  0.112 mol  3.136 g N 2


2
 N2: 89.6% m/m
n H  0.188 mol  0.376 g H 2
2
 H2: 10.4 % m/m

3.1. d) La masa del protón es ligeramente inferior a la del neutrón.

3.2.a) Mayor proporción del 35Cl que de 37Cl.

3.3. b) 52

3.4. a) 0 y +2

3.5. c) Hidrógeno, tritio y deuterio.

3.6. c) Incrementa la electronegatividad.

3.7. b) 220,1 g/mol

3.8. a) 1/16

3.9. a) Q y J forman el compuesto de mayor carácter iónico.

3.10. a) 75 % de Cl2 y 25 % de H2

3.11. d) 0,372

3.12. c) 18

3.13. b) Ca

3.14. c) 7,1 x 10-9

3.15. b) NH4Cl

3.16. d) 9 L

3.17. b) 0,2 mol/L


+
3.18. c) VO
2

3.19. b) El valor de Kw a 60 °C es igual a 10-13,02.

638
3.20. c)

3.21. d) El átomo de Li es paramagnético.

3.22. c) El momento dipolar del BeF2 es cero por ser una molécula simétrica.

3.23. a) 234Th

3.24. b) 10 mL de ácido acético 0,1 mol/L + 5,0 mL de NaOH 0,1 mol/L

3.25. b) ∆ E° = 0,92 V y la reacción es espontánea.

4.1.
A B
CH2
CH3
CH3CH2COCl

C D
O CH3
CH3 CH3

HO

4.2.
1) CH3MgBr - K2Cr2O7, 1) PhMgBr HA 1) B2H6
ó H2SO4 ó calor ó
CH3Li PhLi BH3
- CrO3
2) H2O 2) H2O 2) NaOH,
- NaOCl H2O2

639
XXII Olimpiada Nacional de Química. 3er Examen Nivel A y B.
RESPUESTAS

1.1. ΔH fus  λ fus M  (98.62 J/g)(58 g/mol)  5719.6 J/mol

2
MRTfus (5.8x10-3 kg/mol)(8.314 J/mol K)(178.7 K 2 )
Kc    2.6921 K kg/mol
ΔH fus 5719.6J/mol

1.2. ΔTfus  K c m  (2.96 K kg/mol)(15 g)(1000 g/kg)/(180 .16 g/mol)(150 g)  1.643 K


Tfus  Tfus - ΔTfus  178.7 K - 1.643 K  177.957 K

1.3. C: 60 g/12 g/mol = 5 ÷ 2.22 = 2.25

H: 4.44 g/1 g/mol = 4.44 ÷ 2.22 = 2.00

O: 35.55 g/16 g/mol = 2.22 ÷ 2.22 = 1.00

La fórmula mínima es: (C2.25H2O)n = 180.16 g/mol 180.16/45 = 4

La fórmula del compuesto es (C2.25H2O)4 = C9H8O4

1.4. ΔTeb  K b m; m  (ΔΔeb /K b )  (56.9 - 56.1)K/(1. 71 k kg/mol)  0.4678 mol/kg

1.5. ln P2  - ΔH V  1 - 1 ;  1    1 - R ln P2


P1 R  T2 T2   T2   T1 ΔH v  P1

ΔH v  λ v M  (501.7 J/g)(58 g/mol)  29098.6 J/mol

1  1 8.314 J/mol K  85300 Pa


    - ln  3.08639x10  3
 T2   329.25 K 29098.6 J/mol  101325 Pa

T2 = 324.00 K

1.6. log10 P  A - B  4.4245 - 1312.3


 0.5144
TC 298.15  (-32.445)

P = 10-0.5144 = 0.3959 bar

1.7. log10 P  A - B  T
-C-B
; P  13.133 kPa  0.1333 bar
TC log10 P - A
- (-31.445) - 1312.3
T  280.018 K
log10 0.133 - 4.4245

640
1.8. ntot = nacetona + ncompuesto = 2.5 mol + 3.5 mol = 6.0 mol

Xacetona = 2.5 mol/6.0 mol = 0.41666; Xcompuesto = 3.5 mol/6.0 mol = 0.58333

P = (0.41666)(35.5 kPa) + (0.58333)(40.3 kPa) = 38.3 kPa

1.9. P tot  X1P1  X 2 P2  38.38 kPa  X1 (35.5)  X 2 (40.3)

X1  X 2  1

Resolviendo las ecuaciones simultáneas: X2 = 0.6 y X1 = 0.4

nacetona = 116 g/58 g/mol = 2 mol; Xacetona = 2 mol/ntot

ntot = 2 mol/Xacetona = 5 mol; ncompuesto = Xcompuestontot = 0.6(5mol) = 3 mol

ncompuesto = Wcompuesto/Mcompuesto

Mcompuesto = Wcompuesto/ncompuesto = 222.0 g/3 mol = 74 g/mol

2.1. Ag+ (ac) + Cl- (ac) → AgCl (s)

2.2. (0,840 L)(0,5 mol/L)(169,91 g/mol) = 71,3622 g AgNO3

2.3. Cu (s) + 2AgNO3 (ac) → Cu(NO3)2 (ac) + 2Ag (s)

2.4. X = mol Ag; Y = mol Cu

1,52 g = X(108 g/mol) – Y(63,5 g/mol)

Y como la relación es 1 mol Cu por 2 mol Ag; Y = 0,5X; sutituyendo en la ecuación anterior:

1,52 g = X(108 g/mol) – (0,5X)(63,5 g/mol); resolviendo la ecuación: X = 0,0199 mol Ag

(0,0199 mol)(107,9 g/mol) = 2,147 g Ag

2.5. Y = 0,5X = (0,5)(0,0199 mol) = 0.00995 mol Cu

2.6. X = mol NaCl; Y = mol KCl

X + Y = 0,40 mol y X(58,5 g/mol) + Y(74,6 g /mol) = 25 g

Resolviendo las ecuaciones simultáneas: X = 0,3006 mol Na Cl y Y = 0,0994 mol KCl

mNaCl = 17,585 g y mKCl = 7,415 g

70,34 % NaCl y 29,66 % KCl

641
2.7. (0,840 L)(0,5 mol/L) – 0,0199 mol = 0,4001 mol AgNO3

3.1. log P0  0,4


0.40P0

3.2. 0,4 = (2,1x104 cm2/mol)(1 cm)(c)


c = 0,4/2,1x104 cm3/mol = 1,9x10-5 mol/cm3 = 1,9 x10-2 mol/L

3.3. [A] = 2x10-2 M – 1,9x10-2 M = 1x10-3 M

[A] 1x10 3 M
pKa  pH  log  8,8  log  7,5
[B] 1,9x10 2 M

4.1.
Frasco Fórmula del
Reacción de identificación
No. compuesto
1 NH4Cl NH4Cl (ac) + NaOH (ac) → NH3 (g) + NaCl (ac) + H2O (l)
2 KCl KCl (ac) + AgNO3 (ac) → AgCl (s) + KNO3 (ac)
3 Ba(OH)2 Ba(OH)2 (ac) + Na2SO4 (ac) → BaSO4 (s) + 2NaOH (ac)
4 AgNO3 AgNO3 (ac) + KI (ac) → AgI (s) + KNO3 (ac)
5 KI KI (ac) + Pb(NO3)2 (ac) → PbI2 (s) + 2KNO3 (ac)
6 NH4I NH4I (ac) + NaOH (ac) → NH3 (g) + NaI (ac) + H2O (l)
7 Na2SO4 Na2SO4 (ac) + Ba(OH)2 (ac) → BaSO4 (s) + 2NaOH (ac)
8 Pb(NO3)2 Pb(NO3)2 (ac) + 2NaOH (ac) → Pb(OH)2 (s) + 2NaNO3 (ac)
9 NaOH 2NaOH (ac) + Pb(NO3)2 (ac) → Pb(OH)2 (s) + 2NaNO3 (ac)
10 Ag2SO4 Ag2SO4 (ac) + 2KI (ac) → 2AgI (s) + K2SO4 (ac)

5.
1 2 3

He Li N

4 5 6

Ne Al Cl

7 8 9

Ti Ni Cu

642
6.1. Kp = (1.7)2 / 0.65 = 4.44

6.2. Las presiones parciales se reducen a la mitad: PA = 0.325 bar; PB = 0.85 bar

Por Le Châtelier, al aumentar el volumen en el sistema se producirán más moléculas de B


(aumentarán las moles de B).

PA = 0.325 – x PB = 0.85 + 2x

(0.85  2x) 2
4.4   resolviendo la ecuación x  0.08683
(0.325 - x)

PA = 0.325 – 0.08683 = 0.23817 bar PB = 0.85 + 2(0.08683) = 1.02366 bar

NOTA INTERESANTE: PT = 1.261 bar (esta presión no es la mitad de la presión inicial, 2.35
bar, porque no se mantiene constante las moles del sistema).

Supongamos que T es 298.15 K

ninicial = (2.35 bar)(5 L) / (0.08314 L bar/mol K)(298.15 K) = 0.4740 mol

nfinal = (1.261 bar)(10 L) / (0.08314 L bar/mol K)(298.15 K) = 0.5087 mol

7. Una red cúbica centrada en las caras contiene 4 átomos. Relacionando la masa molar con el
número de átomos, la densidad y la constante de Avogadro se obtiene el volumen del cubo.

(58,7 g/mol)(4 átomos)


V  4,36x10  23 cm 3
(8,94 g/cm 3 )(6,022x10 23 átomos/mol)

a  3 4,36x10  23 cm 3  3.52x10  8 cm

8.1.

cis R cis

o R o meso

Z D RyS

8.2.
A B C
Br MgBr OH

CH3 CH3

643
I II III

Mg O HA
1)

CH3 CH3
(u otro
deshidratante)
2) H2O

8.3.
A B C
CH3 O O
N
HO Br
OH
O2N

XXII Olimpiada Nacional de Química. Examen Internacional.


RESPUESTAS

1.1. N: 11.97 g/14 g/mol = 0.855 mol 0.855 mol/0.855 mol = 1


H: 3.45 g/1 g/mol = 3.44 mol 3.44 mol/0.855 mol = 4
O: 41.03 g/16 g/mol = 2.56 mol 2.56 mol/0.855 mol = 3

NH4O, se trata de una sal de amonio.

1.2. NH4MO3, 100% - (11.97% + 3.45% + 41.03%) = 43.55% M.

43.55 g/0.855 mol = 50.94 g/mol ⇒ Masa molar de M

Por lo tanto M = V, entonces el compuesto es NH4VO3

1.3. Especie más reductora es: SO2

1.4. Especie más oxidante es: Cl2

1.5. Indica sobre la línea correspondiente la especie química identificada para cada letra

M=V A = NH4VO3 B = V2O5 C = VO2+ D = VO3- E = V2+

F = VO2+ G = V3+ I = VCl4 J = VCl3 K = VCl2

644
1.6. Ecuaciones de las reacciones 1 a 14.

1 2NH4VO3 (s) → 2NH3 (g) + V2O5 (s) + H2O (g)

2 V2O5 (s) + 2H3O+ (ac) → 2VO2+ (ac) + 3H2O (l)

3 2VO2+ (ac) + 3Zn (s) + 8H3O+ (ac) → 2V2+ (ac) + 3Zn2+ (ac) + 12H2O (l)

4 NH4VO3 (s) → NH4+ (ac) + VO3- (ac)

5 VO3- (ac) + 2H3O+ (ac) → VO2+ (ac) + 3H2O (l)

6 2VO3- (ac) + SO2 (g) + 4H3O+ (ac) → 2VO2+ (ac) + SO42- (ac) + 6H2O (l)

7 VO2+ (ac) + V2+ (ac) + 2H3O+ (ac) → 2V3+ (ac) + 3H2O (l)

8 V (s) + 2Cl2 (g) → VCl4 (l)

9 2VCl4 (l) → 2VCl3 (s) + Cl2 (g)

10 2VCl3 (s) + H2 (g) → 2VCl2 (s) + 2HCl (g)

11 VCl4 (l) + 3H2O (l) → VO2+ (ac) + 4Cl- (ac) + 2H3O+ (ac)

12 2VCl3 (s) → 2V3+ (ac) + 6Cl- (ac)

13 VCl2 (s) → V2+ (ac) + 2Cl- (ac)

14 VCl3 (s) → VCl2 (s) + VCl4 (l)

********************************************************************************
2.1. VN  YN VTot  0.78(1 m 3 )  0.78 m3
2 2

PVN (105 Pa)(0.78 m 3 )(28 g/mol)


mN  2
  881.06 g N 2
2
RT (8.314 Pa m 3 /mol K)(298.15 K)

3
2.2. m Aire  PVAire  (105 Pa)(1 m )(28.82 g/mol)  1162.65 g Aire
3
RT (8.314 Pa m /mol K)(298.15 K)

mN
% mN  2
x100  75.78%
2
m Aire

2.3. De la ecuación de Antoine se calcula la temperatura de ebullición del nitrógeno en la Cd. de


México:

645
 B   264.651 
T  - C   (3.7362 - log0.7799)  - (-6.788)  75.63 K
 (A - logP)   

2.4. Suponiendo la reacción completa: ½ N2(g) + 3/2 H2(g)  NH3(g)

Por cada mol de nitrógeno se producen 2 moles de amoniaco.

n = moles de nitrógeno = 1000 g/28 g/mol = 35.714 mol

moles de amoniaco = 2 x 35.7134 mol = 71.428 mol

masa de amoniaco = (71.428 mol)(17 g/mol) = 1214.3 g

2.5. NH3(g)  ½ N2(g) + 3/2 H2(g)

Para la reacción de formación del NH3 (g), Gor = -16.5 kJ/mol

Para la disociación cambia de signo; Gor = 16.5 kJ/mol

Kpdisociación = exp(-Gor/RT) = exp(-16500 J/[(8.3144 J/mol K)(298.15 K) = 1.2858x10-3

2.6. Para el nitrógeno: nN2 = 1000 g/28 g/mol = 35.714 mol


Para el amoniaco: nNH3 = 2 nN2 = 2 x 35.714 mol = 71.428 mol NH3 (100%)
Si se disocia el 5% queda el 95% sin disociar: nNH3 = 0.95(71.428 mol) = 67.857 mol
Es decir, la masa de amoniaco es mNH3 = (67.857 mol)(17 g/mol) = 1153.57 g

********************************************************************************
3.1.
I) A
1) O3 O
3) LiAlH4 OH
2) CH3SCH3 4) H2O

O OH

646
II) CH3
CH3 CH3
CH3 Br

1) Br2, h
2) EtO Na 3) R-CO3H

4) CH3OH, H

B
OCH3

CH3

OH

Br MgBr
OH
III)

2) Mg
1) HBr

O
3)

4) H2O

OH
O
C
CH3 6) H2N-NH2, KOH
OH 5) PCC
, calor
HO

647
3.2.
O
I)
CH3O OCH3

O O

H , calor,
HO OH - H2 O

O O
CH3O OCH3

O O

1) LiAlH4
2) H2O

O O

OH OH

H , calor

C
OO

+
HO OH

648
II)
A
O
HO
MeO 1) 4 MeMgBr
OMe
2) H2O
OH
O

HCl

C B
Cl

AlCl3 Cl

********************************************************************************

4.
Tabla “B”
Muestra pH Azúcares HMF Vitamina
Reductores ppm C
% µg en ppm
disolución en jugo de
medida naranja
I 4.0 12 20 36.8 353
II 4.0 4.8 < 0.03 33.5 643
III 3.5 4.1 < 0.03 26.3 505
IV 3.6 8.2 37 20.6 49.4
Norma mexicana 3 a 4 3.5 a 12 < 20 >300

De acuerdo con tus resultados, el mejor jugo es: II

La peor calidad la tiene la muestra: IV

Una muestra fue adulterada con un azúcar reductor (glucosa); lo más probable es que se trate de
la muestra: I
1 Vit C + 2 [Fe(CN)6]3- → 1 Vit Cred + 2 [Fe(CN)6]4-

********************************************************************************

5.1.
Ánodo: Cu (s) →Cu2+ (ac) + 2e-

649
Cátodo: 2H+ (ac) + 2e- → H2 (g)

5.2. (600 mA)(1800 s) = 1080 C

1080 C/1.602x10-19 C/e- = 6.74x1021 e- =Número de electrones

5.3. 14 g – 13.6446 g = 0.3554 g

6.74x10 21 e 
Átomos Cu   3.37x10 21
2e  /Átomos Cu

0.3554 g
Masa de un átomo Cu  21
 1.055x10 - 22 g
3.37x10 átomos Cu

5.4.
63.546 g/mol
Constante de Avogadro experimental   6.0233x10 23 mol-1
1.055x10- 22 g

5.5. Átomos de Cu = moléculas de H2

3.37x1021 moléculas H2 ≡ 0.056 mol H2 ≡ 0.01129 g H2

650
XXIII Olimpiada Nacional de Química. 1er Examen Nivel A y B.
RESPUESTAS

1) B 6) A 11) D 16) A 21) D 26) B


2) C 7) C 12) A 17) C 22) D 27) D
3) B 8) C 13) B 18) B 23) B 28) D
4) A 9) D 14) C 19) A 24) B 29) A
5) A 10) B 15) A 20) A 25) B 30) B

XXIII Olimpiada Nacional de Química. 2do Examen Nivel A y B.


RESPUESTAS

1. D) [H2SO4] = 0,54 mol/L; [H+] = 0,61 mol/L; pH = 0,21

2. B) No se cumple el producto iónico necesario para que se forme precipitado.

3. C) 4,7

4. D) Se obtiene CuY2− y MnY2− en concentración 0,001 mol/L cada uno.

********************************************************************************
5.1. Wtotal = Wsacarosa + Wagua = 135 g + 315 g = 450 g

% sacarosa = (135 g / 450 g) x 100 = 30 %

5.2. Wsacarosa = 135 g; MMsacarosa (C12H22O11) = 342 g/mol

nsacarosa = W/MM = 135 g / 342 g/mol = 0.39474 mol

Vsolución = Wsolución /densidadsolución = 450 g / 1.127 g cm-3 = 399.3 cm3

Concentración molar = 0.39474 mol / 0.3993 L = 0.98858 mol/L

5.3. Wdisolvente = 315 g = 0.315 kg

molidad = molsoluto/kgdisolvente = 0.39474 mol / 0.315 kg = 1.25 mol/kg

5.4. Wsacarosa = 135 g = 1.35x105 mg; Wsolución = 0.450 kg

ppm = 1.35x105 mg / 0.450 kg = 3x105 ppm

5.5. nsacarosa = 0.39474 mol; nagua = 315 g / 18 g/mol = 17.5 mol; ntotales = 17.89474 mol

Fracción molsacarosa = 0.39474 mol / 17.89474 mol = 0.02206

5.6. Por cada 100 g de solución hay 60 g de sacarosa y 40 g de agua.

651
nsacarosa = 60 g/342 g/mol = 0.17544 mol

molalidad = 0.17544 mol / 0.040 kg = 4.386 mol/kg

Tfus = Kc m = (1.86 K kg/mol)(4.386 mol/kg) = 8.16 K

Tfus = Tfus - Tfus = 0 oC - 8.16 oC = -8.16 oC

5.7. Vsolución = Wsolución /densidadsolución = 100 g / 1.286 g/cm3 = 77.76 cm3 = 0.7776 L

nsacarosa = 0.17544 mol

Concentración molar = 0.17544 mol / 0.7776 L = 0.2256 mol/L = 225.6 mol/m3

 = RTC = (8.314 Pa m3/mol K)(293.15 K)( 225.6 mol/m3) = 5.498x106 Pa

********************************************************************************
6. D) Ninguna es cierta.

7. B) C y E

8. A) 1/16

9. B) Los cloratos son sales del ácido clórico y ejemplos de estas sales son NaClO3, KClO3,
Ca(ClO3)2.

10. C) Fosfina 20; permanganato de potasio12

11. (Sólo nivel B) C) 20 m2

12. (Sólo nivel B) B) Entre 20 g cm−3 y 180 g cm−3

********************************************************************************

652
11. (Sólo nivel A)
A B C D

Br Br
Br
Br

Br OH

Br

E F G+H

OH OH
+

OH O CH2

12. (Sólo nivel A)


A B C

O O

XXIII Olimpiada Nacional de Química. 3er Examen Nivel A y B.


RESPUESTAS

1.1. 3Pb (s) + 8HNO3 (ac) → 3Pb(NO3)2 (ac) + 2NO (g) + 4H2O (l)
Ag (s) + 4HNO3 (ac) → 3AgNO3 (ac) + NO (g) + 2H2O (l)

1.2. Pb2+ (ac) + 2IO31- (ac) → Pb(IO3)2 (s)


Ag1+ (ac) + IO31- (ac) → AgIO3 (s)

1.3. Lluvia de oro, precipitación con cloruros.

1.4. 0,250 g = (108 g/mol)X + (207,2 g/mol)Y X = mol Ag; Y = mol Pb


0,6605 = (282,9 g/mol)X + (557 g/mol)Y

Resolviendo las ecuaciones: X = 0,00157 mol Ag; Y = 0,00039 mol Pb

Plata: 169,6 mg, 67,82%; Plomo: 80,81 mg, 32,32%

2.1. Na2CO3 (s) + 2HCl (ac) → 2NaCl (ac) + CO2 (g) + H2O (l)
NaHCO3 (s) + HCl (ac) → NaCl (ac) + CO2 (g) + H2O (l)

653
2.2. Neutro.

2.3. 0,500 g = (106 g/mol)X + (84 g/mol)Y X = mol Na2CO3; Y = mol NaHCO3
0,450 g = 2X(58,5 g/mol) + (58,5 g/mol)Y

Resolviendo las ecuaciones: X = 0,0023 mol Na2CO3; Y = 0,0030 mol NaHCO3

Na2CO3: 243,8 mg, 48,8%; NaHCO3: 252 mg, 50,4%

2.4. [CO32-] = 0,00235 M; [HCO31-] = 0,00297 M

pH = pKa + log(0,00235/0,00297) = 10,3 – 0,1 = 10,2

1 mol 1000 mL
2.5. a) (0,045 g)( )( )  4,25 mL
106 g 0,1 mol
1 mol 1 mol
b) 2(0,045 g)( )  0,035 g( )  0,00127 mol HCl
106 g 84 g
(0,00127 mol)(1000 mL/0,1 mol) = 12.7 mL

2.6. Para el primer volumen: Fenolftaleína

Para el segundo volumen: Rojo de metilo

■■■■■■■■■■■■■■■■■■■■■■■■■■■■■■■■■■■■■■■■■■■■■■■■■■■■■■■■■■■■■■■■■■

3.1. naire = PV/RT = (101325 Pa x 48 m3)/(8.314 Pa m3/mol K x 268.15 K) = 2181.57 mol

maire = nM = (2181.57 mol)(29 g/mol) = 63265.7 g ≈ 63.27 kg

3.2. Q = mCpΔT = (1000 g)(1.012 J/g oC)[20 oC –(-5 oC)] = 25300 J

3.3. Masa de aire en un m3:

m = PVM/RT = (101325 Pa x 1 m3 x 29 g/mol)/(8.314 Pa m3/mol K x 268.15 K) = 1318.03 g

Q = mCpΔT = (1318.03 g)(1.012 J/g oC)[20 oC –(-5 oC)] = 33346.16 J

El calor de combustión del carbón es: -393505.2 J/mol

ncarbón = Q/Qcomb = (33346.16 J)/(393505.2 J/mol) = 0.0847413 mol

mcarbón = (ncarbón)(Mcarbón) = (0.0847413 mol x 12 g/mol) = 1.0169 g

3.4. El agua debe de proveer 16736 kJ al enfriarse y congelarse a la temperatura de 0 oC. El calor
cedido por el agua es el sensible y el latente.

654
Para cada cubeta de agua se tiene:

Qcedido = mCpΔT = 10000 g x 4.184 J/g oC x (0-20) oC= -836800 J

Qcongelar = -Qlatente = –mλfusión = -(10000 g/18 g/mol)x(6025 J/mol) = -3347222 J

Qtotal por cubeta = -836800 J + (-3347222 J) = -4184022 J = -4184.022 kJ

Número de cubetas = 16736 kJ/4184.022 kJ = 3.99999 cubetas ≈ 4 cubetas

3.5. La galleta contiene 75% de granola, es decir, 25 g x 0.75 = 18.75 g de granola

Que proporcionan: (18.75 g)(397.5 kJ/21 g) = 354.91 kJ

La galleta contiene 25% de chocolate, es decir, 25 g x 0.25 = 6.25 g de chocolate

Que proporcionan: (6.25 g)(2664 kJ/100 g) = 166.5 kJ

Una galleta proporciona: 354.91 kJ +166.5 kJ = 521.41 kJ = 124.62 kcal

Número de galletas = 200 kcal/(124.62 kcal/galleta) = 16.048 galletas.

■■■■■■■■■■■■■■■■■■■■■■■■■■■■■■■■■■■■■■■■■■■■■■■■■■■■■■■■■■■■■■■■■■

4.1. N2O (I+); NO (II+); N2O3 (III+); NO2 (IV+); N2O4 (IV+); N2O5 (V+).

4.2. (4.99x10-23 g/ 1 molécula)(6.02x1023 moléculas/1 mol) =30 g/mol

4.3. (28.8 km)(1 mol/10 km)(30 g/1 mol) = 86.4 g

4.4. 86.4 g/28.8 km = 3 g/km

Si la excede. Esta cantidad es 37.5 veces mayor a la establecida por la Norma Euro-6.

5.1. CaCO3 (s) + 2HCl (ac) → CaCl2 (ac) + H2O (l) + CO2 (g)

5.2. (185 mL disol.)(1.081 g disol./1 mL disol.)(15 g HCl/100 g disol.) = 30 g HCl

5.3. CaCO3 (s) + 2HCl (ac) → CaCl2 (ac) + H2O (l) + CO2 (g)

La mas inicial de los componentes = masa disol. HCl + masa CaCO3 añadido.

Masa disol. HCl = (185 mL disol.)(1.082 g disol./1 mL disol.) = 200g

Masa CaCO3 añadido = (100 g/mol)X X = moles de CaCO3 añadido, y este es el


reactivo limitante

El balance de materia establece que la masa final del sistema debe ser:

655
200 g + (100g/mol)X – (44 g/mol)X

La masa final de HCl es igual a la masa inicial de HCl menos lo que reaccionó:

Masa final de HCl = [30 g – (2 x 36.5 g/mol x X)]

La masa de HCl es el 5.5% de la disolución final, por lo tanto el valor de X será:

0.055 = (30 - 73X)/(200 + 100X - 44X); X = 0.25 mol

La masa de la disolución final es

200 g + (100 g/mol)(0.25 mol) – (44 g/mol)(0.25 mol) = 214 g

5.4. Conocido el valor de X = 0.25 mol, la masa del CaCO3 añadido es: 100X = 25 g

Si consideran 212 g, el valor de X necesario para calcular la masa del trozo de CaCO3
añadido es: 200 – 100X - 44X = 212 g; X = 0.21 mol; Masa CaCO3 = 100X = 21 g

5.5. La cantidad de HCl que quedó sin reaccionar es el 5% de 30 g = 1.5 g

El volumen de la disolución se mantiene constante.

(1.5 g/185 mL)(1 mol HCl/36.5 g)(1000 mL/1 L) = 0.22 mol HCl

(185 mL)(0.22 mol/1000 mL)(1000 mL/4.5 mol) = 9.04 mL de KOH

■■■■■■■■■■■■■■■■■■■■■■■■■■■■■■■■■■■■■■■■■■■■■■■■■■■■■■■■■■■■■■■■■■

6. (Sólo Nivel B).


1 2 3
He Li N
Helio Litio Nitrógeno

4 5 6
Ne Al Cl
Neón Aluminio Cloro

7 8 9
Ti Ni Cu
Titanio Níquel Cobre

■■■■■■■■■■■■■■■■■■■■■■■■■■■■■■■■■■■■■■■■■■■■■■■■■■■■■■■■■■■■■■■■■■

656
6. (Sólo Nivel A).
A B C

O O

7. (Sólo Nivel A).


A B C D E
O HO
EtO2C CO2Et HO2C CO2H HO2C

OH OH
OH OH

8. (Sólo Nivel A).


A B C D
CN Cl
CN
CN
CN

E F G
Cl
Cl

Cl Cl

657
9. (Sólo Nivel A).
a b c d e
CH3I O
NaOH AlCl3 MgCl
(Me2SO4)
Cl

f A B
CH3O
HCl, H2O OCH3

C D E
O
CH3O O Br O

CH3O

XXIII Olimpiada Nacional de Química. Examen Internacional.


RESPUESTAS

Problema 1.
1.1. XA = 3 mol/10 mol = 0.3; XB = 7 mol/10 mol = 0.7

P = PA + PB = (0.3 x 150 mmHg) + (0.7 x 350 mmHg) = 290 mmHg

1.2. YB = PB/P = 245 mmHg/290 mmHg = 0.845

1.3. 1/P = YA/PA* + YB/PB* = (0.3/150 mmHg) + (0.7/350 mmHg) = 4.0 x10-3 mmHg-1

P = 1/4.0 x10-3 mmHg-1 = 250 mmHg

1.4. XB = YB/PB* = (0.7)(250 mmHg/350 mmHg) = 0.50

1.5. XB = (P – PA*)/(PB*-PA*) = (270 mmHg – 150 mmHg)/(350 mmHg – 150 mmHg) = 0.6

1.6. YB = (XBPB*)/P = (0.6 x 350 mmHg)/270 mmHg) = 0.777

1.7. nB(líq) = XBnlíq = (0.6)(4.375 mol) = 2.625 mol

1.8. nB(vap) = YBnvap = (0.777)(5.625 mol) = 4.375

Problema 2.
2.1. 1 g N2 = 0.0357 mol; R = 62.36 mmHg L/mol K
658
P = (0.0357 mol)(62.36 mmHg L/mol K)(298.15 K)/ 1 L = 663.76 mmHg

2.2. n = (1x10-7)(1)/[(8.314)(223.15)] = 5.39x10-14 mol

2.3. n = (10-4)(1)/[(62.36)(223.15)] = 7.186x10-9 mol = 4.328x1015 moléculas

Moléculas adsorbidas = 2.164x1015 moléculas

(2.164x1015 moléculas)(0.162x10-7 cm)2 = 0.568 cm2

(0.568 cm2)(100)/2 cm2 = 28.4%

A) Se alcanza a recubrir la totalidad de la superficie metálica.

********************************************************************************
Problema 3.
A B C D
O O
CN NH2
Cl NH2

Problema 4.
A B C

O
H H
N N N

Problema 5.
B E
HO OH
(CH2)7
O Br
O
(CH2)5 (CH2)7
OH (CH2)5
HO
EtO

C F O
Br Br
(CH2)7
O O
O
(CH2)5 (CH2)7
Br (CH2)5
HO
EtO

D Br G
Br
(CH2)7
O OH
O
(CH2)5 (CH2)7
Br (CH2)5
EtO
EtO

659
Problema 6.
A B C

O O

Br OH OH

O OH

D CAPSAICINA
O
O
O
N
Cl
H

OH

********************************************************************************

Problema 7.
7.1. Mprom. = (431.9798 x 0.75) + (437.9221 x 0.25) = 433.46

7.2. Ba (Z = 56), Cs (Z = 55), Xe (Z = 54)

E = 56 + 55 + 54 = 165

7.3. Familia 15 (5A).

7.4. +5, +3, +1, 0.

7.5. +1 y/o +3.

7.6. El de un metal.

Problema 8.
d) Cl (g) + e- → Cl- (g)

Problema 9.
Moles de C: 4.84 x10-3; %C: 23.25

Moles de H: 0.01936; %H: 7.74

Moles de Cl: 3.226 x10-3; %Cl: 22.87

Fórmula empírica: NiC6H24N6Cl2

660
Problema 10.

X
F O O
O N
F
X S
F P Cl O
Cl
F
Cl

Problema 11.
Li2O (B) SO3 (A) MgO (B) P2O5 (A) Al2O3 (AN)

Cl2O (B) CrO3 (A) Mn2O7 (A) Fe2O3 (AN) CO2 (A)

Problema 12.
K4[Co(CN)6] K2[CoCl4]

4-
CN 2-
NC CN Cl
Co
NC CN Co
CN Cl
Cl Cl
Octaédrica Tetraédrica

_↿_ ___ _↿_ _↿_ _↿_

↿⇂ ↿⇂ ↿⇂ ↿⇂ ↿⇂
1 electrón desapareado 3 electrones desapareados

Problema 13.
1 mol = 12 onzas 12C

(12 onzas 12C)(28.3495 g/ 1 onza)(6.02x1023 partículas/ 12 g 12C) = 1.71 x 1025 partículas

Problema 14.
Número de iones sodio: 4

Número de iones cloruro: 4

********************************************************************************

661
Problema 15.
15.1. [HOx]org = 1,78[HOx]ac

[HOx]org + [HOx]ac = 10 g

Resolviendo las 2 ecuaciones con 2 incógnitas se tiene:

[HOx]org = 6,4 y [HOx]ac =3,6

Masa: 6,4 g; Rendimiento: 64%

15.2.

(g HOx org )(M HOx ) 1,78 x 100


  0,593 y g HOxorg + g HOxac = 10 g
(g HOx ac )(M HOx ) 300

1) 1,593 g HOxac = 10 g ⇒ g HOxorg = 3,72 g y g HOxac = 6,277 g

2) 1,593 g HOxac = 6,277 g ⇒ g HOxorg = 2,34 g y g HOxac = 3,94 g

3) 1,593 g HOxac = 3,94 g ⇒ g HOxorg = 1,47 g y g HOxac = 2,47 g

g HOxorg = 3,72 g + 2,34 g + 1,47 g = 7, 53 g = 75.3%

15.3. Es mejor varias extracciones con volúmenes pequeños que una sola extracción con el mismo
volumen.

Problema 16.
16.1.
Tipo de hidrógeno Número de picos
CH2 CH3
CH3 3
H2C CH
CH2 5
H3C CH2 CH3
CH 7

3 4 Tipo de hidrógeno Número de picos


H3 C C CH
6 tipos de hidrógenos Todas son señales
O N C O
simples
C CH2 HC C
2 5
H 3C O CH3
1 6

662
XXIV Olimpiada Nacional de Química. 1er Examen Nivel A y B.
RESPUESTAS

1) A 13) D 25) C
2) B 14) A 26) B
3) C 15) C 27) A
4) A 16) B 28) C
5) C 17) A 29) C
6) D 18) D 30) A
7) C 19) B
8) A 20) B
9) D 21) X
10) C 22) B
11) A 23) A
12) B 24) A

XXIV Olimpiada Nacional de Química. 2do Examen Nivel A y B.


RESPUESTAS

1.1.
PV 1 x 33.5
n   1.39 mol
RT 0.082 x 293

1.39 mol x 29 g/mol = 40.31 g

1.2.
PV 1.5 x 33.5
n   2.091 mol
RT 0.082 x 293

2.091 mol x 4 g/mol = 8.366 mol

1.3.
40.41 – 8.36 = 32.05 g

2.1.
PV 1 x 33.5
n   1.37 mol
RT 0.082 x 298

60% de 1.37 mol = 0.82 mol 40% de 1.37 mol = 0.55 mol

0.82 mol x 4 g/mol = 3.28 g 0.55 mol x 29 g/mol = 15.95 g

3.28 g + 15.95 g = 19.23 g

663
m 19.23 g
   0.574 g /L
V 33. 5 L

2.2.
1.37 mol = 19.23 g
1 mol = X = 14.03 g

3.
PV PV
m aire  m He  4000 g
RT RT

4000 RT 4000 x 0.082x293


V   3844.16 L
P(m aire  m He ) 1(29 - 4)

4. C) Un compuesto iónico de fórmula M2Q.

5. A) 1/16

6. A) Mayor proporción del cloro 35 que de cloro 37.

7. D) Todas son ciertas.

8. C) Incrementa la electronegatividad.

9. D) Pirámide triangular

10. D) La molécula de agua puede actuar como ácido y como base de Brönsted-Lowry.

11. C) Sus átomos y moléculas son paramagnéticos.

12. C) Los dos ácidos necesitan igual volumen de la disolución de sosa.

13. C) NH3

14. C) el azufre se oxida y el nitrógeno se reduce

15. C) 12,35

16. B) III+ y 6

17. A) Igual a la inicial

18. C) Zn

19. D) 2,5
664
20. C) 93,08

21. C) III+

22. B) Cu (s)

23. A) 0,298

24. B) Pb2+ (ac)

25. D) 1,85 mol/L

26.

H2O2 Fe2(SO4)3

X XX

Agua oxigenada o peróxido de hidrógeno Sulfato de hierro(III)

Na2B4O3·10H2O KMnO4

Tetraborato de sodio decahidratado Permanganato de potasio

CH3CH2CH3 CH3CH2CH2CH3

Propano Butano

C6H12 (CH3)2CHOH

Isopropanol

Ciclohexano

C6H6 (CH3)2C=O

Acetona o Propanona
Tolueno

665
27.
RESPUESTAS
ISÓMERO DIMETILBENCENO COMPUESTO I COMPUESTO II

CH3 CH3 CH3

CH3 CH3 CH3

NO2 O2N

28.a.
RESPUESTA
O

Cl O

AlCl3

28.b.
RESPUESTA
CH 1) BH3, THF OH K2Cr2O7
CH2 H2SO4 OH
2) NaOH, H2O2 O

1) O3, CH2Cl2 1) Mg, THF


2) CO2 1) NaOH, H2O
2) NaBH4 calor
3) HCl, H2O
2) HCl, H2O

PBr3 Br NaCN CN
OH
PBr5
HBr, H2SO4

Br2, PPh3

666
29.
B)

XXIV Olimpiada Nacional de Química. 3er Examen Nivel A y B.


RESPUESTAS

1.1. ΔHo = 2(33.9)-9.7 = 58.1 d) 58.1 kJ/mol

1.2. ΔSo = 2(240)-304.3 = 175.7 c) +175.7 J/mol K

1.3. ΔGo = ΔH - TΔS = 58.1-(298.15x0.1757) = 5.715 kJ/mol a) 5.71 kJ/mol

1.4. ΔG = R T ln K

ΔG 4503
ln K     1.786
RT 8.314 x 308.15

K  e 1.786  0.1675 b) 0.1675

2
PNO
1.5. Kp  2
 0.11367
PN2O4

PN O  PNO  1
2 4 2
Sistema de 2 ecuaciones con 2 incógnitas. Al resolverla:
PN 2O4
 0.715 ; PNO  0.285
2
d) 0.715 bar

■■■■■■■■■■■■■■■■■■■■■■■■■■■■■■■■■■■■■■■■■■■■■■■■■■■■■■■■■■■■■■■■■■
2.1. B) entre 3 y 4 gramos

2.2. C) entre 3.6 y 4 kcal.

2.3. D) más de 4 kcal.

2.4. B) entre 100 y 150 L.

2.5. C) entre 201 y 250 g.

2.6. D) más de 500 g.

2.7. B) entre 1500 y 2000 g.

667
■■■■■■■■■■■■■■■■■■■■■■■■■■■■■■■■■■■■■■■■■■■■■■■■■■■■■■■■■■■■■■■■■■

3.1. Ánodo X

3.2. 2 I- - 2e- → I2

3.3. 2 H2O + 2e- → H2 + 2 OH-

3.4. H2S + I2 → S + 2 I- + 2 H+

3.5. Detectar la aparición de exceso de yodo cuando reaccionó todo el H2S.

3.6. ((2 x 60) + 35)(0.002) = 0.31 C

3.7. (0.31) / (96500 x 2) = 1.61 x 10-6 mol

1000 x (1.61 x 10-6 x 34.1)/2 = 0.275 mg L-1

3.8. Superior al máximo permitido X

■■■■■■■■■■■■■■■■■■■■■■■■■■■■■■■■■■■■■■■■■■■■■■■■■■■■■■■■■■■■■■■■■■

4.1. Fracción en masa de cloro: 74.47 / 100 = 0.7447

Fracción en masa de R: 1 – 0.7447 = 0.2553

n ElementoMM Elemento
4.2. Composición porcentual de un elemento  x100
MM Compuesto
n Cl MM Cl n R MM R
Cloro: 74.47 %  x100 R: 25.53 %  x100
MM RCln MM RCln

n Cl MM Cl n R MM R
MM RCln  x100 MM RCln  x100
74.47 % 25.53 %

n Cl MM Cl n R MM R n Cl (35.45)(25.53)
 MM R 
74.47 25.53 (1)(74.47)

nCl = 1 mol MMR = 12.15 g/mol


nCl = 2 mol MMR = 24.31 g/mol (Esta MM corresponde a un metal)
nCl = 3 mol MMR = 36.46 g/mol
nCl = 4 mol MMR = 48.61 g/mol

La relación es: 1 de R por 2 de Cl.

La masa molar es: 24.31 g/mol.

El metal es: magnesio (Mg).

668
4.3. La fórmula del compuesto C es: MgCl2.

4.4. Fracción en masa de L = 46.76 / 100 = 0.4676


Fracción en masa de O = 1 – 0.4676 = 0.5324

n O MM O n L MM L
4.5. Oxígeno: 53.24 %  x100 L: 46.76 %  x100
MM LqOw MM LqOw

n O (16) n L MM L n O (16)(46.76)
 MM L 
53.24 46.76 n L (53.24)

nL = 1 mol y nO = 1 mol MML = 14.05 g/mol (Esta MM corresponde a un no metal)


nL = 1 mol y nO = 2 mol MML = 28.11 g/mol (Esta MM corresponde a un no metal)
nL = 1 mol y nO = 3 mol MML = 42.16 g/mol
nL = 2 mol y nO = 1 mol MML = 7.03 g/mol
nL = 2 mol y nO = 2 mol MML = 14.05 g/mol (Esta MM corresponde a un no metal)
nL = 2 mol y nO = 3 mol MML = 21.08 g/mol

Como L es el principal componente de la arena la MML es 28.11 g/mol, que corresponde al


silicio.

El valor de “q” es 1 y el de “w” es 2.

La masa molar de L es: 28.11 g/mol.

El no metal L es: silicio (Si).

4.6. La fórmula de D es: SiO2.

4.7. Z% en masa = 12.47% y Si% en masa = 100 – 12.47 = 87.53%

4MM Z (1)(28.11)
Z: 12.47 %  x100 Si: 87.53 %  x100
MM SiZ4 MM SiZ4

(4)MM Z (1)(28.11)
MM SiZ4  x100 MM SiZ4  x100
12.47 % 87.53 %

(4)MM Z (1)(28.11) (1)(28.11)(12.47)


 MM Z   1 g/mol
12.47 87.53 (4)(87.53)

La masa molar de Z es: 1 g/mol.

El elemento Z es: hidrógeno (H).

La fórmula del compuesto B es: SiH4.

669
4.8. El compuesto A contiene Mg (R) y Silicio (L); y un 63.36% de Mg.

Por lo tanto hay un 36.64% de Si.

n Mg MM Mg n Si (MMSi )
Mg: 63.36 %  x100 Si: 36.64 %  x100
MM R x Ly MM R x Ly

n Mg (24.31) n Si (28.11)
MM R x Ly  x100 MM R x Ly  x100
63.36 % 36.64 %

n Mg (24.31) n Si (28.11) n Mg (63.36)(28.11)


  2
63.36 36.64 n Si (24.31)(36.64)

La relación estequiométrica es: 2 de Mg por 1 de Si.

La fórmula del compuesto A es: Mg2Si.

4.9. ECUACIÓN 1: 2 Mg + Si → Mg2Si

ECUACIÓN 2: Mg2Si + 4 HCl → 2 MgCl2 + SiH4

ECUACIÓN 3: SiH4 + 2 O2 → 2 H2O + SiO2

FIN DEL EXAMEN NIVEL B.

■■■■■■■■■■■■■■■■■■■■■■■■■■■■■■■■■■■■■■■■■■■■■■■■■■■■■■■■■■■■■■■■■■

5.1.a.
RESPUESTAS

A B C
O

O H O N O N
NO2
OCH3 H H
OCH3 OH

670
5.1.b.
RESPUESTA
ESTRUCTURA ENANTIÓMERO R DE LA PROLINA

H
O N

H
OH

5.2.
RESPUESTAS

(a) NBS (b) (c) Jones


control temperatura
K2Cr2O7, H2SO4
O
PCC, Clorocromato
H2O, NaOH de piridinio

N Br Collins
CrO3, piridina
Swern
Cl-CO-CO-Cl
O DMSO, Et3N

(d) (e) (f) Lindlar


PCl3 H2 / Pd / BaSO4
Jones PCl5 Quinolina
K2Cr2O7, H2SO4 SOCl2
DIBAL-H
KMnO4, H2O Cl
Al
Calor
N N H

Cl N Cl NaBH(OAc)3

671
5.3.a.
RESPUESTA:

H2 1) NaNO2, HCl
Pd(C) 2) H2SO4, H2O, calor

NO2 NH2 OH

5.3.b.

RESPUESTA:

H2 Ac2O Ac2O, HNO3 NaOH


Pd(C) H2O NO2
NO2 CALOR
NO2 NH2 N N NH2
H H
O O 1) NaNO2, HCl
2) NaBH4

NO2

H3PO2 (Ácido hipofosforoso)


EtOH

XXIV Olimpiada Nacional de Química. Examen Internacional.


RESPUESTAS

I. QUÍMICA ANALÍTICA.

I.1. BrO3- + 5 Br- + 6 H+ → 3 Br2 + 3 H2O


KBrO3 + 5 KBr + 6 HCl → 3 Br2 + 3 H2O + 6 HCl

I.2. H2O + AsO+ + Br2 → AsO2+ + 2 HBr


H2O + SbO+ + Br2 → SbO2+ + 2 HBr

672
I.3. El AsO+.

 0,0049 L  0,0001 mol KBrO3  3 mol Br2  1 mol AsO 


I.4.       1,47x10 5 mol AsO /L
 0,1 L  1 L  1 mol KBrO 3  1 mol Br2 

I.5. 9,1 mL – 4,9 mL = 4,2 mL

 0,0042 L  0,0001 mol KBrO3  3 mol Br2  1 mol SbO  


      1,26 x10 5 mol SbO  /L
 0,1 L  1 L  1 mol KBrO 3  1 mol Br2 

0,4
I.6.    31746 mol1cm 1L
(1 cm)(1,26x10-5 mol/L)
 (13 mL  9,1 mL)(0,0001 mol KBrO3 /L  3 mol Br2 
I.7.     1,17x10 5 mol Br2 /L
 100 mL  1 mol KBrO 3 

I.8. As(III): (1,47x10-5 mol/L)(74,9 g/mol)(1000000 μg/1 g) = 1101,03 μg/L. NO ES APTA.

Sb(III): (1,26x10-5 mol/L)(121,8 g/mol)(1000000 μg/1 g) = 1534,68 μg/L. NO ES APTA.

(0,6)(10 mL)
I.9. ε   200 mol-1cm-1L
(2 cm)(5 mL)(0,001 mol KBrO3 /L)(3 mol Br2 /1 mol KBrO3 )

II. FISICOQUÍMICA.

II.1.1.  = (Tc – Tf)/Tc = (400 – 300)K/400 K = 0.25 a) 0.250

II.1.2.  = (Qabs + Qced)/Qabs

0.45 = (45.78 + Qced)/45.78

Qced = (0.45 x 45.78) – 45.79 = -25.179 b) -25.18 kJ

II.1.3.  = (Tc – Tf)/Tc

0.35 = (400 – Tf)/400

-Tf = (0.35 x 400) – 400 = - 260 K d) 260 K

II.1.4. PiVi = PfVf

Pf = (8 atm x 10 L)/20 L = 4 atm b) 4 atm

II.1.5. TiVi(-1) = TfVf(-1);

673
2
(  1) 2
Ti Vi 5 (400)[(20) ] 5
Vf  (  1)   41.056 c) 41.1 L
Tf 300

II.1.6. TiPi(1-) = TfPf(1-).

7 2
 (1  ) 
Ti Pi 
2
[(400) 5 ][(20) 5 ]
Pf  (1 ) 
 5
7
 1.46 b) 1.46 atm
Tf
300 5

II.1.7. P3V3 = P4V4 = K1 = (1.46142)(41.1) = 60.0

P1V1γ = P4V4γ = K2 = (8)[(10)1.4)] = 200.95

V4γ-1 = 200.95 / 60.0 = 3.3492

V4 = 3.3492 1/γ-1 = 20.528 a) 20.5 L

II.1.8. P4V4 = P3V3

P4 = P3V3/V4 = (1.4616)(41.02) / 20.5 = 2.92 c) 2.92 atm

II.2.1. La energía de activación para el proceso sin catalizador es + 21.0 kJ /mol

II.2.2. La energía de activación para el proceso con catalizador es +14.2 kJ /mol

II.2.3. La reacción de hidrogenación del eteno es exotérmica (- 6 kJ/mol)

II.2.4. 100 g / 28 g/mol = 3.57 mol

La energía es: 3.5 mol x -6 kJ = -21.42 kJ/mol

II.2.5. PV = nRT

P = nRT/V = [(53.57)(8.314x102)(293) / 20 = 65.248 bar

P inicial es 65.248 bar


II.2.6. C2H4 + H2 → C2H6

100 g de eteno = 3.57 mol


10 g de hidrógeno = 20 mol
Después de la reacción se forman 3.57 mol de etano y quedan sin reaccionar 46.43 mol de
hidrógeno.

Moles gaseosas al concluir la reacció: 50 mol.

674
III. QUÍMICA INORGÁNICA.

III.1. ELEMENTO 1:____Mg_____ ELEMENTO 2:_____Mn_____

III.2. Ca2+

III.3. Cl-

III.4. LiF

III.5. e) La primera afinidad electrónica del Na.


207
III.6. Hg

III.7.
a) C, D, H, L

b) F, I

c) K

d) B

e) H

III.8.
a) R: d) NO2

b) R: g) NH4+ y e) (CH3)2NOH

III.9. a) ଶଷସ ଶଷସ ସ


ଽଵܺ ; g) ଽ଴ܺ ; f) ଶܺ

III.10. a) SbO; b) NaSbO4; c) Na2SbO4; g) H2SbO3

III.11. c) [Cr(CO)4Cl2]Cl

III.12. c) K2[PtCl4] + 2 equivalentes de NO2- y d) trans-K2[PtCl2(CN)2] + 2 equivalentes de NO2-

III.13. Na, O2-, Cr2+

III.14. Densidad del LiF(s): _2.63 g/cm3_

675
IV. QUÍMICA ORGÁNICA.
IV.1.
RESPUESTA
Br
Br2 Br HNO3 Br Br2 Br
AlCl3 H2SO4 O N AlCl3 O N Br
2 2

1) Fe, HCl, CALOR


2) NaOH

Br Br
Br 1) NaNO2, HCl Br
2) H3PO2
H Br H2 N Br
NaBH4
EtOH

IV.2.
RESPUESTAS

A B C

OCH3
O O
O
O OH OH

D E

N O N
CH3 CH3

IV.3.
d) (2) (3) (6) (4) (5) (1)

676
IV.4.
RESPUESTAS

CEMBRENO DIHIDROCEMBRENO

OCTAHIDROCEMBRENO

H
O
O
O H
1) O3 O H O
O H
2) Me2S

H O O

677
678

Vous aimerez peut-être aussi